Está en la página 1de 228

Taller de resolucin

de problemas de concurso
Universidad de Puerto Rico
Colegio Universitario de Cayey
Dr. David A. SANTOS

Versin del 30 de enero de 2007

ndice general

Prefacio

1. Tcnicas elementales
1.1. Contradiccin . . . . . . . . . . . . . .
Tarea . . . . . . . . . . . . . . . . . . . . . .
1.2. Principio de las pichoneras de Dirichlet
Tarea . . . . . . . . . . . . . . . . . . . . . .
1.3. Paridad . . . . . . . . . . . . . . . . .
Tarea . . . . . . . . . . . . . . . . . . . . . .
1.4. Induccin . . . . . . . . . . . . . . . .
Tarea . . . . . . . . . . . . . . . . . . . . . .
1.5. Buen orden . . . . . . . . . . . . . . .
Tarea . . . . . . . . . . . . . . . . . . . . . .
1.6. Condiciones extremas . . . . . . . . . .
Tarea . . . . . . . . . . . . . . . . . . . . . .

.
.
.
.
.
.
.
.
.
.
.
.

.
.
.
.
.
.
.
.
.
.
.
.

.
.
.
.
.
.
.
.
.
.
.
.

.
.
.
.
.
.
.
.
.
.
.
.

.
.
.
.
.
.
.
.
.
.
.
.

.
.
.
.
.
.
.
.
.
.
.
.

.
.
.
.
.
.
.
.
.
.
.
.

.
.
.
.
.
.
.
.
.
.
.
.

.
.
.
.
.
.
.
.
.
.
.
.

.
.
.
.
.
.
.
.
.
.
.
.

.
.
.
.
.
.
.
.
.
.
.
.

.
.
.
.
.
.
.
.
.
.
.
.

.
.
.
.
.
.
.
.
.
.
.
.

.
.
.
.
.
.
.
.
.
.
.
.

.
.
.
.
.
.
.
.
.
.
.
.

.
.
.
.
.
.
.
.
.
.
.
.

.
.
.
.
.
.
.
.
.
.
.
.

.
.
.
.
.
.
.
.
.
.
.
.

.
.
.
.
.
.
.
.
.
.
.
.

.
.
.
.
.
.
.
.
.
.
.
.

.
.
.
.
.
.
.
.
.
.
.
.

.
.
.
.
.
.
.
.
.
.
.
.

.
.
.
.
.
.
.
.
.
.
.
.

.
.
.
.
.
.
.
.
.
.
.
.

.
.
.
.
.
.
.
.
.
.
.
.

.
.
.
.
.
.
.
.
.
.
.
.

.
.
.
.
.
.
.
.
.
.
.
.

.
.
.
.
.
.
.
.
.
.
.
.

.
.
.
.
.
.
.
.
.
.
.
.

.
.
.
.
.
.
.
.
.
.
.
.

.
.
.
.
.
.
.
.
.
.
.
.

.
.
.
.
.
.
.
.
.
.
.
.

.
.
.
.
.
.
.
.
.
.
.
.

.
.
.
.
.
.
.
.
.
.
.
.

.
.
.
.
.
.
.
.
.
.
.
.

.
.
.
.
.
.
.
.
.
.
.
.

.
.
.
.
.
.
.
.
.
.
.
.

.
.
.
.
.
.
.
.
.
.
.
.

1
1
2
2
4
4
6
6
7
8
9
9
10

2. lgebra y aritmtica
2.1. Identidades algebraicas
Tarea . . . . . . . . . . . . .
2.2. Los enteros . . . . . .
Tarea . . . . . . . . . . . . .
2.3. Aritmtica modular . .
Tarea . . . . . . . . . . . . .

.
.
.
.
.
.

.
.
.
.
.
.

.
.
.
.
.
.

.
.
.
.
.
.

.
.
.
.
.
.

.
.
.
.
.
.

.
.
.
.
.
.

.
.
.
.
.
.

.
.
.
.
.
.

.
.
.
.
.
.

.
.
.
.
.
.

.
.
.
.
.
.

.
.
.
.
.
.

.
.
.
.
.
.

.
.
.
.
.
.

.
.
.
.
.
.

.
.
.
.
.
.

.
.
.
.
.
.

.
.
.
.
.
.

.
.
.
.
.
.

.
.
.
.
.
.

.
.
.
.
.
.

.
.
.
.
.
.

.
.
.
.
.
.

.
.
.
.
.
.

.
.
.
.
.
.

.
.
.
.
.
.

.
.
.
.
.
.

.
.
.
.
.
.

.
.
.
.
.
.

.
.
.
.
.
.

.
.
.
.
.
.

.
.
.
.
.
.

.
.
.
.
.
.

.
.
.
.
.
.

.
.
.
.
.
.

.
.
.
.
.
.

.
.
.
.
.
.

11
11
15
16
20
20
24

.
.
.
.
.
.
.
.
.

26
26
30
31
32
33
35
37
39
44

4. Sumas y recurrencias
4.1. Progresiones aritmticas . . . . . . . . . . . . . . . . . . . . . . . . . . . . . . . . . . . . . . . . . . . . . .
Tarea . . . . . . . . . . . . . . . . . . . . . . . . . . . . . . . . . . . . . . . . . . . . . . . . . . . . . . . . . . . .
4.2. Progresiones geomtricas . . . . . . . . . . . . . . . . . . . . . . . . . . . . . . . . . . . . . . . . . . . . . .

45
45
47
48

.
.
.
.
.
.

.
.
.
.
.
.

.
.
.
.
.
.

.
.
.
.
.
.

.
.
.
.
.
.

.
.
.
.
.
.

.
.
.
.
.
.

.
.
.
.
.
.

.
.
.
.
.
.

3. Combinatoria
3.1. Las reglas de la multiplicacin y la suma
Tarea . . . . . . . . . . . . . . . . . . . . . .
3.2. Mtodos combinatorios . . . . . . . . .
3.2.1. Permutaciones sin repeticin . .
3.2.2. Permutaciones con repeticin .
3.2.3. Combinaciones sin repeticin .
3.2.4. Combinaciones con repeticin .
3.3. Principio de inclusin-exclusin . . . .
Tarea . . . . . . . . . . . . . . . . . . . . . .

.
.
.
.
.
.
.
.
.

.
.
.
.
.
.
.
.
.

.
.
.
.
.
.
.
.
.

.
.
.
.
.
.
.
.
.

.
.
.
.
.
.
.
.
.

.
.
.
.
.
.
.
.
.

.
.
.
.
.
.
.
.
.

II

.
.
.
.
.
.
.
.
.

.
.
.
.
.
.
.
.
.

.
.
.
.
.
.
.
.
.

.
.
.
.
.
.
.
.
.

.
.
.
.
.
.
.
.
.

.
.
.
.
.
.
.
.
.

.
.
.
.
.
.
.
.
.

.
.
.
.
.
.
.
.
.

.
.
.
.
.
.
.
.
.

.
.
.
.
.
.
.
.
.

.
.
.
.
.
.
.
.
.

.
.
.
.
.
.
.
.
.

.
.
.
.
.
.
.
.
.

.
.
.
.
.
.
.
.
.

.
.
.
.
.
.
.
.
.

.
.
.
.
.
.
.
.
.

.
.
.
.
.
.
.
.
.

.
.
.
.
.
.
.
.
.

.
.
.
.
.
.
.
.
.

.
.
.
.
.
.
.
.
.

.
.
.
.
.
.
.
.
.

.
.
.
.
.
.
.
.
.

.
.
.
.
.
.
.
.
.

.
.
.
.
.
.
.
.
.

.
.
.
.
.
.
.
.
.

.
.
.
.
.
.
.
.
.

.
.
.
.
.
.
.
.
.

.
.
.
.
.
.
.
.
.

.
.
.
.
.
.
.
.
.

.
.
.
.
.
.
.
.
.

NDICE GENERAL

III

Tarea . . . . . . . . . . . . . . . . . . . . .
4.3. Cancelacin telescpica . . . . . . . .
Tarea . . . . . . . . . . . . . . . . . . . . .
4.4. Recursiones y ecuaciones funcionales
Tarea . . . . . . . . . . . . . . . . . . . . .

.
.
.
.
.

.
.
.
.
.

.
.
.
.
.

.
.
.
.
.

.
.
.
.
.

.
.
.
.
.

.
.
.
.
.

.
.
.
.
.

.
.
.
.
.

.
.
.
.
.

.
.
.
.
.

.
.
.
.
.

.
.
.
.
.

.
.
.
.
.

.
.
.
.
.

.
.
.
.
.

.
.
.
.
.

.
.
.
.
.

.
.
.
.
.

.
.
.
.
.

.
.
.
.
.

.
.
.
.
.

.
.
.
.
.

.
.
.
.
.

.
.
.
.
.

.
.
.
.
.

.
.
.
.
.

.
.
.
.
.

.
.
.
.
.

.
.
.
.
.

.
.
.
.
.

.
.
.
.
.

.
.
.
.
.

.
.
.
.
.

.
.
.
.
.

.
.
.
.
.

.
.
.
.
.

.
.
.
.
.

.
.
.
.
.

51
52
58
59
62

.
.
.
.

.
.
.
.

.
.
.
.

.
.
.
.

.
.
.
.

.
.
.
.

.
.
.
.

.
.
.
.

.
.
.
.

.
.
.
.

.
.
.
.

.
.
.
.

.
.
.
.

.
.
.
.

.
.
.
.

.
.
.
.

.
.
.
.

.
.
.
.

.
.
.
.

.
.
.
.

.
.
.
.

.
.
.
.

.
.
.
.

.
.
.
.

.
.
.
.

.
.
.
.

.
.
.
.

.
.
.
.

.
.
.
.

.
.
.
.

.
.
.
.

.
.
.
.

.
.
.
.

.
.
.
.

.
.
.
.

.
.
.
.

.
.
.
.

.
.
.
.

.
.
.
.

63
63
66
68
74

6. Desigualdades
6.1. Valor absoluto y desigualdad del tringulo . . .
Tarea . . . . . . . . . . . . . . . . . . . . . . .
6.2. El cuadrado de todo real es positivo . . . . . .
Tarea . . . . . . . . . . . . . . . . . . . . . . .
6.3. Desigualdades de las medias . . . . . . . . . .
Tarea . . . . . . . . . . . . . . . . . . . . . . .
6.4. Desigualdad de Cauchy-Schwarz-Bunyakovsky
Tarea . . . . . . . . . . . . . . . . . . . . . . .
6.5. Desigualdad del reordenamiento . . . . . . . .
Tarea . . . . . . . . . . . . . . . . . . . . . . .

.
.
.
.
.
.
.
.
.
.

.
.
.
.
.
.
.
.
.
.

.
.
.
.
.
.
.
.
.
.

.
.
.
.
.
.
.
.
.
.

.
.
.
.
.
.
.
.
.
.

.
.
.
.
.
.
.
.
.
.

.
.
.
.
.
.
.
.
.
.

.
.
.
.
.
.
.
.
.
.

.
.
.
.
.
.
.
.
.
.

.
.
.
.
.
.
.
.
.
.

.
.
.
.
.
.
.
.
.
.

.
.
.
.
.
.
.
.
.
.

.
.
.
.
.
.
.
.
.
.

.
.
.
.
.
.
.
.
.
.

.
.
.
.
.
.
.
.
.
.

.
.
.
.
.
.
.
.
.
.

.
.
.
.
.
.
.
.
.
.

.
.
.
.
.
.
.
.
.
.

.
.
.
.
.
.
.
.
.
.

.
.
.
.
.
.
.
.
.
.

.
.
.
.
.
.
.
.
.
.

.
.
.
.
.
.
.
.
.
.

.
.
.
.
.
.
.
.
.
.

.
.
.
.
.
.
.
.
.
.

.
.
.
.
.
.
.
.
.
.

.
.
.
.
.
.
.
.
.
.

.
.
.
.
.
.
.
.
.
.

.
.
.
.
.
.
.
.
.
.

.
.
.
.
.
.
.
.
.
.

.
.
.
.
.
.
.
.
.
.

.
.
.
.
.
.
.
.
.
.

.
.
.
.
.
.
.
.
.
.

.
.
.
.
.
.
.
.
.
.

.
.
.
.
.
.
.
.
.
.

75
75
79
80
83
84
88
89
91
91
95

7. Geometra plana
7.1. ngulos . . . . . . . . . . . . . . . . . . . . . . . . .
Tarea . . . . . . . . . . . . . . . . . . . . . . . . . . . . . .
7.2. Congruencia de tringulos y desigualdad del tringulo .
Tarea . . . . . . . . . . . . . . . . . . . . . . . . . . . . . .
7.3. Trapecios y paralelogramos . . . . . . . . . . . . . . .
Tarea . . . . . . . . . . . . . . . . . . . . . . . . . . . . . .
7.4. Permetros y reas . . . . . . . . . . . . . . . . . . . .
Tarea . . . . . . . . . . . . . . . . . . . . . . . . . . . . . .
7.5. Teorema de Pitgoras . . . . . . . . . . . . . . . . . .
Tarea . . . . . . . . . . . . . . . . . . . . . . . . . . . . . .
7.6. Proporcionalidad y semejanza . . . . . . . . . . . . .
Tarea . . . . . . . . . . . . . . . . . . . . . . . . . . . . . .
7.7. Construcciones con regla y comps . . . . . . . . . . .
Tarea . . . . . . . . . . . . . . . . . . . . . . . . . . . . . .
7.8. Repaso de Trigonometra . . . . . . . . . . . . . . . .
Tarea . . . . . . . . . . . . . . . . . . . . . . . . . . . . . .
7.9. Repaso de Geometra Analtica . . . . . . . . . . . . .
Tarea . . . . . . . . . . . . . . . . . . . . . . . . . . . . . .
7.10. Vectores . . . . . . . . . . . . . . . . . . . . . . . . .
Tarea . . . . . . . . . . . . . . . . . . . . . . . . . . . . . .
7.11. Baricentros . . . . . . . . . . . . . . . . . . . . . . .
Tarea . . . . . . . . . . . . . . . . . . . . . . . . . . . . . .
7.12. Transformaciones geomtricas . . . . . . . . . . . . .
Tarea . . . . . . . . . . . . . . . . . . . . . . . . . . . . . .
7.13. Teoremas de Ceva y de Menelao . . . . . . . . . . . .
Tarea . . . . . . . . . . . . . . . . . . . . . . . . . . . . . .
7.14. Puntos y rectas notables de un tringulo . . . . . . . .
Tarea . . . . . . . . . . . . . . . . . . . . . . . . . . . . . .
7.15. Potencia de un punto con respecto a un crculo . . . . .

.
.
.
.
.
.
.
.
.
.
.
.
.
.
.
.
.
.
.
.
.
.
.
.
.
.
.
.
.

.
.
.
.
.
.
.
.
.
.
.
.
.
.
.
.
.
.
.
.
.
.
.
.
.
.
.
.
.

.
.
.
.
.
.
.
.
.
.
.
.
.
.
.
.
.
.
.
.
.
.
.
.
.
.
.
.
.

.
.
.
.
.
.
.
.
.
.
.
.
.
.
.
.
.
.
.
.
.
.
.
.
.
.
.
.
.

.
.
.
.
.
.
.
.
.
.
.
.
.
.
.
.
.
.
.
.
.
.
.
.
.
.
.
.
.

.
.
.
.
.
.
.
.
.
.
.
.
.
.
.
.
.
.
.
.
.
.
.
.
.
.
.
.
.

.
.
.
.
.
.
.
.
.
.
.
.
.
.
.
.
.
.
.
.
.
.
.
.
.
.
.
.
.

.
.
.
.
.
.
.
.
.
.
.
.
.
.
.
.
.
.
.
.
.
.
.
.
.
.
.
.
.

.
.
.
.
.
.
.
.
.
.
.
.
.
.
.
.
.
.
.
.
.
.
.
.
.
.
.
.
.

.
.
.
.
.
.
.
.
.
.
.
.
.
.
.
.
.
.
.
.
.
.
.
.
.
.
.
.
.

.
.
.
.
.
.
.
.
.
.
.
.
.
.
.
.
.
.
.
.
.
.
.
.
.
.
.
.
.

.
.
.
.
.
.
.
.
.
.
.
.
.
.
.
.
.
.
.
.
.
.
.
.
.
.
.
.
.

.
.
.
.
.
.
.
.
.
.
.
.
.
.
.
.
.
.
.
.
.
.
.
.
.
.
.
.
.

.
.
.
.
.
.
.
.
.
.
.
.
.
.
.
.
.
.
.
.
.
.
.
.
.
.
.
.
.

.
.
.
.
.
.
.
.
.
.
.
.
.
.
.
.
.
.
.
.
.
.
.
.
.
.
.
.
.

.
.
.
.
.
.
.
.
.
.
.
.
.
.
.
.
.
.
.
.
.
.
.
.
.
.
.
.
.

.
.
.
.
.
.
.
.
.
.
.
.
.
.
.
.
.
.
.
.
.
.
.
.
.
.
.
.
.

.
.
.
.
.
.
.
.
.
.
.
.
.
.
.
.
.
.
.
.
.
.
.
.
.
.
.
.
.

.
.
.
.
.
.
.
.
.
.
.
.
.
.
.
.
.
.
.
.
.
.
.
.
.
.
.
.
.

.
.
.
.
.
.
.
.
.
.
.
.
.
.
.
.
.
.
.
.
.
.
.
.
.
.
.
.
.

.
.
.
.
.
.
.
.
.
.
.
.
.
.
.
.
.
.
.
.
.
.
.
.
.
.
.
.
.

.
.
.
.
.
.
.
.
.
.
.
.
.
.
.
.
.
.
.
.
.
.
.
.
.
.
.
.
.

.
.
.
.
.
.
.
.
.
.
.
.
.
.
.
.
.
.
.
.
.
.
.
.
.
.
.
.
.

.
.
.
.
.
.
.
.
.
.
.
.
.
.
.
.
.
.
.
.
.
.
.
.
.
.
.
.
.

.
.
.
.
.
.
.
.
.
.
.
.
.
.
.
.
.
.
.
.
.
.
.
.
.
.
.
.
.

.
.
.
.
.
.
.
.
.
.
.
.
.
.
.
.
.
.
.
.
.
.
.
.
.
.
.
.
.

.
.
.
.
.
.
.
.
.
.
.
.
.
.
.
.
.
.
.
.
.
.
.
.
.
.
.
.
.

.
.
.
.
.
.
.
.
.
.
.
.
.
.
.
.
.
.
.
.
.
.
.
.
.
.
.
.
.

.
.
.
.
.
.
.
.
.
.
.
.
.
.
.
.
.
.
.
.
.
.
.
.
.
.
.
.
.

.
.
.
.
.
.
.
.
.
.
.
.
.
.
.
.
.
.
.
.
.
.
.
.
.
.
.
.
.

96
96
110
111
116
116
122
123
124
126
127
128
134
136
141
141
150
152
157
157
165
166
170
170
175
176
182
182
190
190

5. Polinomios y ecuaciones
5.1. Ecuaciones . . . .
Tarea . . . . . . . . . . .
5.2. Polinomios . . . .
Tarea . . . . . . . . . . .

.
.
.
.

.
.
.
.

.
.
.
.

.
.
.
.

.
.
.
.

.
.
.
.

.
.
.
.

.
.
.
.

.
.
.
.

.
.
.
.

IV

NDICE GENERAL
Tarea . . . . . . . . . . . . . . . . . . . . . . . . . . . . . . . . . . . . . . . . . . . . . . . . . . . . . . . . . . . . 192

A. Indicaciones y respuestas
194
Indicaciones y respuestas . . . . . . . . . . . . . . . . . . . . . . . . . . . . . . . . . . . . . . . . . . . . . . . . . 194

Prefacio

Escrib estas notas en el verano del 1996, en un taller de resolucin de problemas de concurso para maestros de secundaria
en Puerto Rico.
El hacerlas accesibles en la red me ha hecho percatar cun tiles han sido para estudiantes de habla castellana, ya que
muchos lectores se han comunicado conmigo. Por eso decidido el revisarlas frecuentemente.
Quisiera pues pedir a los lectores el que me comunicasen errores que hallaren, etc.
Bastante del material que aqu aparece es traduccin de material que he escrito en lengua gringa. De semana en semana, de
mes en mes, ir aadiendo material y las respuestas de varios ejercicios. Todava necesito aadir material en geometra, teora
de grafos, enumeracin y anlisis. Invito tambin a los lectores a contribuir material.
David A. SANTOS
dsantos@ccp.edu

VI

Aviso Legal
La loi dans sa majestueuse galit, interdit tous, aux riches comme aux pauvres de dormir sous les ponts, de mendier dans
la rue et de voler du pain. Anatole France (Les Lys Rouge - 1894)
This material may be distributed only subject to the terms and conditions set forth in the Open Publication License, version
1.0 or later (the latest version is presently available at
http://www.opencontent.org/openpub/.
THIS WORK IS LICENSED AND PROVIDED AS IS WITHOUT WARRANTY OF ANY KIND, EXPRESS OR IMPLIED, INCLUDING, BUT NOT LIMITED TO, THE IMPLIED WARRANTIES OF MERCHANTABILITY AND FITNESS
FOR A PARTICULAR PURPOSE OR A WARRANTY OF NON-INFRINGEMENT.
THIS DOCUMENT MAY NOT BE SOLD FOR PROFIT OR INCORPORATED INTO COMMERCIAL DOCUMENTS
WITHOUT EXPRESS PERMISSION FROM THE AUTHOR(S). THIS DOCUMENT MAY BE FREELY DISTRIBUTED
PROVIDED THE NAME OF THE ORIGINAL AUTHOR(S) IS(ARE) KEPT AND ANY CHANGES TO IT NOTED.

Captulo

Tcnicas elementales
1.1. Contradiccin
1 Ejemplo Sean x, y, z, w enteros satisfaciendo

1 1 1 1
+ + + = 1.
x y z w
Demustrese que al menos uno de ellos es par.
Resolucin: Presmase que todos x, y, z, w son nones. Luego
yzw + xzw + xyw + xyz = xyzw.
El lado siniestro es par, por ser la suma de cuatro enteros nones. El lado diestro es non, siendo el producto de
cuatro enteros nones. Esto resulta en una contradiccin.
2 Ejemplo El producto de 34 enteros es igual a 1. Demustrese que la suma de stos no puede ser 0.

Resolucin: Forzosamente los enteros debern ser 1. Ya que el producto es 1, debe de haber un nmero par
de 1s. Si la suma de estos enteros fuese 0, entonces debera haber tantos +1s como 1s. Luego as habrn de
haber diecisiete 1s y diecisiete +1s, lo que conlleva a una contradiccin.
3 Ejemplo Demustrese, sin recurrir a una calculatriz, que 6

1
35 < .
10

1
1
Resolucin: Presmase que 6 35 . Entonces 6
35, o sea, 59 10 35. Al cuadrar uno y
10
10

1
otro lado, 3481 3500, lo que no tiene sentido. Luego entonces se concluye que 6 35 < .
10
4 Ejemplo Sea a1 , a2 , . . . , an una permutacin arbitraria de los enteros 1, 2, . . . , n, donde n es non. Demustrese que el producto

(a1 1)(a2 2) (an n)


es par.
Resolucin: Obsrvese primero que la suma de un nmero impar de enteros impares impar es. Slo tiene que
demostrarse que al menos una de las diferencias ak k es par. Presmase al contrario, que todas las diferencias
ak k impares son. Es evidente que
S = (a1 1) + (a2 2) + + (an n) = 0,
1

Captulo 1
ya que las ak s son un reordenamiento de 1, 2, . . . , n. S es, por suposicin, la suma de un nmero impar de enteros
impares, resultando en el entero par 0. Esto es imposible, as que nuestra suposicin inicial es falsa y por lo tanto
al menos una de las diferencias ak k es par, lo que por consiguiente, hace par al producto.

5 Ejemplo Demustrese que

2 es irracional.

a
2 = , con enteros positivos a, b. Esto conlleva a 2b2 = a2 . Ahora bien, tanto a2
b
como b2 tienen un nmero par de primos en su factorizacin (contando repeticiones). Luego 2b2 tiene un nmero
impar de primos en su factorizacin y a2 tiene un nmero par de primos en su factorizacin. Esto contradice el
hecho de que todo entero positivo mayor que 1 puede descomponerse en factores primos de forma nica.

Resolucin: Presmase que

6 Ejemplo Demestrese que 2003 no es la suma de dos cuadrados.

Resolucin: Primero se demostrar que la suma de dos cuadrados nunca deja residuo 3 al ser dividida por
4. De esto se obtiene el resultado de inmediato. Cada entero es o bien par, (de la forma 2k) o non (de la forma
2k + 1). Se tiene que
(2k)2

4(k2 ),

(2k + 1)2

4(k2 + k) + 1.

Luego, el cuadrado de cada entero o bien deja residuo 0 o bien deja residuo 1 al ser dividido por 4. La suma de
dos enteros entonces dejar residuo 0, 1, o 2 al ser dividida por 4.
7 Ejemplo Si a, b, c son enteros impares, demustrese que la ecuacin ax2 + bx + c = 0 no posee una solucin racional.

Resolucin: Si la ecuacin poseyere la solucin racional


2

p
q

p
, con p, q relativamente primos, entonces
q

+b

p
+ c = 0 = ap2 + bpq + cq2 = 0.
q

Si ambos p y p fuesen nones, entonces ap2 +bpq +cq2 sera tambin non, y por lo tanto 6= 0. De manera semejante,
si uno entre p y q fuese impar y el otro par, luego o bien ap2 + bpq o bien bpq + cq2 sera par, y ap2 + bpq + cq2
impar, otra contradiccin. Luego, tal raz racional d f racpq es ficticia.

Tarea
b. Demustrese que BC > AC.
8 Problema En ABC, b
A>B
9 Problema Sea 0 < < 1. Demustrese que

> .

10 Problema Sea =
0,999 . . . en donde hay al menos 2000 nueves. Demustrese que

la expansin decimal de

tambin comienza con al menos 2000 nueves.

11 Problema Demostrar que no existen enteros a, b, c, d tales que

x4 + 2x2 + 2x + 2 = (x2 + ax + b)(x2 + cx + d).

1.2. Principio de las pichoneras de Dirichlet


12 Ejemplo Las nueve casillas de un cuadrado 3 3 son llenadas aleatoriamente por 1s, 0s, o 1s. Demustrese que entre

las ocho sumas resultantes (tres columnas, tres filas y dos diagonales), hay al menos dos de ellas idnticas.

Resolucin: Hay siete sumas posibles, cada una un entero del conjunto {3, 2, 1, 0, 1, 2, 3}. Por el principio
de las pichoneras, dos de las ocho sumas del cuadrado debern de coincidir.

Principio de las pichoneras de Dirichlet

13 Ejemplo Cincuenta y un puntos se distribuyen sobre un cuadrado 1 1. Demustrese que hay al menos tres puntos que

pueden ser cubiertos por un cuadrado

1 1
.
5 5

1 1
Resolucin: Divdase al cuadrado en veinticinco subcuadrados , cada uno de ellos con sus lados para5 5
51
lelos al cuadrado original. Uno de estos subcuadrados posee V W = 3 al menos.
25
14 Ejemplo (Putnam 1978) Sea A cualquier conjunto de veinte enteros escogidos de la progresin aritmtica 1, 4, . . . , 100.

Demustrese que hay al menos dos enteros diferentes en A cuya suma es 104.
Resolucin: Agrpese los treinticuatro enteros de esta progresin en los dicienueve grupos
{1}, {52}, {4, 100}, {7, 97}, {10, 94}, . . ., {49, 55}.
Como se habr de escoger veinte y se tiene diecienueve conjuntos, se habr de tomar dos enteros, al menos,
perteneciendo al mismo conjunto, y estos sumarn a 104.
15 Ejemplo Demustrese que entre siete enteros positivos distintos 126, siempre se puede conseguir dos de ellos a y b

satisfaciendo

b < a 2b.
Resolucin: Divdase el conjunto {1, 2, 3, . . . , 126} en los seis subconjuntos
{1, 2}, {3, 4, 5, 6}, {7, 8, . . ., 13, 14}, {15, 16, . . ., 29, 30},
{31, 32, . . ., 61, 62} y {63, 64, . . ., 126}.
Dos de los siete enteros yacern en el mismo subconjunto y satisfacern las desigualdades mencionadas.
16 Ejemplo No importa cuales cincuenta y cinco enteros se seleccionen de

{1, 2, . . ., 100},
demustrese que habr dos de ellos cuya diferencia ser 10.
Resolucin: Obsrvese primero que si elegimos n + 1 enteros de cualquiera ristra de 2n enteros consecutivos,
entonces habr dos cuya diferencia ser n. Esto es patente al aparear los 2n enteros consecutivos
{a + 1, a + 2, a + 3, . . ., a + 2n}
en los n pares
{a + 1, a + n + 1}, {a + 2, a + n + 2}, . . ., {a + n, a + 2n}.
Agrpese pues los cien enteros como sigue:
{1, 2, . . .20}, {21, 22, . . ., 40},
{41, 42, . . ., 60}, {61, 62, . . ., 80}
y
{81, 82, . . ., 100}.
Si seleccionamos cincuenta y cinco enteros, entonces, forzozamente habremos de seleccionar once del mismo
grupo. Del dicho grupo, por la observacin anterior (con n = 10), habr dos cuya differencia ser 10.

Captulo 1

17 Ejemplo (AHSME 1994) Mrquese a un disco con la etiqueta 1, a dos discos con la etiqueta 2, a tres discos con la

etiqueta 3, . . . , a cincuenta discos con la etiqueta 50. Pngase a estos 1 + 2 + 3 + + 50 = 1275 discos en una caja. Se
sacan luego discos de la caja, al azar y sin remplazo. Cul es el nmero mnimo de discos que se debe sacar para garantizar al
menos diez discos con la misma etiqueta?
Resolucin: Si se saca todos los 1+2+ +9 = 45 discos con etiquetas 1, . . . , 9 y cualquiera nueve discos
con etiquetas 10, . . . , 50, se habr sacado 45 + 9 41 = 414 discos. El 415-avo disco sacado garantizar que
haya al menos diez discos con la misma etiqueta.
18 Ejemplo Dado cualquier subconjunto A de diez enteros del conjunto {1, 2, . . ., 98, 99} demustrese que siempre habr dos
subconjuntos disjuntos de A cuyos elementos tienen la misma suma.

Resolucin: Hay 210 1 = 1023 subconjuntos no nulos que se pueden formar con un conjunto de diez elementos. A cada uno de estos subconjuntos le asociamos su suma. La mxima suma que puede ser obtenible es
90 + 91 + + 99 = 945 < 1023. Luego, hay dos subconjuntos, digamos S, T (no necesariamente disjuntos) cuya
suma de elementos es idntica. Luego, S \ (S T ) y T \ (S T ) tambin tienen suma idntica de elementos.
19 Ejemplo Dados cualesquiera 9 enteros cuyos factores primos yagan en el conjunto {3, 7, 11}, demustrese que habr dos
cuyo producto es un cuadrado perfecto.

Resolucin: Para que un entero sea un cuadrado, todos los exponentes de los primos de su factorizacin en
primos deben ser pares. Todo entero cuyos factores yagan en el conjunto dado es de la forma 3a 7b 11c . Los tros
(a, b, c) yacen en exactamente uno de los 8 patrones de paridad (par, par, par), (par, par, non), (par, non, par), (par,
non, non), (non, par, par), (non, par, non), (non, non, par), (non, non, non). En un grupo de nueve tales enteros,
habr pues dos cuyo patrn de paridad sea idntico. Luego el producto de estos dos enteros es un cuadrado, ya
que la suma de cada exponente ser par.

Tarea
20 Problema Si se toman n + 1 enteros del conjunto {1, 2, . . . ,2n}, demustrese que
siempre habr dos que son relativamente primos.

21 Problema Si se toman n + 1 enteros del conjunto {1, 2, . . . ,2n}, demustrese que


siempre habr dos tales que el menor dividir (sin dejar residuo) al mayor.

23 Problema (AHSME 1991) Una mesa circular tiene exactamente sesenta sillas en

torno. Hay N personas ya sentadas de manera que la prxima persona a sentarse por
fuerza se sentar al lado de alguien. Cul es el valor mnimo de N?

24 Problema Cinco puntos cualesquiera son colocados sobre


un cuadrado de lado 1.

Demustrese que dos de ellos estn a una distancia de a lo sumo

2/2.

22 Problema Pruebe que entre n + 1 enteros, siempre habr dos cuya diferencia ser

divisible por n.

1.3. Paridad
25 Ejemplo Dos esquinas diametralmente opuestas son cortadas de un tablero de ajedrez, que como se recordar, tiene 64
casillas. Demustrese que es imposible recubrir totalmente a las 62 casillas restantes de 31 domins.

Resolucin: Cada domin cubre cuadrados de diferente color. Al eliminar dos casillas diametralmente opuestas, se eliminan dos casillas del mismo color. Por lo tanto quedan 32 casillas de un color y 30 de otras y luego los
31 domins no las pueden cubrir a todas.
26 Ejemplo Los 28 domins de un juego se enfilan observando las reglas del domin. Si al principio de la cadena se observa

un 6 qu entero se observar al final de la cadena?

Paridad

Resolucin: Se observar tambin a un 6. Cada nmero debe ocurrir un nmero par de veces de manera que
se puedan enfilar. De los ocho 6s: se tiene uno al principio de la cadena, seis de ellos se aparearn entre s en
medio de la cadena y finalmente, el restante quedar al final de la cadena.
27 Ejemplo Demustrese que para ninguna seleccin de signos en

1 2 10,
se obtendr una suma 0.
Resolucin: La suma 1 + 2 + + 10 = 55, un entero impar. Ya que la paridad no es afectada por la eleccin
de signo, para cualquier seleccin de signo 1 2 10 nunca ser par, y en particular, nunca ser 0.
28 Definicin Llmase punto reticular en el plano al punto coordenado (m, n) en el plano cuyas coordenadas m y n son ambas

enteras.
29 Definicin El punto medio del segmento de recta que comienza en (x, y) y termina en (x1 , y1 ) es el punto
x + x y + y 
1

30 Ejemplo Se seleccionan cinco puntos reticulares en el plano, al azar. Demostrar que existe dos de entre ellos que forman

un segmento de recta cuyo punto medio es tambin un punto reticular.


Resolucin: Hay cuatro patrones de paridad posibles para cada punto reticular en el plano: (par, par), (par,
non), (non, non), (non, par). Por el principio de las pichoneras, dos de los cinco puntos reticulares compartirn el
mismo patrn de paridad, y luego, su punto medio ser tambin reticular.
Para los ejemplos siguientes necesitaremos algunas definiciones de tetromins, las cuales se daremos en las figuras al calce.

Figura 1.1: L-tetromin

Figura 1.4: Tetromin torcido

Figura 1.2: T-tetromin

Figura 1.3: Tetromin recto

Figura 1.5: Tetromin cuadrado

31 Ejemplo Se posee tan slo una copia de los cinco tetromins arriba mostrados, pudiendo as cubrir 20 cuadrados. Demustrese que es imposible arreglarlos de tal manera que se cubra a un rectngulo.

Resolucin: Si tal rectngulo existiese, tendra 20 cuadrados. Colorese el rectngulo a la manera de un


tablero de ajedrez, con diez cuadrados rojos y diez negros. El T-tetromin siempre cubre un nmero impar de
cuadrados rojos y los otros siempre cubren un nmero par de cuadrados rojos. As pues el nmero de cuadrados
rojos cubiertos es impar, contradiccin.

Captulo 1

32 Ejemplo Demustrese que un tablero de ajedrez (8 8) no se puede recubrir totalmente con 15 tetromins rectos y un

L-tetromin.

Resolucin: Colorese las filas 1, 3, 5, 7 en blanco y las filas 2, 4, 6, 8 en azul. Un tetromin recto siempre
cubrir un nmero par de cuadrados blancos y un L-tetromin siempre cubrir un nmero impar de cuadrados
blancos. Si el recubrimiento fuese posible entonces se cubrir tan slo un nmero impar de cuadrados blancos,
contradiccin.

Tarea
33 Problema Veinticinco nios y veinticinco nias son sentados alrededor de una mesa
circular. Demustrese que ambos vecinos de al menos una persona sern nias.

36 Problema Demustrese que un tablero 10 10 nunca se podr recubrir totalmente


con 25 tetromins rectos.

34 Problema Se sueldan 2001 varillas (rectas) formando un camino. Demostrar que no

37 Problema Demustrese que un tablero 8 8 nunca se podr recubrir totalmente con


15 T-tetromins y un tetromin cuadrado.

existe ninguna lnea rectano pasando por un punto de soldadura del caminoque intersecte a todos los 2001 segmentos del camino.
35 Problema Se escribe los 1, 2, . . . ,2001 enteros en una pizarra. Se borran de dos en

dos, remplazndolos con el valor absoluto de su diferencia. Demostrar que el ltimo nmero obtenido nunca ser 0.

38 Problema Una urna tiene 900 boletas, numeradas del 100 al 999. Se sacan boletas al
azar y sin remplazo, y se suman sus dgitos. Cul es el nmero menor de boletas que se
necesitar sacar para garantizar que al menos tres de estas boletas tengan la misma suma
de dgitos?

1.4. Induccin
El principio de induccin matemtica resta en la siguiente observacin intuitiva. Supongamos que tenemos que efectuar
una tarea que requiere cierto nmero de pasos sucesivos. Supongamos que siempre lograremos completar el paso n si ya hemos
completado el paso n 1. As pues, si acaso pudisemos comenzar (completando un paso base), entonces podramos completar
todos los pasos a partir del paso base.
As pues, en el principio de induccin matemtica, tratamos de comprobar la veracidad de una asercin P(n) estableciendo
primero su validez en un caso base k0 (usualmente k0 = 1). Luego tratamos de establecer si informacin sobre la validez de
P(n 1) conlleva a informacin favorable sobre P(n).
39 Teorema (Principio de induccin matemtica) Si un conjunto S de enteros positivos posee al 1, y tambin se verifica

que el entero n + 1 est toda vez que el entero n est, entonces S = N.


40 Corolario Si el conjunto A de enteros positivos contiene al entero m y tambin contiene al entero n + 1 siempre que

contenga a n, donde n > m, entonces A es el conjunto de todos los enteros positivos mayores o iguales a m.
41 Corolario (Induccin robusta) Si el conjunto A de enteros positivos contiene al entero m y tambin contiene a n + 1
siempre que contenga a m + 1, m + 2, . . . , n, donde n > m, entonces A es el conjunto de todos los enteros positivos mayores o
iguales a m.
42 Ejemplo Demostrar que 2n > n, n N.

Resolucin: La asercin es cierta para n = 0, ya que 20 > 0. Presmase que 2n1 > n 1 para n > 1. Ahora
bien,
2n = 2(2n1 ) > 2(n 1) = 2n 2 = n + n 2.
Pero n 1 > 0 = n 2 0, ya que n + n 2 n + 0 = n y entonces,
2n > n.
Esto establece el resultado por induccin.

Tarea

43 Ejemplo Demostrar que todo cuadrado se puede descomponer en n subcuadrados, no necesariamente del mismo tamao,
para toda n = 4, 6, 7, 8, . . ..

Resolucin: Al dividir al cuadrado en cuatro, como en la figura 1.6, se incrementa el nmero de cuadrados
por tres. As pues, si n es asequible, tambin lo es n + 3. As pues, si se demuestra que n = 6, n = 7 y n = 8 son
factibles, entonces se conseguir toda descomposicin en n 6 cuadrados. Pero esto se deduce de las figuras 1.7
y 1.8 (para n = 7, se descompone uno de los subcuadrados de la figura 1.6), terminando la demostracin.

Figura 1.6: Ejemplo 43.

Figura 1.7: Ejemplo 43.

Figura 1.8: Ejemplo 43.

44 Ejemplo Demostrar que

33n+3 26n 27
es un mltiplo de 169 para todo nmero natural n.
Resolucin: Sea P(n) la asercin T N with 33n+3 26n 27 = 169T . Demostrarase que P(1) es cierta y
que P(n 1) = P(n). Para n = 1 se asevera que 36 53 = 676 = 169 4 es divisible por 169, lo cual es evidente.
Ahora bien, P(n 1) se traduce en la existencia de un N N tal que 33(n1)+3 26(n 1) 27 = 169N, i.e., para
n > 1,
33n 26n 1 = 169N
para algn entero N. Luego
33n+3 26n 27 = 27 33n 26n 27 = 27(33n 26n 1) + 676n
lo que simplifica a
27 169N + 169 4n,
que claramente mltiplo de 169 es. Esto establece el resultado mediante induccin.

Tarea
45 Problema Demostrar que para todo entero n 1, la cantidad

(1 + 2)2n + (1 2)2n

es un entero par y que

(1 + 2)2n (1 2)2n = b 2

para algn entero b.

48 Problema La sucesin de Fibonacci est dada por

f0 = 0, f1 = 1, fn+1 = fn + fn1 , n 1,

46 Problema Si k es impar, demostrar que 2n+2 divide a


n

k2 1
para todo natural n.

47 Problema Si se toman n + 1 enteros del conjunto {1, 2, . . . ,2n}, demustrese que


siempre habr dos tales que el menor dividir (sin dejar residuo) al mayor.

esto es, cada nmero luego del segundo es la suma de los dos precedentes. As la sucesin
de Fibonacci comienza por
0, 1, 1, 2, 3, 5, 8, 13, 21, . . . .

Captulo 1

Demustrese mediante induccin matemtica , que para entero n 1, se tiene


fn1 fn+1 =

fn2

53 Problema Sea n 1 un entero y sea C un conjunto constituido de 2n + 1 enteros


positivos no nulos, no necesariamente distintos. Supngase que C tiene la propiedad siguiente: Si x C , entonces existe particicin de C \ {x} en dos subconjuntos A y B de
n elementos cada uno tal que la suma de los elementos en A es igual a la suma de los
elementos en B. Demostrar que todos los elementos de C son idnticos.

+ (1) .

49 Problema En el pas Pesimista, las monedas slo vienen en cantidades de 3 y 5 pe-

sos. Demostrar que toda cantidad de 8 o ms pesos se puede pagar con estas monedas.
54 Problema Sean a1 , a2 , . . . ,an y b1 , b2 , . . . ,bn enteros naturales no nulos. Supngase

que

50 Problema Utilcese induccin para demostrar que para todo nmero natural n > 0

1
4

1
9

1
(n + 1)2

a1 + a2 + + an = b1 + b2 + + bn < mn.

n+2
.
2(n + 1)

Demostrar que siempre es posible suprimir unos cuantos trminos de uno y otro lado
(pero no todos!) y conservar la igualdad de la suma de los trminos restantes.
51 Problema Utilcese induccin para demostrar que para entero natural n, la candidad
n3 + (n + 1)3 + (n + 2)3 es siempre divisible por 9. Puede avalarse de la identidad
55 Problema Mediante induccin, demuestre que todo tringulo equiltero puede ser
descompuesto en n subtringulos equilteros (no necesariamente del mismo tamao) para toda n 6.

(a + b)3 = a3 + 3a2 b + 3ab2 + b3 .


52 Problema Demostrar que para todo entero n 1,

1+

1
1
1
+ + 2 2 .
n
22
n

56 Problema Sea s un entero estrictamente positivo. Demostrar que en el intervalo ce-

rrado [s; 2s] hay una potencia de 2.

1.5. Buen orden


57 Axioma (Axioma del buen orden) Todo conjunto no vacuo S de nmeros naturales posee un elemento mnimo.
58 Ejemplo Demustrese que no existe ningn entero natural en el intervalo ]0; 1[.

Resolucin: Presmase al contrario que el conjunto S de enteros naturales en ]0; 1[ no es nulo. Por el axioma
del buen orden, este conjunto debe poseer un elemento mnimo, llammosle m. Ahora bien, 0 < m2 < m < 1, y
por tanto m2 S . Pero esto quiere decir que S tiene un entero positivo m2 estrictamente menor que su mnimo
elemento m, contradiccin, y por lo tanto S = .
59 Ejemplo Si a, b, c son enteros tales que a6 + 2b6 = 4c6 , demustrese que a = b = c = 0.

Resolucin: Es evidente que podemos restringirnos al caso donde todas las incgnitas son mayores o iguales
a 0. Escjase un tro a, b, c satisfaciendo la ecuacin y con
max(a, b, c) > 0
tan pequeo como fuere posible. Si a6 + 2b6 = 4c6 entonces a deber ser par, a = 2a1 . Esto conlleva a 32a61 + b6 =
2c6 . Luego tambin b = 2b1 y as 16a61 + 32b61 = c6 . Finalmente esto d c = 2c1 , por lo cual a61 + 2b61 = 4c61 .
Pero max(a1 , b1 , c1 ) < max(a, b, c) lo que contradice la minimalidad de max(a, b, c). As pues se debe tener que
max(a, b, c) = 0 y todas las incgnitas son 0.
60 Ejemplo (IMO 1988) Si a, b son enteros positivos tales que

a2 + b2
a2 + b2
es entero, entonces demuestre que
cuadrado es.
1 + ab
1 + ab

a2 + b2
= k es un contra-ejemplo de un entero que no es cuadrado, con max(a, b)
1 + ab
tan pequeo como fuere posible. Sin prdida de generalidad presmase que a < b, ya que si a = b entonces then
Resolucin: Supngase que

0<k=
forzando k = 1, un cuadrado.

2a2
a2 + 1

< 2,

Tarea

9
Ahora bien, a2 + b2 k(ab + 1) = 0 es una ecuacin cuadrtica en b, con suma de races ka y producto de races
a2 k. Sean b1 , b sus races, de modo que b1 + b = ka y b1 b = a2 k.
Si a, k son enteros positivos, el suponer que b1 < 0 es incompatible con a2 + b21 = k(ab1 + 1). Como k no es
cuadrado, el suponer b1 = 0 es incompatible con a2 + 02 = k(0 a + 1). Adems
b1 =

a2 k b2 k
<
< b.
b
b

Entonces hemos encontrado otro entero b1 para el cual


Entonces pues k cuadrado es.

a2 + b21
= k y que es menor que max(a, b), contradiccin.
1 + ab1

Tarea
62 Problema Demostrar que cuartetos de enteros estrictamente positivos (x, y, z, w) satisfaciendo la ecuacin x2 + y2 = 3(z2 + w2 ) no existen.

61 Problema Demostrar que la serie infinita de cuadrados

1, 4, 9, 16, . . . ,
no contiene ninguna progresin aritmtica infinita.

1.6. Condiciones extremas


63 Ejemplo (Problema de Sylvester) Un conjunto de n puntos en el plano posee la propiedad que toda lnea pasando por dos
de ellos siempre pasa por un tercero de ellos. Demustrese que los puntos estn alineados.

Resolucin: Si los puntos no estuviesen alienados, entre todos los pares (p, L) de puntos p no sobre la lnea L
habr uno minimizando la distancia d entre p y L. Sea f el pie de la perpendicular de p a L, como en la figura
1.9. Por hiptesis hay al menos tres puntos a, b, c sobre L. Dos de stos, digamos a y b, estn del mismo lado que
f y uno de ellos, digamos b, es ms cercano a f . La distancia de b a la recta ap es menor que d, contradiccin.

A
b

qb
b

b
b

Figura 1.9: Ejemplo 63


Figura 1.10: Ejemplo 64.

64 Ejemplo De 2n puntos en el plano, n son rojos, n son azules y ningn tro de entre ellos es colineal. Sortanse los puntos

en n pares de tal manera que cada par tiene un punto rojo y otro azul y se forman n segmentos uniendo cada par de puntos.
Existir un apareamiento para el cual ningn par de segmentos se intersecte?
Resolucin: S existe. El nmero de apareamientos es finito, luego ha de existir una manera de parear los
puntos de modo que la distancia total de los segmentos sea mnima. Sostenemos que bajo estas condiciones ningn
par de segmentos se intersecar.

10

Captulo 1
Supngase que bajo las condiciones de mnima distancia arriba estipuladas existe un par de segmentos AB y CD
que se intersecan en el punto P, como en la figura 1.10. En virtud de la desigualdad del tringulo
AD + BC < AP + PD + BP + PC = AB +CD,
minimizando la ya minimizada distancia, contradiccin.

Tarea
65 Problema (BMO 1975) Siete puntos se encuentran sobre un disco cerrado de radio
1. Si las distancias mutuas entre todos estos puntos son 1, demostrar que el centro del

disco pertenece a esta coleccin de puntos.

Captulo

lgebra y aritmtica
2.1. Identidades algebraicas
Una de las identidades ms tiles en la resolucin de problemas es la diferencia de cuadrados
x2 y2 = (x y)(x + y).
Muchas expresiones se pueden factorizar si se convierten en diferencias de cuadrados. Por ejemplo
x4 + x2 y2 + y4

x4 + 2x2 y2 + y4 x2 y2

(x2 + y2 )2 (xy)2

(x2 xy + y2)(x2 + xy + y2).

Tambin
a4 + 4b4

= a4 + 4a2 b2 + 4b4 4a2b2


= (a2 + 2b2)2 (2ab)2
= (a2 2ab + 2b2)(a2 + 2ab + 2b2)

Otra identidad til es la de diferencia de cubos


x3 y3 = (x y)(x2 xy + y2).
Si n es un entero positivo tenemos en general el siguiente teorema.
66 Teorema Sea n un entero positivo. Entonces

xn yn = (x y)(xn1 + xn2 y + xn3y2 + + xyn2 + yn1 ).


Demostracin: Primero demostraremos que si a 6= 1, entonces
1 + a + a2 + an1 =

1 an
.
1a

Pngase
S = 1 + a + a2 + + an1.
11

12

Captulo 2
Entonces
aS = a + a2 + + an1 + an.

Luego

S aS = (1 + a + a2 + + an1) (a + a2 + + an1 + an) = 1 an,

y al ver que

(1 a)S = S aS = 1 an,
x
se obtiene el resultado. Poniendo ahora a = , se ve que
y
2

x
x
1+ +
y
y
de donde se obtiene

x
1
y

lo que equivale a

x
+ +
y

x
1
y

n
x
y

1 yx

n1 

x
x
1+ +
y
y

n1

+ +

x
y

x x2
xn1
1 + + 2 + + n1
y y
y

= 1


= 1

x
y

xn
.
yn

Multiplicando por yn uno y otro lado

x n1
x x2
xn1
y 1
y
1 + + 2 + + n1
y
y y
y

=y

xn
1 n ,
y

lo que da
el resultado pedido.

yn xn = (y x)(yn1 + yn2x + + yxn2 + xn1 ),

El segundo factor tiene n trminos y cada trmino tiene grado (peso)

n 1.

67 Corolario Sean x, y enteros con x 6= y y sea n un entero positivo. Entonces x y divide a xn yn .

Por ejemplo, sin necesidad de hacer clculos, el corolario anterior nos dice que 781 = 1996 1215 divide a 19965 12155.
Otros resultados tiles son los siguientes
68 Teorema Si n es un entero positivo impar

xn + yn = (x + y)(xn1 xn2 y + xn3y2 xn4 y3 + + x2 yn3 xyn2 + yn1 ).


69 Corolario Sean x, y enteros con x 6= y y sea n un entero positivo impar. Entonces x + y divide a xn + yn .

Por ejemplo 129 = 27 + 1 divide a 2861 + 1 y 1001 = 1000 + 1 = 999 + 2 = = 500 + 501 divide a
11997 + 21997 + + 10001997.
70 Ejemplo Si a2 + b2 = 1 y ab = 2, halle (a + b)2 , (a b)2 , a4 + b4 .

Resolucin: Tenemos
(a + b)2 = a2 + b2 + 2ab = 5,
(a b)2 = a2 + b2 2ab = 3,
y
a4 + b4 = (a2 + b2)2 2a2b2 = 7.

Identidades algebraicas

13

71 Ejemplo Hallar todos los primos de la forma n3 1, donde n es un entero positivo.

Resolucin: Como n3 1 = (n 1)(n2 + n + 1) y como n2 + n + 1 > 1, deberemos tener n 1 = 1. Luego el


nico primo de la forma deseada es 23 1 = 7.
72 Ejemplo Demostrar que el nico primo de la forma n4 + 4 es el 5.

Resolucin: Podemos restringirnos a enteros positivos. Vemos que


n4 + 4 = n4 + 4n2 + 4 4n2 = (n2 + 2)2 (2n)2 = (n2 2n + 2)(n2 + 2n + 2).
Si este producto es un nmero primo entonces el factor ms pequeo debe ser igual a 1. As n2 2n + 2 = 1, o sea
(n 1)2 = 0, esto es n = 1. As, el nico primo de esta forma es 14 + 4 = 5.
73 Ejemplo Dado que 1979 es primo, demostrar que si

u
1 1
1
= 1 + + + +
.
b
2 3
1978
entonces 1979 divide a u.
Resolucin: Rearreglemos la suma de la siguiente manera

1
1
1
+
+
1978
2 1977

1
1
1
1
+
+
+ +
+
3 1976
989 990

1+

1979
1979
1979
+
+ +
.
1 1978 2 1977
989 990

Al sumar todas las fracciones arriba en la derecha , vemos que el denominador divide a 1978!. Como 1979 es
primo, ningn factor de 1978! cancela al 1979 del numerador. Luego, 1979 divide al numerador de la fraccin.
74 Ejemplo Demostrar la siguiente identidad de Cataln:

1 1 1
1
1
1
1
1
+ + +

=
+
+ + .
2 3 4
2n 1 2n n + 1 n + 2
2n

Resolucin: La cantidad de la izquierda es

1 1 1
1
1
+ + + +
+
2 3 4
2n 1 2n

1 1 1
1
2
+ + + +
2 4 6
2n

1+

=
como queramos demostrar.

1 1 1
1
1
+ + + +
+
2 3 4
2n 1 2n

1
1 1 1
1
2
1 + + + + +
2
2 3 4
n

1 1 1
1
1
1 + + + + +
+
2 3 4
2n 1 2n

1 1 1
1
1 + + + + +
2 3 4
n
1
1
1
+
+ + ,
n+1 n+2
2n
1+

14

Captulo 2

75 Ejemplo Si tan x + cotx = a, exprese tan3 x + cot3 x como un polinomio en a.

Resolucin: Primero observemos que


a2 = (tan x + cotx)2 = tan2 x + cot2 x + 2,
de donde a2 2 = tan2 x + cot2 x. As
tan3 x + cot3 x = (tan x + cotx)(tan2 x tanx cot x + cot2 x) = a(a2 3).

76 Ejemplo Factorizar

1 + x + x2 + + x80 .
Resolucin: Pongamos S = 1 + x + x2 + + x80 . Entonces xS = x + x2 + x3 + + x80 + x81 = S 1 + x81. De
aqu
1 + x + x2 + + x80 =

x81 1
.
x1

Luego
x81 1 x81 1 x27 1 x9 1 x3 1
= 27

.
x1
x 1 x9 1 x3 1 x 1
Por lo tanto
1 + x + x2 + + x80 = (x54 + x27 + 1)(x18 + x9 + 1)(x6 + x3 + 1)(x2 + x + 1).

77 Ejemplo Hallar la raz cuadrada de

Resolucin: Observe que

Luego

5 + 2 6.

5 + 2 6 = 3 + 2 2 3 + 2 = ( 2 + 3)2 .

5 + 2 6 = 2 + 3.

78 Ejemplo Simplificar

1
1
1
1

+
+ +

+
.
99 + 100
1+ 2
2+ 3
3+ 4

Resolucin: Como 1 = n + 1 n = ( n + 1 n)( n + 1 + n), entonces

= n + 1 n.

n+ n+1

Tarea

15
Por lo tanto
1

1+ 2
1

2+ 3
1

3+ 4
..
.

y as

99 + 100

2 1

3 2

4 3

..
.

..
.

100 99,

1
1
1
1

+
+ +

+
= 100 1 = 9.
99 + 100
1+ 2
2+ 3
3+ 4

79 Ejemplo Demostrar que para todo entero positivo n, la expresin

2903n 803n 464n + 261n


es siempre divisible por 1897.
Resolucin: Por el Teorema 66, 2903n 803n es divisible por 2903 803 = 2100 = 7 300 y 261n 464n es
divisible por 203 = (29) 7. Por lo tanto, la expresin es divisible por 7. Adems 2903n 464n es divisible por
2903 464 = 2439 = 9 271 y 803n + 261n es divisible por 803 + 261 = 542 = 2 271. As pues, como la
expresin es divisible por 7 y por 271 y como estos son relativamente primos, la expresin es pues divisible por
7 271 = 1897.

Tarea
80 Problema Dado que 9877892 = 975727108521, halle el valor de 9877902 .

81 Problema Calcule (123456789)2 (123456791)(123456787) mentalmente.

87 Problema Demostrar que si k es un entero positivo impar

1k + 2k + + nk
es divisible por
1 + 2 + + n.

82 Problema Halle a + a

dado que a + a

= 4.

88 Problema Demostrar que 1492n 1770n 1863n +2141n es divisible por 1946 para
todo entero positivo n.

83 Problema Demostrar que el entero


89 Problema Dividir x128 y128 por

11 . . . 11

| {z }

(x + y)(x2 + y2 )(x4 + y4 )(x8 + y8 )(x16 + y16 )(x32 + y32 )(x64 + y64 ).

221 1 s

es compuesto.

90 Problema Halle la suma de los factores primos de 216 1.

84 Problema Demostrar que 7 divide a


5555

2222

91 Problema Dado que 1002004008016032 tiene un factor primo p > 250000, hllelo.
2222

+ 5555

85 Problema Demostrar que 100 divide a 1110 1.

92 Problema Si a3 b3 = 24, a b = 2, halle el valor de (a + b)2 .

93 Problema Hallar

94 Problema Hallar

11 + 72.

86 Problema Demostrar que 271958 108878 + 101528 es exactamente divisible por

26460.

10 + 4i 6.

16

Captulo 2

95 Problema Evale la suma

101 Problema Demostrar que

1
1
1

+
3
3
3
1+ 3 2+ 3 4
4+ 3 6+ 3 9
9 + 3 12 + 3 16

a3 + b3 + c3 3abc = (a + b + c)(a2 + b2 + c2 ab bc ca).

102 Problema Demostrar que


96 Problema Factorice 1 + x + x2 + x3 + + x624 .

(x + y)5 x5 y5 = 5xy(x + y)(x2 + xy + y2 ).

97 Problema Expandir el producto


103 Problema Demostrar que

(1 + x)(1 + x2 )(1 + x4 )(1 + x8 ) (1 + x1024 ).

(x + a)7 x7 a7 = 7xa(x + a)(x2 + xa + a2 )2 .

98 Problema Demostrar que si 2n 1 es un nmero primo, entonces n es un nmero


primo. Primos de esta forma se llaman primos de Mersenne.

104 Problema Demostrar que

A = x9999 + x8888 + x7777 + + x1111 + 1

99 Problema Demostrar que si 2n + 1 es un nmero primo, entonces n es una potencia

de 2. Primos de esta forma se llaman primos de Fermat.

es divisible por B = x9 + x8 + x7 + + x2 + x + 1.

100 Problema Demuestre que

105 Problema La diferencia


1
a2 + b2 + c2 ab bc ca =
(a b)2 + (b c)2 + (c a)2 .
2

57 40 2
57 + 40 2

es un entero. Hllelo.

2.2. Los enteros


Una de las propiedades ms tiles de los enteros es la expresada por el algoritmo de divisin:
106 Teorema (Algoritmo de divisin) Sean a, b enteros con a > 0. Entonces existen enteros q y r con

b = aq + r, 0 r < a.
Por ejemplo, 39 = 4 9 + 3. Vemos pues que el algoritmo de divisin discrimina a los enteros segn el residuo que dejan al ser
divididos por a. Por ejemplo, si a = 2, descomponemos a los enteros en las dos familias
A0 = {. . . 4, 2, 0, 2, 4, . . .},
A1 = {. . . , 5, 3, 1, 1, 3, 5, . . .}.
As pues todo entero es de la forma 2k o 2k + 1. Observe que todo entero de la forma 2k + 1 es tambin de la forma 2t 1.
Si a = 4 entonces descomponemos a los enteros en las cuatro familias
B0 = {. . . , 8, 4, 0, 4, 8, . . .},
B1 = {. . . , 7, 3, 1, 5, 9, . . .},
B2 = {. . . , 6, 2, 2, 6, 10, . . .},
B3 = {. . . , 5, 1, 3, 7, 11, . . .}.
As pues, los enteros son de la forma 4k, 4k + 1, 4k + 2 o 4k + 3. Observe que todo entero de la forma 4k + 1 es tambin de la
forma 4t 3 y que todo entero de la forma 4k + 3 es tambin de la forma 4t 1.
107 Ejemplo Sea r el residuo cuando 1059, 1417 y 2312 se dividen por d > 1. Halle el valor de d r.

Resolucin: Por el algoritmo de divisin, existen enteros q1 , q2 , q3 con 1059 = dq1 + r, 1417 = dq2 + r y 2312 =
dq3 + r. Restando obtenemos 1253 = d(q3 q1 ), 895 = d(q3 q2 ) y 358 = d(q2 q1 ). Como 7 179, 895 = 5
179, 358 = 2 179, vemos que d = 179. Como 1059 = 5 179 + 164, r = 164. Finalmente, d r = 15.

Los enteros

17

108 Ejemplo Demostrar que el cuadrado de todo entero es de la forma 4k o de la forma 4k + 1.

Resolucin: Si el entero es par, es decir de la forma 2a, su cuadrado es (2a)2 = 4a2, que es de la forma 4k. Si
el entero es impar, digamos 2t + 1, entonces (2t + 1)2 = 4(t 2 + t) + 1, que es de la forma 4k + 1.
109 Ejemplo Demostrar que ningn entero en la sucesin

11, 111, 1111, 11111, . . .


es el cuadrado de un entero.
Resolucin: Como es obvio que 11 no es un cuadrado, nos ocuparemos de los dems enteros en la sucesin.
Para n > 2,
. . . 1} = |11 .{z
. . 11} 00 + 12 1 = 100 11
. .11} +12 1.
|11{z
| .{z
n 1 s

n2 1 s

n2 1 s

As pues, todo nmero en esta sucesin es de la forma 4k 1. Pero por el ejercicio anterior, 4k 1 no puede ser el
cuadrado de ningn entero. Esto completa la demostracin.
110 Ejemplo Demuestre que n2 + 23 es divisible por 24 para un nmero infinito de nmeros n.

Resolucin: Tenemos que n2 + 23 = n2 1 + 24 = (n 1)(n + 1) + 24. Luego, las familias n = 24m 1, m =


0, 1, 2, 3, . . . producen infinitos valores de n2 + 23 que son divisibles por 24.
111 Ejemplo Demostrar que todos los enteros en la sucesin

49, 4489, 444889, 44448889, 44


. .44} |88 .{z
. . 88} 9
| .{z
n 4 s

n1 8 s

son cuadrados.
Resolucin: Observe que
. . 88} 9
44
. . 44} |88 .{z
| .{z
n 4 s

. .88} 10 + 9
44
. . 44} 10n + 88
| .{z
| .{z
n 4 s

n1 8 s

=
=
=
=

n1 8 s

4
8
(10n 1) 10n + (10n1 1) 10 + 9
9
9
4
4
1
2n
n
10 + 10 +
9
9
9
1
2
n
(2 10 + 1)
9

2 10n + 1 2
3

Nos falta demostrar que esta ltima cantidad es entera, esto es, que 3 divide a 2 10n + 1 = 2 00
. .00} 1. Pero la
| .{z
suma de los dgitos de esta ltima cantidad es 3, y por lo tanto este entero es divisible por 3.

n1 0 s

112 Ejemplo Demostrar que el cuadrado de todo primo mayor que 3 deja residuo 1 al ser dividido por 12.

Resolucin: Si p > 3 es primo, entonces p es de la forma 12k 1, 12k 5. Ahora bien


(12k 1)2 = 12(12k2 2k) + 1

18

Captulo 2
y
(12k 5)2 = 12(12k2 10k + 2) + 1.
Esto demuestra la asercin.

113 Ejemplo Demostrar que si ambos p y 8p 1 son primos, entonces 8p + 1 es compuesto.

Resolucin: Si p = 3, 8p 1 = 23 y 8p + 1 = 25, luego la aseveracin se cumple para p = 3. Si p > 3, p es de


la forma 3k + 1 o 3k + 2. Si p = 3k + 1, 8p 1 = 24k 7 y 8p + 1 = 24k 6, que es divisible por 6 y por lo tanto
no es primo. Si p = 3k + 2, 8p 1 = 24k 15 no es primo.
114 Ejemplo Demostrar que si n es un entero positivo tal que 2n +1 es un cuadrado, entonces n +1 es la suma de dos cuadrados

consecutivos.
Resolucin: Como 2n + 1 es un cuadrado impar, tenemos 2n + 1 = (2t + 1)2 para algn entero t. Resolviendo
para n,
(2t + 1)2 1
= 2t 2 + 2t.
n=
2
Luego n + 1 = t 2 + (t + 1)2 , la suma de dos cuadrados consecutivos.
115 Ejemplo Demostrar que si 3n + 1 es un cuadrado, entonces n + 1 es la suma de tres cuadrados.

Resolucin: Es claro que 3n + 1 no es un mltiplo de 3, luego 3n + 1 = (3k 1)2. De aqu


n+1 =

(3k 1)2 1
+ 1 = 3k2 2k + 1 = k2 + k2 + (k 1)2,
3

como queramos demostrar.


116 Ejemplo Hallar todos los enteros con dgito inicial 6 tales que si se les suprime este dgito incial,el nmero resultante es

1/25 del nmero original.


Resolucin: Sea x el entero buscado. Entonces x = 6 10n + y donde y es un entero positivo. La condicin del
problema estipula que
1
y=
(6 10n + y) ,
25
o sea,
10n
= 25 10n2.
y=
4
Esto requiere n 2 y por lo tanto y = 25, 250, 2500, 25000, etc.. Luego x = 625, 6250, 62500, 625000, etc..
117 Ejemplo Sea A un entero positivo y A sea el entero positivo resultante de alguna permutacin especfica de los dgitos de

A. Demostrar que si A + A = 1010 entonces A es divisible por 10.

Resolucin: Claramente, A y A debern tener 10 dgitos cada uno. Pongamos pues


A = a10 a9 a8 . . . a1
y
A = b10 b9 b8 . . . b1 ,
donde ak , bk , k = 1, 2, . . . , 10 son los dgitos de A y A respectivamente. Ahora, como A + A = 10000000000, deberemos tener que a1 + b1 = a2 + b2 = = ai + bi = 0 y
ai+1 + bi+1 = 10, ai+2 + bi+2 = = a10 + b10 = 9,

Los enteros

19

para algn subndice i, 0 i 9. Note que si i = 9 no hay ninguna suma de las ai+2 + bi+2 , ai+3 + bi+3 , . . . y si
i = 0 no hay ninguna suma de las a1 + b1, . . . , ai + bi .
Sumando,
a1 + b1 + a2 + b2 + + ai + bi + ai+1 + bi+1 + + a10 + b10 = 10 + 9(9 i).

Ahora bien, si i es par, 10 + 9(9 i) es impar y si i es impar 10 + 9(9 i) es par. Pero como
a1 + a2 + + a10 = b1 + b2 + + b10,
tenemos

a1 + b1 + a2 + b2 + + ai + bi + ai+1 + bi+1 + + a10 + b10 = 2(a1 + a2 + + a10),

un entero par. Colegimos que i es impar, lo que necesariamente implica a1 = b1 = 0, esto es, A y A son ambos
divisibles por 10.
118 Ejemplo Cuntos ceros hay al final de 999!?

Resolucin: El nmero de ceros est determinado por la potencia mayor de 10 que divide a 999!. Como hay
menos mtiplos de 5 en {1, 2, . . . , 999} que mltiplos de 2, el nmero de ceros est pues determinado por la
potencia mayor de 5 que divide a 999!. Esta es

999
999
999
999
+
+
+
= 199 + 39 + 7 + 1 = 246.
5
52
53
54

Por lo tanto, 999! termina en 246 ceros.


119 Ejemplo La suma de enteros positivos es 1996. Cul es el valor mximo de su producto?

Resolucin: Tenemos enteros positivos a1 , a2 , . . . , an con a1 + a2 + + an = 1996. Es claro que para maximizar
a1 a2 an , ninguna de las ak s puede ser igual a 1. Demostraremos que para obtener un producto mximo deberemos tener la mayora de las ak = 3 y a lo sumo dos a j = 2. Supongamos que a j > 4. Si substituimos a j por los dos
trminos a j 3 y 3 la suma no se afecta, pero el producto incrementa pues a j < 3(a j 3). As pues las ak s son
iguales a 2, 3 4. Pero como 2 + 2 + 2 = 3 + 3 y 2 2 2 < 3 3, si hay tres o ms 2s, los podemos substituir
con 3s. Como 1996 = 3(665) + 1 = 3(664) + 4, el producto mximo es pues 3664 4.
120 Ejemplo Demostrar que el producto de cuatro enteros consecutivos es siempre divisible por 24.

Resolucin: Sean n 1, n, n + 1, n + 2 los cuatro enteros consecutivos. Uno de ellos es divisible por 3, uno de
ellos es de la forma 4k (y por lo tanto divisible por 4) y otro de ellos es de la forma 4a + 2 (y por ende divisible
por 2). Luego el producto es divisible por 3 4 2 = 24.
121 Ejemplo Demostrar que el producto de cuatro enteros consecutivos, diferentes de 0, jams es un cuadrado.

Resolucin: Sean n 1, n, n + 1, n + 2 cuatro enteros consecutivos. Entonces su producto P es


P = (n 1)n(n + 1)(n + 2) = (n3 n)(n + 2) = n4 + 2n3 n2 2n.
Ahora bien,
(n2 + n 1)2 = n4 + 2n3 n2 2n + 1 = P + 1 > P.
Como P 6= 0 y P es 1 ms que un cuadrado, P no puede ser un cuadrado.
122 Ejemplo Hallar todos los enteros positivos de la forma

1
r+ ,
r
donde r es un nmero racional.

20

Captulo 2
Resolucin: Demostraremos que la expresin r + 1/r es entero slo cuando r = 1, en cuyo caso r + 1/r = 2.
Sea pues
1
r + = k,
r
k un entero positivo. Luego

k k2 4
r=
.
2
Como k es un entero, r puede ser entero si y slo si k2 4 es un cuadrado de la misma paridad que k. Ahora, si
k 3,
(k 1)2 < k2 4 < k2 ,
esto es, k2 4 est entre dos cuadrados consecutivos y por lo tanto no puede ser un cuadrado. Si k = 1,
no es real. Si k = 2, k2 4 = 0. Luego, r + 1/r = 2, esto es, r = 1. Esto termina la demostracin.

k2 4

123 Ejemplo Para cuntos enteros n en {1, 2, 3, . . . , 100} es el dgito de las decenas de n2 impar?

Resolucin: En el subconjunto {1, 2, . . . 10} hay slo dos valores de n (4 y 6) para los cuales el dgito de las
decenas de n2 es impar. Ahora bien, (n + 10)2 = n2 + 20n + 100 tiene la misma paridad en su dgito de las decenas
que el dgito de las decenas de n2 . Luego, hay 2 10 = 20 enteros n para los cuales se verifica la condicin
prescrita.

Tarea
130 Problema Demostrar que la suma de todos los enteros de n dgitos, n 3, es

124 Problema Sea a el entero

a = 111 . . . 1

| {z }

494 99 . . . 9 55 00 . . . 0 .

|{z} |{z}

m 1 s

n3 9 s

y sea b el entero

n2 0 s

b = 1 000 . . . 0 5.

| {z }
m1 0 s

131 Problema Demostrar que para todo entero positivo n,

Demostrar que ab + 1 es un cuadrado perfecto.


11 . . . 1 22 . . . 2

|{z} |{z}
2n 1 s

125 Problema Demostrar que el cuadrado de un entero es de la forma 3k o 3k + 1. Lue-

go demostrar que si los lados de un tringulo rectngulo son enteros, entonces 3 divide a
alguno de los lados.

n 2 s

es un cuadrado.

132 Problema Demostrar que para todo nmero a 6= 0, a 6= i 3 se verifica la frmula


de Reyley (1825):

126 Problema Hallar la suma

5 + 55 + 555 + + 5 . . . 5 .

|{z}
n 5 s

127 Problema Qu dgitos aparecen en el producto

3... 3 6... 6 ?

|{z} |{z}
666 3 s


a=

a6 + 45a5 81a2 + 27
6a(a2 + 3)2

3 
+

a2 + 30a2 9
6a(a2 + 3)

3 
+

6a3 + 18a
(a2 + 3)2

3
.

Si a es racional, esto demuestra que todo nmero racional puede expresarse como la suma
de tres cubos de nmeros racionales.

666 6 s

133 Problema Demostrar que para n 2, la expresin


128 Problema Demostrar que no existe ningn entero con la propiedad de que si su

n3 + (n + 2)3
4

dgito inicial se suprime, el entero resultante es 1/35 del entero inicial.


129 Problema Cul es la potencia mayor de 7 que divide a 1000!?

es un entero compuesto.

2.3. Aritmtica modular


Comenzaremos primero con la siguiente definicin. Si a 6= 0 es un entero, decimos que a divide al entero b (escrito a|b) si
existe un entero k con ak = b. Por ejemplo, 11|99 porque 11 9 = 99.

Aritmtica modular

21

Las siguientes propiedades de divisibilidad son obvias. Sean a, b, c, x, y enteros. Entonces


ab 6= 0, a|b, b|c = a|c

(2.1)

a 6= 0, a|b, a|c = a|(xb + yc)

(2.2)

Por ejemplo, 11|99 y 33|330 implica que 11|330.

Tambin, 7|21 y 7|49 implica que 7 divide a 3 21 2 49 = 35.


Si a no divide a b escribimos a 6 |b. Note adems que a|c, b|c no necesariamente implica que ab|c. Por ejemplo, 2|6, 6|6
pero claramente 12 = 2 6 6 |6.
Dado un entero n 2, el algoritmo de divisin distribuye los enteros en una de n clases dependiendo del residuo que deje el
entero al ser dividido por n. Si u y v dejan el mismo residuo al ser divididos por n, o de manera equivalente, si u v es divisible
por n, entonces decimos que u y v son congruentes mdulo n y escribimos u v mod n. Por ejemplo, 3 13 26 7 mod
10.
Notamos de paso que si u v mod n, entonces u = v + an para algn entero a. Por ejemplo, 3 24 mod 7 y 3 = 24 + (3)7.
El siguiente teorema es de suma utilidad.
134 Teorema Sea n 2 un entero. Si x y mod n y u v mod n entonces

ax + bu ay + bv mod n.
Demostracin Como n|(x y), n|(u v) entonces hay enteros s,t con ns = x y, nt = u v. Luego
a(x y) + b(u v) = n(as + bt),
es decir,
n|(ax + bu ay bv).
Esto ltimo es equivalente a
ax + bu ay + bv mod n.

135 Corolario Sea n 2 un entero. Si x y mod n y u v mod n entonces

xu yv mod n.
Demostracin Pongamos a = u, b = y en el teorema anterior.
136 Corolario Sea n > 1 un entero, x y mod n y j un entero positivo. Entonces x j y j mod n.
137 Ejemplo Hallar el residuo cuando 61987 es dividido por 37.

Resolucin: 62 1 mod 37. As pues, 61987 6 61986 6(62 )993 6(1)993 6 31 mod 37.
138 Ejemplo Hallar el residuo cuando

12233 455679 + 876533


es dividido por 4.
Resolucin: 12233 = 12200 +32 +1 1 mod 4. De manera semejante, 455679 = 455600 +76 +3 3, 87653 =
87600 + 52 + 1 1 mod 4. As
12233 455679 + 876533 1 3 + 13 4 0 mod 4.
O sea, que 12233 455679 + 876533 es divisible por 4.

22

Captulo 2

139 Ejemplo Hallar el ltimo dgito de 3100 .

Resolucin: Queremos hallar 3100 mod 10. Observemos que 32 1 mod 10. Luego, 3100 = (32 )50 (1)50 1
mod 10. As, el ltimo dgito es el 1.
140 Ejemplo Demostrar que 7|(22225555 + 55552222).

Resolucin: 2222 3 mod 7, 5555 4 mod 7 y 35 5 mod 7. Ahora bien, 22225555 +55552222 35555 +42222
(35 )1111 + (42 )1111 51111 51111 0 mod 7, lo que demuestra la asercin.
7

141 Ejemplo Hallar el dgito de las unidades de 77 .


7

Resolucin: Tenemos que hallar 77 mod 10. Ahora bien, como 72 1 mod 10, entonces tenemos 73 72 7
7 3 mod 10 y 74 (72 )2 1 mod 10. Adems, 72 1 mod 4 y por lo tanto 77 (72 )3 7 3 mod 4, lo que
quiere decir que hay un entero t tal que 77 = 3 + 4t. Ensamblando todo esto,
7

77 74t+3 (74 )t 73 1t 3 3 mod 10.


As el ltimo dgito es un 3.
142 Ejemplo Demostrar que 7 divide a 32n+1 + 2n+2 para todo nmero natural n.

Resolucin: Observemos que 32n+1 3 9n 3 2n mod 7 y 2n+2 4 2n mod 7. Luego


32n+1 + 2n+2 7 2n 0 mod 7,
para todo nmero natural n.
143 Ejemplo Qu dgitos debe substituirse por a y b en 30a0b03 de tal manera que el entero resultante sea divisible por 13 ?

Resolucin: Como 30a0b03 = 3 + 100b + 10000a + 3000000, observamos que 30a0b03 3 + 9b + 3a + 3


6 + 9b + 3a mod 13. Para que 30a0b03 sea divisible por 13 necesitamos 9b + 3a 7 mod 13. Aqu claro est,
debemos tener 0 a, b 9. Por inspeccin vemos que a = 8, b = 1; a = 5, b = 2; a = 2, b = 3; a = 9, b = 5; a =
6, b = 6; a = 3, b = 7; a = 0, b = 8 Luego 3080103, 3050203, 3020303, 3090503, 3060603, 3030703, 3000803 son
todos divisibles por 13.
144 Ejemplo Hallar los cuadrados mod 13.

Resolucin: Observemos primero que slo necesitamos cuadrar los enteros hasta 6, porque r2 (13 r)2 mod
13. Cuadrando los enteros no negativos hasta el 6, obtenemos 02 0, 12 1, 22 4, 32 9, 42 3, 52 12, 62 10
mod 13. Por lo tanto, los cuadrados mod 13 son 0, 1, 4, 9, 3, 12, y 10.
145 Ejemplo Demostrar que la ecuacin x2 5y2 = 2 no tiene soluciones enteras.

Resolucin: Si x2 = 2 5y2, entonces x2 2 mod 5. Pero 2 no es un cuadrado mod 5.


146 Ejemplo Demostrar la siguiente observacin de Euler: 232 + 1 es divisible por 641.

Resolucin: Observemos que 641 = 27 5 + 1 = 24 + 54 . Luego 27 5 1 mod 641 y 54 24 mod 641. Ahora
bien, 27 5 1 mod 641 nos da 54 228 = (5 27 )4 (1)4 1 mod 641. Esta tima congruencia y 54 24
mod 641 nos da 24 228 1 mod 641, lo que significa que 641|(232 + 1).

Aritmtica modular

23

147 Ejemplo Hallar un nmero infinito de enteros n tal que 2n + 27 sea divisible por 7.

Resolucin: Observemos que 21 2, 22 4, 23 1, 24 2, 25 4, 26 1 mod 7 y as 23k 1 mod 3 para todos


los enteros positivos k. Luego 23k + 27 1 + 27 0 mod 7 para todos los enteros positivos k. Esto produce una
familia infinita de enteros n = 3k, k = 1, 2, . . . tal que 2n + 27 es divisible por 7.
148 Ejemplo Existen acaso enteros positivos x, y tales que x3 = 2y + 15?

Resolucin: No. Los cubos mod 7 son 0, 1, y 6. Ahora bien, cada potencia de de 2 es congruente a 1, 2, 4 mod
7. As pues, 2y + 15 2, 3, or 5 mod 7. Esto es imposible.
149 Ejemplo Demostrar que 2k 5, k = 0, 1, 2, . . . nunca deja residuo 1 cuando es dividido por 7.

Resolucin: 21 2, 22 4, 23 1 mod 7 y este ciclo de tres se repite. As pues, 2k 5 deja residuos 3, 4, 6 al


ser dividido por 7.
150 Ejemplo (USAMO 1979) Determine todas las soluciones no negativas

(n1 , n2 , . . . , n14 )
de la ecuacin diofntica
n41 + n42 + + n414 = 1599
de haberlas.
Resolucin: No hay tales soluciones. Todas las cuartas potencias mod 16 son o bien 0 o bien 1 mod 16.
Esto significa que
n41 + + n414
es a lo sumo 14 mod 16. Pero 1599 15 mod 16.
Usando congruencias y el sistema de numeracin decimal podemos obtener varias reglas de divisibilidad. La ms famosa
es quizs la siguiente.
151 Teorema Regla de los 9s Un nmero natural n es divisible por 9 si y slo si la suma de sus dgitos es divisible por 9.

Demostracin Sea n = ak 10k + ak1 10k1 + + a1 10 + a0 la expansin en base-10 de n. Como 10 1 mod 9, tenemos
10 j 1 mod 9. Colegimos que n = ak 10k + + a110 + a0 ak + + a1 + a0 , de donde resulta la asercin.
152 Ejemplo (AHSME 1992) Los enteros de dos dgitos desde el 19 hasta el 92 se escriben consecutivamente para obtener el

entero
192021222324 89909192.
Cul es la potencia mayor de 3 que divide a este nmero?
Resolucin: Por la regla de los 9s este nmero es divisible por 9 si y slo si
19 + 20 + 21 + + 92 = 372 3
lo es. Por lo tanto, el nmero es divisible por 3 pero no por 9.
153 Ejemplo (IMO 1975) Cuando 44444444 se escribe en notacin decimal, la suma de sus dgitos es A. Sea B la suma de los

dgitos de A. Hallar la suma de los dgitos de B. (A y B se escriben en notacin decimal.)

24

Captulo 2
Resolucin: Tenemos que 4444 7 mod 9, y por lo tanto 44443 73 1 mod 9. As 44444444 = 44443(1481)
4444 1 7 7 mod 9. Sea C la suma de los dgitos de B.
Por la regla de los 9s, 7 44444444 A B C mod 9. Ahora bien, 4444 log10 4444 < 4444 log10 104 = 17776.
Esto significa que 44444444 tiene a lo sumo 17776 dgitos, as la suma de los dgitos de 44444444 es a lo sumo
9 17776 = 159984, de aqu A 159984. Entre los nmeros naturales 159984 el que tiene la suma mximal de
sus dgitos es 99999, de donde colegimos que B 45. De todos los enteros naturales 45, 39 tiene la mxima
suma dgitoal, es decir 12. As la suma de los dgitos de B es a lo sumo 12. Pero como C 7 mod 9, se sigue que
C = 7.

Las congruencias mod 9 a veces pueden ser usadas para verificar multiplicaciones. Por ejemplo, 8759612753 6= 2410520633,
ya que si esto fuese cierto entonces
(8 + 7 + 5 + 9 + 6 + 1)(2 + 7 + 5 + 3) 2 + 4 + 1 + 0 + 5 + 2 + 0 + 6 + 3 + 3 mod 9.
Pero esto dice que 0 8 8 mod 9, que es patentemente falso.
Se puede establecer un criterio de divisibilidad por 11 de una manera semejante. Sea n = ak 10k + ak1 10k1 + + a110 +
a0 . Como 10 1 mod 11, tenemos 10 j (1) j mod 11. Por lo tanto n (1)k ak + (1)k1 ak1 + a1 + a0 mod 11,
o sea, n es divisible por 11 si y slo si la suma alternante de sus dgitos es divisible por 11. Por ejemplo, 912282219
9 1 + 2 2 + 8 2 + 2 1 + 9 7 mod 11 y as 912282219 no es divisible por 11, mientras que 8924310064539 8 9 +
2 4 + 3 1 + 0 0 + 6 4 + 4 3 + 9 0 mod 11, y as 8924310064539 es divisible por 11.

Tarea
154 Problema Si 62ab427 es un mltiplo de 99, hallar los dgitos a y b.

163 Problema Demostrar que si 9|(a3 + b3 + c3 ), entonces 3|abc, para enteros a, b, c.

155 Problema Demostrar que un nmero natural es divisible por 2n , n = 1, 2, 3, . . . si y


slo si el nmero formado por sus ltimos n dgitos es divisible por 2n .

164 Problema Describa todos los enteros n tal que 10|n10 + 1.

165 Problema Demostrar que si


156 Problema Hallar el ltimo dgito de

2333333334 9987737 + 12 21327 + 12123 99987.


157 Problema Demostrar que la ecuacin

x2 + 3xy 2y2 = 122

a b, a2 b2 , a3 b3 , a4 b4 , . . .
son todos enteros, entonces a y b son tambin enteros.
166 Problema Hallar los ltimos dos dgitos de 3100 .

167 Problema (AHSME 1992) Cul es el tamao del subconjunto mayor S de

no posee soluciones enteras.

{1, 2, . . . ,50} tal que ningn par de elementos distintos de S tenga una suma divisible
por 7?

158 Problema Hallar cuntas n, 1 n 25 poseen la propiedad que n2 + 15n + 122 es

168 Problema Demostrar que la ecuacin x2 7y = 3 no tiene soluciones enteras.

divisible por 6.

159 Problema Demostrar que en cualquier subconjunto de 55 elementos tomado del

conjunto {1, 2, 3, . . . ,100}, siempre se encontrarn dos elementos que diferirn por 9.
160 Problema (AIME, 1994) La sucesin creciente

3, 15, 24, 48, . . . ,


consiste de aquellos mtiplos de 3 que son uno menos de un cuadrado. Cul es el residuo
cuando el 1994avo trmino de esta sucesin se divide por 1000?

169 Problema Demostrar que si 7|a2 + b2 entonces 7|a y 7|b.

170 Problema Demostrar que no hay enteros con

800000007 = x2 + y2 + z2 .
171 Problema Demostrar que la suma de los dgitos, en notacin decimal, de un cua-

drado, no puede ser igual a 1991.


172 Problema Demostrar que
n

161 Problema Demostrar que para cualesquiera a, b, c Z, n N, n > 3, existe un en-

tero k tal que n 6 |(k + a), n 6 |(k + b), n 6 |(k + c).

162 Problema (AIME 1983) Sea an = 6n + 8n . Determine el residuo cuando a83 se di-

vide por 49.

7|42 + 22 + 1
para todos los nmeros naturales n.
173 Problema Cuntos cuadrados hay mod 2n ?

Tarea

25

174 Problema Demostrar que los no-mltiplos de 3 son potencias de 2 mod 3n .

178 Problema (Putnam 1952) Sea


n
X

175 Problema (USAMO 1986) Cul es el menor entero n > 1, para el cual

12 + 22 + + n2
n

f (x) =

1/2

ak xnk

k=0

un polinomio de grado n con coeficientes enteros. Si a0 , an y f (1) son todos nones, demostrar que f (x) = 0 no tiene races racionales.

es un entero?
179 Problema (AHSME 1991) Un entero de n dgitos es lindo si sus n dgitos son una
176 Problema Hallar todos los enteros a, b, a > 1 y todos los primos p, q, r que satisfa-

cen la ecuacin
pa = qb + ra

permutacin del conjunto {1, 2, . . . ,n} y sus primeros k dgitos forman un entero que es
divisible por k para toda k, 1 k n. Por ejemplo, 321 es lindo de tres dgitos ya que 1
divide a 3, 2 divide a 32 y 3 divide a 321. Cuntos enteros lindos de seis dgitos hay?

(a, b, p, q, r no son necesariamente diferentes).


180 Problema Un viejo recibo est algo borroso. Dice que 88 pollos costaban un total
n

177 Problema Si n > 1 es un entero, demostrar que n n + n 1 es divisible por

de $x4,2y, donde x, y son dgitos ilegibles. Cunto costaba cada pollo?

(n 1) .
181 Problema Demostrar que un entero que consiste de 3n dgitos idnticos es divisible

por 3n .

Captulo

Combinatoria
3.1. Las reglas de la multiplicacin y la suma
182 Definicin (Cardinalidad de un conjunto) Si S es un conjunto, entonces su cardinalidad es el nmero de elementos que

ste tiene. Se denotar la cardinalidad de S por card (S).


Comenzaremos nuestro estudio de la combinatoria con los siguientes dos principios fundamentales.
183 Regla (Regla de la suma: Forma disyuntiva) Sean E1 , E2 , . . . , Ek , conjuntos finitos mutuamente disjuntos, esto es Ek

E j = si k 6= j. Entonces

card (E1 E2 Ek ) = card (E1 ) + card(E2 ) + + card(Ek ) .


184 Regla (Regla del producto) Sean E1 , E2 , . . . , Ek , conjuntos finitos. Entonces

card (E1 E2 Ek ) = card (E1 ) card (E2 ) card (Ek ) .


185 Ejemplo Cuntos pares ordenados de enteros (x, y) hay tales que 0 < |xy| 5?

Resolucin: Pngase Ek = {(x, y) Z2 : |xy| = k} para k = 1, . . . , 5. El nmero deseado es


card (E1 ) + card(E2 ) + + card (E5 ) .
Entonces
E1

{(1, 1), (1, 1), (1, 1), (1, 1)}

E2

{(2, 1), (2, 1), (1, 2), (1, 2), (1, 2), (1, 2), (2, 1), (2, 1)}

E3

{(3, 1), (3, 1), (1, 3), (1, 3), (1, 3), (1, 3), (3, 1), (3, 1)}

E4

{(4, 1), (4, 1), (2, 2), (2, 2), (1, 4), (1, 4), (1, 4), (1, 4), (2, 2), (2, 2), (4, 1), (4, 1)}

E5

{(5, 1), (5, 1), (1, 5), (1, 5), (1, 5), (1, 5), (5, 1), (5, 1)}

Por lo tanto 4 + 8 + 8 + 12 + 8 = 40 es el nmero deseado.

26

Las reglas de la multiplicacin y la suma

27

186 Ejemplo Se escribe los divisores positivos de 400 en manera creciente:

1, 2, 4, 5, 8, . . ., 200, 400.
Cuntos enteros hay en esta sucesin? Cuntos de stos son cuadrados?
Resolucin: Como 400 = 24 52 , cualquier divisor de 400 habr de ser de la forma 2a 5b en donde 0 a 4
y 0 b 2. Luego hay 5 maneras de elegir a a y 3 maneras de elegir a b, dando un total de 5 3 = 15 divisores
positivos.
Para ser un cuadrado, un divisor de 400 deber tener la forma 2 5 con {0, 2, 4} y {0, 2}. As pues, hay
3 2 = 6 divisores de 400 que son adems cuadrados.
Arguyendo de la manera mostrada en el ejemplo 186, se obtiene el siguiente teorema.
187 Teorema Tenga el entero positivo n la factorizacin en primos
a

n = pa11 pa22 pk k ,
en donde los primos pi son distintos y las enteros ai son 1. Si d(n) denota el nmero de divisores positivos de n, entonces
d(n) = (a1 + 1)(a2 + 1) (ak + 1).
188 Ejemplo (AHSME 1977) Cuntos caminosconsistiendo de una sucesin de segmentos verticales u horizontales y cada

segmento uniendo un par adyacente de letrasen la figura 3.1 deletrean CONT EST ?

O C

O C

Figura 3.1: Problem 188.

O C

Figura 3.2: Problem 188.

Resolucin: Divdase el diagrama como en la figura 3.2. Ya que cada uno de los caminos requeridos debe
utilizar la T de abajo a la derecha, se contarn los caminos que comienzan con esta T hasta llegar a la C. Ya
que hay seis filas ms a las que se podr ir y en cada etapa se puede ir o hacia arriba o hacia la derecha, existen
pues 26 = 64 caminos. La otra porcin del diagrama contribuir 64 caminos ms. Pero la columna del medio es
compartida por ambas porciones, as que hay un total de 64 + 64 1 = 127 caminos.
189 Ejemplo Se escriben los enteros del 1 al 1000 en sucesin. Hllese la suma de todos los dgitos.

28

Captulo 3
Resolucin: Al escribir los enteros del 000 al 999 (con tres dgitos), se utilizan 3 1000 = 3000 dgitos. Cada
uno de los 10 dgitos est representado de una manera igual y uniforme, as que cada dgito se utiliza 300 veces.
La suma de los dgitos del 000 al 999 es pues
(0 + 1 + 2 + 3 + 4 + 5 + 6 + 7 + 8 + 9)(300) = 13500.
Por lo tanto, la suma de los dgitos al escribir del 1 al 1000 es 13500 + 1 = 13501.

Aliter: Pareando los enteros del 0 al 999 de la siguiente manera


(0, 999), (1, 998), (2, 997), (3, 996), . . . , (499, 500),
se ve que cada par tiene suma de dgitos 27 y que hay 500 pares. Aadiendo 1 por la suma de los dgitos de 1000,
el total requerido es pues
27 500 + 1 = 13501.

190 Ejemplo Determine cuntos enteros positivos de 3-dgitos pueden escribirse en notacin decimal que no tengan al 0 en su

expansin. Hllese tambin la suma de estos nmeros de 3-dgitos.


Resolucin: Hay 9 9 9 = 729 enteros positivos de 3-dgitos no poseyendo al 0 en su expansin decimal. Si
100x + 10y + z es uno de estos enteros, entonces para cada seleccin fija de una variable, hay 9 9 = 81 selecciones
de las otras dos variables. Luego, la suma requerida es
81(1 + 2 + + 9)100 + 81(1 + 2 + + 9)10 + 81(1 + 2 + + 9)1 = 404595.

191 Ejemplo Determine cuntos enteros positivos de 3-dgitos pueden escribirse en notacin decimal poseyendo al menos un

0 en su expansin. Hllese tambin la suma de estos nmeros de 3-dgitos.


Resolucin: Utilizando el ejemplo 190, hay 900 729 = 171 tales enteros. La suma de todos los enteros de tres
dgito es
100 + 101 + + 998 + 999.
Para obtener esta suma, observe que hay 900 sumandos y que la suma no cambia al cambiar el orden de los
sumandos:

dando S =

100

101

999

999

998

100

2S

1099

1099 +

1099

900(1099),

900(1099)
= 494550. La suma requerida es pues 494550 404595 = 89955.
2

192 Ejemplo Todos los enteros positivos se escriben en sucesin

123456789101112131415161718192021222324 . . .
Qu dgito ocupa el 206790avo lugar?

Las reglas de la multiplicacin y la suma

29

Resolucin: Observemos que


1 9 + 2 90 + 3 900 + 4 9000 = 38819
y que
1 9 + 2 90 + 3 900 + 4 9000 + 5 90000 = 488819.
Por lo tanto, el dgito buscado est entre los nmeros de cinco dgitos. Si 5x+38819 206790, entonces x 33595
(el entero x es cunto nos adentramos en los nmeros de cinco dgitos). As pues, para llegar hasta el 206790avo
dgito debemos de ir hasta el 33595avo nmero de cinco dgitos, es decir 43594 (el primero es 10000). Luego,
hasta el dgito final de 43594 (el 4 de las unidades) hemos utilizado 38819 + 5 33595 = 206794 dgitos. Luego, el
4 ocupa la posicin 206794ava, el 9 la 206793ava, el 5 la 206792ava, el 3 la 206791ava y el 4 la 206790ava. El
dgito requerido es el 4.
193 Ejemplo Cuntos enteros del 1 al 1000000 tienen al menos un 1 en su expansin decimal?

Resolucin: Hay
8

enteros positivos de 1-dgito,

8 9 = 72

enteros positivos de 2-dgitos,

8 9 9 = 648

enteros positivos de 3-dgitos,

8 9 9 9 = 5832

enteros positivos de 4-dgitos,

8 9 9 9 9 = 52488

enteros positivos de 5-dgitos,

8 9 9 9 9 9 = 472392

enteros positivos de 6-dgitos,

no poseyendo el dgito 1. As pues hay


8 + 72 + 648 + 5832 + 52488 + 472392 = 531440
entre el 1 y el 999999 no poseyendo el dgito 1. Luego hay 999999 531440 = 468559 poseyendo al menos un 1 y
as 468559 + 1 = 468560 enteros entre el 1 y el 1000000 teniendo al menos un 1 en su expansin decimal.
Aliter: Analizaremos los enteros del 0 al 999999. Al resultado final le sumaremos 1, ya que 1000000 tiene un 1 en
su expansin.
Dividamos este conjunto de un milln de enteros como sigue: en 100000 decenas
{0, 1, 2, . . ., 9}
{10, 11, 12, . . ., 19}
..
.
{999990, 999991, . . ., 999999}.
En 10000 centenas
{0, 1, 2, . . ., 99}
{100, 101, 102, . . ., 199}
..
.

30

Captulo 3
{999900, 999901, . . ., 999999},
etc., hasta llegar a diez 100000enas
{0, 1, 2, . . ., 99999}
{100000, 100001, 100002, . . ., 199999}
..
.
{900000, 900001, . . ., 999999}.
En la primera decena hay solamente un nmero, el 1, que tiene un 1 en su numeracin decimal. En la segunda
decena, los diez enteros tienen un 1 en su numeracin decimal.
En la primera centena, cada decena, excepto la segunda, contendr exactamente un entero que tiene un 1 en su
expansin. La segunda decena, claro est, tiene sus diez enteros con 1s en sus expansiones. En consecuencia, la
primera centena tiene
10 + 9 1
enteros que tienen el 1 en sus expansiones.
En el primer millar, cada centena excepto la segunda tendr exactamente 10 +9 1 enteros con el 1 en su expansin.
La segunda centena, que consiste de los enteros 100, 101, . . .199 tendr sus 100 enteros con el 1 en su expansin.
As pues, el primer millar tendr exactamente
100 + 9(10 + 9 1) = 102 + 9 10 + 9
enteros con el 1 en su expansin.
En la primera decena de millar, cada millar excepto el segundo, tendr exactamente 102 + 9 10 + 9 enteros con el
1 en su expansin. El segundo millar tendr sus 103 enteros con el 1 en su expansin. Luego, en la primera decena
de millar hay
103 + 9(102 + 9 10 + 9) = 103 + 9 102 + 92 10 + 93
enteros con el 1 en su expansin.
Un razonamiento semejante nos lleva a concluir que en la primera centena de millar hay 104 + 9(103 + 9 102 +
92 10 + 93) = 104 + 9 103 + 92 102 + 93 10 + 94 enteros con el 1 en su expansin y en los primeros millones hay
105 + 9 104 + 92 103 + 93 102 + 94 10 + 95 =

106 96
= 468559
10 9

enteros con el 1 en su expansin. Esto quiere decir que en los enteros del 0 al 999999 hay 468559 enteros con el 1
en su expansin y en los enteros del 1 al 1000000 hay 468559 + 1 = 468560 enteros con el 1 en su expansin.

Tarea
194 Problema Se marcan n puntos, 1, 2, . . . ,n sobre una circunferencia, que colocamos
a igual distancia unos de los otros. Si el punto marcado 15 est directamente opuesto al
marcado 49, cuntos puntos hay en total?

195 Problema Cuntos de los factores de 295 hay que sean mayores que 1, 000, 000?

196 Problema Se escribe la sucesin de enteros positivos.

1, 2, 3, 4, 5, 6, 7, 8, 9, 10, 11, 12, 13, 14, 15, 16, 17, 18, 19, 20, . . .

198 Problema Para escribir un libro se utilizaron 1890 dgitos. Cuntas pginas tiene

el libro?
199 Problema Hallar d(1260), (1260) y (1260).

200 Problema Los enteros del 1 al 1000 se escriben en orden sobre un crculo. Comen-

zando con 1, cada quinceavo nmero es marcado (esto es, 1, 16, 31, etc.). Este proceso
se repite hasta que se marque un nmero por segunda vez. Cuntos nmeros sin marcar
quedan?

Qu dgito ocupa la posicin 3000-ava?


201 Problema Cuntos enteros entre 1 y 3012 son divisibles por 5 o por 7 pero no por

ambos nmeros?
8 9 2

197 Problema Cuntos divisores positivos tiene 2 3 5 ? Cul es la suma de estos

divisores?

Mtodos combinatorios

31

202 Problema Escribir la versin de cuatro conjuntos del principio de inclusin-

y que

exclusin.
max(x, y, z) = x + y + z mn(x, y) mn(y, z) mn(z, x) + mn(x, y, z).
203 Problema Cuntos nmeros primos hay entre 1 y 100?

Qu relacin nota entre estas frmulas y el principio de inclusin-exclusin?


204 Problema Sean x, y, z nmeros reales. Demostrar que
205 Problema Cuntos enteros entre 1 y 1000000 no son ni cuadrados, ni cubos, ni

max(x, y) = x + y mn(x, y)

cuartas, ni quintas potencias?

3.2. Mtodos combinatorios


La gran mayora de los problemas de conteo pertenecen a una de cuatro categoras, que explicaremos mediante el siguiente
ejemplo.
206 Ejemplo Considrese el conjunto {a, b, c, d}. Seleccinese dos letras de entre estas cuatro. Dependiendo de la interpreta-

cin se obtendr una de las siguientes cuatro respuestas.

Permutaciones con repeticin. El orden en que se listan las letras importa y se permite repetir letras. En este caso hay
4 4 = 16 selecciones posibles:
aa

ab

ac

ad

ba

bb

bc

bd

ca

cb

cc

cd

da db dc

dd

Permutaciones sin repeticin. El orden en que se listan las letras importa y no se permite repetir letras. En este caso
hay 4 3 = 12 selecciones posibles:
ab
ba
ca

ac

ad

bc

bd

cb

da db

cd
dc

Combinaciones con repeticin. El orden en que se listan las letras no importa y se permite repetir letras. En este caso
43
hay
+ 4 = 10 selecciones posibles:
2
aa ab ac

ad

bb bc

bd

cc

cd
dd

Combinaciones sin repeticin. El orden en que se listan las letras no importa y no se permite repetir letras. En este caso

32

Captulo 3

hay

43
= 6 selecciones posibles:
2
ab ac

ad

bc

bd
cd

Consideraremos ahora ejemplos de cada situacin.

3.2.1. Permutaciones sin repeticin


207 Definicin Definimos el smbolo ! (factorial), como sigue: 0! = 1, y para entero n 1,

n! = 1 2 3 n.
n! se lee n factorial.
208 Ejemplo Se tiene

1! =

1,

2! =

1 2 = 2,

3! =

1 2 3 = 6,

4! =

1 2 3 4 = 24,

5! =

1 2 3 4 5 = 120.

209 Definicin Sean x1 , x2 , . . . , xn n objetos distintos . Una permutacin de estos objetos es simplemente un rearreglo de ellos.
210 Ejemplo Hay 24 permutaciones de las letras de la palabra MAT H:

MAT H

MAHT

MTAH

MT HA MHTA

MHAT

AMT H

AMHT

AT MH

AT HM

AHT M

AHMT

TAMH

TAHM

T MAH

T MHA

T HMA

T HAM

HAT M

HAMT

HTAM

HT MA

HMTA

HMAT

211 Teorema Sean x1 , x2 , . . . , xn n objetos distintos. Hay n! permutaciones de ellos.

Demostracin: La primera posicin de puede elegir de n maneras, la segunda en n 1 maneras, la tercera en


n 2, etc. Esto da
n(n 1)(n 2) 2 1 = n!.

Mtodos combinatorios

33

212 Ejemplo Un librero tiene 5 libros alemanes, 7 libros espaoles y 8 libros franceses. Cada libro es diferente de cada otro.

Cuntos arreglos hay de estos libros?


Cuntos arreglos hay de estos libros si los libros deben estar agrupados por lengua?
Cuntos arreglos hay de estos libros si todos los libros franceses deben estar juntos?
Cuntos arreglos hay de estos libros si ningn libro francs est junto a otro libro francs?
Resolucin:
Se permutan 5 + 7 + 8 = 20 objetos. Luego, el nmero
de arreglos buscados es 20! = 2432902008176640000.
Agrpese los libros por lengua lo que garantizar que
los libros de una misma lengua estarn juntos. Se permutan los 3 grupos en 3! maneras. Se permutan los libros alemanes en 5! maneras, los libros espaoles en 7!
maneras y los libros franceses en 8! maneras. Luego el
nmero total de maneras es 3!5!7!8! = 146313216000.
Alniense los libros alemanes y los libros espaoles
primero. Al alinear estos 5 + 7 = 12 libros, se crean
12 + 1 = 13 espacios (incluyendo el espacio frente al
primer libro, los espacios entre los libros y el espacio
luego del ltimo libro). Para asegurar que todos los libros franceses yagan uno al lado del otro, pegamos
los libros franceses y ponemos este bulto en uno de los
espacios. Ahora, los libros franceses se permutan en 8!
maneras y los libros no-franceses en 12! maneras. Lue-

go el total number of permutaciones es


(13)8!12! = 251073478656000.
Alniense los libros alemanes y los libros espaoles
primero. Al alinear estos 5 + 7 = 12 libros, se crean
12 + 1 = 13 espacios (incluyendo el espacio frente al
primer libro, los espacios entre los libros y el espacio
luego del ltimo libro). Para asegurar que ningn libro
francs yaga junto a otro libro francs, los ponemos en
estos espacios. El primer libro francs puede ponerse
en cualquiera de 13 espacios, el segundo, en cualquiera de 12, etc., el octavo libro francs puede ponerse en
algn de los restantes 6 espacios. Ahora, los libros nofranceses pueden ser permutados en 12! maneras. As el
de permutaciones es
(13)(12)(11)(10)(9)(8)(7)(6)12!,
which es 24856274386944000.

3.2.2. Permutaciones con repeticin


Consideramos ahora permutaciones con objetos repetidos.
213 Ejemplo En cuntas maneras se pueden permutar las letras de la palabra

MASSACHUSETT S

Resolucin: Pngase subndices a las repeticiones, formando


MA1 S1 S2 A2CHUS3 ET1 T2 S4 .
Hay ahora 13 objetos distintos, los que pueden ser permutados en 13! maneras diferentes, gracias al teorema 211.
En cada una de estas 13! permutaciones, A1 A2 puede permutarse en 2! maneras, S1 S2 S3 S4 puede permutarse en 4!
maneras y T1 T2 puede permutarse en 2! maneras. As pues, el sobre-estimado de 13! es corregido y la cuenta final
es
13!
= 64864800.
2!4!2!

Razonando de manera anloga al ejemplo 213, se podr demostrar el siguiente teorema.

34

Captulo 3

214 Teorema Si hay k tipos de objetos: n1 del tipo 1; n2 del tipo 2; etc. Entonces el nmero de maneras en que estos n1 + n2 +
+ nk objetos pueden permutarse es
(n1 + n2 + + nk )!
.
n1 !n2 ! nk !
215 Ejemplo En cuntas maneras se pueden permutar las letras de la palabra MASSACHUSETT S en tal manera que la

partcula MASS est siempre junta, con las letras en este orden?
Resolucin: La partcula MASS se puede considerar como un bloque de una letra, agregando luego las 9 letras
A, C, H, U, S, E, T, T, S. En A, C, H, U, S, E, T, T, S hay cuatro Ss y dos T s, dando un total de permutaciones
de
10!
= 907200.
2!2!

216 Ejemplo De cuntas maneras se puede escribir el 9 como la suma de tres enteros positivos? Se tendr cuenta del orden,

as pues 1 + 7 + 1 se cuenta aparte de 7 + 1 + 1, etc.


Resolucin: Primero buscaremos soluciones del tipo
a + b + c = 9, 1 a b c 7
y veremos las permutaciones de cada tro. Se tiene
(a, b, c)

Nmero de permutaciones

(1, 1, 7)

3!
=3
2!

(1, 2, 6) 3! = 6
(1, 3, 5) 3! = 6
3!
=3
2!
3!
=3
2!

(1, 4, 4)
(2, 2, 5)

(2, 3, 4) 3! = 6
3!
=1
3!

(3, 3, 3)
El nmero deseado es as

3 + 6 + 6 + 3 + 3 + 6 + 1 = 28.

217 Ejemplo En cuntas maneras pueden arreglarse las letras de la palabra MURMUR de tal manera que letras idnticas no

sean adyacentes?
Resolucin: Si comenzramos con, dgase, MU entonces las R se pueden disponer de la manera siguiente:
M

,
R ,

Mtodos combinatorios

35

R .

En el primer caso hay 2! = 2 de poner las M y la U restantes, en el segundo hay 2! = 2 y en el tercero hay solamente
1!. As, si se comenzase con MU se tendra 2 + 2 + 1 = 5 arreglos posibles. En general, se puede elegir la primera
letra de 3 maneras y la segunda en 2 maneras. As pues el nmero de number of maneras pedidas es 3 2 5 = 30.

3.2.3. Combinaciones sin repeticin

 

218 Definicin Sean n, k enteros no negativos, con 0 k n. El smbolo

n
(lase n tomando k) se define como
k

 

n
n!
n (n 1) (n 2) (n k + 1)
=
=
.
k
k!(n k)!
1 2 3 k

Obsrvese que en esta ltima fraccin hay k factores tanto en el numerador como en el denominador. Obsrvese tambin los siguientes valores de frontera
 

 

n
n
=
= 1,
0
n

219 Ejemplo Se tiene

 

6
3
 
11
2
 
12
7


110
109


110
0

 

n
n
=
= n.
1
n1

654
= 20,
123
11 10
= 55,
12
12 11 10 9 8 7 6
= 792,
1234567

=
=
=

= 110,
= 1.

Ya que n (n k) = k, se satisface para enteros n, k, 0 k n, la siguiente identidad de simetra


 

n
n!
n!
n
=
=
=
k
k!(n k)! (n k)!(n (n k))!
nk

sta se puede interpretar como sigue: el nmero de maneras de sacar k boletos de un sombrero que tiene n boletos
distintos es l mismo nmero de maneras de elegir n k boletos que permanezcan dentro del sombrero.
220 Ejemplo

11
11
=
= 55,
9
2

12
12
=
= 792.
5
7

221 Definicin Considrese n distintos. Una k-combinacin es una seleccin de k, (0 k n) objetos de entre los n sin tomar

en cuenta se orden.

36

Captulo 3

222 Ejemplo Las 2-combinaciones de la lista {X,Y, Z,W } son

XY, XZ, XW,Y Z,YW,W Z.


223 Ejemplo Las 3-combinaciones de la lista {X,Y, Z,W } son

XY Z, XYW, XZW,YW Z.
224 Teorema Considrese
n objetos distintos y sea k un entero satisfaciendo 0 k n. El nmero de k-combinaciones de
 

estos n objetos es

n
.
k

Demostracin: Tmese cualesquiera k objetos. Si se tuviese en cuenta el order, entonces se producira una lista
con
n(n 1)(n 2) (n k + 1)

entradas, ya que hay n maneras de elegir el primer objeto, n 1 maneras de elegir el segundo, etc. Esta seleccin
particular de k objetos se puede permutar en k! maneras. Luego, el nmero total de k-combinaciones es
 

n
n(n 1)(n 2) (n k + 1)
=
.
k!
k

225 Ejemplo Cuntos subconjuntos de tres elementos tiene el conjunto {a, b, c, d, f }?

Resolucin: Vea que aqu el orden carece de importancia. Lo que queremos es el nmero de maneras de
seleccionar tres elementos de cinco, el cual es
 

5
543
=
= 10.
3
123

226 Ejemplo De cuntas maneras podemos seleccionar un comit de tres personas de entre diez profesores?


Resolucin:

10
= 120
3

227 Ejemplo De cuntas maneras podemos seleccionar un comit de siete personas con un presidente de entre veinte perso-

nas?
Resolucin: Primero se selecciona
  al presidente de 20 maneras. Luego a los otros 6 de entre los 19 restantes.
19
As el nmero de maneras es 20
= 542640. O tambin podemos seleccionar a las 7 personas primero, de
6


20
20
maneras, y luego al presidente de entre los 7, de 7 maneras. As el nmero de maneras es 7
= 542640,
7
7
que concuerda con lo anterior.
228 Ejemplo De cuntas maneras podemos seleccionar un comit de siete personas con un presidente y un secretario de entre
veinte personas? El secretario no sirve de presidente.


18
20
Resolucin: De 20 19
= 76
= 3255840 maneras.
5
7

Mtodos combinatorios

37

229 Ejemplo Se toman tres enteros diferentes del conjunto {1, 2, . . ., 20}. En cuntas formas se pueden tomar de tal manera

que su suma sea divisible por 3?

Resolucin: En {1, 2, . . . , 20} hay


6

nmeros que dejan residuo 0 al ser divididos por 3,

nmeros que dejan residuo 1 al ser divididos por 3,

nmeros que dejan residuo 2 al ser divididos por 3.

La suma de tres enteros ser divisible por 3 cuando (a) los tres enteros son divisibles por 3; (b) uno de los enteros
es divisible por 3, uno deja residuo 1 y el tercero deja residuo 2 al ser divididos por 3; (c) los tres dejan residuo 1
al ser divididos por 3; (d) los tres dejan residuo 2 al ser divididos por 3. Luego el nmero de maneras es
 

   

6
6
+
3
1

7
1

 

 

7
7
7
+
+
= 384.
1
3
3

230 Ejemplo Para contar el nmero de caminos de longitud mnima de A a B en la figura 3.3 obsrvese que cualquier camino de
longitud mnima deber consistir de 6 movimientos horizontales y de 3 verticales, para un total de 6 + 3 = 9 movidas. De estas
9 desplazamientos,
una vez se hallan elegido las 6 que sern horizontales, las 3 verticales quedan completamente determinadas.
 
9
= 84 caminos.
Hay as
6
231 Ejemplo Para contar el nmero de caminos
 de
 longitud mnima de A a B en la figura 3.4 que pasan por el punto O, se

cuenta el nmero de pasos de A a O (que son

 

5
3

= 20 en nmero) y el nmero de caminos que hay de O a B ( que son


  

4
5
= 4 en nmero). El nmero total de caminos lo es pues
3
3

4
= (20)(4) = 80.
3
B

B
O

Figura 3.3: Example 230.

Figura 3.4: Example 231.

3.2.4. Combinaciones con repeticin


232 Teorema (De Moivre) Sea n un entero positivo. El nmero de soluciones enteras positivas de

x1 + x2 + + xr = n

38

Captulo 3

es

n1
.
r1

Demostracin: Escrbase n como


n = 1 + 1 + + 1 + 1,
en donde hay n 1s y n 1 +s. Para descomponer a n en r sumandos slo se necesita escoger r 1 +s del total
de n 1, lo cual demuestra el teorema.
233 Ejemplo En cuntas maneras se puede escribir 9 como la suma de tres enteros estrictamente positivos? Aqu 1 + 7 + 1 se
tomar como diferente de 7 + 1 + 1.

Resolucin: Este es el ejemplo 216. Se buscan soluciones ntegras de


a + b + c = 9, a > 0, b > 0, c > 0.
Por el teorema 232 esto es

 

91
8
=
= 28.
31
2

234 Ejemplo En cuntas maneras se puede escribir 100 como la suma de cuatro enteros estrictamente positivos?

Resolucin: Se buscan soluciones ntegras de


a + b + c + d = 100,
que gracias al teorema 232 son en total

99
= 156849.
3

235 Corolario Sea n un entero positivo. El nmero de soluciones enteras no negativas de

y1 + y2 + + yr = n
es

n+r1
.
r1

Demostracin: Pngase xr 1 = yr . Entonces xr 1. La ecuacin


x1 1 + x2 1 + + xr 1 = n
es equivalente a
x1 + x2 + + xr = n + r,
que por el teorema 232, tiene

soluciones.

n+r1
r1

Principio de inclusin-exclusin

39

236 Ejemplo Hllese el nmero de cuartetos (a, b, c, d) de enteros que satisfagan

a + b + c + d = 100, a 30, b > 21, c 1, d 1.


Resolucin: Pngase a + 29 = a, b + 20 = b. Entonces se desea el nmero de soluciones enteras estrictamente
positivas de
a + 29 + b + 21 + c + d = 100,
o sea, de
a + b + c + d = 50.


Por el teorema 232 este nmero es

49
= 18424.
3

237 Ejemplo En cuntas maneras se puede escribir 1024 como el producto de tres enteros positivos?

Resolucin: Vase que 1024 = 210 . Se necesita una descomposicin de la forma 210 = 2a 2b 2c , esto es , se
necesitan soluciones ntegras de
a + b + c = 10, a 0, b 0, c 0.


10 + 3 1
12
Por el teorema 235 stas son
=
= 66 en nmero.
31
2

3.3. Principio de inclusin-exclusin


La regla de la adicin 183 da la cardinalidad de la reunin de conjuntos disjuntos. En esta seccin se mostrar como deducir
la cardinalidad de la reunin de conjuntos no necesariamente disjuntos.
El principio de inclusin-exclusin es atribudo tanto a Sylvester como a Poincar.
238 Teorema (Inclusin-exclusin para dos conjuntos)

card (A B) = card (A) + card(B) card(A B)


Demostracin: En el diagrama de Venn 3.5, sea R1 el nmero de elementos simultneamente en ambos conjuntos
(i.e., en A B), sea R2 el nmero de elementos en A pero no en B (i.e., en A \ B) y por R3 el nmero de elementos
en B pero no en A (i.e., en B \ A). Se tiene R1 + R2 + R3 = card (A B), lo cual demuestra el teorema.
C
R4
R6

A
R2

R1

R3

Figura 3.5: Inclusin-exclusin para dos conjuntos.

R2

R3
R5

R7
R1

Figura 3.6: Inclusin-exclusin para tres conjuntos.

239 Ejemplo De 40 personas, 28 fuman y 16 mascan tabaco. Adems, se sabe que 10 tanto fuman como mascan tabaco.
Cuntas personas ni fuman ni mascan tabaco?

40

Captulo 3
Resolucin: Si X es un conjunto finito, denotaremos por card (X) su cardinalidad, esto es, el nmero de elementos que hay en el conjunto. Sea A el conjunto de personas que fuman y B el conjunto de personas que mascan
tabaco. Como card (A B) = card (A) + card (B) card (A B) = 28 + 16 10 = 34, hay 34 personas o que fuman
o que mascan tabaco. Por lo tanto, el nmero de personas que ni fuma ni masca tabaco es 40 34 = 6.

240 Ejemplo Cuatrocientos nios forman un crculo y los numeramos 1, 2, . . . , 400. Sea k, 1 k 400 un entero fijo. Marcamos cada k nios detenindonos cuando marcamos a un nio por segunda vez. Por ejemplo, si k = 6, comenzamos marcando
los nins 6, 12, 18, . . ., 396. Luego nos toca marcar al nio 2, pues el sexto luego 396 es el 2. Seguimos marcando a los nios
8, 14, 20, . . ., 398. Nos toca ahora marcar a los nios 4, 10, 16, . . ., 400. El prximo nio a marcar es el sexto, que lo marcamos
pues por segunda vez. Notamos que dejamos sin marcar a los nios 1, 3, 5, 7, 9, 11, . . .399esto es, los enteros de la forma
6k 1, 6k + 3 entre 1 y 400. Para cuntos valores de k sern marcados todos los nios al menos una vez?

Resolucin: Vemos que si k tiene un factor mayor que 1 en comn con 400, entonces no marcamos a todos
los nios. As pues, las ks requeridas son aquellas ks entre 1 y 400 inclusive que son relativamente primas a
400. Ahora bien, 400 = 24 52 . Para contar las ks que no tienen factores primos en comn con 400, contaremos
las que s tienen factores en comn con 400 y las restaremos a 400. Sea A el conjunto los mltiplos de 2 en
{1, 2, 3, . . ., 400} y B el conjunto de mltiplos de 5 en {1, 2, 3, . . ., 400}. Por inclusin- exclusin card (A B) =
card (A) + card (B) card(A B). Ahora bien,

card (A) =

400
400
400
= 200, card (B) =
= 80, card (A B) =
= 40.
2
5
10

Luego card (A B) = 240 enteros en {1, 2, . . .400} no son relativamente primos a 400 y 400 240 = 160 lo son.
As pues, slo 160 ks provocan que todos los nios sean marcados.

Sea n = pa11 pa22 pas s , donde las ps son primos distintos. Si (n) denota el nmero de enteros k, 1 k n relativamente
primos a n, entonces por inclusin-exclusin se puede demostrar que

(n) = (pa11 pa11 1 )(pa22 pa221 ) (pas s psas 1 ).


241 Teorema (Inclusin-exclusin en tres conjuntos)

card (A B C) =

card (A) + card(B) + card (C)


card (A B) card(B C) card (C A)
+card (A B C)

Principio de inclusin-exclusin

41

Demostracin: Combinando la propiedad asociativa y distributiva,

card (A B C) =

card (A (B C))

card (A) + card(B C) card (A (B C))

card (A) + card(B C) card ((A B) (A C))

card (A) + card(B) + card(C) card (B C)


card (A B) card (A C)
+card ((A B) (A C))

card (A) + card(B) + card(C) card (B C)


(card (A B) + card(A C) card(A B C))

card (A) + card(B) + card(C)


card (A B) card (B C) card(C A)
+card (A B C).

Esto demuestra el teorema. Vase tambin la figura 3.6.

242 Ejemplo De 200 polticos entrevistados en la legislatura, 75 usan cocana, 85 usan herona y 100 utilizan barbitricos.

Entre los 200, 30 usan cocana y herona, 50 usan herona y barbitricos y 40 utilizan cocana y barbitricos. Finalmente, 10
indulgen en el uso de las tres substancias. Cuntos de estos 200 polticos no usan drogas?

Resolucin: Sean A, B,C el conjunto de polticos entre los 200 que utilizan cocana, herona y barbitricos,
respectivamente. Se nos es dado que card (A) = 75, card (B) = 85, card (C) = 100, card(A B) = 30, card(B C) =
50, card(C A) = 40, card (A B C) = 10. Por el principio de inclusin-exclusin

card (A B C) = 75 + 85 + 100 30 50 40 + 10 = 150

polticos utilizan al menos una droga. Luego, 200 150 = 50 no utilizan ninguna droga.

243 Ejemplo Cuntos nmeros entre 1 y 600 inclusive no son divisibles ni por 3, ni por 5, ni por 7?

42

Captulo 3
Resolucin: Dentese por Ak aquellos enteros en el intervalo [1; 600] que son divisibles por k = 3, 5, 7. Entonces
card (A3 )

card (A5 )

card (A7 )

card (A15 )

card (A21 )

card (A35 )

card (A105 ) =

600

3
600

5
600

7
600

15
600

21
600

35
600

105

200,

120,

85,

40

28

17

Por inclusin-exclusin hay 200 + 120 + 85 40 28 17 + 5 = 325 enteros en [1; 600] divisibles por al menos
uno de los enteros en {3, 5, 7}. Los no divisibles por los enteros en {3, 57} son 600 325 = 275 en total.
C

sin 9

9550

14266

3
A

14406

14266

9550

9550

14266

sin 7

Figura 3.7: Ejemplo 244.

sin 8

Figura 3.8: Ejemplo 245.

244 Ejemplo En un grupo de 30 personas , 8 hablan ingls , 12 hablan castellano y 10 hablan francs. Se sabe que 5 hablan

ingls y castellano , 5 castellano y francs, y 7 ingls y francs. Tres personas hablan los tres idiomas. Cuntas personas no
hablan ninguno de estos idiomas?
Resolucin: Sea A el conjunto de los anglfonos, B el conjunto de los hispanfonos y C el conjunto de los francfonos. Llenamos sucesivamente los diagramas de Venn en la figura 3.7. En la interseccin de los tres ponemos
8. En la regin comn de A y B que no ha sido llenada ponemos 5 2 = 3. En la regin comn entre A y C que no
ha sido llenada ponemos 5 3 = 2. En la regin comn de B y C que no ha sido llenada ponemos 7 3 = 4. En la
parte restante de A ponemos 8 2 3 2 = 1, en la parte restante de B ponemos 12 4 3 2 = 3, y en la parte
restante de C ponemos 10 2 3 4 = 1. Las regiones disjuntas cuentan 1 + 2 + 3 + 4 + 1 + 2 + 3 = 16 personas.
Fuera de estos tres crculos hay 30 16 = 14.
245 Ejemplo Considrese el conjunto de enteros naturales de cinco dgitos escritos en notacin decimal.
1. Cuntos hay?

5. Cuntos tienen exactamente dos 9s?

2. Cuntos no tienen un 9 en su expansin decimal?

6. Cuntos tienen exactamente tres 9s?

3. Cuntos tienen al menos un 9 en su expansin decimal?

7. Cuntos tienen exactamente cuatro 9s?

4. Cuntos tienen exactamente un 9?

8. Cuntos tienen exactamente cinco 9s?

Principio de inclusin-exclusin

43

9. Cuntos no tienen ni un 8 ni un 9 en su expansin decimal?


10. Cuntos no tienen ni un 7, ni un 8, ni un 9 en su expansin
decimal?

11. Cuntos tienen o bien un 7, o un 8, o un 9 en su expansin


decimal?

Resolucin:
1. Hay 9 selecciones posibles para el primer dgito y 10 selecciones posibles para los dgitos restantes. El nmero
de selecciones es as 9 104 = 90000.

2. Hay 8 selecciones posibles para el primer dgito y 9 selecciones posibles para los dgitos restantes. El nmero
de selecciones es as 8 94 = 52488.

3. Esto es la diferencia 90000 52488 = 37512.

4. Se condiciona en el primer dgito. Si el primer dgito


es un 9 entonces los otros cuatro dgitos restantes debern de ser distintos de 9, lo que da 94 = 6561 nmeros.
Si el primer dgito no es un 9, entonces hay
8 seleccio4
nes para este primer dgito. Adems hay
= 4 ma1
neras de seleccionar donde el 9 estar, y hay 93 maneras de llenar los 3 espacios restantes. En este caso hay 8 4 93 = 23328 tales nmeros. En total hay
6561 + 23328 = 29889 enteros de cinco dgitos con
exactamente un 9 en expansin decimal.
5. Se condiciona en el primer dgito. Si el primer dgito
es un 9 uno de los cuatro restantes
ser
un 9, y selec4
cin de posicin se puede lograr en
= 4 maneras.
1
Los otros tres dgitos restantes debern de ser distintos
de 9, lo que da 4 93 = 2916 tales nmeros. Si el primer dgito no es un 9, entonces hay
8 selecciones para

4
este primer dgito. Adems hay
= 6 maneras de
2
seleccionar donde los otros dos 9s estarn, y hay 92
maneras de llenar los dos espacios restantes. En este
caso hay 8 6 92 = 3888 tales nmeros. En total hay
2916 + 3888 = 6804 enteros de cinco dgitos con exactamente dos 9s en su expansin decimal.
6. De nuevo, condicionamos en el primer dgito. Si el primer dgito es un 9 entonces dos de los restantes cuatro
sern
9s, y la seleccin de posicin se puede lograr en
4
= 6 maneras. Los otros dos dgitos restantes de2

bern de ser distintos de 9, lo que da 6 92 = 486 tales nmeros. Si el primer dgito no es un 9, entonces
hay
8 selecciones para este primer dgito. Adems hay
4
= 4 maneras de seleccionar donde los tres 9s es3
tarn, y se tiene 9 maneras de llenar los espacios restantes. En este caso hay 8 4 9 = 288 tales nmeros. En
total hay 486 + 288 = 774 enteros de cinco dgitos con
exactamente tres 9s en expansin decimal.
7. Si el primer dgito es un 9 entonces tres de los restantes cuatro
sern
9s, y la seleccin de posicin se puede
4
lograr en
= 4 maneras. Los otros restantes dgi3
tos debern de ser distintos de 9, lo que da 4 9 = 36
tales nmeros. Si el primer dgito no es un 9, entonces
hay
8 selecciones para este primer dgito. Adems hay
4
= 4 maneras de seleccionar donde los cuatro 9s
4
estarn, as llenando todos estos espacios. En este caso
hay 8 1 = 8 tales nmeros. En total hay 36 + 8 = 44
enteros de cinco dgitos con exactamente tres 9s en expansin decimal.
8. Obviamente existe solamente 1 entero tal.

Obsrvese que 37512 = 29889 + 6804 +


774 + 44 + 1.
9. Hay 7 selecciones para el primer dgito y 8 selecciones
para los restantes 4 dgitos, lo que da 7 84 = 28672
tales enteros.
10. Hay 6 selecciones para el primer dgito y 7 selecciones
para los restantes 4 dgitos, lo que da 6 74 = 14406
tales enteros.
11. Se utiliza inclusin-exclusin. De la figura 3.8, los nmeros dentro de los crculos suma a 85854. Luego el
nmero deseado es 90000 85854 = 4146.

246 Ejemplo

Cuntas soluciones enteras tiene la ecuacin


a + b + c + d = 100,
que satisfacen las siguientes restricciones:
1 a 10, b 0, c 2, 20 d 30?

44

Captulo 3


Resolucin: Se utiliza inclusin-exclusin. Hay

80
= 82160 soluciones ntegras
3

a + b + c + d = 100, a 1, b 0, c 2, d 20.
Sea A el conjunto de soluciones
a 11, b 0, c 2, d 20
y B el conjunto de soluciones


a 1, b 0, c 2, d 31.


70
69
59
Entonces card (A) =
, card (B) =
, card (A B) =
y as
3
3
3


70
69
59
card (A B) =
+

= 74625.
3
3
3
El nmero total de soluciones
a + b + c + d = 100
con
1 a 10, b 0, c 2, 20 d 30
es as

80
70
69
59

+
= 7535.
3
3
3
3

Tarea
247 Problema De cuntas maneras diferentes puede un estudiante adivinar un examen

mentos, demuestre que

completo de verdadero/falso que tenga dieciocho preguntas?

  
2n =

248 Problema En contando el nmero total de subconjuntos de un conjunto de n ele-

 

n
n
n
n
n
+
+
+ +
+
.
0
1
2
n1
n

Captulo

Sumas y recurrencias
4.1. Progresiones aritmticas
Consideremos el siguiente problema.
249 Ejemplo Si la sucesin de trminos 6, 10, 14, 18, 22, . . . sigue la misma ley de formacin, hallar los trminos 10mo, 50avo , 100avo.

Puede hallar el n-simo trmino?


Resolucin: Observemos que cada trmino se obtiene sumndole 4 al trmino anterior. As:
10 = 6 + 4, 14 = 10 + 4, 18 = 14 + 4, 22 = 18 + 4, . . .
Pero an podemos ir ms lejos. Podemos expresar cada trmino en trminos del primero. Luego
6

6 + 0 4 primer termino

10 =

6 + 1 4 segundo termino

14 =

6 + 2 4 tercer termino

18 =

6 + 3 4 cuarto termino

22 =

6 + 4 4 quinto termino.

Si el patrn de formacin es respetado para los trminos subsiguientes entonces deberamos tener: dcimo trmino
= 6 + 9 4 = 42, cincuentavo trmino = 6 + 49 4 = 202 y cienavo trmino = 6 + 99 4 = 402. De igual manera
deducimos que el ensimo trmino es = 6 + 4(n 1).
Una progresin como la del ejemplo previo, en donde la diferencia de trminos consecutivos es constante se llama progresin aritmtica. As
18, 12, 6, 0, 6, 12, . . .
es una progresin aritmtica de diferencia comn 6, mientras que 1, 3, 7, 97, . . . no lo es, ya que trminos sucesivos no guardan
diferencia constante.
En general, si comenzamos con un nmero arbitrario, digamos a y si guardamos una diferencia comn de d, entonces
obtenemos la progresin aritmtica a, a + d, a + 2d, a + 3d, . . ., con trmino en la posicin n igual a a + (n 1)d.
250 Ejemplo Hallar el 35avo trmino de una progresin aritmtica cuyo 27avo trmino es 186 y cuyo 45avo trmino es 312.

45

46

Captulo 4
Resolucin: Tratemos de expresar la data que sabemos en trminos del primer trmino y la diferencia comn.
Como no se nos da el primer trmino, llammosle a y a la diferencia comn llammosla d. As el primer trmino
es a, el segundo a + d, el tercero a + d + d = a + 2d, el cuarto a + 2d + d = a + 3d, etc. As el 27avo trmino
debe ser a + 26d y el 45avo trmino debe ser a + 44d. Pero la data del problema estipula que a + 26d = 186
y a + 44d = 312. De aqu (a + 44d) (a + 26d) = 312 186 = 126, i.e., 18d = 126 o d = 7. Pero entonces
186 = a + 26d = a + 26 7 = a + 182, de donde a = 4. Finalmente el 35avo trmino, o sea, a + 34d est dado por
a + 34d = 4 + 34(7) = 242.

Veremos ahora como sumar progresiones aritmticas.


251 Ejemplo Sumar la progresin aritmtica

7 + 15 + 23 + + 807.
Resolucin: Vemos que los trminos estn en progresin aritmtica: 7, 7 + 8 1, 7 + 8 2, . . ., 7 + 8 100. Observe
que si S = 7 + 15 + 23 + + 807, entonces tambin S = 807 + 799 + 791 + + 7. As: 2S = (7 + 807) + (15 +
799) + (23 + 791) + + (807 + 7) = 101 814 = 82214. Finalmente, S = 41107.
252 Ejemplo Sumar 5/2, 1, 1/2, . . . hasta 19 trminos.

Resolucin: La diferencia comn es 3/2. Luego el primer trmino es 5/2 = 5/2 + 0(3/2), el segundo
1 = 5/2 + 1(3/2), el tercero 1/2 = 5/2 + 2(3/2), . . . , el diecinueveavo trmino 5/2 + 18(3/2) = 49/2.
As, la suma que queremos es
S = 5/2 + 1 1/2 49/2.
Operando como en los ejemplos anteriores,
2S

(5/2 49/2) + (1 46/2) + (1/2 43/2) + + (49/2 + 5/2)

44/2 44/2 44/2 44/2

19(44/2)

418.

Colegimos luego que S = 209.


La siguiente frmula ocurre con regularidad y el lector har bien en aprender a deducirla. Si
An = 1 + 2 + 3 + + n
entonces tambin
An = n + (n 1) + + 1.

Sumando estas dos cantidades,

(n 1) +

(n + 1) +

+ (n + 1)

An

An

2An

(n + 1)

n(n + 1),

puesto que hay n sumandos. De esto colegimos


1 + 2 + + n =

n(n + 1)
.
2

(4.1)

Tarea

47

253 Ejemplo Hallar el valor de la suma

1 2 + 3 4 + 5 6 + + 99 100.
Resolucin: Observe que 1 = 1 2 = 3 4 = = 99 100. Luego agrupando la suma en cincuenta pares
que suman 1, tenemos
1 2 + 3 4 + 5 6 + + 99 100 = 50.

254 Ejemplo Hallar la suma

12 22 + 32 42 + 52 62 + + 992 1002.
Resolucin: Como x2 (x + 1)2 = 2x 1, tenemos
(12 22 ) + (32 42 ) + (52 62 ) + + (992 1002) = (3 + 7 + 11 + + 199) = 5050.

Tarea
255 Problema Hallar los trminos 14 y 110 de la progresin 43, 42, 41, . . ..

269 Problema (AHSME 1994) Sumar la serie

2
1
20 + 20 + 20 + + 40.
5
5

256 Problema Hallar los trminos 20 y 310 de la progresin 43, 40, 37, . . ..

257 Problema Hallar los trminos 12 y 1090 de la progresin 0,6, 1,2, 1,8, . . ..

258 Problema Hallar los trminos 13 y 150 de la progresin a 2d, a d, a, . . ..

270 Problema (AIME 1984) Hallar el valor de a2 + a4 + a6 + + a98 si a1 , a2 , a3 , . . .


es una progresin aritmtica con diferencia comn 1 y con a1 +a2 +a3 + +a98 = 137.
271 Problema Demostrar que

1
2
3
1995
+
+
+ +
1996 1996 1996
1996

259 Problema Hallar los trminos 10 y 51 de la progresin x y, x + y, x + 3y, . . ..

260 Problema Sumar 64, 96, 128, . . . hasta cuarenta trminos.

261 Problema Sumar 1/2, 1/2 x, 1/2 2x, . . . hasta treinta trminos.

es un entero.
272 Problema (AHSME 1991) Sea Tn = 1 + 2 + 3 + + n y

Pn =
262 Problema Sumar x y, x + y, x + 3y, . . . hasta diez trminos.

T2
T3
T4
Tn

.
T2 1 T3 1 T4 1
Tn 1

Hallar P1991 .
263 Problema Hallar el trmino 15 de una progresin aritmtica cuyo trmino 14 es 9
y cuyo trmino 16 es 90.

273 Problema Considere la tabla:

1=1
2+3+4 = 1+8

264 Problema Hallar el trmino 22 de una progresin aritmtica cuyo trmino 11 es

1 y cuyo trmino 16 es 55.

5 + 6 + 7 + 8 + 9 = 8 + 27
10 + 11 + 12 + 13 + 14 + 15 + 16 = 27 + 64

265 Problema Hallar el nmero de trminos y la diferencia comn de una serie aritm-

tica cuya suma es 30, cuyo primer trmino es 9 y cuyo ltimo trmino es 14.

Descubra el patrn de formacin, conjeture una ley general y demustrela.


274 Problema Los enteros naturales impares se agrupan en parntesis de la siguiente

266 Problema Sumar

1 + 2 + 3 + + 100.

manera:
(1)
(3, 5)

267 Problema Demostrar que

(7, 9, 11)

n2 (n2 + 1)
1 + 2 + 3 + + (n 1) + n =
.
2
2

268 Problema Demostrar que

1 + 3 + 5 + + 2n 1 = n2 .

(13, 15, 17, 19)


(21, 23, 25, 27, 29)
...............................
Halle la suma del sexto, sptimo, y octavo grupos. Conjeture y demuestre una frmula
para la suma del ensimo parntesis.

48

Captulo 4

4.2. Progresiones geomtricas


Consideremos ahora la progresin 2, 6, 18, 54, 162, . . .. Notamos que cada trmino es el triple del anterior. Una progresin
de este tipo se llama progresin geomtrica. Decimos que 3 es la razn comn de esta progresin geomtrica. Vemos que el
primer trmino est dado por 2(3)0 , el segundo por 2(3)1 , el tercero por 2(3)2 , el cuarto por 2(3)3 , el quinto por 2(3)4 , etc. As
pues, el cincuentavo trmino est dado por 2(3)49 , el 100-avo es 2(3)99 y el ensimo trmino est dado por 2(3)n1 .
275 Ejemplo Hallar el dcimo trmino de la progresin geomtrica

1/2, 1, 2, . . ..
1
Resolucin: Vemos que la razn comn es 2. Luego el primer trmino est dado por (2)0 , el segundo est
2
1
1
1
1
2
3
dado por (2) , el tercero por (2) , el cuarto por (2) , etc. El patrn indica pues que el dcimo trmino
2
2
2
1
est dado por (2)9 .
2
En general si una progresin geomtrica tiene primer trmino a y razn comn r, entonces va de la siguiente manera: a, ar, ar2 , ar3 , . . .
y el ensimo trmino est dado por arn1 .
276 Ejemplo Hallar el segundo trmino de una progresin geomtrica con cuarto trmino 24 y sptimo trmino 192.

Resolucin: No conocemos el primer trmino, llammosle a. Tampoco conocemos la razn comn, llammosle
r. Luego el primer trmino est dado por a, el segundo por ar, el tercero por ar2 , el cuarto por ar3 y siguiendo el
patrn, el sptimo trmino es ar6 . La data es que 24 = ar3 y 192 = ar6 . De aqu tenemos
r3 =

ar6 192
=
= 8,
ar3
24

de donde r = 2. Luego a(2)3 = 24 y as a = 3. Luego el segundo trmino est dado por ar = 6.


277 Ejemplo Hallar la suma de la serie geomtrica

1 + 2 + 4 + + 1024.
Resolucin: Pongamos
S = 1 + 2 + 4 + + 1024.
Entonces
2S = 2 + 4 + 8 + + 1024 + 2048.
As
S = 2S S = (2 + 4 + 8 + 2048) (1 + 2 + 4 + + 1024) = 2048 1 = 2047.

278 Ejemplo Hallar la suma de la serie geomtrica

x=

1 1
1
1
+ + + + 99 .
3 32 33
3

Resolucin: Tenemos
1
1
1
1
1
x = 2 + 3 + + 99 + 100 .
3
3
3
3
3

Progresiones geomtricas

49

Luego
2
x
3

=
=

1
x x
3
1 1
1
1
( + 2 + 3 + + 99 )
3 3
3
3
1
1
1
1
( 2 + 3 + + 99 + 100 )
3
3
3
3
1
1

.
3 3100

De esto colegimos
x=

1
1

.
2 2 399

279 Ejemplo Sumar

a = 1 + 2 4 + 3 42 + + 10 49.
Resolucin: Tenemos
Luego 4a a nos da

4a = 4 + 2 42 + 3 43 + + 9 49 + 10 410.
3a = 1 4 42 43 49 + 10 410.

Al sumar esta ltima serie geomtrica,


a=

10 410 410 1

.
3
9

280 Ejemplo Hallar la suma

Sn = 1 + 1/2 + 1/4 + + 1/2n.


Interpretar el resultado cuando n crece indefinidamente.
Resolucin: Tenemos
1
Sn Sn = (1 + 1/2 + 1/4 + + 1/2n) (1/2 + 1/4 + + 1/2n + 1/2n+1) = 1 1/2n.
2
As
Sn = 2 1/2n.
Calculamos ahora los siguientes valores:
S1

= 2 1/20

= 1

S2

= 2 1/2

= 1,5

S3

= 2 1/22

= 1,875

S4

= 2 1/23

= 1,875

S5

= 2 1/24

= 1,9375

S6

= 2 1/25

= 1,96875

S10

= 2 1/29

= 1,998046875

50

Captulo 4
Vemos que a medida que tomamos ms trminos de la serie, nos acercamos cada vez ms a 2.

Supongamos que los procedimientos que utilizamos para sumar series geomtricas finitas son vlidos para las infinitas.
Entonces podremos decir que
S = 1 + 1/2 + 1/22 + 1/23 +
hasta infinitos suma a 2, ya que
1
S S = (1 + 1/2 + 1/22 + 1/23 + ) (1/2 + 1/22 + 1/23 + 1/24 + )) = 1
2
puesto que los trminos luego del primero encuentran su opuesto en la serie de la derecha.
La manipulacin formal que utilizamos supra debe manejarse con sumo cuidado. Por ejemplo,
S = 2 + 22 + 23 +
es obviamente infinitamente grande, pues cada trmino es mayor que 1 y esto incrementa el valor de la serie cada vez ms. Sin
embargo, al operar formalmente como lo hicimos anteriormente obtenemos
S = 2S S = (22 + 23 + 24 + ) (2 + 22 + 23 + ) = 2,
i.e., la suma de nmeros positivos da un resultado negativo, lo cual es un disparate maysculo.
Qu sucede entonces? Entra ahora pues el concepto de convergencia. Diremos que la serie geomtrica
a + ar + ar2 +
converge hacia un valor finito S si cada suma parcial
Sn = a + ar + ar2 + + arn1
se acerca ms y ms a S a medida que n aumenta. Como vimos en los ejemplos anteriores, esto suceder cuando |r| < 1. Si las
sumas parciales no se acercan a un valor finito definitivo cuando n aumenta, entonces decimos que la serie geomtrica diverge.
Del ejemplo anterior se vislumbra que esto sucede cuando |r| 1.
La teora de series geomtricas convergentes puede utilizarse para convertir un decimal peridico a una fraccin.
281 Ejemplo Hallar la fraccin que representa

x = 0,122222222222222 . . .
Resolucin: Observemos que
x=
Pero
S=

1
2
2
2
+ 2 + 3 + 4 + .
10 10
10
10

2
2
2
2
1
+
+
+ =
= .
102 103 104
90 45

As
0,1222222222 . . . =

1
1
11
+
= .
10 45 90

282 Ejemplo Hallar la fraccin que representa

x = 1,304040404040 . . ..
Resolucin: Tenemos
x = 1+

30
40
40
40
+
+
+
+ .
102 104 106 108

Tarea

51
Ahora bien, la serie geomtrica infinita
S=

40
40
40
+ 6 + 8 +
4
10
10
10

suma a
S=
As
x = 1+

40
2
=
.
9900 495

30
2
1291
+
=
.
102 495
990

283 Ejemplo Una mosca est en el origen (0, 0) y viaja sobre el plano una pulgada hacia el este, 1/2 pulgada hacia el norte,

1/4 de pulgada hacia el oeste, 1/8 de pulgada hacia el sur, 1/16 de pulgada hacia el este, etc. Si la mosca hiciese esto ad
infinitum, donde terminara?
Resolucin: Sea (X,Y ) el punto donde la mosca terminara. Vemos que
X = 1

1
1
1
4
+

+ =
4 16 64
5

y
1 1
1
1
2
+

+ = .
2 8 32 128
5
Luego la mosca termina en el punto (4/5, 2/5).
Y=

Tarea
284 Problema Hallar los trminos 13, 22 y ensimo de la progresin geomtrica

3/2, 3/8, 3/32, . . .

285 Problema Si el trmino 15 de una progresin geomtrica es 100 y el trmino 20

2. Qu valor tendr
1/10 + 1/10 + 1/100 +
hasta infinito?
3. Qu valor tendr
9/10 + 9/100 + 9/1000 +

es 125, hallar el segundo trmino.

hasta infinito? Discuta por qu


286 Problema Hallar la suma de las siguientes series geomtricas:

1 = 0,9999999999999999999 . . .

1.
1/2 1/4 + 1/8 + 1/512 1/1024.
2.
6 + 18 + 54 + . . .

4. Qu valor tendr
1 + 1 + 1 +
hasta infinito? Discuta.
5. Qu valor tendr

hasta 20 trminos.

1 1 + 1 1 +

3.
1/10 + 1/100 + 1/1000 + . . .
hasta 10 trminos.

hasta infinito?

289 Problema Hallar

287 Problema Hallar la suma de la serie

k
.
2k

k=1

3 + 5 4 + 7 42 + 9 43 + + 99 448 .
288 Problema

1. Qu valor tendr
1/2 + 1/4 + 1/8 +

hasta infinito?

290 Problema Hallar todos los nmeros complejos x, y tales que x, x+2y, 2x+y formen
una progresin aritmtica a la vez que (y + 1)2 , xy + 5, (x + 1)2 formen una progresin
geomtrica.

291 Problema Los lados de un tringulo rectngulo forman una progresin geomtrica.
Halle las tangentes de los ngulos agudos.

52

Captulo 4

292 Problema Sean a, b las races de la ecuacin x2 3x + A = 0 y sean c, d las ra-

293 Problema Los nmeros a1 , a2 , . . . ,an forman una progresin geomtrica. Si

ces de la ecuacin x2 12x + B = 0. Se sabe que a, b, c, d forman, en este orden, una


progresin geomtrica creciente. Halle A y B.

s = a1 + a2 + + an ; T =

1
1
1
+
+ + ,
a1
a2
an

hallar su producto P = a1 a2 an en trminos de s y T.

4.3. Cancelacin telescpica


A veces podemos sumar una serie
a1 + a2 + a3 + + an
si podemos encontrar una sucesin {vk } con ak = vk vk1 . Entonces
a1 + a2 + a3 + + an = v1 v0 + v2 v1 + + vn1 vn2 + vn vn1 = vn v0 .
Si tal sucesin {vk } existe, diremos entonces que la serie a1 + a2 + + an es una serie telscopica.
294 Ejemplo Simplificar

1+

1
1
1
1
1+
1+
1 +
.
2
3
4
99

Resolucin: Sumando cada fraccin tenemos:


3 4 5
100

,
2 3 4
99
lo que simplifica a 100/2 = 50.
295 Ejemplo Dado que

(log2 3) (log3 4) (log4 5) (log511 512)


es un entero, hllelo.
Resolucin: Poniendo todo en una base comn, digamos a > 0, a 6= 1:
(log2 3) (log3 4) (log4 5) (log511 512) =

loga 3 loga 4 loga 5


loga 512 loga 512

=
.
loga 2 loga 3 loga 4
loga 511
loga 2

Pero
loga 512
= log2 512 = log2 29 = 9,
loga 2
de donde obtenemos el resultado.
296 Ejemplo Simplificar

99

(2 + 1) 22 + 1 22 + 1 22 + 1 22 + 1 .

Cancelacin telescpica

53

Resolucin: Utilizando la identidad x2 y2 = (x y)(x + y) y llamando P al producto:


(2 1)P =

99

99

(2 1) (2 + 1) 22 + 1 22 + 1 22 + 1 22 + 1

22 1 22 + 1 22 + 1 22 + 1 22 + 1

99

99

22 1 22 + 1 22 + 1 22 + 1

22 1 22 + 1 22 + 1 22 + 1

=
..
.

99

..
.

99

(22 1)(22 + 1)

22

=
de donde
P = 22

100

100

1,

1.

297 Ejemplo Simplificar

S = log tan 1 + log tan 2 + logtan 3 + + log tan 89 .


Resolucin: Observe que (90 k) + k = 90 . Luego, sumando el trmino k-simo con el 90 k-simo obtenemos que la suma dada es
S

log(tan 1 )(tan 89 ) + log(tan 2 )(tan 88 )


+ log(tan 3 )(tan 87 ) + + log(tan 44 )(tan 46 ) + logtan 45 .

Como tan k = 1/ tan(90 k), tenemos


S = log 1 + log1 + + log 1 + logtan 45 .

Finalmente, como tan 45 = 1, colegimos

S = log 1 + log1 + + log 1 = 0.

298 Ejemplo Hallar el valor exacto del producto

P = cos

2
4
cos
cos .
7
7
7

Resolucin: Multiplicando a uno y otro lado por sen y haciendo uso de la identidad sen 2x = 2 sen x cos x
7
obtenemos

2
4
sen P = (sen cos ) cos
cos
7
7
7
7
7
1
2
2
4
=
(sen
cos ) cos
2
7
7
7
1
4
4
=
(sen
cos )
4
7
7
1
8
=
sen .
8
7

54

Captulo 4

Como sen

8
= sen , deducimos que
7
7
1
P= .
8

299 Ejemplo Demostrar que

1 3 5
9999
1

<
.
2 4 6
10000 100
Resolucin: Pongamos
A=

9999
1 3 5

2 4 6
10000

B=

2 4 6
10000

.
3 5 7
10001

Es claro que x2 1 < x2 para todo nmero real x. De esto deducimos


x
x1
<
.
x
x+1
Por tanto
1/2

<

2/3

3/4

<

4/5

5/6

<

6/7

..
.

..
.

..
.

9999/10000 <

10000/10001

Como todas estas desigualdades son de nmeros positivos, podemos multiplicar una y otra columna para obtener
1 3 5
9999
2 4 6
10000

<
,
2 4 6
10000 3 5 7
10001
o A < B. Luego A2 = A A < A B. Pero entrelazando los factores de A y B,
1 2 3 4 5 6 7
9999 10000
1

=
,
2 3 4 5 6 7 8
10000 10001 10001

y por consiguiente, A2 < A B = 1/10001. De aqu A < 1/ 10001 < 1/100.


AB =

Para nuestro siguiente ejemplo necesitaremos la siguiente definicin. El smbolo n! (lase n factorial) significa
n! = 1 2 3 n.
Por ejemplo 1! = 1, 2! = 1 2 = 2, 3! = 1 2 3 = 6, 4! = 1 2 3 4 = 24. Observe que (k + 1)! = (k + 1)k!. Tendremos por
convencin 0! = 1.
300 Ejemplo Sumar

1 1! + 2 2! + 3 3! + + 99 99!.

Cancelacin telescpica

55

Resolucin: De (k + 1)! = (k + 1)k! = k k! + k! deducimos (k + 1)! k! = k k!. As


1 1!

= 2! 1!

2 2!

= 3! 2!

3 3!

= 4! 3!

..
.

..
.

98 98

= 99! 98!

..
.

99 99! = 100! 99!


Sumando una y otra columna,
1 1! + 2 2! + 3 3! + + 99 99! = 100! 1! = 100! 1.

301 Ejemplo Hallar una forma cerrada para la suma

An = 1 + 2 + + n.
Resolucin: Observemos que
k2 (k 1)2 = 2k 1.
Luego
12 02

= 211

22 12

= 221

32 22

= 231

..
.

..
.

n2 (n 1)2

..
.

= 2n1

Sumando una y otra columna,


n2 02 = 2(1 + 2 + 3 + + n) n.
Resolviendo para la suma,
1 + 2 + 3 + + n = n2 /2 + n/2 =

n(n + 1)
.
2

302 Ejemplo Hallar la suma

12 + 22 + 32 + + n2 .
Resolucin: Observemos que
k3 (k 1)3 = 3k2 3k + 1.

56

Captulo 4
Luego
13 03

3 12 3 1 + 1

23 13

3 22 3 2 + 1

33 23

..
.

..
.

3 32 3 3 + 1

n3 (n 1)3

..
.

3 n2 3 n + 1

Sumando una y otra columna,


n3 03 = 3(12 + 22 + 32 + + n2 ) 3(1 + 2 + 3 + + n) + n.
Pero por el ejercicio anterior se tiene
n3 03 = 3(12 + 22 + 32 + + n2 )

3
n(n + 1) + n.
2

Resolviendo para la suma,


12 + 22 + 32 + + n2 =

n3 1
n
+ n(n + 1) .
3 2
3

Simplificando obtenemos
12 + 22 + 32 + + n2 =

n(n + 1)(2n + 1)
.
6

303 Ejemplo Sumar la serie

1
1
1
1
+
+
+ +
.
12 23 34
99 100
Resolucin: Observe que
1
1
1
=
.
k(k + 1) k k + 1
Luego
1
12
1
23
1
34
..
.

Sumando una y otra columna obtenemos

1
99 100

=
=
=
..
.
=

1 1

1 2
1 1

2 3
1 1

3 4
..
.
1
1

99 100

1
1
1
1
1
99
+
+
+ +
= 1
=
.
12 23 34
99 100
100 100

304 Ejemplo Sumar

1
1
1
1
+
+
+ +
.
1 4 4 7 7 10
31 34

Cancelacin telescpica

57

Resolucin: Observe que


1
1
1
1
1
=

.
(3n + 1) (3n + 4) 3 3n + 1 3 3n + 4

Luego

1
14
1
47
1
7 10
1
10 13
..
.

Sumando una y otra columna obtenemos

1
34 37

=
=
=
=
..
.
=

1
1

3 12
1
1

12 21
1
1

21 30
1
1

30 39
..
.
1
1

102 111

1
1
1
1
1
1
12
+
+
+ +
=
= .
1 4 4 7 7 10
31 34 3 111 37

305 Ejemplo Sumar

1
1
1
1
+
+
+ +
.
1 4 7 4 7 10 7 10 13
25 28 31
Resolucin: Observe que
1
1
1
1
1
=

.
(3n + 1) (3n + 4) (3n + 7) 6 (3n + 1)(3n + 4) 6 (3n + 4)(3n + 7)
Luego
1
147
1
4 7 10
1
7 10 13
..
.
1
25 28 31
Sumando una y otra columna obtenemos

1
1

614 647
1
1
=

6 4 7 6 7 10
1
1
=

6 7 10 6 10 13
.. ..
. .
=

1
1

6 25 28 6 28 31

1
1
1
1
1
1
9
+
+
+ +
=

=
.
1 4 7 4 7 10 7 10 13
25 28 31 6 1 4 6 28 31 217

306 Ejemplo Sumar

1 2 + 2 3 + 3 4 + + 99 100.
Resolucin: Observemos que
1
1
k(k + 1) = (k)(k + 1)(k + 2) (k 1)(k)(k + 1).
3
3

58

Captulo 4
Por lo tanto
1
1
123 012
3
3
1
1
=
234 123
3
3
1
1
=
345 234
3
3
.. ..
. .

12

23
34
..
.

99 100 =

1
1
99 100 101 98 99 100
3
3

Sumando una y otra columna,


1 2 + 2 3 + 3 4 + + 99 100 =

1
1
99 100 101 0 1 2 = 333300.
3
3

Tarea
307 Problema Hallar una frmula para

315 Problema (AIME 1993) Durante una campaa poltica reciente, un candidato hizo

una trayectoria que presumimos yace en el plano. En el primer da l viaj hacia el este,
en el segundo, l viaj hacia el norte, en el tercero hacia el oeste, en el cuarto hacia el
sur, en el quinto hacia el este, etc. Si el candidato viaj n2 /2 millas en el n-simo da, a
cuantas millas estaba l de su punto de partida en el 40avo da?

Dn = 1 2 + 3 4 + + (1)n1 n.
308 Problema Simplificar

(1 + i)2004
.
(1 i)2000

316 Problema Demostrar que

13 + 23 + 33 + + n3 =

309 Problema Si

a + ib = 1 + 2i + 3i2 + 4i3 + + 1995i1994 + 1996i1995 ,

n(n + 1) 2
2

317 Problema Simplificar

con a y b nmeros reales, hallar a y b.

23 1 33 1 43 1
1003 1

.
23 + 1 33 + 1 43 + 1
1003 + 1

310 Problema Simplificar

1
22

1
32

1
42

1
992

318 Problema Sean a1 , a2 , . . . ,an nmeros arbitrarios. Demostrar que

a1 + a2 (1 + a1 ) + a3 (1 + a1 )(1 + a2 ) + a4 (1 + a1 )(1 + a2 )(1 + a3 )


311 Problema Simplificar

1023
X

log2

1
1+
k

+ + an1 (1 + a1 )(1 + a2 )(1 + a3 ) (1 + an2 )

= (1 + a1 )(1 + a2 )(1 + a3 ) (1 + an ) 1.

k=2

312 Problema Hallar el valor exacto de

319 Problema Demostrar que

csc 2 + csc 4 + csc8 + + csc2n = cot 1 cot 2n .

1
1
1
1
+
+
+ +
.
log2 1996!
log3 1996! log4 1996!
log1996 1996!

320 Problema Sea 0 < x < 1. Demostrar que


313 Problema (AHSME 1996) La sucesin

1, 2, 1, 2, 2, 1, 2, 2, 2, 1, 2, 2, 2, 2, 1, 2, 2, 2, 2, 2, 1, 2, . . .

x2

1 x2n+1

x
.
1x

n=1

consiste de 1s separados por bloques de 2s, con n bloques de 2s en el n-simo bloque.


Hallar la suma de los primeros 1234 trminos de esta sucesin.
321 Problema Demostrar que
314 Problema (AIME 1985) Calcular el producto x1 x2 x8 si x1 = 97 y xn =

n/xn1 , n > 1.

tan

+ 2 tan 99 + 22 tan 98 + + 298 tan 2 = cot 100 .


2100
2
2
2
22

Recursiones y ecuaciones funcionales

59

322 Problema Demostrar que


n
X

324 Problema Demostrar que

1
n2 + n
k
= 2
.
2 n +n+1
k4 + k2 + 1

arctan
n=0

k=1

325 Problema Demostrar que

323 Problema Evale el radical

1 2 4 + 2 4 8 + 3 6 12 +
1 3 9 + 2 6 18 + 3 9 27 +

1
= .
4
1 + n + n2

1/3

1
1
1
1
1998 < 1 + + + + +
< 1999.
2
3
4
1000000

4.4. Recursiones y ecuaciones funcionales


Veremos ahora mtodos para hallar formas cerradas de ciertas recursiones. En general aplicaremos las tcnicas de la seccin
anterior.
326 Ejemplo Sea x0 = 7 y xn = 2xn1 , n 1. Hallar una frmula cerrada para xn .

Resolucin: Tenemos
x0

= 7

x1

= 2x0

x2

= 2x1

x3

= 2x2

..
.

..
.

xn

= 2xn1

..
.

Multiplicando una y otra columna,


x0 x1 xn = 7 2nx1 x2 xn1 .
Cancelando factores comunes,
xn = 7 2 n .

327 Ejemplo Sea x0 = 7 y xn = xn1 + n, n 1. Hallar una frmula cerrada para xn .

Resolucin: Tenemos
x0

= 7

x1

= x0 + 1

x2

= x1 + 2

x3

= x2 + 3

..
.

..
.

xn

= xn1 + n

..
.

60

Captulo 4
Sumando una y otra columna,
x0 + x1 + x2 + + xn = 7 + x0 + x2 + + xn1 + (1 + 2 + 3 + + n).
Cancelando y simplificando,
xn = 7 +

n(n + 1)
.
2

328 Ejemplo Sea x0 = 7 y xn = 2xn1 + 1, n 1. Hallar una frmula cerrada para xn .

Resolucin: Tenemos
x0

x1

2x0 + 1

x2

2x1 + 1

x3

2x2 + 1

..
.

..
.

..
.

xn1

2xn2 + 1

xn

2xn1 + 1

Aqu no nos funcionan los mtodos anteriores, as que nos valdremos del siguiente artificio. Multipliquemos a la
k-sima fila por 2nk , obteniendo
2 n x0

2n 7

2n1x1

2n x0 + 2n1

2n2x2

2n1 x1 + 2n2

2n3x3

2n2 x2 + 2n3

..
.

..
.

..
.

22 xn2

23 xn3 + 22

2xn1

22 xn2 + 2

xn

2xn1 + 1

Sumando una y otra columna y cancelando,


xn = 7 2n + (1 + 2 + 22 + + 2n1) = 7 2n + 2n 1 = 2n+3 1.
Aliter: Pongamos un = xn + 1 = 2xn1 + 2 = 2(xn1 + 1) = 2un1. Luego la recursin un = 2un1 la resolvemos
como en nuestro primer ejemplo, obteniendo de esta forma un = 2n u0 = 2n (x0 + 1) = 2n 8 = 2n+3 . Finalmente
xn = un 1 = 2n+3 1.
329 Ejemplo Una sucesin satisface u0 = 3, u2n+1 = un , n 1. Halle una forma cerrada para ella.

Recursiones y ecuaciones funcionales

61
1/2

Resolucin: Pongamos vn = log un . Entonces vn = log un = log un1 =


n

tenemos vn = v0 /2n, o sea, log un = (log u0 )/2n . Luego un = 31/2 .

1
vn1
log un1 =
. Como vn = vn1 /2,
2
2

330 Ejemplo Halle una forma cerrada para

a0 = 5, a j+1 = a2j + 2a j , j 0.
Resolucin: Tenemos
a j+1 + 1 = a2j + 2a j + 1 = (a j + 1)2 .
j

Pongamos v j+1 = a j+1 + 1. Entonces v j+1 = a j+1 + 1 = (a j + 1)2 = v2j . De aqu v j+1 = v20 , o sea
j

a j+1 = v j+1 1 = v20 1 = (a0 + 1)2 1 = 62 1.

331 Ejemplo Una escalera tiene n escalones. Un duende puede subir la escalera de escaln en escaln o saltndose un escaln.

Hallar una recursin para el nmero de maneras en que el duende puede subir la escalera.
Resolucin: Sea un el nmero de maneras en que puede el duende subir una escalera de n escalones. El duende
puede llegar al ltimo escaln o bien desde el penltimo o bien desde el antepenltimo escaln. Luego
un = un1 + un2.
Es claro que u1 = 1, u2 = 2.
332 Ejemplo Si f (x) = f0 (x) =

1
y fn (x) = f ( fn1 (x)) para n > 0, hallar f1996 (1/3).
1x

Resolucin: Observe que


f1 (x) = f ( f0 (x)) =

1
x1
=
,
1
x
1 1x

f2 (x) = f ( f1 (x)) =

1
=x
1 x1
x

y
1
= f0 (x).
1x
Luego esta recursin es cclica de orden 3. Esto implica que
f3 (x) =

f0 (x) = f3 (x) = f6 (x) = ,


f1 (x) = f4 (x) = f7 (x) =
y
f2 (x) = f5 (x) = f8 (x) = .
Como 1996 = 1995 + 1 deja residuo 1 al ser dividido por 3,
f1996 (1/3) = f1 (1/3) = 4.

333 Ejemplo Hallar todas las funciones que satisfacen

f (x + y) + f (x y) = 4x2 + 4y2 .

62

Captulo 4
Resolucin: Tomando y = 0, obtenemos f (x) + f (x) = 4x2 o f (x) = 2x2 . Veamos que f (x) = 2x2 satisface la
ecuacin funcional:
f (x + y) + f (x y) = 2(x + y)2 + 2(x y)2 = 2x2 + 4xy + 2y2 + 2x2 4xy + 2y2 = 4x2 + 4y2 .

Tarea
334 Problema Sea x1 = 1, xn+1 = x2n xn + 1, n > 0. Demostrar que

1
= 1.
xn

n=1

335 Problema (AIME 1994) La funcin f tiene la propiedad que para todo nmero real

341 Problema Una sucesin a1 , a2 , . . . satisface a1 = 2 y

am+n = 4mn am an m, n.
Hallar el valor mnimo de n para el cual an tiene al menos 3000 dgitos.

342 Problema Sea k un entero no-negativo fijo y supngase que

x,
f (x) + f (x 1) = x2 .

f (2x) = 2k1

Si f (19) = 94, hallar el residuo cuando f (94) se divide por 1000.

1
f (x) + f (x + ) .
2

Demustrese que
336 Problema Hallar una forma cerrada para

f (3x) = 3k1

x0 = 1; xn = xn1 + n2 , n > 0.

337 Problema Sea f (x) = f0 (x) =

1 x2 , fn (x) = f ( fn1 (x)), n > 0. Hallar

2
1
f (x) + f (x + ) + f (x + ) .
3
3

343 Problema Hallar todas las funciones f que satisfagan

f99 (4).
f (x)2 f

1x
1+x

= 64x.

338 Problema Sea f una funcin con las siguientes propiedades:

1)
2)
3)
4)
5)

f (n) est definida para todo entero positivo n;


f (n) es un entero;
f (2) = 2;
f (mn) = f (m) f (n) para todo m y n;
f (m) > f (n) si m > n.
Demostrar que f (n) = n.

344 Problema (AHSME 1979) La funcin f satisface

f (x + y) = f (x) + f (y) xy + 1
para todos los nmeros reales x, y. Si f (1) = 1, hallar todos los enteros n 6= 1 tales que
f (n) = n.

339 Problema Demostrar que existe una funcin f nica del conjunto R+ de los reales

positivos a R+ tal que

345 Problema (AHSME 1981) La funcin f no est definida para x = 0, pero si x 6= 0

f ( f (x)) = 6x f (x), f (x) > 0 x > 0.


340 Problema Si u0 = 1/3 y un+1 = 2u2n 1, hallar una frmula para un .

satisface

f (x) + 2 f

1
x

= 3x.

Para cuntos valores de x se cumple f (x) = f (x)?

Captulo

Polinomios y ecuaciones
5.1. Ecuaciones
A continuacin veremos como resolver algunas ecuaciones. La mayora de ellas son ecuaciones cuadrticas disfrazadas.
Para resolver ecuaciones cuadrticas, el mtodo ms eficiente es quizs la complecin del cuadrado. sta es preferible a la
frmula cuadrtica ya que crea malicia para identificar patrones. Por ejemplo, para resolver
x2 6x + 3 = 0,
escribimos
x2 6x + 3 =

x2 6x + 9 6

(x 3)2 ( 6)2

(x 3 + 6)(x 3 6).

De aqu x = 3 6. De manera semejante, para resolver 2x2 + 6x + 5 = 0 escribimos


2x2 + 6x + 5 =
=
=

2x2 + 6x +

9 1
+
2 2

3
1
( 2x + )2 (i )2
2
2

3
1
3
1
( 2x + i )( 2x + + i ).
2
2
2
2

3
1
Luego, x = i .
2
2
346 Ejemplo Resolver

9 + x4 = 10x2.
Resolucin: Observe que
x4 10x2 + 9 = (x2 9)(x2 1).
1
Luego x = y x = 1.
3
347 Ejemplo Resolver

9x 3x+1 4 = 0.
63

64

Captulo 5
Resolucin: Observe que 9x 3x+1 4 = (3x 4)(3x + 1). Como no existe ningn nmero real con 3x + 1 = 0,
este factor se descarta. As 3x 4 = 0 nos da x = log3 4.

348 Ejemplo Resolver

(x 5)(x 7)(x + 6)(x + 4) = 504.


Resolucin: Reordenemos los factores y multipliquemos para obtener
(x 5)(x 7)(x + 6)(x + 4) = (x 5)(x + 4)(x 7)(x + 6) = (x2 x 20)(x2 x 42).
Pongamos y = x2 x. As (y 20)(y 42) = 504 o y2 62y + 336 = (y 6)(y 56) = 0. Luego y = 6, 56, lo que
implica
x2 x = 6
y
x2 x = 56.
De aqu x = 2, 4, 7, 8.
349 Ejemplo Resolver

12x4 56x3 + 89x2 56x + 12 = 0.


Resolucin: Reordenando
12x4 + 12 56(x3 + x) + 89x2 = 0.

(5.1)

Dividiendo por x ,
12(x2 +

1
1
) 56(x + ) + 89 = 0.
x2
x

Pongamos u = x + 1/x. Luego u2 2 = x2 + 1/x2 . Usando esto, (1) se convierte 12(u2 2) 56u + 89 = 0, de
donde u = 5/2, 13/6. Por lo tanto
1 5
x+ =
x 2
y
1 13
x+ = .
x
6
Concluimos que x = 1/2, 2, 2/3, 3/2.
350 Ejemplo Hallar las soluciones reales de
p

x2 5x + 2 x2 5x + 3 = 12.
Resolucin: Observe que

x2 5x + 3 + 2 x2 5x + 3 15 = 0.
Poniendo u = x2 5x+3 obtenemos u +2u1/2 15 = (u1/2 +5)(u1/2 3) = 0. Luego u = 9 (descartamos u1/2 +5 =
0, por qu?). Por lo tanto x2 5x + 3 = 9 o x = 1, 6.
351 Ejemplo Resolver

3x2 4x + 34

3x2 4x 11 = 9.

(5.2)

Resolucin: Tenemos idnticamente


(3x2 4x + 34) (3x2 4x 11) = 45.

(5.3)

Ecuaciones

65

Dividiendo cada miembro de (3) por los miembros correspondientes de (2) obtenemos
p

3x2 4x + 34 +

Sumando (2) y (4)

3x2 4x 11 = 5.

(5.4)

3x2 4x + 34 = 7,

5
de donde x = , 3.
3
352 Ejemplo Resolver la ecuacin

Resolucin: Pngase u =

3
14 + x + 3 14 x = 4.

3
14 + x, v = 3 14 x. Entonces

64 = (u + v)3 = u3 + v3 + 3uv(u + v) = 14 + x + 14 x + 12(196 x2)1/3 ,


de donde
3 = (196 x2)1/3 ,
que al resolver nos da x = 13.
353 Ejemplo Halle el valor exacto de cos 2 /5.

Resolucin: Usando la identidad


cos(u v) = cos u cos v senu sen v
par de veces obtenemos
y

cos 2 = 2 cos2 1

(5.5)

cos 3 = 4 cos3 3 cos .

(5.6)

Pongamos x = cos 2 /5. Como cos 6 /5 = cos4 /5, gracias a las dos identidades (5) y (6), vemos que x satisface
la ecuacin
4x3 2x2 3x + 1 = 0,
o sea,
(x 1)(4x2 + 2x 1) = 0.
Como x = cos 2 /5 6= 1, y cos 2 /5 > 0, x es la raz positiva de la ecuacin cuadrtica 4x2 + 2x 1 = 0, es decir

2
51
cos
=
.
5
4

354 Ejemplo Cuntas soluciones reales tiene la ecuacin

sen x =

x
?
100

Resolucin: Vemos que x = 0 es una solucin. Adems si x > 0 es una solucin, x < 0 lo es tambin. As pues,
slo contaremos las soluciones positivas.
Para que x sea una solucin, se debe tener |x| = 100| sen x| 100. Por lo tanto podemos restringir x al intervalo
]0; 100]. Dividamos este intervalo en subintervalos de longitud 2 (con un ltimo intervalo ms corto):
]0; 100] =]0; 2 ]]2 ; 4 ]]4 ; 6 ] ]28 ; 30 ]]30 ; 100].

66

Captulo 5
De las grficas de y = sen x, y = x/100 vemos que en el intervalo ]0; 2 ] existe slo una solucin. En cada intervalo
de la forma ]2 k; 2(k +1) ], k = 1, 2, . . . , 14 existen dos soluciones. El intervalo ]30 ; 100] tiene una onda completa
de longitud (ya que 31 < 100) en la cual hay dos soluciones. Por consiguiente existen 1 + 2 14 + 2 = 31 As
pues, hay 31 soluciones positivas y por ende 31 soluciones negativas. Como 0 es tambin una solucin, el total de
soluciones reales es por lo tanto 31 + 31 + 1 = 63.

355 Ejemplo Resolver el sistema de ecuaciones

x+y+u =

4,

y + u + v = 5,
u+v+x =
v+x+y

0,

= 8.

Resolucin: Sumando las cuatro ecuaciones y diviendo por 3,


x + y + u + v = 3.
Esto implica
4+v

3,

5 + x

3,

0+y

3,

8 + u =

3.

De aqu x = 2, y = 3, u = 5, v = 7.
356 Ejemplo Resolver el sistema de ecuaciones

(x + y)(x + z) = 30,
(y + z)(y + x) = 15,
(z + x)(z + y) = 18.
Resolucin: Pongamos u = y + z, v = z + x, w = x + y. Entonces el sistema se convierte en
vw = 30, wu = 15, uv = 18.
2 2 2

(5.7)

Multiplicando todas estas ecuaciones obtenemos u v w = 8100, esto es, uvw = 90. Combinando este resultado
con cada una de estas ecuaciones en (7), obtenemos u = 3, v = 6, w = 5, o u = 3, v = 6, w = 5. Luego
y+z

= 3,

y+z

3,

z+x

= 6, o z + x

6,

x+y =

5,

x + y = 5,

de donde x = 4, y = 1, z = 2 o x = 4, y = 1, z = 2..

Tarea

Tarea

67

357 Problema Resolver

372 Problema Si la ecuacin

x
+3
a

b 6a
a
= + .
x
a
b

x+

x+

x + = 2

hiciere sentido, hallar el valor de x.

358 Problema Resuelva

(x 7)(x 3)(x + 5)(x + 1) = 1680.

373 Problema Resolver la ecuacin

x + x2 1 x x2 1
+
= 98.

x x2 1 x + x2 1

359 Problema Resuelva

x4 + x3 4x2 + x + 1 = 0.

374 Problema Sean a,b, c constantes reales con abc 6= 0. Resolver el sistema de ecua-

360 Problema Resolver la ecuacin

ciones

(x2 8x+15)/(x2)

|x 3|

x2 (y z)2 = a2 ,
= 1.

y2 (z x)2 = b2 ,
z2 (x y)2 = c2 .

361 Problema Resolver la ecuacin

0,5 logx (x2 x)

= 3log9 4 .

375 Problema Resolver el sistema

log2 x + log4 y + log4 z = 2,


362 Problema Resolver la ecuacin

log3 x + log9 y + log9 z = 2,


2sen

2x

+ 5 2cos

2x

= 7.

363 Problema Resolver la ecuacin

log4 x + log16 y + log16 z = 2.


376 Problema Resuelva el sistema


log1/3




5
5
cos x +
+ log1/3 cosx
= 2.
6
6

x3 + 3x2 y + y3 = 8,
2x3 2x2 y + xy2 = 1.

364 Problema Cuntas soluciones reales tiene la ecuacin


377 Problema Encuentre una solucin real para la ecuacin

sen x = ln x?

(x2 9x 1)10 + 99x10 = 10x9 (x2 1).

365 Problema Resolver la ecuacin


378 Problema Resolver el sistema de ecuaciones

|x + 1| |x| + 3|x 1| 2|x 2| = x + 2.

x2 yz = 3,

366 Problema Hallar las races reales de

y2 zx = 4,

z2 xy = 5.

x+34 x1+
x + 8 6 x 1 = 1.
379 Problema Resolver el sistema

367 Problema Resolver la ecuacin

2x + y + z + u = 1
6x4 25x3 + 12x2 + 25x + 6 = 0.
x + 2y + z + u = 12
x + y + 2z + u = 5

368 Problema Una progresin geomtrica de nmeros reales satisface que la suma de

sus primeros cuatro trminos es 15 y la suma de los cuadrados de estos trminos es 85.
Hallar esta progresin.

x + y + z + 2u = 1

380 Problema Resolver el sistema de ecuaciones

369 Problema Resolver la ecuacin

x2 + x + y = 8,
x(2x + 1)(x 2)(2x 3) = 63.
y2 + 2xy + z = 168,
2

z + 2yz + 2xz = 12480.

370 Problema Hallar el valor de

30 31 32 33 + 1.

371 Problema Si la ecuacin

x
hiciere sentido, hallar el valor de x.

381 Problema Hallar las races reales de la ecuacin

..
xx

=2

x+2

x+2

p
x + + 2

{z

n radicales

x + 2 3x = x.

68

Captulo 5
x y = 2.

382 Problema Resolver la ecuacin

1
1+

= x.

1
1

1+
1+

385 Problema Resolver el sistema de ecuaciones

.
.
.

1+ 1x

x1 x2 = 1, x2 x3 = 2, . . . , x100 x101 = 100, x101 x1 = 101.

Aqu la expresin de la fraccin se repite n veces.


386 Problema Resuelva para x
383 Problema Resolver el sistema

x+2+y+3+

x + x + 11 +
x + x 11 = 4.

(x + 2)(y + 3) = 39,

(x + 2)2 + (y + 3)2 + (x + 2)(y + 3) = 741.


387 Problema Dos estudiantes trataron de resolver la ecuacin cuadrtica x2 +bx +c =
0. Maguer ambos estudiantes ejecutaron todos los pasos correctamente, el primero copi
mal el coeficiente b y obtuvo las soluciones x = 6, 1. El segundo copi mal c y obtuvo
las soluciones x = 2, 3. Cules son las soluciones correctas?

384 Problema Resolver el sistema de ecuaciones

x4 + y4 = 82,

5.2. Polinomios
Recordemos que un polinomio es una expresin de la forma
p(x) = a0 + a1 x + a2x2 + + an xn .
Aqu los coeficientes ak de p(x) pueden ser cualquier nmero complejo. Si las ak s pertenecen exclusivamente al conjunto de
los nmeros enteros diremos que p(x) Z[x]. Si las ak s son nmeros reales entonces escribiremos p(x) R[x]. Finalmente,
escribiremos p(x) C[x] si las ak s son nmeros complejos.
388 Ejemplo Hallar la suma de todos los coeficientes obtenidos luego de expandir y simplificar el producto

(1 x2 + x4 )109 (2 6x + 5x9)1996 .
Resolucin: Pongamos
p(x) = (1 x2 + x4 )109 (2 6x + 5x9)1996 .
Vemos que p(x) un polinomio de grado 4 109 + 9 1996 = 18400. As pues, p(x) es tambin la expresin
p(x) = a0 + a1 x + a2x2 + + a18400x18400 .
Vemos entonces que la suma de los coeficientes de p(x) es
p(1) = a0 + a1 + a2 + + a18400,
que tambin es p(1) = (1 12 + 14 )109 (2 6 + 5)1996 = 1. As pues, la suma deseada es igual a 1.
389 Ejemplo Pngase

(1 + x4 + x8 )100 = a0 + a1 x + a2x2 + + a800x800 .


Hallar:
(A) a0 + a1 + a2 + a3 + + a800.
(B) a0 + a2 + a4 + a6 + + a800.
(C) a1 + a3 + a5 + a7 + + a799.
(D) a0 + a4 + a8 + a12 + + a800.
(E) a1 + a5 + a9 + a13 + + a797.
Resolucin: Pongamos
p(x) = (1 + x4 + x8 )100 = a0 + a1x + a2 x2 + + a800 x800 .

Polinomios

69

Entonces
(A)
a0 + a1 + a2 + a3 + + a800 = p(1) = 3100 .
(B)
a0 + a2 + a4 + a6 + + a800 =

p(1) + p(1)
= 3100 .
2

(C)
a1 + a3 + a5 + a7 + + a799 =

p(1) p(1)
= 0.
2

(D)
a0 + a4 + a8 + a12 + + a800 =

p(1) + p(1) + p(i) + p(i)


= 2 3100.
4

(E)
a1 + a5 + a9 + a13 + + a797 =

p(1) p(1) ip(i) + ip(i)


= 0.
4

Otra propiedad de los polinomios que es a menudo til es el algoritmo de divisin: si dividimos p(x) por a(x) obtendremos
polinomios q(x), r(x) con
p(x) = a(x)q(x) + r(x).
Aqu 0 grado r(x) < grado a(x). Por ejemplo, al dividir x5 + x4 + 1 por x2 + 1 obtenemos
x5 + x4 + 1 = (x3 + x2 x 1)(x2 + 1) + x + 2,
de donde el cociente es q(x) = x3 + x2 x 1 y el residuo es r(x) = x + 2.
390 Ejemplo Hallar el residuo cuando (x + 3)5 + (x + 2)8 + (5x + 9)1997 se divide por x + 2.

Resolucin: Como estamos dividiendo por un polinomio de grado 1, el residuo es un polinomio de grado 0, es
decir, una constante. As pues, existe un polinomio q(x) y una constante r con
(x + 3)5 + (x + 2)8 + (5x + 9)1997 = q(x)(x + 2) + r
Si ponemos x = 2 obtenemos
0 = (2 + 3)5 + (2 + 2)8 + (5(2) + 9)1997 = q(2)(2 + 2) + r = r,
de donde el residuo es r = 0.
391 Ejemplo Un polinomio deja residuo 2 cuando se divide por x 1 y residuo 4 cuando se divide por x + 2. Hallar el

residuo cuando este polinomio se divide por x2 + x 2.


Resolucin: De la informacin dada existen polinomios q1 (x), q2 (x) con p(x) = q1 (x)(x 1) 2 y p(x) =
q2 (x)(x + 2) 4. Luego p(1) = 2 y p(2) = 4. Ahora bien, como x2 + x 2 = (x 1)(x + 2) es un polinomio de
grado 2, el residuo r(x) al dividir p(x) por x2 + x 1 es de grado 1 o menor, es decir r(x) = ax + b para constantes
a, b que debemos determinar. Por el algoritmo de divisin
p(x) = q(x)(x2 + x 1) + ax + b.
Luego
2 = p(1) = a + b
y
4 = p(2) = 2a + b.
De estas ecuaciones vemos que a = 2/3, b = 8/3. Luego el residuo es r(x) = 2x/3 8/3.

70

Captulo 5

392 Ejemplo Sea f (x) = x4 + x3 + x2 + x + 1. Hallar el residuo cuando f (x5 ) se divide por f (x).

Resolucin: Observe que f (x)(x 1) = x5 1 y


f (x5 ) = x20 + x15 + x10 + x5 + 1 = (x20 1) + (x15 1) + (x10 1) + (x5 1) + 5.
Cada sumando en parntesis es divisible por x5 1 y por ende por f (x). Luego el residuo es 5.
El algoritmo de divisin nos ayuda a demostrar el siguiente resultado, a menudo conocido como el Teorema del factor.
393 Teorema (Teorema de Ruffini) El polinomio p(x) es divisible por x a si y slo si p(a) = 0.

Demostracin: Como x a es un polinomio de grado 1, el residuo al dividir p(x) por x a es un polinomio de


grado 0, es decir, una constante. As
p(x) = q(x)(x a) + r.
De aqu p(a) = q(a)(a a) + r = r. El teorema se deduce de esto.
394 Ejemplo Si p(x) un polinomio cbico con p(1) = 1, p(2) = 2, p(3) = 3, p(4) = 5. Hallar p(6).

Resolucin: Pongamos g(x) = p(x) x. Entonces g(x) es un polinomio de grado 3 y g(1) = g(2) = g(3) = 0.
Luego g(x) = c(x 1)(x 2)(x 3) para alguna constante c que debemos determinar. Pero g(4) = c(4 1)(4
2)(4 3) = 6c y g(4) = p(4) 4 = 1, de donde c = 1/6. Finalmente
p(6) = g(6) + 6 =

(6 1)(6 2)(6 3)
+ 6 = 16.
6

395 Ejemplo El polinomio p(x) tiene coeficientes enteros y p(x) = 7 para cuatro valores enteros diferentes de x. Demostrar

que p(x) 6= 14 para ningn entero x.


Resolucin: El polinomio g(x) = p(x) 7 se anula para cuatro enteros diferentes a, b, c, d. Luego, por el
Teorema del factor,
g(x) = (x a)(x b)(x c)(x d)q(x),
para algn polinomio q(x) con coeficientes enteros. Supongamos que p(t) = 14 para algn entero t. Entonces
g(t) = p(t) 7 = 14 7 = 7. De aqu
7 = g(t) = (t a)(t b)(t c)(t d)q(t),
esto es, hemos factorizado a 7 como el producto de al menos cuatro factores enteros distintos, lo que es imposible,
pues 7 es a lo sumo 7(1)1 el producto de tres enteros distintos. De esta contradiccin colegimos que no existe tal
entero t.
396 Ejemplo Hallar un polinomio cbico p(x) que se anule cuando x = 1, 2, 3 y que satisfaga p(4) = 666.

Resolucin: El polinomio debe tener la forma p(x) = a(x 1)(x 2)(x 3), donde a es una constante. Como
666 = p(4) = a(4 1)(4 2)(4 3) = 6a, a = 111. Luego el polinomio deseado es p(x) = 111(x1)(x2)(x3).

397 Ejemplo Hallar un polinomio cbico p(x) con p(1) = 1, p(2) = 2, p(3) = 3, p(4) = 5.

Polinomios

71

Resolucin: Utilizaremos el siguiente mtodo debido a Lagrange. Sea


p(x) = a(x) + 2b(x) + 3c(x) + 5d(x),
donde a(x), b(x), c(x), d(x) son polinomios cbicos con las siguientes propiedades: a(1) = 1 y a(x) se anula para
x = 2, 3, 4; b(2) = 1 y b(x) se anula cuando x = 1, 3, 4; c(3) = 1 y c(3) = 1 se anula para x = 1, 2, 4 y d(4) = 1,
d(x) anulndose cuando x = 1, 2, 3.
Utilizando el mtodo del problema anterior hallamos
a(x) =

(x 2)(x 3)(x 4)
,
6

(x 1)(x 3)(x 4)
,
2
(x 1)(x 2)(x 4)
c(x) =
2
b(x) =

y
d(x) =

(x 1)(x 2)(x 3)
.
6

As

1
p(x) = (x 2)(x 3)(x 4) + (x 1)(x 3)(x 4)
6
3
5
(x 1)(x 2)(x 4) + (x 1)(x 2)(x 3).
2
6
El lector podr verificar que este polinomio cumple con las condiciones estipuladas.
Por ltimo discutiremos las frmulas de Vite y las identidades de Newton-Girard. Para introducir el tpico consideremos
primero el siguiente ejemplo.
398 Ejemplo Hallar el producto

(x + 1)(x 2)(x + 4)(x 5)(x + 6).


Resolucin: Vemos que el producto es un polinomio de grado 5. Para obtener el coeficiente de x5 tomamos una
x de cada binomio. As pues el coeficiente de x5 es 1. Para formar el trmino de x4 tomamos una x de cuatro de los
binomios y una constante del binomio restante. As pues, el coeficiente de x4 es
1 2 + 4 5 + 6 = 4.
Para formar el trmino de x3 tomamos tres x de tres de los binomios y dos constantes de los dos binomios restantes.
As el coeficiente de x3 es
(1)(2) + (1)(4) + (1)(5) + (1)(6) + (2)(4) + (2)(5) + (2)(6)
+(4)(5) + (4)(6) + (5)(6) = 33.
De manera semejante, el coeficiente de x2 es
(1)(2)(4) + (1)(2)(5) + (1)(2)(6) + (1)(4)(5) + (1)(4)(6) + (2)(4)(5)
+(2)(4)(6) + (4)(5)(6) = 134
y el coeficiente de x es
(1)(2)(4)(5) + (1)(2)(4)(6) + (1)(2)(5)(6) + (1)(4)(5)(6) + (2)(4)(5)(6) = 172.
Finalmente, el trmino constante es (1)(2)(4)(5)(6) = 240. El producto pedido es entonces
x5 + 4x4 33x3 134x2 + 172x + 240.

72

Captulo 5

Del ejemplo anterior vemos que cada trmino tiene un peso de 5, pues de cada uno de los cinco binomios o bien tomamos
el trmino de x o bien tomamos la constante.
Si a0 6= 0 y
a0 xn + a1 xn1 + a2 xn2 + + an1x + an
es un polinomio con races 1 , 2 , . . . , n entonces podemos escribir

a0 xn + a1xn1 + a2 xn2 + + an1 x + an = a0 (x 1 )(x 2 )(x 3 ) (x n1 )(x n ).


De esto deducimos las frmulas de Vite:

a1 X
=
k ,
a0
n

k=1

a2
=
a0

a3
=
a0

a4
=
a0

j k ,

1 j<kn

j k l ,

1 j<k<ln

j k l s ,

1 j<k<l<sn

..........
..........
...........
a
n
(1)n = 1 2 n .
a0

399 Ejemplo Hallar la suma de las races, la suma de las races tomadas de dos en dos, la suma de los cuadrados de las races

y la suma de los recprocos de las races de la ecuacin


2x3 x + 2 = 0.
Resolucin: Sean a, b, c las races de 2x3 x + 2 = 0. Por las frmulas de Vite la suma de las races es
0
a+b+c= = 0
2
y la suma de las races tomadas de dos en dos es
ab + ac + bc =

1
.
2

Para hallar a2 + b2 + c2 recurrimos a la siguiente identidad


a2 + b2 + c2 = (a + b + c)2 2(ab + ac + bc).
Luego
a2 + b2 + c2 = 02 2(1/2) = 1.
Finalmente, como abc = 2/2 = 1, vemos que
1 1 1 ab + ac + bc 1/2
+ + =
=
= 1/2.
a b c
abc
1

Polinomios

73

400 Ejemplo Sean , , las races de x3 x2 + 1 = 0. Hallar

1
1
1
+ 2 + 2.
2

Resolucin: De x3 x2 + 1 = 0 deducimos 1/x2 = 1 x. Luego


1
1
1
+
+
= (1 ) + (1 ) + (1 ) = 3 ( + + ) = 3 1 = 2.
2 2 2

Conjunto con las frmulas de Vite tenemos las identidades de Newton-Girard para las sumas de potencias sk = 1k + 2k + +
nk de las races:
a0 s1 + a1 = 0,
a0 s2 + a1s1 + 2a2 = 0,
a0 s3 + a1 s2 + a2 s1 + 3a3 = 0,
etc..
401 Ejemplo Si a, b , c son las races de x3 x2 + 2 = 0, hallar

a 2 + b 2 + c2
a 3 + b 3 + c3
y
a 4 + b 4 + c4 .
Resolucin: Primero observamos que
a2 + b2 + c2 = (a + b + c)2 2(ab + ac + bc) = 12 2(0) = 1.
Como x3 = x2 2, obtenemos
a3 + b3 + c3 = a2 2 + b2 2 + c2 2 = a2 + b2 + c2 6 = 1 6 = 5.
Finalmente, de x3 = x2 2 obtenemos x4 = x3 2x, de donde
a4 + b4 + c4 = a3 2a + b3 2b + c3 2c = a3 + b3 + c3 2(a + b + c) = 5 2(1) = 7.

402 Ejemplo (USAMO 1973) Determine todas las soluciones, reales o complejas del sistema de ecuaciones

x + y + z = 3,
x2 + y2 + z2 = 3,
x3 + y3 + z3 = 3.
Resolucin: Sean x, y, z las races del polinomio
p(t) = (t x)(t y)(t z) = t 3 (x + y + z)t 2 + (xy + yz + zx)t xyz.
Ahora bien, xy + yz + zx = (x + y + z)2/2 (x2 + y2 + z2 )/2 = 9/2 3/2 = 3 y de
x3 + y3 + z3 3xyz = (x + y + z)(x2 + y2 + z2 xy yz zx)
se desprende que xyz = 1. Luego
p(t) = t 3 3t 2 + 3t 1 = (t 1)3 .
Luego x = y = z = 1 es la nica solucin del sistema anterior.

74

Captulo 5

Tarea
403 Problema Sea

410 Problema Suponga que

(1 + x + x2 )n = a0 + a1 x + + a2n x2n .

xn + a1 xn1 + a2 xn2 + + an = (x + r1 )(x + r2 ) (x + rn )


donde r1 , r2 , . . . ,rn son nmeros reales. Demuestre que

Hallar
a0 + a2 + a4 + + a2n .

404 Problema Demostrar que las tres races de x 1 = 0 son = 1/2 + i 3/2, 2 =

3
2
1/2 i 3/2 y = 1. Demostrar que + + 1 = 0.

(n 1)a21 2na2 .

411 Problema Si 1 , 2 , . . . , 100 son las races de

x100 10x + 10 = 0,

405 Problema Sea

(1 + x2 + x4 )100 = a0 + a1 x + + a400 x400 .

hallar la suma

100
1100 + 2100 + + 100
.

Hallar
a0 + a3 + a6 + + a399 .
406 Problema El polinomio p(x) satisface p(x) = p(x). Cuando p(x) es dividido
por x 3 el residuo es 6. Cul es el residuo cuando p(x) es dividido por x2 9?

412 Problema Hallar un polinomio p(x) de grado 4 con p(1) = 1, p(2) =


2, p(3) = 4, p(4) = 5, p(5) = 8.
413 Problema Sean , , las races de x3 x 1 = 0. Halle

1
1
1
+ 3+ 3
3

407 Problema La ecuacin x4 16x3 + 94x2 + px + q = 0 tiene dos races dobles. Ha-

llar p + q.
y
408 Problema (USAMO 1984) El producto de dos de las races de

x4 18x3 + kx2 + 200x 1984 = 0

414 Problema Los nmeros reales , safisfacen

3 3 2 + 5 17 = 0,

es 32. Determine el valor de k.

3 3 2 + 5 + 11 = 0.

409 Problema Si p(x) es un polinomio de grado n tal que p(k) = 1/k, k = 1, 2, . . . ,n +

1, hallar el valor de p(n + 2).

5 + 5 + 5.

Demuestre que + = 2.

Captulo

Desigualdades
6.1. Valor absoluto y desigualdad del tringulo
Se presume que R est dotado de una relacin > que satisface los siguientes axiomas.
415 Axioma (Ley de tricotoma) (x, y) R2 exactamente una de las siguientes se cumple:

x > y,

o y > x.

x = y,

416 Axioma (Transitividad) (x, y, z) R3 ,

si x > y

y y>z

entonces x > z.

417 Axioma (Conservacin de las desigualdades en la adicin) (x, y, z) R3 ,

si x > y

entonces x + z > y + z.

418 Axioma (Conservacin de las desigualdades en la multiplicacin por factores positivos) (x, y, z) R3 ,

si x > y

y z>0

entonces xz > yz.

x < y es lo mismo que y > x. x y quiere decir y > x, o bien y = x, etc.

Alerta al vocabulario! Se dir que un nmero x es positivo si x 0 y que es estrictamente positivo


si x > 0. De manera semejante, se dir que un nmero y es negativo si y 0 y que es estrictamente
negativo si y < 0. Este uso difiere de algunos autores, que suelen utilizar palabrotas en neo-lengua
como no-negativo y no-positivo.

419 Definicin (La funcin signum) Sea x un nmero real. La funcin signum se define y se denota por
8
>
>
1 if x < 0,
>
>
<

signum (x) =

0
>
>
>
>
:

+1

El siguiente lema es inmediato.


75

if x = 0,
if x > 0.

76

Captulo 6

420 Lema La funcin signum es multiplicativa, esto es, si (x, y) R2 entonces signum (x y) = signum (x) signum(y).
421 Definicin (Valor absoluto) Sea x R. El valor absoluto de x se define y se denota por

|x| = signum (x) x.


Las siguientes propiedades del valor absoluto se deducen de inmediato de su definicin.
422 Teorema Sea x R. Entonces
8
>
<

1. |x| =

>
: x

if x < 0,
if x 0.

2. |x| 0,
3. |x| = max(x, x),
4. |x| = |x|,
5. |x| x |x|.

6. x2 = |x|
7. |x|2 = |x2 | = x2
8. x = signum (x) |x|
423 Teorema ((x, y) R2 ),

|xy| = |x| |y| .

Demostracin: Se tiene
|xy| = signum (xy) xy = (signum(x) x) (signum (y) y) = |x| |y| ,
en donde se ha utilizado el lema 420.
424 Teorema Sea t 0. Entonces

|x| t t x t.

Demostracin: O bien, |x| = x, o bien, |x| = x. Si |x| = x,


|x| t x t t 0 x t.
Si |x| = x,

|x| t x t t x 0 t.

425 Teorema Si (x, y) R2 , entonces max(x, y) =

x + y + |x y|
x + y |x y|
y mn(x, y) =
.
2
2

Demostracin: Obviamente, max(x, y) + mn(x, y) = x + y. Ahora, o bien |x y| = x y y entonces x y, lo


que significa que max(x, y) mn(x, y) = x y, o bien |x y| = (x y) = y x, lo que significa que y x y

Valor absoluto y desigualdad del tringulo

77

entonces max(x, y) mn(x, y) = y x. En cualquier caso, max(x, y) mn(x, y) = |x y|. Resolviendo el sistema
de ecuaciones
max(x, y) + mn(x, y) =

x+y

max(x, y) mn(x, y) =

|x y|,

para max(x, y) y para mn(x, y) se obtiene el resultado.


426 Teorema (Desigualdad del tringulo) Sea (a, b) R2 . Entonces

|a + b| |a| + |b|.

(6.1)

Demostracin: De 5 en el teorema 422, por adicin,


|a| a |a|
to
|b| b |b|,
obteniendo
(|a| + |b|) a + b (|a| + |b|),
de donde sigue el teorema, en aplicando 424.
Por induccin, se obtiene la siguiente generalizacin,
427 Corolario Sean x1 , x2 , . . . , xn nmeros reales. Entonces

|x1 + x2 + + xn | |x1 | + |x2 | + + |xn | .


Demostracin: Aplicando el teorema 426 n 1 veces
|x1 + x2 + + xn |

|x1 | + |x2 + xn1 + xn |

|x1 | + |x2 | + |x3 + xn1 + xn |

|x1 | + |x2 | + + |xn1 + xn |

|x1 | + |x2 | + + |xn1 | + |xn | .

..
.

428 Corolario Sea (a, b) R2 . Entonces,

||a| |b|| |a b| .

Demostracin: Se tiene
|a| = |a b + b| |a b| + |b|,

(6.2)

78

Captulo 6
dando
|a| |b| |a b|.
De manera semejante,
|b| = |b a + a| |b a| + |a| = |a b| + |a|,
dando
|b| |a| |a b| = |a b| |a| |b| .
As,
|a b| |a| |b| |a b|
y se aplica ahora el teorema 424.

429 Ejemplo (Desigualdad de Weierstrass) Sea n > 0 un entero. Sea xi 0 para toda i [1; n]. Demostrar que
n
Y
k=1

(1 + xk ) 1 +

n
X

xk .

k=1

Estudiar el caso de igualdad.


Resolucin: Expandiendo el producto
n
Y

(1 + xk ) = 1 +

k=1

n
X
k=1

xk +

n
X
1i< jn

xi x j + 1 +

n
X

xk ,

k=1

la igualdad cumplindose porque xk 0. Cuando n = 1 la igualdad es obvia. Si n > 1 la igualdad se cumple para
n
X

xi x j = 0.

1i< jn

430 Ejemplo Demostrar que para todo x > 0,


n
X
k=1

1
1
1
<
.
(x + k)2
x x+n

Resolucin: Obsrvese que k 1, (x + k)2 > (x + k)(x + k 1) y por lo tanto


1
1
1
1
<
=

.
2
(x + k)
(x + k)(x + k 1) x + k 1 x + k
De aqu,
1
1
1
1
1
+
+
+ +
+
(x + 1)2 (x + 2)2 (x + 3)2
(x + n 1)2 (x + n)2

<

1
1
1
+
+
x(x + 1) (x + 1)(x + 2) (x + 2)((x + 3))
1
1
+ +
+
(x + n 2)(x + n 1) (x + n 1)(x + n)
1
1
1
1
1
1

x x+1 x+1 x+2 x+2 x+3


1
1
1
1
+ +

x+n2 x+n1 x+n1 x+n


1
1

.
x x+n

Valor absoluto y desigualdad del tringulo


431 Ejemplo Sean xi R tales que

X
i=1

79

|xi | = 1 y que

xi = 0. Demustrese que

i=1

n
X
xi 1
1

1
.



i 2
n
i=1

Resolucin: Para 1 i n, se tiene

2

1 2
4 4
1
4
(i n)(i 1)
1 1 1 1 2 1 + 1

0.
1

1
+
+ 0
i

2
n
n
i
n
n
i
i
n
n
i2 n



n
n
X
xi 1 X 2
1


1+
xi ,

=


i 2
i
n

Thus

ya que

i=1

i=1

xi = 0. Ahora bien,

i=1

n
n
n
X
2

2
1
1


1 1 |xi | 1 1
|x
|
=
1+
xi
1

.

i
i


i
n
n
n
n
i=1

i=1

i=1

Tarea
432 Problema Sean x, y nmeros reales. Demostrar que

demostrar que

1
1
< x+1 x < .

2 x
2 x+1

0 x < y x2 < y2 .

Utilcese este hecho para demostrar que si n > 1 es un entero, entonces


433 Problema Sean t 0, nmeros reales. Demostrar que

|x| t (x t)

or (x t).

434 Problema Sean (x, y) R2 . nmeros reales. Demostrar que max(x, y) =

mn(x, y).

1
1
1
2 n + 1 2 < 1 + + + + < 2 n 1
n
2
3
439 Problema Demostrar que si n > 2 es un entero, entonces

nn/2 < n!.


435 Problema Sean x, y, z nmeros reales. Demostrar que

max(x, y, z) = x + y + z mn(x, y) mn(y, z) mn(z, x) + mn(x, y, z).

440 Problema Demostrar que

1 3 5
9999
1

<
.
2 4 6
10000
100

436 Problema Sean b > 0 y B > 0. Demostrar que

a
A
a
a+A
A
<
=
<
< .
b
B
b
b+B
B

n
X

441 Problema Sea n 2 un entero y a1 , a2 , . . . ,an reales tal que

An ms, si p y q son enteros positivos tales que


7
p
11
< <
,
10
q
15

Demostrar que

X
| ai a j |

hallar el valor mnimo de q.


i< j

437 Problema Sea a < b. Demostrar que

|x a| < |x b| x <

ai = 0.
i=1

a+b
.
2

438 Problema Si x > 0, usando la identidad

1
x+1 x =
,
x+1+ x

n
2

n
X

| ai | .
i=1

442 Problema Dado un conjunto de nmeros reales {a1 , a2 , . . . ,an } demostrar que existe un ndice m {0, 1, . . . ,n} tal que



X
X

ak
ak max |ak | .


1kn
1km

m<kn

Si m = 0 la primera suma se toma como 0 y si m = n, la segunda se toma como 0.

80

Captulo 6

6.2. El cuadrado de todo real es positivo


443 Teorema El cuadrado de todo real es positivo, esto es, a R, entonces a2 0. An ms, si a 6= 0, entonces a2 > 0.

Demostracin: Si a = 0, entonces 02 = 0 y no hay nada que demostrar. Presmase ahora que a 6= 0. Por tricotoma, o bien a > 0 o bien a < 0. Presmase primero que a > 0. Gracias al axioma 418 con x = z = a y y = 0, se
tiene
aa > a0 = a2 > 0,
mostrando el teorema cuando a > 0.
Presmase ahora que a < 0. Entonces a > 0 y se aplica lo venido de demostrar:
a > 0 = (a)2 > 0 = 1 a2 > 0 = a2 > 0,
obteniendo nuevamente el resultado.
El teorema 443 probar ser extremadamente til y ser la base de las desigualdades clsicas de este captulo. Se comenzar con
la desigualdad de las medias, de la que se darn varias demostraciones y generalizaciones en las pginas subsiguientes.
444 Teorema (Desigualdad de las medias aritmtica y geomtrica) Sean a 0, b 0. Entonces

a+b
ab
,
2
con igualdad si y slo si a = b.
Demostracin: Por el teorema 443,

2
a b 0,

con igualdad si y slo si a = b. Expandiendo,

a+b
a b 0 a 2 ab + b 0 ab
,
2

demostrando el teorema.
445 Teorema (Desigualdad de las medias armnica y geomtrica) Sean a > 0, b > 0. Entonces

2
1 1
+
a b

ab

si y slo si a = b.
Demostracin: Por el teorema 444,
r

1 1
+

1 1
2
a b
ab,
1
1
a b
2
+
a b

demostrando el teorema.
446 Ejemplo Demostrar que la suma de todo real positivo y su recproco es al menos 2.

El cuadrado de todo real es positivo

81

Resolucin: Sea x > 0. Entonces


r

1=

1
1 x+ x
1
= 2 x + ,
x
x
2
x

demostrando la asercin. Ntese que hay igualdad si y slo si


x=

1
x2 = 1 x = 1,
x

ya que se presume x > 0.


447 Ejemplo Sean a, b dos reales no nulos. Determinar el valor mnimo de

a6 a4 a2 b6 b4 b2
+ + + + + .
b6 b4 b2 a6 a4 a2
Resolucin: Se tiene
a6 a4 a2 b6 b4 b2
+ + + + +
b6 b4 b2 a6 a4 a2

 6
a

b6

b6
a6

2+2+2

6,

 4
a

+
4

b4
a4

 2
a

b2

b2
a2

con igualdad si y slo si a = b.


448 Ejemplo Demostrar que si x > 0, y > 0, entonces

x
y
1
+
.
x4 + y2 x2 + y4
xy
Resolucin: Se tiene
x
y
+
x4 + y2 x2 + y4

y
+
2
2 x2 y4
y
x
+
2
2x y 2xy2
1
.
xy

x4 y2

449 Ejemplo Sean u1 , u2 , u3 , u4 reales positivos. Aplicando la desigualdad de las medias aritmtica y geomtrica de dos nme-

ros, establecer la desigualdad de las medias aritmtica y geomtrica de cuatro nmeros:


(u1 u2 u3 u4 )1/4

u1 + u2 + u3 + u4
4

u1 + u2
u3 + u4
u1 u2
y u3 u4
. Ahora, aplicando la desigualdad de las medias
2
2

aritmtica y geomtrica de dos nmeros dos veces a u1 u2 y u3 u4 se obtiene

u1 +u2

4
+ u3 +u
u1 u2 + u3 u4

2
u1 u2 u3 u4
2
.
2
2
Resolucin: Se tiene

Simplificando se obtiene el resultado deseado.

(6.3)

82

Captulo 6

450 Ejemplo Sean u, v, w reales positivos. Utilizando el ejemplo 449, demostrar la desigualdad de las medias aritmtica y
geomtrica de tres nmeros:
u+v+w
(6.4)
(uvw)1/3
3

Resolucin: Por el ejemplo 449,




uvw

 u + v + w 1/4

u + v + w + u+v+w
3
.
4

u+v+w u+v+w
+
,
4
12

Luego de masajear la desigualdad anterior, se obtiene


(uvw)1/4

 u + v + w 1/4

lo que equivale a
(uvw)1/4
Multiplicando uno y otro lado por

 u + v + w 1/4

 u + v + w 1/4

u+v+w
.
3

se obtiene

(uvw)1/4

 u + v + w 3/4

de donde se destila el resultado.


451 Ejemplo Demostrar que si a, b, c son reales positivos entonces

(a + b)(b + c)(c + a) 8abc.

Resolucin:
El resultado se obtiene en seguida en multiplicando las desigualdades a + b 2 ab, b + c 2 bc
y c + a 2 ca.
452 Ejemplo Sean a, b, c, d, nmeros reales ligados por la relacin a2 + b2 + c2 + d 2 = ab + bc + cd + da. Demostrar que

a = b = c = d.
Resolucin: Se tiene,
a2 ab + b2 bc + c2 dc + d 2 da = 0,
o sea,
a2
b2 b2
c2 c2
d2 d2
a2
ab + + bc + + dc + + da + = 0.
2
2
2
2
2
2
2
2
Descomponiendo en factores,
1
1
1
1
(a b)2 + (b c)2 + (c d)2 + (d a)2 = 0.
2
2
2
2
Como la suma de nmeros positivos es cero solamente cuando cada nmero es cero, se obtiene a = b, b = c, c =
d, d = a, lo que demuestra la asercin.

Se nota de paso que de la identidad


a2 + b2 + c2 ab bc ca =

1
(a b)2 + (b c)2 + (c a)2
2

(6.5)

se obtiene la desigualdad
a2 + b2 + c2 ab + bc + ca.

(6.6)

El cuadrado de todo real es positivo

83

453 Teorema Sean (a, b, x, y) R4 con x > 0, y > 0. Entonces

a2 b2 (a + b)2
+

.
x
y
x+y
Hay igualdad si y slo si

a b
= .
x
y

Demostracin: Como el cuadrado de todo real es positivo,


(ay bx)2 0

a2 y2 2abxy + b2x2 0

a2 y(x + y) + b2x(x + y) (a + b)2 xy

a2 b2 (a + b)2
+

.
x
y
x+y

Se cumple igualdad si y slo si ay = bx.


Iterando el teorema 453, se obtiene el corolario siguiente.
454 Corolario Sean ak , bk nmeros reales con bk > 0.

a21 a22
a2 (a1 + a2 + + an )2
+ + + n
,
b1 b2
bn
b1 + b2 + + bn
con igualdad si y slo si

an
a1 a2
=
= = .
b1 b2
bn

Tarea
455 Problema Sean (x1 , x2 , . . . ,xn ) Rn tales que

461 Problema Demostrar que si a, b, c son reales positivos, entonces

a b
c
+ + 3.
b
c
a

x21 + x22 + + x2n = x31 + x32 + + x3n = x41 + x42 + + x4n .


Demostrar que xk {0, 1}.

462 Problema Sean a, b, c nmeros reales. Demostrar que

456 Problema Sea n 2 un entero. Sea (x1 , x2 , . . . ,xn ) Rn tal que

x21 + x22 + + x2n = x1 x2 + x2 x3 + + xn1 xn + xn x1 .

a3 + b3 + c3 3abc = (a + b + c)(a2 + b2 + c2 ab bc ca).


Utilizar esta identidad para demostrar nuevamente que

u+v+w
3
uvw
3

Demostrar que x1 = x2 = = xn .
457 Problema Demostrar que si r s t entonces

r2 s2 + t 2 (r s + t)2 .

para reales positivos u, v, w.

463 Problema (AIME 1983) Minimizar la funcin

x 7

458 Problema Si 0 < a b, demostrar que

1 (b a)2
a+b
1 (b a)2

ab
8
b
2
8
a
459 Problema Demostrar que si a, b, c son reales positivos, entonces

(a2 + 1)(b2 + 1)(c2 + 1) 8abc.


460 Problema La suma de dos reales positivos es 100. Maximizar su producto.

9x2 sen2 x + 4
x senx

en el intervalo ]0; [.

464 Problema Demostrar que si 0 x 1 entonces x x2

si 0 a, b, c, d 1 entonces alguno de entre los productos


a(1 b), b(1 c),

es

1
.
4

1
. Luego demostrar que
4

c(1 d), d(1 a)

84

Captulo 6

465 Problema Demostrar que si x, y, son nmeros reales positivos, entonces

x2 + y2 + 1 > x

y2 + 1 + y

470 Problema Sean x, y, z nmeros reales estrictamente positivos. Demostrar que

x2 + 1.
x2 z 2
y2 x2
z 2 y2
+
+
0
y+z
z+x
x+y

466 Problema Demostrar que si a, b, c son las longitudes de los lados de cualquier trin-

gulo, entonces
y determinar cuando se cumple la igualdad.

(a + b) (b + c) (c + a) 8 (a + b c) (b + c a) (c + a b) .
467 Problema Sean x, y, z reales estrictamente positivos. Demostrar que

471 Problema Sean a, b, c nmeros reales estrictamente positivos tales que abc = 1.
Demustrese que

x2
y2
z2
x
y
z
+ 2+ 2 + + .
y
z
x
y2
z
x

a1+

468 Problema Sean a, b, c las longitudes de los lados de un tringulo. Demostrar que

1
b

b1+

1
c

c1+

1
a

1.

3 (ab + bc + ca) (a + b + c) < 4 (ab + bc + ca) .


472 Problema Sean a, b, c,

2

469 Problema Sean a, b, c las longitudes de los lados de un tringulo. Demostrar que

nmeros reales estrictamente positivos tales que

a2 + b2 + c2 > 2 a4 + b4 + c4 . Demostrar que a, b, c son longitudes de los lados de un tringulo.

a + b c + b + c a + c + a b a + b + c.

6.3. Desigualdades de las medias


473 Teorema (Desigualdad de las medias aritmtica y geomtrica) Sean a1 , . . . , ak nmeros reales positivos. Su media

geomtrica es a lo sumo su media aritmtica, esto es,

a + + ak
k a a 1
,
1
k
k
con igualdad a1 = = ak .
Se darn varias demostraciones de tan importante resultado, tanto en esta seccin como en secciones subsiguientes. Estas
ilustrarn varias tcnicas tiles.
Demostracin primera: La primera demostracin es un argumento por induccin un tanto truculento debido a
Cauchy. Se demostrar la desigualdad primero para potencias de 2 y luego se interpolar entre potencias de 2.
Los casos k = 2 y k = 4 se han demostrado en el teorema 444 y en el ejemplo 449. Presmase ahora que la
desigualdad es vlida para k = 2n1 > 2, esto es, presmase que para reales positivos x1 , x2 , . . . , x2n1 se tiene
(x1 x2 x2n1 )1/2

n1

x1 + x2 + + x2n1
.
2n1

(6.7)

Se demostrar ahora la desigualdad para 2k = 2n . Considrese


ahora reales
positivos y1 , y2 , . . . , y2n . Ntese que

n
n1
n1
n1
n1
n
hay 2 2
= 2 (2 1) = 2
enteros en el intervalo 2
+ 1 ; 2 . Se tiene,
(y1 y2 y2n )1/2

(y1 y2 y2n1 )1/2

n1

n1

y2n1 +1 y2n

1/2n1


+ y2n1 +1 y2n 1/2


2
y1 + y2 + + y2n1 y2n1 +1 + + y2n
+
2n1
2n1
2
y1 + + y2 n
,
2n
(y1 y2 y2n1 )1/2

n1

en donde la primera desigualdad sigue del caso k = 2 y la segunda de la hiptesis inductiva (6.7). Queda as
demostrado el teorema para potencias de 2.

Desigualdades de las medias

85

Presmase ahora que 2n1 < k < 2n y considrense los k reales positivos a1 , a2 , . . . , ak . El truco es de aumentar
esta coleccin hasta tener 2n nmeros y utilizar el resultado obtenido para potencias de 2. El relleno ser el
promedio de los nmeros a1 , a2 , . . . , ak . As pues, considrese los 2n nmeros reales
a1 , a2 , . . . , ak , ak+1 , . . . , a2n
con ak+1 = . . . = a2n =


a1 a2 ak

a1 + a2 + + ak
. Por lo ya demostrado para 2n se tiene
k

 a + a + + a 2n k 1/2n
1
2
k

a1 + a2 + + ak + (2n k)

a + a + + a 
1
2
k

2n

de donde

(a1 a2 ak )

1/2n

 a + a + + a 1k/2n
1
2
k

lo que implica
(a1 a2 ak )1/2
Resolviendo para

a + a + + a 
a1 + a2 + + ak
1
2
k
+ (2n k)
k
k

,
2n

 a + a + + a 1k/2n
1
2
k

a + a + + a 
1
2
k

a1 + a2 + + ak
da la desigualdad deseada.
k

Demostracin segunda: La segunda demostracin es tambin por induccin. Como en la demostracin anterior,
el caso k = 2 ya se ha establecido en el teorema 444. Pngase
Ak =

a1 + a2 + + ak
,
k

Gk = (a1 a2 ak )1/k .

Obsrvese que
ak+1 = (k + 1)Ak+1 kAk .
La hiptesis inductiva es Ak Gk y se quiere demostrar que Ak+1 Gk+1 . Pngase
A=

ak+1 + (k 1)Ak+1
,
k

G = ak+1 Ak1
k+1

1/k

Por la hiptesis inductiva, A G. Ahora bien,


A + Ak
=
2

(k + 1)Ak+1 kAk + (k 1)Ak+1


+ Ak
k
= Ak+1 .
2

As pues,
Ak+1

A + Ak
2

(AAk )1/2

(GGk )1/2 .

=
Se ha establecido que

k1
Ak+1 Gk+1
k+1 Ak+1

lo cual completa la induccin.

k1
Gk+1
k+1 Ak+1

1/2k

1/2k

= Ak+1 Gk+1 ,

86

Captulo 6
Demostracin tercera: La tercera demostracin depende de la nocin de continuidad. Se har una serie de
substituciones que conservarn la suma
a1 + a2 + + an ,
pero que incrementarn el producto
a1 a2 an .
Al final se ver que todas las ai sern iguales y que su media aritmtica A ser igual a su media geomtrica
a1 + a2 + + an
nA
>
= A, lo cual es imposible. De igual manera, las
G. Si todas las ai fuesen > A entonces
n
n
ai no pueden ser todas < A. Luego deben de haber dos subndices i, j, tales que ai < A < a j . Pngase ai = A,
aj = ai + a j A. Obsrvese que ai + a j = ai + aj , de donde reemplazar las as originales con las as primas no
altera la media aritmtica. Por otra parte,
ai aj = A (ai + a j A) = ai a j + (a j A) (A ai) > ai a j
ya que a j A > 0 y A ai > 0.
Ntese ahora que hay a lo sumo n as que reemplazar. El procedimiento terminar eventualmente logrando igualar
a todas las as a la media aritmtica e incrementando a la media geomtrica. Habr desigualdad estricta si al
menos dos de las as son desiguales.

474 Ejemplo Sea f (x) = (a + x)5 (a x)3 , x [a; a]. Hallar el valor mximo de f por medio de la desigualdad de la media.

Resolucin: Si x [a; a], entonces a + x 0 y a x 0, por lo tanto se puede utilizar la desigualdad de las
a+x
ax
medias con n = 8, a1 = a2 = = a5 =
y a6 = a7 = a8 =
. Se deduce que
5
3
 a + x 5  a x 3


a + x

+3

 a x  8

 a 8

de donde

55 33 a8
,
48
a+x ax
en donde se satisface la igualdad si y slo si
=
.
5
3
f (x)

475 Ejemplo Para todo entero n > 1 se tiene

1 3 5 (2n 1) < nn ,
ya que por la desigualdad de las medias,

1 3 5 (2n 1) <

1 + 3 + 5 + + (2n 1)
n

 2 n
n

= nn .

Obsrvese que como los factores son desiguales habr desigualdad estricta.

1 n
476 Ejemplo La sucesin xn = 1 +
, n = 1, 2, . . . es estrictamente creciente. De hecho, el conjunto de n + 1 nmeros

1
1
1
1, 1 + , 1 + , . . . , 1 + ,
n
n
n
tiene media aritmtica
1+

1
n+1

Desigualdades de las medias

87

y media geomtrica

1+

1
n

Por lo tanto,

n/(n+1)

1+
esto es

1
1
> 1+
n+1
n

1+

1
n+1

n+1

n/(n+1)

> 1+

1
n

o
xn+1 > xn ,
demostrando la asercin.
477 Ejemplo Hallar el volumen de la caja rectangular mayor con lados paralelos a los ejes que se pueda inscribir en el elipsoide
 x 2

 y 2

 z 2

= 1.

Aqu, a > 0, b > 0, c > 0.


Resolucin: Sean 2x, 2y, 2z las dimensiones de esta caja. Se quiere maximizar 8xyz. Poniendo n = 3, x1 =
x2 =

z2
y2
, x3 = 2 , en la desigualdad de las medias, se tiene
2
b
c

x2
,
a2

 x + x + x 3
x2 y2 z2
1
1
2
3
=
x
x
x

= .
1 2 3
a 2 b 2 c2
3
27

Por lo tanto, el volumen mximo es


8abc
8xyz .
3 3

478 Definicin Dados los reales a1 > 0, a2 > 0, . . . , an > 0, su media armnica est dada por

n
.
1
1
1
+ + +
a1 a2
an
Como colorario al teorema de las medias se obtiene
479 Corolario (Desigualdad de las medias armnica y geomtrica) Considrense los reales b1 > 0, b2 > 0, . . . , bn > 0. En-

tonces

n
(b1 b2 bn )1/n .
1
1
1
+ + +
b1 b2
bn

Demostracin: Esto se sigue en poniendo ak =

1 1
1

b1 b2
bn

1
en el teorema 473. Entonces,
bk
1/n

1
1
1
+ + +
b1 b2
bn

.
n

88

Captulo 6

Combinando el teorema 473 y el corolario 479, se deduce


480 Corolario (Desigualdad de las medias armnica, geomtrica y aritmtica) Sean b1 > 0, b2 > 0, . . . , bn > 0. Entonces

n
b1 + b2 + + bn
(b1 b2 bn )1/n
.
1
1
1
n
+ + +
b1 b2
bn
481 Ejemplo Sea ak > 0 y pngase s = a1 + a2 + + an. Demostrar que
n
X
k=1

s
n2

s ak
n1

y que
n
X
k=1

ak
n

.
s ak
n1

s
. Entonces
s ak

Resolucin: Pngase bk =

n
X
1
k=1

bk

n
X
s ak

k=1

y por el corolario 480,

Pn

n1

= n1

s
s ak
,
n

k=1

demostrando la primera desigualdad.

Como

s
ak
1 =
, se tiene
s ak
s ak
n
X
k=1

ak
s ak

n
X
k=1

n
X
k=1
2

=
demostrando la segunda desigualdad.

Tarea
482 Problema Demostrar que si n > 1 es un entero,

n! <

n + 1 n
2

s
1
s ak

s
n
s ak

n
n
n1
n
,
n1

Desigualdad de Cauchy-Schwarz-Bunyakovsky

89

6.4. Desigualdad de Cauchy-Schwarz-Bunyakovsky


483 Teorema (Identidad de Lagrange) Sean ak , bk nmeros reales. Entonces
n
X

!2

n
X

ak bk

k=1

a2k

k=1

n
X

b2k

k=1

(ak b j a j bk )2 .

1k< jn

Demostracin: Como para j = k se tiene ak b j a j bk = 0, se puede relajar la estrictura de la desigualdad en la


ltima suma. Ahora bien,
X

(ak b j a j bk )2

1k< jn

(a2k b2j 2ak bk a j b j + a2j b2k )

1k jn

a2k b2j 2

ak bk a j b j +

1k jn
n X
n
X

1k jn
!2
n
X

k=1 j=1

k=1

a2k b2j

ak bk

a2j b2k

1k jn

demostrando el teorema.
484 Teorema (Desigualdad de Cauchy-Bunyakovsky-Schwarz) Sean xk , yk nmeros reales, 1 k n. Entonces


!1/2
!1/2
n
n
n
X

X
X


2
2
xk yk
xk
yk
,



k=1

k=1

k=1

con igualdad si y slo si se cumple la proporcin


(a1 , a2 , . . . , an ) = t(b1 , b2 , . . . , bn )
para alguna constante real t.
Demostracin primera: La desigualdad sigue de inmediato de la identidad de Lagrange,
n
X

!2

xk yk

k=1

(teorema 483), ya que

n
X

x2k

k=1

n
X

y2k

k=1

(xk y j x j yk )2

1k< jn

(xk y j x j yk )2 0.

1k< jn

Demostracin segunda: Pngase a =

n
X

x2k , b =

k=1

at 2 + bt + c = t 2

n
X

n
X

xk yk y c =

k=1

x2k 2t

k=1

n
X

n
X

y2k . Considrese el polinomio cuadrtico

k=1

xk yk +

k=1

n
X
k=1

y2k =

n
X
k=1

(txk yk )2 0,

en donde se tiene desigualdad porque se suman cuadrados de nmeros reales. Luego este polinomio cuadrtico es
positivo para todo real t de donde se desprende que tiene races complejas y su discriminante b2 4ac es negativo.
As,
!
!
!
4

n
X
k=1

dando la desigualdad deseada.

xk yk

n
X
k=1

x2k

n
X
k=1

y2k ,

90

Captulo 6
Demostracin tercera: Gracias al corolario 454,
x21 + x22 + + x2n

x21 y21 x22 y22


x2 y2
+ 2 + + n2n
2
yn
y1
y2
(x1 y1 + x2 y2 + + xn yn )2
,
y21 + y22 + + y2n

de donde se obtiene la desigualdad deseada.


Demostracin cuarta:

La desigualdad es obvia si

n
X

x2k

k=1

n
X

y2k

= 0, as que presmase al contrario.

k=1

Utilizando la desigualdad del tringulo y la desigualdad de la media,




n

X

xk yk

Pn
P



n
2
2
k=1
x
y
k=1 k
k=1 k

n
X
k=1
n
X
k=1

|xk |

|yk |
P

1
2

n
2
k=1 xk

x2
Pn k 2
k=1 xk

n
2
k=1 yk

y2
Pn k 2
k=1 yk

1,

de donde se deduce la desigualdad requerida.


485 Ejemplo Demostrar que si x1 , x2 , . . . , xn , son nmeros reales estrictamente positivos, entonces

(x1 + x2 + . . . + xn )

1
1
1
+ + ... +
x1 x2
xn

n2 .

Resolucin: Por CBS,

1
1
1
(x1 + x2 + . . . + xn )
+ + ...+
x1 x2
xn

n
X

1
xi
xi

i=1

!2

n2 .

486 Ejemplo (USAMO 1978) Sean a, b, c, d, e reales satisfaciendo

a + b + c + d + e = 8,

a2 + b2 + c2 + d 2 + e2 = 16.

Maximizar el valor de e.
Resolucin: Por CBS,

(a + b + c + d)2 (1 + 1 + 1 + 1) a2 + b2 + c2 + d 2
De aqu,

(8 e)2 4 16 e2 e (5e 16) 0 0 e


El valor mximo e =

6
16
es alcanzado cuando a = b = c = d = .
5
5

4 a 2 + b 2 + c2 + d 2 .
16
.
5

Desigualdad del reordenamiento

91

487 Teorema (Desigualdad de Minkowski) Sean xk , yk nmeros reales. Entonces


!1/2
!1/2
!1/2
n
n
n
X
X
X

(xk + yk )2

x2k

k=1

y2k

k=1

k=1

Demostracin: Se tiene,
n
X

(xk + yk )2

n
X

k=1

x2k + 2

k=1
n
X

k=1

x2k + 2

n
X

xk yk +

k=1
n
X

n
X

k=1
!1/2

x2k

k=1

n
X

!1/2

x2k

n
X

k=1

y2k
n
X

!1/2

y2k

k=1
!1/2 2
,
y2k

n
X

y2k

k=1

k=1

donde la desigualdad deseada se sigue por CBS.

Tarea
488 Problema Si ak , bk , ck , k = 1, . . . ,n, son reales positivos, demustrese que

c2 + d 2

4 n n n 2
X
X
X

X
n

ak bk ck

a4k

k=1

b4k

k=1

c2k

k=1

491 Problema Sean a, b, c, d nmeros reales estrictamente positivos tales que a2 + b2 =

3

. Demostrar que
c3
d3
+
1.
a
b

k=1

492 Problema Sean x > 1, y > 1, z > 1 tales que


489 Problema Sea xi 0 para 1 i n. Demostrar que

x1 + x2 + + xn

x21 + x22 + + x2n


n

1/2

x+y+z

,
yz + zx. Demostrar que

xyz 3 (x + y + z).

490 Problema Sea n 3 un entero y considrese reales estrictamente positivos

a1 , a2 , . . . ,an , tales que

a21 + a22 + . . . + an

x1+
y 1 + z 1.

493 Problema Sean x, y, z nmeros reales estrictamente positivos tales que xyz xy +

con igualdad si y slo si x1 = x2 = = xn .


2 2

1 1 1
+ + = 2. Demostrar que
x
y
z


4

494 Problema Sean a1 , a2 , . . . ,an nmeros reales estrictamente positivos. Pngase


n
X

S1 =

> (n 1) a41 + a42 + . . . + an .

n
X

ai y S2 =
i=1

Demostrar que para todo tro i, j, k, dos a dos distintos, los nmeros ai , a j , ak son las
longitudes de los lados de un tringulo.

a2i .

i=1

Demustrese que

n
X

S2 a2k
S1 .
S1 ak

k=1

6.5. Desigualdad del reordenamiento


495 Definicin Dado un conjunto de nmeros reales {x1 , x2 , . . . , xn }, dentese por

x1 x2 xn
el reordenamiento decreciente de las xi y por
el reordenamiento creciente de las xi .

x1 x2 xn

496 Definicin Dadas dos sucesiones de nmeros reales {x1 , x2 , . . . , xn } y {y1 , y2 , . . . , yn } de la misma longitud n, se dice que

son similarmente sorteadas si ambas son crecientes o si ambas son decrecientes. Se dice que son diferentemente sorteadas si
una es creciente y la otra es decreciente.

92

Captulo 6

497 Ejemplo Las sucesiones 1 2 n and 12 22 n2 son similarmente sorteadas, mientras que las sucesiones

1
1
1
2 2 y 13 23 n3 son diferentemente sorteadas.
12
2
n

498 Teorema (Desigualdad del reordenamiento) Dados nmeros reales {a1, a2 , . . . , an } y {b1 , b2 , . . . , bn } se tiene
X
X
X

As la suma

1kn

ak b k

1kn

ak bk

ak b k .

1kn

ak bk se minimiza cuando las sucesiones son diferentemente sorteadas y se maximiza cuando las sucesiones

1kn

son similarmente sorteadas.

Obsrvese que

X
1kn

ak b k =

ak b k .

1kn

Demostracin: Sea { (1), (2), . . . , (n)} un reordenamiento de {1, 2, . . . , n}. Si hay dos subndices i, j, tales
que las sucesiones halan en direcciones opuestas, por ejemplo ai > a j y b (i) < b ( j) , entonces considrese las
sumas
S

a1 b (1) + a2b (2) + + ai b (i) + + a j b ( j) + + anb (n)

a1 b (1) + a2b (2) + + ai b ( j) + + a j b (i) + + anb (n)

Entonces
S S = (ai a j )(b ( j) b (i) ) > 0.
Esta ltima desigualdad demuestra que mientras ms cerca estn las as y las bs de halar en la misma direccin,
mayor ser la suma. Esto demuestra el resultado.
499 Ejemplo Dar una demostracin de la desigualdad de las medias utilizando la desigualdad del reordenamiento. Esto es, si

a1 , . . . , an son reales positivos, demustrese que

a1 + + an
n
a1 an
,
n
con igualdad si y slo si a1 = = an .
Resolucin: Si alguna de las ak es cero, no hay nada que demostrar. Presmase pues que todas son estrictamente positivas. Pngase
a1
a1 a2
a1 a2 an
x1 =
, x2 =
, . . . , xn =
= 1,
1/n
2/n
(a1 a2 an )
(a1 a2 an )
(a1 a2 an )n/n
y
1
1
1
= 1.
y1 = , y2 = , . . . , yn =
x1
x2
xn
Obsrvese que para 2 k n,
xk yk1 =

a1 a2 ak
(a1 a2 an )(k1)/n
ak

=
.
a1 a2 ak1
(a1 a2 an )k/n
(a1 a2 an )1/n

Las xk y las yk son diferentemente sorteadas, y en virtud de la desigualdad de reordenamiento,


1 + 1 + + 1 =

x1 y1 + x2 y2 + + xn yn

x1 yn + x2 y1 + + xn yn1
=

a1
a2
an
+
+ +
,
(a1 a2 an )1/n (a1 a2 an )1/n
(a1 a2 an )1/n

Desigualdad del reordenamiento

93

o sea,
n

a1 + a2 + + an
,
(a1 a2 an )1/n

de donde se obtiene el resultado.


500 Ejemplo Minimizar f : x 7

i h
sin3 x cos3 x
+
sobre 0 ;
.
cosx
sin x
2

h
. Las sucesiones sin3 x, cos3 x y
2
pues, por la desigualdad del reordenamiento,

Resolucin: Sea x 0 ;

f (x) sin3 x

1
1
,
cos x sin x

son similarmente sorteadas. As

1
1
+ cos3 x
= sin2 x + cos2 x = 1.
sin x
cos x

Por otra parte,


f

 

= 1.

Luego, el mnimo deseado es 1.


501 Ejemplo Demostrar que (a, b, c) R3 ,

a2 + b2 + c2 ab + bc + ca.
Resolucin: Sin prdida de generalidad, supngase que a b c. Entonces a b c es similarmente sorteada
que ella misma, y as por la desigualdad del reordenamiento,
a2 + b2 + c2 = aa + bb + cc ab + bc + ca.
La desigualdad deseada tambin se sigue de inmediato de la identidad

b+c
a + b + c ab bc ca = a
2
2

3
+ (b c)2 .
4

Se puede tambin utilizar la desigualdad de las medias tres veces


a2 + b2 2ab;

b2 + c2 2bc;

c2 + a2 2ca,

y sumar.
502 Ejemplo Demostrar que si (a, b, c) R3 , con a 0, b 0, c 0, entonces se cumplen las siguientes desigualdades:

a3 + b3 + c3 max(a2 b + b2c + c2a, a2 c + b2a + c2b),


a3 + b3 + c3 3abc,

a 3 + b 3 + c3

1 2
a (b + c) + b2(c + a) + c2(a + b) .
2

Resolucin: Sin prdida de generalidad, supngase que a b c. Entonces a b c es similarmente sorteado


con a2 b2 c2 y en virtud de la desigualdad del reordenamiento,
a3 + b3 + c3 = aa2 + bb2 + cc2 a2 b + b2c + c2 a,
y
a3 + b3 + c3 = aa2 + bb2 + cc2 a2 c + b2a + c2b.

94

Captulo 6
Sumando,
a3 + b3 + c3 = aa2 + bb2 + cc2
Otra vez, si a b c entonces

1 2
a (b + c) + b2(c + a) + c2(a + b) .
2

ab ac bc,

y as
a3 + b3 + c3 = a2 b + b2c + c2a = (ab)a + (bc)b + (ac)c (ab)c + (bc)a + (ac)b = 3abc.
Esta ltima desigualdad tambin resulta de la desigualdad de las medias ya que
(a3 b3 c3 )1/3

a 3 + b 3 + c3
,
3

u otra vez, de la identidad


a3 + b3 + c3 3abc = (a + b + c)(a2 + b2 + c2 ab bc ca),
y la identidad del ejemplo 501.
503 Ejemplo (Desigualdad de Chebyshev) Dados reales {a1 , a2 , . . . , an } y {b1 , b2 , . . . , bn } prove that
!
!
1 X
1 X
1 X
1 X

1kn

ak b k

ak

1kn

bk

1kn

ak b k .

1kn

Resolucin: Se aplica la desigualdad del reordenamiento n veces:


a1 b 1 + a2b 2 + + an b n

a1 b1 + a2 b2 + + an bn

a1 b 1 + a2b 2 + + anb n

a1 b 1 + a2b 2 + + an b n

a1 b2 + a2 b3 + + an b1

a1 b 1 + a2b 2 + + anb n

a1 b 1 + a2b 2 + + an b n

a1 b3 + a2 b4 + + an b2

a1 b 1 + a2b 2 + + anb n

a1 b 1 + a2b 2 + + an b n

a1 bn + a2 b1 + + an bn1

a1 b 1 + a2b 2 + + anb n

..
.

Sumando se obtiene la desigualdad deseada.

504 Ejemplo (Desigualdad de Nesbitt) Sean a, b, c reales estrictamente positivos. Demostrar que

a
b
c
3
+
+
.
b+c c+a a+b 2
Resolucin: Presmase que a b c. Pngase s = a + b + c. Entonces
a b c = s a s b s c =
y por tanto las sucesiones a, b, c y
reordenamiento dos veces,

1
1
1

sa sb sc

1
1
1
,
,
estn similarmente sorteadas. Usando la desigualdad del
sa sb sc

a
b
c
a
b
c
+
+

+
+
;
sa sb sc sc sa sb

a
b
c
a
b
c
+
+

+
+
.
sa sb sc sb sc sa

Desigualdad del reordenamiento

95

Sumando estas dos desigualdades,

a
b
c
+
+
sa sb sc

de donde
2

b+c c+a c+a


+
+
,
sa sb sc

a
b
c
+
+
3,
b+c c+a a+b

estableciendo el resultado.

Tarea
505 Problema (IMO, 1978) Sea ak , 1 k n una sucesin de enteros distintos estrictamente positivos. Demostrar que
n
X
a

k
k2

k=1

n
X
1

509 Problema Sean a, b, c reales estrictamente positivos y sea et n > 0 un entero. De-

mostrar que
bn
cn
an1 + bn1 + cn1
an
+
+

.
b+c
c+a a+b
2

k=1

510 Problema Sean x1 , x2 , . . . ,xn reales estrictamente positivos. Demostrar que

506 Problema Sean a, b, c, d reales positivos, satisfaciendo ab + bc + cd + da = 1. De-

a3
b3
c3
d3
1
+
+
+
.
b+c+d
c+d +a d +a+b a+b+c
3
507 Problema (IMO 1975) Sean x1 x2 . . . xn , y y1 y2 . . . yn nmeros reales.

Considrese una permutacin (z1 , z2 , . . . ,zn ) de (y1 , y2 , . . . ,yn ) . Demostrar que


n
X

i=1

n
X

(xi yi )2

(xi zi )2 .

i=1

508 Problema Sean a1 , a2 , . . . ,an reales estrictamente positivos. Demostrar que

a2n1
a21
a2
a2
+ 2 + ... +
+ n a1 + a2 + . . . + an .
a2
a3
an
a1

x11 x22 . . . xxnn (x1 x2 . . . xn )

mostrar que

x1 +x2 +...+xn
n

511 Problema (IMO 1998) Sean x, y, z reales estrictamente positivos tales que xyz = 1.

Demostrar que
y3
z3
3
x3
+
+
.
4
(1 + y) (1 + z) (1 + z) (1 + x) (1 + x) (1 + y)

Captulo

Geometra plana
Se intentar aqu recoger una cantidad de resultados y mtodos tiles para resolver problemas de tipo concurso en Geometra. Una gran parte de estos resultados son clsicos y se estudiarn con mtodos clsicos: esto es, sin introducir las nociones de
distancia o distancia dirigida. A algunos resultados se darn mltiples demostraciones, para evidenciar la utilidad de diversos
puntos de vista. Se utilizarn tanto mtodos sintticos, como vectoriales, analticos y trigonomtricos.
El deseo no es de dar una presentacin rigurosa y axiomtica de la Geometra, sino ms bien, presentar una serie de
resultados tiles para la resolucin de problemas tipo olimpada. Por tanto, la progresin de tpicos no ser necesariamente
lineal.

7.1. ngulos
Se supondrn por conocidas las nociones elementales de punto, recta, rayo, plano y segmento de recta. Los puntos generalmente se denotarn por maysculas, e.g. P, Q, etc. Las rectas generalmente se denotarn por maysculas con flechas supra,

e.g. L , M . La recta que contiene los puntos A y B se denotar por AB. El segmento de recta con punto inicial A y punto final
B se denotar por [AB] y su longitud o distancia positiva entre A y B por AB, notando que AB = BA. El rayo con punto inicial A
y que pasa por B se denotar por [AB[.
512 Definicin Se dir que dos figuras son congruentes, si coinciden cuando una es sobreimpuesta a la otra.
513 Presuncin Dos puntos distintos determinan una recta nica.

B
b

Figura 7.1: Un punto.

Figura 7.2: Una recta.

Figura 7.3: Un rayo.

b
b

Figura 7.4: ngulo.

Figura 7.5: ngulos convexo (en rojo) y cncavo


(en verde).

514 Presuncin Dado un punto y una recta, o bien el punto yace sobre la recta, o bien el punto no yace sobre la recta.

96

ngulos

97

515 Presuncin Dos puntos yacen sobre una recta nica.


516 Presuncin Dos rectas sobre el mismo plano o bien se intersecan en un punto nico o bien no se intersecan, en cuyo caso


se denominan paralelas. Si la recta L es paralela a la recta L se escribir L k L .

517 Presuncin El paralelismo es una relacin de equivalencia, esto es,

es reflexiva, ya que toda recta es paralela a s misma,


es simtrica, ya que si una recta es paralela a otra, la otra lo es a la primera y
es transitiva, ya que si una recta es paralela a una segunda recta, y esta a otra tercera recta, la primera lo es a la tercera.
518 Presuncin Dado un punto P y una recta L que no contenga a P, existe una recta nica L que contiene a P y que satisface

L k L .

519 Definicin (Puntos colineales) Tres puntos o ms puntos se dicen colineales si yacen en la misma recta.
520 Presuncin (Relacin de Chasles) Si A, B, C son tres puntos colineales y si B est entre A y C entonces

AC = AB + BC.
521 Definicin (Rectas concurrentes) Tres o ms rectas se dicen concurrentes si pasan por un punto en comn.
522 Definicin (ngulo) Sean [OA[ y [OB[ dos rayos de origen comn O. La regin barrida por el rayo [OA[ cuando este gira

sobre el vrtice O hasta llegar al rayo [OB[ se denomina ngulo dirigido y se denota por ([OA[
, [OB[). Tambin se utiliza la

b
notacin AOB o O.

Como el rayo inicial puede tanto girar en sentido levgiro como en sentido dextrgiro, hay ambigedad al
nombrar ngulos. As pues la misma notacin se puede utilizar tanto para nombrar el ngulo convexo o cncavo.
La mayora de las veces la notacin se referir al ngulo producido cuando el rayo inicial viaja en sentido levgiro,
pero en caso contrario se harn notar las excepciones con el smbolo . Vase la figura 7.5.
Se utilizar tanto el grado como el radin para medir ngulos. Recuerdse que se verifica la identidad
A
r
= ,
180

(7.1)

en donde A es la medida del ngulo en grados y r es la medida del ngulo en radianes. Se utilizar la misma notacin para
denotar tanto a un ngulo como a su medida.

523 Definicin (ngulo adyacente) Si el rayo [OB[ est entre los rayos [OA[ y [OB[, se dice que los ngulos ([OA[
, [OB[) y

([OB[ , [OC[) son adyacentes y se cumple

([OA[
, [OC[) = ([OA[
, [OB[) + ([OB[
, [OC[).

524 Definicin (Revolucin) Una revolucin es el ngulo obtenido al rotar un rayo hasta que yaga otra vez sobre s mismo.

Mide 360 o 2 radianes.

525 Definicin (ngulo llano) Un ngulo llano es el ngulo formado por un rayo y el rayo con el mismo punto inicial pero
viajando en direccin opuesta. Mide 180 o radianes.

98

Captulo 7

526 Definicin (ngulo recto) Un ngulo recto es la mitad de un ngulo llano. Mide 90 o

radianes.
2

527 Definicin (ngulo agudo) Un ngulo agudo es aqul que mide menos que un ngulo recto.
528 Definicin (ngulo obtuso) Un ngulo obtuso es aqul que mide ms que un ngulo recto pero menos que un ngulo

llano.
529 Definicin (ngulo reflejo) Un ngulo reflejo es aqul que mide ms que un ngulo llano.
530 Definicin (ngulo complementario) Dos ngulos se dicen complementarios si la suma de sus medidas es un ngulo

recto.
531 Definicin (ngulo suplementario) Dos ngulos se dicen suplementarios si la suma de sus medidas es un ngulo llano.

bM

Figura 7.6: ngulos adyacentes.

bB

Figura 7.7: Revolucin.

Figura 7.8: ngulo llano.

Figura 7.9: ngulo recto.

O
A

Figura 7.10: ngulos


opuestos por el vrtice.

Es evidente que si un ngulo recto se descompone en dos ngulos adyacentes, el uno es suplementario al
otro. Recprocamente, si dos ngulos adyacentes son suplementarios, entonces forman un ngulo recto.
532 Definicin (ngulos opuestos por el vrtice) Cada par de ngulos opuestos cuando dos rectas se intersecan se llaman

ngulos opuestos por el vrtice.


533 Teorema Dos ngulos opuestos por el vrtice son congruentes.

Demostracin: Vase la figura 7.1. Por formar ngulos llanos adyacentes,


OA = = B
OA + A
OB = AOB
OB .

+B
= A
AOB

Si dos rectas se cortan y uno de los ngulos en el corte es recto, entonces todos los dems sern ngulos rectos, en virtud del
teorema 7.1. De aqu la siguiente definicin.
534 Definicin (Rectas perpendiculares) Dos rectas L y L se dicen perpendiculares, denotado por L L , si el ngulo entre

ellas es recto.

535 Definicin (Transversal) Una transversal es una recta que cruza a otras dos o ms rectas. Estas ltimas rectas pueden ser

o no paralelas.

ngulos

99

536 Definicin (ngulos correspondientes) ngulos homlogos en rectas cortadas por una transversal son llamados ngu-

los correspondientes. Vase a figura 7.11.


537 Definicin (ngulos alternos internos) ngulos entre rectas cortadas por una transversal y en lados opuestos de la

transversal son llamados ngulos alternos internos. Vase a figura 7.12.


538 Definicin (ngulos alternos externos) ngulos fuera de rectas cortadas por una transversal y en lados opuestos de la

transversal son llamados ngulos alternos externos. Vase a figura 7.13.

D
b

O
b

O
b

Figura 7.11: ngulos correspondientes.

Figura 7.12: ngulos alternos internos.

D
b

O
b

Figura 7.13: ngulos alternos externos.

Se presumir que el lector conoce los siguientes resultados.


539 Presuncin Si una transversal corta a dos paralelas, lo ngulos correspondientes son congruentes. Recprocamente, si una

transversal cortare dos rectas y si los ngulos correspondientes fueren congruentes, entonces las rectas cortadas son paralelas.
540 Presuncin Si una transversal corta a dos paralelas, lo ngulos alternos externos son congruentes. Recprocamente, si una
transversal cortare dos rectas y si los ngulos alternos externos fueren congruentes, entonces las rectas cortadas son paralelas.
541 Presuncin Si una transversal corta a dos paralelas, lo ngulos alternos internos son congruentes. Recprocamente, si una
transversal cortare dos rectas y si los ngulos alternos internos fueren congruentes, entonces las rectas cortadas son paralelas.
542 Definicin (Tringulo) Un tringulo es una figura en el plano, obtenida al unir, por segmentos de recta, tres puntos no

alineados en el plano. Un tringulo se dice issceles si dos de sus lados tienen la misma longitud, y equiltero si su tres lados
tienen la misma longitud
543 Definicin Un tringulo rectngulo es aqul que posee un ngulo recto. El lado opuesto al ngulo recto se llama hipotenusa, y los otros dos lados catetos.

Se presumir conocido lo siguiente.


544 Presuncin Si un tringulo es issceles, los ngulos opuestos a los lados congruentes son congruentes. Recprocamente,
si dos de los ngulos de un tringulo son congruentes, el tringulo es issceles. Un tringulo equiltero es equingulo, esto es,

cada uno de sus tres ngulos mide radianes.


3
ABC
y BCA,
se denotarn, respectivamente, por A,
b B
b o a veces, por
b y C,
Dado un tringulo ABC, sus ngulos interiores CAB,
letras griegas, , , .

100

Captulo 7

545 Teorema La suma de los ngulos internos de un tringulo es un ngulo llano (180 o radianes).

Demostracin: Constryase por A una recta paralela a BC. Para simplificar la nomenclatura, presmase que el
punto X est sobre esta recta, a la izquierda de A, y el punto Y est sobre esta recta, a la derecha de A, como en la
figura 7.15. Por ser ngulos alternos externos a rectas paralelas,
= B,
b
XAB

= C.
b
YAC

Por ser ngulos adyacentes en una lnea recta


+A
b + YAC
=B
b + C,
b
b+ A
= XAB

como se quera demostrar.


A

b
b

Figura 7.14: Tringulo ABC rectngulo en B. [AC] es la hipotenusa. [BC] y [AB] son los catetos.

546 Definicin (ngulos exteriores de un tringulo) Un ngulo exterior o externo de un tringulo es el ngulo suplemen-

tario formado al extender un lado del tringulo. Vase la figura 7.16.


La siguiente asercin es obvia.
547 Teorema La medida de un ngulo exterior de un tringulo es la suma de las medidas de los dos ngulos internos opuestos

del tringulo.
X

Figura 7.16: Un ngulo externo.

548 Ejemplo Hallar la suma de ngulos de los vrtices


b+ B
b+ D
b+ C
b+E
b+F
b
A

en la figura 7.17.

H
b

X
D

Figura 7.15: Teorema 545.

I
b

Figura 7.17: Ejemplo 548.

ngulos

101

Resolucin: Adanse los vrtices G, H, e I, como se muestra en la figura. Por ser ngulos opuestos por el
vrtice, G, H, e I son tambin los ngulos internos del tringulo en el centro, luego pues se tiene
b+ B
b = ,
b+ G
A

y as
de donde

b + Ib = ,
Cb + D

= ,
Eb + Fb + H

b+H
+ Ib= ,
G

b+ B
b+ D
b+H
b+ C
b+E
b+F
b+G
+ Ib = 3 ,
A
b+ B
b+ D
b+ C
b+E
b+F
b = 3 = 2 .
A
b

D
b

b 2 c3
B

c1
b

a2

a1

c2
b1

Figura 7.18: Ejemplo 549.

549 Ejemplo Se construye una sucesin de tringulos issceles, comenzando con AB = BC, luego BC = CD, etc., tal como en
= x > 0, demostrar que tan slo se puede construir
la figura 7.18. Si BAC

T U
2x
tringulos.

Resolucin: Sea n el nmero de tales tringulos que puede ser construido. Considrese la figura . Sea a1 = x.
Como los tringulos han de ser issceles, b1 = a1 = x. Esto conlleva a c1 = a1 b1 y as
a2

c1 = 2x,

b2

a2 = 2x,

c2

a2 b2 = 4x,

a3

b1 c2 = 3x,

b3

3x,

..
.
an

nx

bn

nx.

102

Captulo 7
En otras palabras, el ensimo tringulo tiene dos ngulos iguales a nx. Se puede construir tringulos en tanto
2nx < , de donde

n = T U.
2x

550 Definicin (Cuadriltero) Un cuadriltero es una figura en el plano obtenida al unir cuatro puntos, no tres de ellos alineados. El cuadriltero es simple si sus lados no se cruzan. Es convexo si para cada dos puntos dentro del cuadriltero, el segmento
de recta unindolos est dentro del cuadriltero.

De ahora en adelante, la palabra cuadriltero tan slo denotar la nocin de cuadriltero simple, a menos
que se indique lo contrario.

b
b

b
b

B
D

Figura 7.19: Cuadriltero simple y convexo.

b
b

E
H

Figura 7.20: Cuadriltero no simple.

I
b

Figura 7.21: Cuadriltero no convexo.

551 Teorema La suma de los ngulos internos de un cuadriltero simple es dos ngulos llanos (360 o 2 radianes).

Demostracin: De la figura 7.19, vemos que si el cuadriltero es convexo, entonces se puede elegir un vrtice
y conectarlo con otro vrtice no consecutivo, creando una diagonal y dos tringulos. Si el cuadriltero es simple,
pero no convexo, como en la figura ??, entonces deber tener un ngulo reflejo. Desde el vrtice de este ngulo
reflejo se traza una diagonal, creando as dos tringulos. Luego, en cada caso, la suma de los ngulos es la de dos
tringulos, esto es, 2 o 360 .
552 Definicin (Polgono) Sean P1 , P2 , . . . , Pn n puntos distintos en el plano, ninguno tro de entre ellos en lnea recta. La
figura de n lados P1 P2 . . . Pn obtenida al unir Pk con Pk+1 , k < n y Pn con P1 , se denomina polgono. El polgono es simple si sus
lados no se cruzan. Es convexo si para cada dos puntos dentro del polgono, el segmento de recta unindolos est dentro del
polgono.

De ahora en adelante, la palabra polgono tan slo denotar la nocin de polgono simple, a menos que se
indique lo contrario.
553 Teorema La suma de los ngulos internos de un polgono de n lados es n 2 ngulos llanos ((n 2)180 o (n 2)

radianes).
Demostracin: Por induccin. Para n = 3 esto es el teorema 545. Presmase primero que el polgono es convexo.
Eljase cualquier vrtice y nase a este vrtice con los otros n 2 vrtices que no inciden en l. De esta manera se
han formado n 2 tringulos, y as, la suma de los ngulos interiores del polgono es la suma de los ngulos de
estos n 2 tringulos, es decir, (n 2) .
Presmase ahora que el polgono es simple, pero no convexo. Entonces al menos uno de sus vrtices tiene un
ngulo reflejo, ya que el polgono no es convexo. Un rayo emanando desde este vrtice deber chocar con otro
vrtice creando una diagonal interior, porque de otro modo el polgono tendra rea infinita. Esta diagonal divide

ngulos

103

al polgono en dos sub-polgonos. O bien ambos sub-polgonos son convexos, en cuyo caso ya se termina la
demostracin por induccin, o bien, al menos uno de ellos no lo es. En el ltimo caso, se divide a este subpolgono en dos regiones, etc. Este procedimiento deber terminar eventualmente en formando un tringulo, ya
que el nmero de vrtices es finito.
Segunda demostracin: Sean k , 1 k n las medidas de los ngulos internos del polgono. Al viajar en
sentido dextrgiro por el permetro del polgono, comenzando desde un punto que no es un vrtice, se hace un
giro de k una vez pasado el k-simo vrtice. Cuando se llega al punto original, se ha dado una revolucin
completa. As
n
X

n
X

k=1

k=1

( k ) = 2 =

k = (n 2).

554 Definicin (ngulos externos de un polgono) Si se extiende un un lado de un polgono simple, el ngulo suplementa-

rio al ngulo del vrtice es el ngulo externo del polgono.


La siguiente asercin es evidente, pues al moverse a travs de los ngulos exteriores de un polgono se ha dado una revolucin.
555 Teorema La suma de los ngulos externos de un polgono de n lados es dos ngulos llanos (360 o 2 radianes).
556 Definicin (Polgono regular) Un polgono regular es aqul cuyos lados son congruentes y cuyos ngulos son congruen-

tes.
La siguiente asercin es entonces obvia.
557 Teorema La medida de un ngulo interior de un polgono regular es

(n 2)180 (n 2)
o
radianes.
n
n

558 Ejemplo En la figura adjunta (figura 7.22), AB k EF. Hllese la suma de ngulos
+ BCD
+ CDE
+ DEF.

ABC

Resolucin: Trcese una recta perpendicular a ambas paralelas, como en la figura 7.23. Como la suma de los
+ EFA
= , se tiene
ngulos interiores de un hexgono es 4 y como FAB
+ BCD
+ CDE
+ DEF
= 4 = 3 .
ABC

b
b

Figura 7.22: Ejemplo 558.

Figura 7.23: Ejemplo 558.

104

Captulo 7

559 Definicin (Arco) Dos puntos A y B en la circunferencia de un crculo dividen al crculo en dos partes, llamadas arcos,

denotado por AB.

Hay ambigedad al nombrar arcos. Normalmente se referir al arco nombrado en sentido levgiro, hacindose notar salvedades
con el smbolo .
560 Definicin (Cuerda) Dados dos puntos en la circunferencia de un crculo, el segmento de recta que los une es llamado

cuerda. Una cuerda que pasa por el centro del crculo se llama dimetro. Un segmento desde el centro del crculo hasta la
circunferencia, esto es, la mitad de un dimetro se llama radio.
561 Definicin (ngulo central) Un ngulo cuyo vrtice es el centro del crculo y cuyos lados son lados del crculo, es

llamado ngulo central.


562 Definicin (ngulo perifrico) Un ngulo cuyo vrtice est en la circunferencia del crculo y cuyos lados son cuerdas es

llamado ngulo perifrico o inscrito en el crculo.


563 Definicin (Tangente a un crculo) Una tangente es una recta que pasa por slo un punto de la circunferencia.
564 Definicin (Secante a un crculo) Una secante es una recta que pasa por dos puntos de una circunferencia.

b
b


Figura 7.24: Arco AB.

Figura 7.25: Cuerda [AB].

d
Figura 7.26: ngulo central B
OA.

b
b

Figura 7.27: ngulo perifrico


d
B
CA.

Se utilizar el siguiente resultado, de fcil demostracin.


565 Presuncin Una recta perpendicular en su extremo a un radio de un crculo le es tangente al crculo. Recprocamente,

una tangente a un crculo es perpendicular al radio trazado desde el punto de contacto.


Se desarrollarn ahora una serie de resultados tiles en la caza de ngulos.
566 Teorema La medida de un ngulo perifrico es la mitad de la medida del ngulo central que subtiende el mismo arco.

Demostracin: Se dividir la demostracin en tres casos: (I) cuando uno de los lados es un dimetro, (II) cuando
el centro del crculo est en el interior del ngulo, (III) cuando el centro del crculo est en el exterior del ngulo.
Vanse las figuras 7.28, 7.29 y 7.30.
= OBA.
Por ser ngulo
En el primer caso, el OAB es issceles en A, ya que OA y OB son radios. As pues BAC
exterior al OAB,
= BAC
+ OBA
= CAB
=
COB

COB
.
2

ngulos

105

En el segundo caso, utilizando el primer caso,


= CAD
+ DAB
=
CAB

COD DOB COB


+
=
.
2
2
2

Para el tercer caso se utiliza el primer y el segundo caso:


= DAB
DAC
=
CAB

DOB DOC COB

=
.
2
2
2

b
b

C
b

C
b

b
b

Figura 7.29: Teorema 566.


Caso II.

Figura 7.30: Teorema 566.


Caso III.

Figura
567.

7.31:

C
Figura 7.28: Teorema 566.
Caso I.

A
Corolario

Figura
568.

7.32:

Corolario

Los siguientes corolarios son ahora inmediatos.


567 Corolario Dos ngulos perifricos que subtienden el mismo arco son congruentes.
568 Corolario Un ngulo perifrico que subtiende a un semicrculo es un ngulo recto.

A
B

B
b

O
b

A
Figura 7.33: Teorema 569.

A
Figura 7.34: Corolario 570.

O
A

b
b

Figura 7.35: Teorema 571

b
b

O
b

Figura 7.36: Teorema 572

569 Teorema El ngulo entre una tangente a un crculo y una cuerda es la mitad del ngulo central subtendido por la cuerda.

Demostracin: En la figura 7.33, trcese el dimetro [BV ]. Por lo tanto [BV ] [PB]. As,
=
PBA

1
= BOA ,
ABV = AOV = BOA
2
2
2 2
2

como se tena que demostrar.


570 Corolario Tangentes trazadas desde un punto exterior hasta un crculo son congruentes.

106

Captulo 7
Demostracin: Se sigue de inmediato de la congruencias OPB
= OPA.

571 Teorema El ngulo entre dos cuerdas intersecndose dentro de una circunferencia es el promedio de los ngulos centrales
de los arcos subtendidos.

Demostracin: En la figura 7.35, sea X = AA BB . Se tiene que demostrar que


=
AXB

OB

AOB
A
+
.
2
2

es suplemento del ngulo exterior del tringulo A XB , extendiendo el lado A X hasta A. As


Ahora AXB

=
AXB

B + XB
A

XA

B + BB
A

AA

BOA

AOB
+
2
2

OB

AOB
A
+
,
2
2

terminando la demostracin.
572 Teorema El ngulo entre dos secantes intersecndose fuera de una circunferencia es el promedio de la diferencia de los
ngulos centrales de los arcos subtendidos.

Demostracin: En la figura 7.36, sea X = AA BB . Se tiene que demostrar que


=
AXB

Del PAB,

=
PAB

OB
 A
AOB

.
2
2

+ PBA

PAB

AB + B
BA

AB + B
BA

=
=

AOA
BOB
OB +
+ A
2
2

=
=

OB

A
AOB
+
2
2

OB

AOB
A

2
2

AOB  A OB

2
2

terminando la demostracin.
573 Definicin Un cuadriltero se llama cclico si sus cuatro vrtices yacen en un crculo.
574 Teorema El cuadriltero simple ABMN es cclico si y slo si sus ngulos opuestos son suplementarios.

ngulos

107

y AMB
subtienden, o bien el mismo arco
Demostracin: Si ABMN es cclico, la asercin es clara, ya que ANB
del crculo, o bien, arcos complementarions. Vase las figuras 7.37 y 7.38.

Presmase ahora que los ngulos opuestos del cuadriltero ABMN son suplementarios. Trcese un crculo circunscrito al ABM de centro O. Se demostrar que N tambin yace en este crculo. Se supondr que N est fuera
tanto como dentro del crculo y se obtendr una contradiccin en cada caso.
Presmase primero que N est fuera del crculo circunscrito al ABM, como en la figura 7.39. Obsrvese que
y BOA
juntos subtienden la circunferencia entera y luego 2ABM
+ BOA
= 2 . Por hiptesis y en considerando
ABM
los ngulos perifricos,

=
=
=

+ MNA

ABM
+ 1 AOB
1 QOP

ABM
2
2
1
QOP.
2

= 0, esto es, a menos que N pertenezca, en efecto, a la


Lo anterior es una contradiccin, a menos que QOP
circunferencia.

Finalmente, presmase que N est en el interior del crculo, tal como en la figura 7.40. En este caso tambin se
+ BOA
= 2 , y as
tendr 2ABM

=
=
=

+ MNA

ABM
+ 1 QOP
+ 1 BOA

ABM
2
2
1
+ QOP,
2

= 0, dando nuevamente la demostracin.


de donde se colige que QOP

N
N

b
b

N
b

b
b

b
bM

b
b

b
b

Figura 7.37: Teorema 574.

Figura 7.38: Teorema 574.

Figura 7.39: Teorema 574.

Figura 7.40: Teorema 574.

575 Teorema (Miquel) En el ABC sean P, Q, R puntos sobre los lados [BC], [CA], [AB], respectivamente. Entonces los
crculos circunscritos de los tringulos ARQ, BPR, CQP pasan por un punto comn.

Demostracin: Sean T 6= R el (otro) punto de interseccin de los crculos circunscritos de los tringulos ARQ
y BPR. Por la colinealidad de los puntos involucrados,
,
T
QA = CQT

,
T
RB = ART

.
T
PC = BPT

108

Captulo 7
Porque los cuadrilteros involucrados son cclicos,
,
T
QA = ART

.
T
RB = BPT

Se deduce que
T
PC

BPT

T
RB

ART

T
QA

CQT

y por consecuencia el cuadriltero CQPT es cclico, de donde T al crculo circunscrito del CQP pertenece.
Vase la figura 7.41.

A
b

bQ
b

B
F

D
b

Q
O

b
b

P
b

O
b
b

A
S

R
Figura 7.41: Teorema 575.

Figura 7.42: Ejemplo 576.

Figura 7.43: Ejemplo 577.

576 Ejemplo El crculo de centro O es tangente a la recta AG e D, a la recta AH en E y a la recta BC en F, como en la figura
= , hllese BOC
en trminos de .
7.42. Si HAG
y que AEO
son rectos.
Resolucin: Obsrvese que ADO
y as DAO
=
Ahora, [AO] biseca al HAG
Luego
biseca al DOF.



, DOA = . Como [BD] y [BF] son tangentes al crculo, [OB]
2
2
2

DOA
1  
=

.
2
2 2 2
= .
El ngulo buscado es por lo tanto 2(BOF)
2 2
=
BOF

577 Ejemplo (Canad, 1975) Se toman cuatro puntos consecutivos A, B,C, D en una circunferencia. Los puntos P, Q, R, S


BC,
CD,
DA.
Demostrar que
de la circunferencia son, respectivamente, los puntos medios de los arcos AB,
PR QS.

ngulos

109


Resolucin: Sea PR QS = T como en la figura 7.43 y sea O el centro de la circunferencia. Obsrvese que
y RDS
juntos comprenden la mitad de la circunferencia. Del PT S,
PBQ
S
PT

+ SPT)

(PST

+ SPR)

(PSQ

1
=
POQ + ROS
2
1
=
2

,
=
2

mostrando la perpendicularidad de las rectas.1


578 Ejemplo Dos crculos se intersecan en A y B. El punto P viaja alrededor de uno de los crculos. Las rectas PA y PB se

extienden de tal manera que corten al otro crculo en C y D, respectivamente. Demustrese que la longitud de la cuerda CD es
independiente de P.
es constante, como en la figura 7.44. Pero esto se desprende de
Resolucin: Es suficiente demostrar que CAD
= APB
+ ADB

CAD

y de que estos ltimos dos ngulos son constantes.


C

B
b

D
Figura 7.44: Ejemplo 578.

579 Ejemplo Tres crculos congruentes pasan por un punto comn P, intersecndose en los puntos P, U, V y W como en la

figura 7.45. Demustrese que P es el ortocentro del UVW .


Resolucin: Sean A, B,C los centros de los crculos, vase la figura 7.46. Note que AVCP y BWCP son rombos
y por tanto, paralelogramos. Luego AV k BW y as, AVW B es un paralelogramo.
Adems PU es una cuerda comn de los crculos con centro en A y B y as, perpendicular a AB. Luego pues, lo es
tambin a VW . De manera semejante se demuestra que PV UW y PW UV , de donde se obtiene el resultado.

580 Ejemplo Sean A, B,C puntos colineales. Constryanse crculos con dimetros en AB, AC y BC. Sea D un punto en el arco
tal que AD AC y sea EF la tangente comn a los arcos AB
y BC.
Demustrese que BEFD es un rectngulo. (Figura 7.47.)
AC
1

Rectal?

110

Captulo 7
Resolucin: Reljese la condicin AC BD y permtase a D recorrer el semicrculo AC. Sea Sea E la interseccin de AD con el semicrculo AB, y sea F la interseccin de DC con el semicrculo BC. Sean M y N los puntos
B = BF
C = 90 , BE DF es un rectngulo. Se tiene

= AE
medios de AB y BC respectivamente. Como ADC
BD = BE
F

BD = BF
E .

B. De igual manera, NBF es issceles, as NBF


B. Por lo
= ME

= NF

El ME B es issceles, as MBE

F y NBD
E . Si MBD
F = 90 entonces ME
F = NF
E = 90 , o en otras

= ME
= NF
= ME
tanto MBD

palabras, E F es tangente a ambos semicrculos AB y BC, y entonces E = E , F = F . Se concluye que BEFD es


un rectngulo.

D
b

b
b

W
V

bW

F
b

Figura 7.45: Ejemplo 579.

Figura 7.46: Ejemplo 579.

Figura 7.47: Ejemplo 580.

Tarea
A B C D
b b b b

b b b b
H G F E

581 Problema En la figura 7.48, ambos tringulos son equilteros. Hllese la medida

de x, en grados.

Figura 7.49: Problema 584.

b
585 Problema Se remueve de una pared un ladrillo que tiene forma poligonal regular.

Se observa que si el ladrillo sufriese un giro de 40 o de 60 en torno a su centro, cabra


otra vez en el hueco original. Cul es el menor nmero de lados de este polgono?

75

65

b+ Cb+ b
b de la
586 Problema Hallar la suma de los ngulos de los vrtices b
A+B
D+E
estrella de cinco puntas de la figura 7.50.

Figura 7.48: Problema 581.

582 Problema Qu ngulo forman las agujas del reloj a cuarto para las cinco?

583 Problema DEFG es un cuadrado que se ha trazado fuera del pentgono regular

ABCDE. Cunto mide E


AF en grados?

584 Problema Tres cuadrados idnticos se construyen consecutivamente, como en la

+ Ad
figura adjunta (figura 7.49). Calclese la suma de ngulos ACH
DH.

D
b

bC

E b

b
A

b
B

Figura 7.50: Problema 586.

587 Problema El ABC es issceles en A. Si P es el punto medio del segmento [AB] y

si AP = PB = BC, demostrar que B


AC =

.
5

Congruencia de tringulos y desigualdad del tringulo


588 Problema (AHSME, 1978) En la figura 7.51,A1 A2 A3 es equiltero y An+3 es el
punto medio del segmento [An An+1 ] para todo entero estrictamente positivo n. Demostrar
2
que A
.
44 A45 A46 =
A3
3
b

111
Figura 7.52: Problema 592.

593 Problema En la figura AB = BC = CD = DE = EF = FG = GA. Hallar D


AE.

bA5

A6 b

G
A1

b
A2

b
A4

B
Figura 7.51: Problema 588.

Figura 7.53: Problema 593.

589 Problema Dos crculos de radios desiguales son tangentes externamente en el pun-

to A. Una tangente exterior comn toca al crculo de menor radio en B y al crculo de

radio mayor en C 6= B. Demostrar que B


AC = .
2

594 Problema ABCDEF en la figura 7.54 es un hexgono convexo equiangular. De-

mostrar que
AB DE = CD FA = EF BC.

590 Problema Dos crculos se intersecan en dos puntos. Demustrese que la recta que

une los centros de dos crculos que se intersecan es bisecada por la cuerda comn de los
crculos.
591 Problema Un crculo est inscrito en el tringulo ABC, siendo D, E, F los puntos
de tangencia a los lados [AB], [CA] y [BC] respectivamente. Demostrar que

1
BD = (AB + BC CA).
2
592 Problema Un crculo es inscrito en el tringulo ABC, rectngulo en C, como en

la figura 7.52. El crculo es tangente a la hipotenusa [AB] en P, en donde AP = 20 y


BP = 6. Hallar el radio del crculo.

bB

B
b

b
b

Figura 7.54: Problema 594.

7.2. Congruencia de tringulos y desigualdad del tringulo


Los siguientes resultados auxiliarn en el desarrollo del tema de esta seccin.
595 Teorema (Desigualdad del tringulo) En todo tringulo no degenerado, la suma de las longitudes de cualesquiera dos

lados es mayor que la longitud del tercer lado.


Demostracin: En la figura 7.55, AB + AC > BC ya que una lnea recta es la distancia ms corta entre dos puntos
en el plano.
596 Ejemplo Sea M un punto en el interior del ABC. Demostrar que

AB + AC > MB + MC.
En consecuencia, si P el permetro del ABC, demostrar que
AM + BM +CM < P.

112

Captulo 7

Resolucin: Sea N el punto en el cual BM corta al [AC]. Entonces


AB + AC = AB + AN + NC > BN + NC = BM + MN + NC > BM + MC,
demostrando la primera asercin.
De manera semejante se puede demostrar que
BA + BC > MA + MC,

CA +CB > MA + MB.

As pues,
AB + AC > MB + MC,

BA + BC > MA + MC,

CA +CB > MA + MB

2(AB + BC +CA) > 2(MA + MB + MC)

AM + BM +CM < P.

A b

bC

///

///

bC

Figura 7.55: Teorema 595.

Figura 7.56: Tringulos congruentes.

597 Ejemplo Sea P el permetro del ABC. Si M es un punto en el interior del tringulo, demostrar que

P
< AM + BM +CM < P.
2
Resolucin: Del MAB se tiene
Del MCA se tiene
Del MBC se tiene

AM + BM > AB.
CM + AM > CA.
BM +CM > BC.

Sumando,
AB + BC +CA < 2(AM + BM +CM) =

P
< AM + BM +CM.
2

La segunda desigualdad se obtiene del problema anterior.


598 Ejemplo Sean a, b, c las longitudes de los lados de un tringulo. Demustrese que

a
b
c
+
+
< 2.
b+c c+a a+b

Congruencia de tringulos y desigualdad del tringulo

113

Resolucin: Se tiene
a
b
c
+
+
b+c c+a a+b

=
<
=
=

2a
2b
2c
+
+
(b + c) + (b + c) (c + a) + (c + a) (a + b) + (a + b)
2a
2b
2c
+
+
a + (b + c) b + (c + a) c + (a + b)
2 (a + b + c)
a+b+c
2.

599 Definicin (Mediatriz de un segmento de recta) La mediatriz de un segmento de recta es la recta que pasa por el punto

medio del segmento y es perpendicular a ste.


600 Teorema Si se erige una perpendicular desde el punto medio de un segmento de recta entonces

cualquier punto en la perpendicular es equidistante de las extremidades del segmento de recta.


cualquier punto que no est en la perpendicular est a distancias desiguales de las extremidades del segmento.
Demostracin: Presmase que en la figura 7.57, [CD] [AB], donde D es el punto medio del [AB] y sea E
cualquier punto en [CD]. Sobrepngase ahora BDE en ADE, utilizando [DE] como eje de simetra. Se tiene

ya que ambos son ngulos rectos. Entonces


= ADE,
BDE
BD coincide con AD. Por hiptesis BD = AD y as, al


doblar, A cae sobre B. Pero esto quiere decier que las rectas BE y AE coinciden y por lo tanto AE = BE.
En la figura 7.58, presmase que CD AB, que D es el punto medio del segmento [AB] y que F est fuera de la

recta CD. Como una recta es la distancia ms corta entre dos puntos,
BE + EF > BF.
Pero BE = AE en virtud de , de donde
BE + EF > BF = AF = AE + EF > BF,
completando la demostracin.
b

E b

F
C

E b

Figura 7.57: Teorema 600.

Figura 7.58: Teorema 600.

Los siguientes corolarios se deducen fcilmente.


601 Corolario Todo punto que equidista de los extremos de un segmento de recta yace en la mediatriz de este segmento de

recta.

114

Captulo 7

602 Corolario Dos puntos equidistantes de los extremos de un segmento de recta determinan la recta perpendicular a este

segmento de recta que pasa por el punto medio del segmento de recta.
603 Corolario Si se trazan rectas desde cualquier punto de la mediatriz de un segmento de recta hasta los extremos ste

entonces
las rectas hacen ngulos iguales con el segmento de recta.
las rectas hacen ngulos iguales con la perpendicular.
= EBD
y
Demostracin: Sobreimpngase BDE en ADE en la demostracin del teorema 600. Luego EAD
= BED.

AED

604 Definicin Los tringulos ABC y DEF se dicen congruentes, denotado por ABC
= DEF si existe una correspon-

dencia tal que sus lados correspondientes y ngulos correspondientes sean iguales.

b = D,
b= F
b B
b = E,
b C
b y por lo tanto ABC
En la figura 7.56 se tiene AB = DE, AC = DF, BC = EF, A
= DEF.

605 Teorema (Criterio LAL) Si dos de los lados de un tringulo y el ngulo comprendido por estos dos lados son congruentes

a los lados y al ngulo homlogos de otro tringulo, ambos tringulos son congruentes.
b = D.
b Sobreimpngase ABC
Demostracin: Presmase que en la figura 7.59 se tiene AB = DE, AC = DF y A
byD
b coincidan, que el segmento [AB] caiga sobre el [DE] y que el [AC] caiga
sobre DEF de tal manera que A
sobre [DF]. Como AB = DE y AC = DF, Bb deber caer sobre E y C sobre F. As, el lado [BC] coincide con el lado
[EF] demostrando el teorema.

606 Teorema (Criterio ALA) Si dos de los ngulos de un tringulo y el lado comprendido por estos dos ngulos son congruen-

tes a los ngulos y al lado homlogos de otro tringulo, ambos tringulos son congruentes.
b=D
byB
b = E.
b Sobreimpngase ABC en DEF
Demostracin: En la figura, 7.59, presmase que AB = DE, A
b = D,
b el lado [AC] deber
de tal manera [AB] coincida con [DE], que A caiga en D y que B caiga en E. Como A
b
b
caer en el lado [DF] y C deber caer en alguna parte sobre [DF]. Como B = E, el lado [BC] deber caer en alguna

parte de la recta EF. Ya que C cae simultneamente en [DF] y [EF], se sigue que deber caer en la interseccin
de estos dos segmentos, esto es, en F.

bC

Figura 7.59: Teoremas 605, 606 y 607.

bF

bF

Figura 7.60: Teorema 607.

607 Teorema (Criterio LLL) Si los tres lados de un tringulo son congruentes a los tres lados homlogos de otro tringulo,

ambos tringulos son congruentes.


Demostracin: Presmase en la figura 7.60 que AB = DE, BC = EF, y que CA = FD. Pngase
 al
 DEF sobre
ABF con el lado [DE] coincidiendo con [AB] y F cayendo sobre F . Trcese el segmento CF . Por hiptesis

Congruencia de tringulos y desigualdad del tringulo




115

=
[AC] = AF y [BC] = BF . As, [AB] CF en su punto medio, por virtud del corolario 602. Entonces BAC

BAF . Pero entonces ABC = ABF por el criterio LAL (teorema 605). Esto implica que ABC = DEF.

608 Definicin Una recta que pasa por el vrtice de un tringulo se llama ceviana de este vrtice. La ceviana es propia si no

coincide con un lado del tringulo.

Se adoptar la convencin de marcar la interseccin de la ceviana con el lado opuesto a su vrtice con una
prima , as AA , BB , CC son cevianas. Vase la figura 7.61.
C

A
b

Figura 7.61: Cevianas.

M
b

Figura 7.62: Cevianas.

Figura 7.63: Puntos menelaicos.

609 Definicin A un punto P que yaga sobre la recta determinada por un lado del ABC se le llama punto menelaico2 de este

lado. Si el punto no es un vrtice del tringulo entonces es un punto menelaico propio.

610 Definicin (Mediana) Una ceviana que va desde un vrtice de un tringulo al punto medio del segmento opuesto se llama

mediana.

Los puntos medios de los lados BC, CA, AB del ABC, se denotarn respectivamente por MA , MB , MC . Las
respectivas medianas sern entonces por [AMA ], [BMB ], [CMC ] y sus respectivas medidas por mA , mB y mC .
611 Definicin (Altura) La ceviana que va desde un vrtice de un tringulo y es perpendicular al segmento opuesto se llama

altura del tringulos.

Los pies de las perpendiculares de los lados BC, CA, AB del ABC, se denotarn respectivamente por H ,
A

HB , HC . Las respectivas alturas sern entonces por [AHA ], [BHB ], [CHC ] y sus respectivas medidas por hA , hB y hC .
b A
D
C
D
C
M b
b
b
b
b
b
b D
P
b / b / b
b
bC
b
B
A
N
B
bO
bO
b

B
b

F
Figura 7.64: Mediatriz de
[AB].

Que no puntos melenudos.

Figura 7.65: Ejemplo 612.

Figura 7.66: Ejemplo 612.

Figura 7.67: Teorema 613.

116

Captulo 7

= OBA
= . Demostrar que CDO es equiltero.
612 Ejemplo En el cuadrado ABCD de la figura 7.65, OAB

12

Constryase el AFB equiltero, como en la figura 7.66. Obsrvese que AO = OB, ya que el
= CBO
= = 5 . Luego DAO
AOB es issceles. Adems DAO
= CBO gracias al criterio LAL, y as
2 12 12
5

=
DO = CO. Por lo tanto es suficiente demostrar que DOA
, ya que entonces DAO sera issceles y se tendra
12
DC = DA = DO = CO, de donde resultara que CDO es equiltero.
Resolucin:

y AFB,
siendo adems la
Como AOB and AFB son ambos issceles, la recta OF biseca a los ngulos AOB


5
5
=
=
mediatriz del segmento [DC]. As pues AOB
=
. Como DAO
=
y OAF =
+ =
12 6
12
2 12 12
12 3
5
5

= AOF
=
y como adems DA = AF, DAO
, completando la
= FAO gracias al criterio LAL. As, DOA
12
12
demostracin.

613 Teorema Todo punto en la bisectriz de un ngulo equidista de los lados del ngulo. Recprocamente, si un punto equidista

de los lados del ngulo entonces yacer sobre la bisectriz angular.

esto es que PBM


=
Demostracin: En la figura 7.67, presmase que P est en la bisectriz BD del ngulo ABC,
Presmase que [PM] [AB], [PN] [BC]. Siendo tringulos rectngulos con ngulos idnticos e hipotenusa
PBN.
idntica se concluye que BPM
= BPN. As PM = PN.
Recprocamente, si [PM] [AB], [PN] [BC] y PM = PN entonces BPM
= BPN, siendo ambos tringulos
rectngulos con un cateto e hipotenusa iguales.

Tarea
614 Problema Demostrar que si los lados de un tringulo son desiguales, los ngulos
opuestos son tambin desiguales y el ngulo mayor est opuesto al lado mayor. Recprocamente, demostrar que si los ngulos de un tringulo son desiguales, los lados opuestos
son tambin desiguales y el lado mayor est opuesto al ngulo mayor.

615 Problema Sea P un punto en el interior del ABC. Demostrar que B


PC > b
A.

618 Problema Considrese n puntos verdes y n puntos amarillos en el plano, satisfa-

ciendo que no tres de ellos son colineales. Demostrar que se puede parear cada punto
verde con uno amarillo de tal manera que ningn segmento se cruce.
619 Problema (AHSME 40) Vase la figura 7.69.En el ABC, b
A = 100 , b
B = 50 ,
b = 30 . Adems, [BH] es una altura y [BM] es una mediana. Hllese la medida de
C

d
M
HC.

616 Problema En el tringulo ABC, se traza la mediana [AMA ]. Demostrar que si

alturas es menor que el permetro del tringulo.

MA

BMA = AMA , entonces el tringulo es rectngulo en A. Vase la figura 7.68.

617 Problema Demostrar que en un tringulo arbitrario, la suma de la longitud de las

b
b

Figura 7.68: Problema 616.

Figura 7.69: Problema 619.

7.3. Trapecios y paralelogramos


620 Definicin (Trapecio) Un trapecio es un cuadriltero en el cual al menos un par de lados son paralelos.
621 Definicin (Paralelogramo) Un paralelogramo es un cuadriltero en el cual ambos pares de lados son paralelos.
622 Teorema Si ABCD es un paralelogramo entonces

los ngulos opuestos son congruentes.

Trapecios y paralelogramos

117

los ngulos interiores adyacentes son suplementarios.


los lados opuestos son congruentes.
las diagonales se bisecan la una a la otra.

Demostracin: Vase la figura 7.70. Por ser ngulos alternos internos a dos paralelas,
= DCA,

BAC

= BCA,

DAC

= CDB,

ABD

= ADB.

CBD

b = DAC
+ BAC
= DCA
+ BCA
=C
by B
+ CBD
= CDB
+ ADB
=D
b = ABD
b y as los ngulos opuestos son
Ahora, A
congruentes. Como la suma de los ngulos interiores de un paralelogramo es 2 se tiene
b+ B
b+ D
b + B)
b + B)
b + D)
b+ C
b = 2(A
b = (A
b = (C
b = ,
2 = A

de donde los ngulos interiores adyacentes son suplementarios.


Ahora, por el criterio ALA, DAC
= BCA y as AD = BC y AB = CD, de donde los lados opuestos son congruentes.
Por la igualdad de ngulos arriba establecida y por el criterio OAB
= OCD, de donde AO = OC y BO = OD,
de donde las diagonales se bisecan la una a la otra.

O
b

Figura 7.70: Paralelogramo.

Figura 7.71: Teorema 623.

En efecto, algunas de las propiedades arriba mencionadas son suficientes para que un cuadriltero paralelogramo sea.
623 Teorema Sea ABCD un cuadriltero convexo. Las siguientes propiedades son mutuamente equivalentes:

1. ABCD es un paralelogramo.
2. un par de lados opuestos son congruentes y paralelos.
3. Los ngulos interiores opuestos son congruentes.
4. Los lados opuestos son iguales.
5. Las diagonales se bisecan la una a la otra.

Demostracin: Refirase a la figuras 7.70 y 7.71.


1 = 2 Que un par de lados opuestos son congruentes se sigue por definicin de paralelogramo. Que este par
de lados opuestos son congruentes se sigue por el teorema 622.

118

Captulo 7

= CBD,
por ser ngulos
2 = 3 Presmase que AD k BC y que AD = BC. En DBA y BDC se tiene ADB

alternos internos entre paralelas por la transversal DB. Por la misma razn, ABD = CDB. As
+ ABD
= CBD
+ CDB
= D
b = B.
b
ADB
b = C.
b
De la misma manera se puede establecer que A

b=C
byB
b = D.
b Como
3 = 4 En la figura 7.71, AD = A D y BC = BC . Presmase que en cuadriltero ABCD, A
b+ B
b+ D
b+ C
b = 2 ,
A

b= D
b Luego los ngulos alternos internos de la transversal
b = B
b = C.
se tiene A
BD a las rectas AD y

BC son iguales, y se concluye que AD k BC.

Habiendo probado que AD k BC, se obtiene entonces que los ngulos alternos internos producidos por la
diagonal [AC] a estas rectas son iguales. Luego, por el criterio ALA, DAC
= BCA. De aqu, AD = BC y
AB = CD.
y BOA
son congruentes y por tanto AOB
4 = 5 Sean AD = BC y AB = CD. Los ngulos verticales DOC
=
COD. De aqu, AO = OC y DO = OB.

= AOB.
Luego
5 = 1 Supngase AO = OC y BO = OD. Por ser ngulos opuestos por el vrtice O, COD

COD = AOB por el criterio LAL. Luego OAB = OCD y siendo ngulos alternos internos de la transver

= BOC
y DOA
= OBC

sal AC, se tiene DC k AB. De igual manera, DOA


= BOC por LAL. Luego ODA

y siendo ngulos alternos internos de la transversal DB, se tiene AD k CB. Se concluye que ABCD es un
paralelogramo.

Las siguientes definiciones y presunciones son anlogas.


624 Definicin (Rombo) Un rombo es un paralelogramo en el cual dos lados consecutivos son congruentes.

En efecto, es fcil demostrar lo siguiente.


625 Presuncin Sea ABCD un cuadriltero simple. Las siguientes propiedades son mutuamente equivalentes:

1. ABCD es un rombo.
2. Los cuatro lados son congruentes.
3. Las diagonales se bisecan la una a la otra en ngulos rectos.
4. Las diagonales se bisecan el ngulo en cada vrtice.
626 Definicin (Rectngulos) Un rectngulo es un paralelogramo en el cual al menos uno de sus ngulos es recto.

Como es bien sabido, en efecto, todos los ngulos de un rectngulo sern rectos.
627 Presuncin Sea ABCD un cuadriltero simple. Las siguientes propiedades son mutuamente equivalentes:

1. ABCD es un rectngulo.
2. Los cuatro ngulos internos son todos rectos.
3. Las diagonales son congruentes y se bisecan la una a la otra.

Trapecios y paralelogramos

119

628 Definicin (Distancia entre rectas paralelas) La distancia entre dos rectas paralelas es la longitud de cualquier segmen-

to perpendicular de una a otra recta.


629 Definicin (Cuadrado) Un cuadrado es un paralelogramo que es tanto un rombo como un rectngulo.

b
b

MC

A
D

bMB

MB
b

bM
A

b
b

b
b

MC

P
b

B
b

A
Figura 7.72: Teorema 630

Figura 7.73: Teorema 633.

Figura 7.74: Teorema 635.

Se utilizarn ahora los resultados obtenidos arriba para demostrar varias propiedades de tringulos y cuadrilteros.
630 Teorema El segmento de recta que une los puntos medios de dos lados de un tringulo es paralelo al tercer lado y mide la

mitad de ste.

Demostracin: Constryase BD k AC e intersecando MB MC en D. Por ser ngulos opuestos por el vrtice MC ,

M
B MC B = DM
B B. Por ser ngulos alternos internos de las paralelas AC k DB se tiene que A = M
C BD. Como por

hiptesis AMC = MC B se concluye por el criterio ALA que AMC MB = BMC D. As, MB MC = MC D y AMB = DB.
DMB BC
Luego BDMBC es un paralelogramo. Finalmente, MB MC =
=
.
2
2
631 Corolario La recta que biseca a un lado de un tringulo y es paralela a otro de sus lados, biseca tambin al tercer lado.

Demostracin: Supngase que en la figura 7.72, la recta MC F es paralela a BC, donde F es la interseccin de



la dicha recta con el lado [AC]. Del teorema 630, MC MB k BC. Luego se tiene MC MB k MC F. Tanto MB como F
yacen sobre [AC], se tiene F = MB .
Aplicando el teorema anterior y su corolario a los tringulos obtenidos por las diagonales de un trapecio, se obtiene el siguiente
teorema.
632 Teorema El segmento de recta que une los puntos medios de los lados no paralelos de un trapecio es paralelo a las bases
y mide el promedio de las longitudes de las bases. Dicho segmento se denomina la mediana del trapecio. Demostrar a su vez,
que la mediana del trapecio biseca las diagonales del trapecio.

Se demostrarn ahora importantes teoremas de concurrencia.


633 Teorema Las medianas de un tringulo concurren. El punto de concurrencia, llamado baricentro divide a la mediana en
razn 2 : 1 comenzando desde el vrtice.

120

Captulo 7

Sea AMA BMB = O. Sean G y H los puntos medios de [OA] y [OB], respectivamente. Del
AB

teorema 630, MA MB =
y MA MB k AB y en considerando el OAB y aplicando nuevamente el teorema 630 se
2
AB

tiene GH =
y HG k AB. Luego MA MB HG es un paralelogramo y por el teorema 622, sus diagonales se bisecan
2
en O. Luego
AG = GO = GMA ,
BH = HO = OMA
Demostracin:

de donde se deduce

2AMA
,
3

AO =

BO =

2BMB
.
3


Supngase ahora que AMA CMC = O . Por lo arriba demostrado,
AO =

2AMA
,
3

AO =

2AMA
= AO = O = O ,
3

Pero entonces

CO =

2CMC
.
3

de donde las tres medianas concurren.

MB

bM
A

O
b

MC

Figura 7.75: Ejemplo 634.

634 Ejemplo Sea P el permetro del ABC. Entonces

3P
< AMA + BMB +CMC < P.
4
Resolucin: Usando el teorema 633,
AO + BO > AB =

2
2
3
AMA + BMB > AB = AMA + BMB > AB.
3
3
2

De manera semejante,
3
BMB +CMC > BC,
2

3
CMC + AMC > CA.
2

Trapecios y paralelogramos

121

Sumando,
3
3
(AB + BC +CA) = AMA + BMB +CMC > (AB + BC +CA) ,
2
4

2(AMA + BMB +CMC ) >


deduciendo la primera desigualdad.

Vase la figura 7.75. Refljese el punto C a travs del punto MC y sea C la imagen de esta reflexin. Ntese que
CMC = MCC y que por lo tanto, CAC B es un paralelogramo. Como CC = 2CMC y AC = BC, se tiene
AC + AC > CC = AC + BC > 2CMC .
De la misma manera,
AC + ABC > 2AMA ,

AB + BC > 2BMB .

Sumando,
2(AB + BC +CA) > 2(AMA + BMB +CMC ) = AMA + BMB +CMC < P,
dando la segunda desigualdad.

bA

HB

HA

A
b

H
C

D
b

HC

B
MC

bMB

b
b

MA

Figura 7.76: Teorema 636.

Figura 7.77: Teorema 637.

O
b

Figura 7.78: Teorema 638.

635 Teorema (Varignon) Sea ABCD un cuadriltero convexo, y sean M, N, O, P, respectivamente, los puntos medios de los
lados [AB], [BC], [CD] y [DA]. Entonces MNOP. es un paralelogramo.

Demostracin: Por el teorema 630, considerando ACD y ABC se tiene


[PO] k [CA] ,

[MN] k [CA] , PO =

CA
= MN.
2

[PM] k [DB] , ON =

DB
= DB.
2

Considerando CDB y ADB se tiene


[ON] k [DB] ,

Se desprende que MNOP es un paralelogramo.


636 Teorema Las bisectrices de los ngulos interiores de un tringulo concurren. El punto de concurrencia es llamado el
incentro del tringulo.

122

Captulo 7
Demostracin: Dentese por I la interseccin de las bisectrices angulares de los ngulos de los vrtices A y B.
Se deber demostrar que IC biseca al ngulo de vrtice C. Sean D, E y F los pies de las perpendiculares desde I
a BC, CA y AB, respectivamente.
Obsrvese que IDB
= IEB y IEA
= IFA. Se sigue que ID = IE = IF. En consecuencia, IDC
= IFC.

Por lo tanto ICD = ICF, como se deba demostrar.

De la demostracin del teorema 636, ID = IE = IF = r, digamos. Luego I es el centro del crculo inscrito en
A

el ABC de radio r. Vase la figura 7.79.

b
b

MC

b
b

bMB
b

b
b

MA
b

O
b

Figura 7.79: El crculo inscrito.


Figura 7.80: El circuncrculo.

637 Teorema Las alturas de un tringulo concurren. El punto de concurrencia es llamado el ortocentro del tringulo.

Demostracin: Por cada uno de los vrtices del ABC trcese una recta paralela al lado opuesto del vrtice,
y frmese el A BC . Refirase a la figura 7.77. Se demostrar que cada altura del ABC es un segmento de
recta yaciendo en una mediatriz perpendicular del A BC . Como ya se ha demostrado que las mediatrices
perpendiculares concurren, se sigue que en tanto las alturas se intersequen, se intersecarn en un slo punto. Pero
que cualquier par de alturas se intersecan en un punto es obvio, ya que no son rectas paralelas!
Se ha demostrar ahora que la altura del ABC en B yace en la mediatriz perpendicular de AC . Los casos de las
otras dos alturas se demuestran de manera semejante. Por definicin, la altura del ABC en B es perpendicular
AC y como, por construccin, AC k AC , esta altura tambin es perpendicular a AC . Falta demostrar que B
es el punto medio de AC . Ntese que tanto ABAC como ACBC son paralelogramos. Se sigue, como los lados
opuestos de un paralelogramo son congruentes, que BA = AC y C B = AC. Luego B es la mediatriz de A B y queda
demostrada la asercin.
638 Teorema Las mediatrices de los lados de un tringulo concurren. El punto de concurrencia es llamado el circuncentro del

tringulo.
Demostracin: Dentense los puntos medios de los lados BC,CA, AB por MA , MB , MC . Sea O el punto de interseccin de las mediatrices perpendiculares de AB y AC. Se tiene que demostrar que OMA BC. Obsrvese
que OBMC
= OAMC , al ser ambos tringulos rectngulos con dos catetos iguales. De manera semejante,

OBMA = OCMA . Notando ngulos correspondientes OM


, ya que forman una lnea recta.
A B = OMAC =
2
Queda demostrado el teorema.

De la demostracin del teorema 638 se sigue que el circuncentro de un tringulo dado equidista de sus
vrtices. Luego, es el centro de su crculo circunscrito.

Tarea

Permetros y reas

123

639 Problema Si las medianas de dos lados de un tringulo son iguales, demostrar que

gulo es issceles.

el tringulo es issceles.

640 Problema Si dos de las alturas de un tringulo son iguales, demostrar que el trin-

641 Problema Si un tringulo no es issceles, demostrar que las bisectrices internas


angulares son desiguales.

7.4. Permetros y reas


Se presumirn conocidos los conceptos de permetro y rea. En particular, se presumirn conocidas las siguientes frmulas.
642 Presuncin (Frmulas de permetro de varias figuras planas) El permetro de un polgono es la suma de las longitudes de sus lados. El permetro o circunferencia de un crculo es 2 r, en donde r es el radio del crculo. La longitud de un arco
de crculo que subtiende un arco central de radianes es r .
643 Presuncin (Frmulas de rea de varias figuras planas) El rea de un rectngulo de lados a y b unidades lineales es

(a + b)h
ab unidades cuadradas. El rea de un trapecio de bases paralelas de longitud a y b unidades lineales y de altura h es
2
bh
unidades cuadradas. En particular, si a = 0 se obtiene que el rea de un tringulo de base b y altura h es
. El rea de un
2

3 2
tringulo equiltero de lado s es
s . El rea de un crculo de radio r es r2 . El rea de un sector circular que subtiende un
4
r2
ngulo central de radianes en un crculo de radio r es
.
2

El rea del tringulo ABC se denotar por [ABC].


b

b
b

Figura 7.81: Ejemplo 644.

b
b

b
b

b
b

Figura 7.82: Ejemplo 645.

b
b

Figura 7.83: Ejemplo 646.

644 Ejemplo Un tringulo de Reuleux es la figura obtenida al trazar arcos de radio s con centro en cada uno de los vrtices

de un tringulo equiltero de lado s, cada arco de 60 , como en la figura 7.81. Hallar el permetro y el rea de un tringulo de
Reuleux.

Resolucin: El permetro es la suma de los tres arcos circulares, cada uno midiendo s , de donde el permetro
3
buscado es
 
3s
= s.
3
El rea deseada es el rea del tringulo equiltero contenido ms tres veces el rea de uno de los segmentos
circulares. El rea de uno de dichos segmentos es

s2 s2 3

,
6
4

124

Captulo 7

de donde el rea buscada es


3


s2 3 s2
s2 s2 3

=
+
3 .
6
4
4
2

645 Ejemplo Tres crculos de radio R son mutuamente tangentes, como se muestra en la figura 7.82. Cul es el rea de la

figura acotada por los tres crculos?


Resolucin: El rea buscada es el rea del tringulo equiltero formado al unir los tres centros de los crculos
menos tres veces el rea de uno de los sectores angulares formados. Como el tringulo equiltero tiene lado de
longitud 2R, su rea es

R2 3.
Como

R2
1
es de la circunferencia, cada sector angular tiene rea de
. Luego el rea buscada es
3
6
6

R2
R2 3
.
2

646 Ejemplo El cuadrado de la figura 7.83 tiene lado 1. La regin sombreada es formada por el rea solapada de cuartos de
crculos centrados en los vrtices del cuadrado. Crculos diagonalmente opuestos son tangentes. Hallar el rea sombreada.

Resolucin: El rea de la figura es el doble del rea en rojo. Primero se halla el radio de cada uno de los
cuartos de crculo. La diagonal del cuadrado mide 2r. Por el Teorema de Pitgoras,
(2r)2 = 12 + 12,

2
de donde r =
. La regin en rojo tiene como rea el rea del cuadrado menos dos de los cuartos de crculos y
2
dos pequeos tringulos issceles en las esquinas. As pues el rea en rojo es
12
El rea de la cruz patea es luego

2
2

1
2

2
1
= 2 .
2
2 4

2 21 .
2

Tarea
C

647 Problema El ABC es equiltero, de lado a. Dos crculos tangentes con centros en

B y C respectivamente se trazan, como en la figura 7.84. Mostrar que el permetro del


rea sombreada es
a+

a
3

y por lo tanto independiente de los radios de los crculos. Demostrar adems que el rea
de esta regin es

a2 3 2
(R + r2 ),
4
6
en donde R y r son los radios de los crculos.

Figura 7.84: Problema 647.

648 Problema (Teorema de Viviani) Sea h la longitud de una altura del tringulo equi-

ltero ABC y sea P cualquier punto en el interior del tringulo. Sean R, S, T los pies de
las perpendiculares desde P hasta los lados [AB], [BC], [CA], respectivamente. Demostrar que
PR + PS + PT = h.

Tarea

125

649 Problema En el trapecio ABCD, [AB] k [CD]. Se traza [MN] k [AB] con M [AD]

654 Problema El punto P est en el interior del tringulo equiltero ABC de lado 3.

650 Problema En la figura 7.85, los seis crculos pequeos tienen radio 1 y cada uno
es tangente a sus dos vecinos y al crculo mayor que los encierra. Cul es el rea de la
regin estrellada acotada por los seis crculos internos?

655 Problema El rectngulo en la figura 7.89 se diseca en nueve cuadrados. Si el cuadrado sombreado tiene rea 1, cul es el rea del rectngulo?

y N [BC]. Si [MN] biseca el rea del trapecio, hallar MN.

La distancia de P a [AB] es a, la distancia de P a [AC] es 2a y la distancia de P a [CB] es


3a. Hallar a.

b
b

b
b
c6
c8

Figura 7.85: Problema 650.

651 Problema Cuatro cilindros de dimetro 1 estn pegados apretadamente por una

cuerda muy fina, como en la figura 7.86. Demostrar que la cuerda tine longitud 4 + .

Demostrar tambin que el rea sombreada entre los cilindros es 1 .


4

b
b

c1

c2

c5

c7

c4

c3

Figura 7.86: Problema 651.


Figura 7.89: Problema 655
652 Problema El cuadrado en la figura 7.87 tiene lado 4. Demostrar que el rea de la

rosa de cuatro ptalos mostrada es


8 16.
La rosa es la interseccin de semicrculos de radio 2 con dimetros en los lados del cuadrado.

656 Problema Dos cuadrados ABCD y EHGF, ambos de lado a, estn colocados en
manera tal que un vrtice de uno est en el centro del otro, como en la figura 7.90. Dea2
mostrar que el rea del cuadriltero EJCK es
y no depende de la posicin de J (o
4
K).

b
b

E
D

Figura 7.87: Problema 652.

b b

b
b
b

Figura 7.90: Problema ??.


653 Problema (AHSME, 1984) Un rectngulo interseca a un crculo, como en la figura

7.88. Si AB = 4, BC = 5 y DE = 3, hallar EF.


657 Problema El hexgono ABCA BC en la figura 7.91 est inscrito en una circunferencia, tal que las diagonales AA , BB y CC son dimetros de la circunferencia y
[ACB] = 1. Calcular [ABCA BC ].

b
b

b
b

Figura 7.88: Problema 653.


Figura 7.91: Problema 657.

126

Captulo 7

658 Problema En la figura 7.92, cada una de las cuerdas divide al crculo exterior en

dos regiones cuyas reas estn en razn 1 : 3. La interseccin de las cuerdas forma un
cuadrado concntrico con el crculo exterior e inscrito en el crculo interior. Demostrar
1
que la razn del rea sombreada al rea del crculo interno es
.
2

B es un cuarto de circunferencia del crculo de centro O que tiene radio


659 Problema A

A y OB son semicrculos congruentes y de dimetro R. Hllese el rea de


R. Los arcos O
la regin sombreada. Vase la figura 7.93.

O
b

Figura 7.93: Problema 659.


Figura 7.92: Problema 658.

7.5. Teorema de Pitgoras

Se discutir ahora lo que es quizs el ms famoso teorema en todas las matemticas.

Figura 7.94: Dimensiones.

a+b

Figura 7.95: Pitgoras.

Figura 7.96: Igual en rea a la figura 7.95.

660 Teorema (Pitgoras) La suma de los cuadrados de las longitudes de los catetos de un tringulo rectngulo es igual a la
longitud del cuadrado de la hipotenusa.

Demostracin: Se presentarn varias demostraciones aqu, casi todas basadas en la diseccin de figuras. En
todas las figuras utilizadas se presumir que los catetos miden a, b con b a y que c es la medida de la hipotenusa,
como en la figura 7.94.
La primera demostracin es atribuida al mismsimo Pitgoras. En la figura 7.95, el cuadrado mayor tiene rea
ab
(a + b)2. El cuadrado interno azul tiene rea c2 . Cada uno de los tringulos rectngulos amarillos tiene rea .
2
Luego

ab
(a + b)2 = c2 + 4
= a2 + b2 = c2 .
2
El clculo algebraico efectuado se puede ver geomtricamente con un reordenamiento de las piezas, como en la
figura 7.96, ya que ambos cuadrados tienen la misma rea y descontando los tringulos amarillos, el rea azul del
uno es la suma de las reas rojas del otro.
La segunda demostracin es atribuida a Bhaskara. El cuadrado mayor en la figura 7.97 tiene rea c2 . Este est
ab
compuesto de cuatro tringulos rectngulos, cada uno de rea
y de un cuadrado magenta, de rea (a b)2 .
2
Luego

ab
c2 = (a b)2 + 4
= c2 = a2 + b2,
2

Tarea

127
dando de nuevo el teorema.
La tercera demostracin es atribuida al presidente yanqui James A. Garfield. El trapecio en la figura 7.98 tiene
bases a y b y altura a + b, de donde su rea es
(a + b)2
.
2
ab
c2
Pero el mismo trapecio puede ser descompuesto en dos tringulos de rea
y uno de rea . As,
2
2

c2
(a + b)2
ab
=2
+
= a2 + b2 = c2 ,
2
2
2
dando otra vez el resultado.
La cuarta demostracin es una de las de los Elementos de Euclides. Primero, ABF
= AEC por el criterio ALA,

ya que AE = AB, AF = AC, y BAF = BAC + CAF = CAB + BAE = CAE. El ABF tiene base [AF] y su altura
AC2
desde B mide AC. Su rea es por lo tanto
. Por otra parte, AEC tiene el lado [AE] y altura desde C igual a
2

AM, en donde M = AB CL y CL k AE. Por lo tanto, el rea del AEC es la mitad del rea del rectngulo AELM.
Esto quiere decir que el rea AC2 del cuadrado cuyos lados tienen longitud AC es igual al rea del rectngulo
AELM.
De manera semejante, el rea BC2 del cuadrado cuyos lados tienen longitud BC es igual al rea BMLD. Finalmente, los dos rectngulos AELM y BMLD componen el cuadrado en la hipotenusa AB.
b

b
b

E
Figura 7.97: Bhaskara.

Figura 7.98: Garfield

b
b

H
K

L D
Figura 7.99: Euclides

661 Teorema (Recproco del Teorema Pitgoras) Si en el tringulo ABC, a2 +b2 = c2 , entonces el tringulo es rectngulo

en C.

Demostracin: Constryase el XY Z tal que XZ = AC = b, Y Z = BC = a y Zb = 90 . Como XY Z es rectngulo


en Z, se puede aplicar el Teorema de Pitgoras y
XY 2 = XZ 2 + ZY 2 = b2 + a2 = c2 = XY = c.
Luego, por el criterio LLL, XY Z
= ABC, de donde Cb = Zb = 90 .

Tarea

128

Captulo 7

662 Problema El ABC es rectngulo en C. Sea D el pie de la perpendicular desde el

vrtice C hasta el lado [AB]. Se inscribe un crculo de radio r1 en el ACD y otro de


radio r2 en el ADB. Si el radio del crculo inscrito al ABC es r, demustrese que
r=

665 Problema En la figura adjunta, los crculos son concntricos, [AB] es tangente al

crculo interno y AB = 20. Hallar el rea del anillo sombreado.

r12 + r22 .

663 Problema Se inscribe un crculo de radio 2 en un cuadrado. Un crculo menor, de

radio r es tangente tanto al crculo mayor como a dos lados del cuadrado dentro del cuadrado. Hallar r.

A
Figura 7.102: Problema 665.

C
666 Problema Se inscribe un crculo dentro de un cuarto de crculo, como en la figura

7.103. Si el crculo mayor tiene radio R, hallar el radio del crculo menor.

Figura 7.100: Problema 663.

664 Problema Dos crculos de radio 2 y de centros O y P son mutuamente tangentes,


como en la figura 7.101. Si [AD] y [BD] son tangentes, hallar BD.

O
b

C
Figura 7.103: Problema 666.

Figura 7.101: Problema 664.

7.6. Proporcionalidad y semejanza


667 Definicin Una proporcin es una aseveracin acerca de la igualdad de dos razones. As, escribimos

a : b = c : d

a c
= .
b d

Las siguientes aseveraciones son fcilmente demostrables.


668 Presuncin Si a : b = c : d y si n > 0 entonces,

1. (a + b) : a = (c + d) : c.
2. (a b) : a = (c d) : c.
3. (a + b) : (a b) = (c + d) : (c d).
4. a : b = (a + c) : (b + d).
5. an : bn = cn : d n .
669 Teorema Considrese una serie de paralelas cortando dos rectas. Si en una de estas rectas, las paralelas cortan segmentos

de igual longitud, tambin cortarn a la otra en segmentos de igual longitud.

Proporcionalidad y semejanza

129

Demostracin: En la figura 7.104, supngase que AB = CD. Se tiene que demostrar que A B = C D . Ahora
bien, en el trapecio ACC A se tiene, por hiptesis que BB k AA . Luego BB biseca a AC. Por lo tanto, tambin
biseca a AC y A B = C D por el teorema 632.
670 Teorema Una recta paralela a un lado de un tringulo que divida a sus otros dos lados, los divide proporcionalmente.

Demostracin: En la figura 7.105, supngase que


CA
= r.
AB
Se tiene que demostrar que

C A
= r.
A B



a
, el cociente de dos enteros positivos. Luego bCA = aAB. Divdase a C A en a
b


partes y a A B en b partes iguales. El resultado se obtiene entonces del teorema 669.

Presmase primero que r =

Si r es irracional, considrese una sucesin de nmeros racionales r1 , r2 , r3 , . . . , convergiendo a r y aplquese el


resultado ya obtenido.
La recproca de este ltimo teorema se demuestra de la misma manera que el corolario 631.
671 Presuncin Si una recta divide a dos lados de un tringulo en segmentos proporcionales entonces la recta es paralela al

tercer lado.

A
B
C

D
b

A
b

B
b

A
C

Figura 7.104: Teorema 669.

Bb

F
b

Figura 7.105: Teorema 670.

Figura 7.106: Teorema 672.

b con [BC]. Entonces


672 Teorema (Teorema de la bisectriz) En el ABC sea D la interseccin de la bisectriz angular del A

BD AB
=
.
CD AC
Demostracin: Trcese [BF] k AD, como en la figura 7.106. Por el teorema 670,
FC
BC
=
.
AC
DC
= BAD.
Luego FA = AB.
Ahora bien, el FAB es issceles en A, ya que por ser ngulos alternos internos FBA
As FC = FA + AC = AB + AC. De aqu,
FC
AB
= 1+
AC
AC

130

Captulo 7
y
BC
BD + DC
BD
=
= 1+
.
DC
DC
DC
Por lo tanto,
1+

AB
BD
= 1+
,
AC
DC

de donde se destila el resultado..


673 Ejemplo Los lados del ABC son AB = c, BC = a y CA = b. Sobre D [AB], se traza una recta paralela al [AC], interse-

cando al [BC] en E. Desde E se traza una recta paralela a [AB] intersecando al [CA] en F. Desde F se traza una recta paralela a
[CA] intersecando al [BC] en H. Desde H se traza una recta paralela a [AB] intersecando al [CA] en I. Desde I se traza una recta
paralela a [BC] intersecando al [AB] en J. Hallar AG y BJ en trminos de BD = t.
Resolucin: Como los lados paralelos de un lado del tringulo cortan a los otros lados en segmentos proporcionales, se tiene
AF
AG CH
CI
BJ
BD BE
=
=
=
=
=
=
,
AB
BC
AC
AB
CB CA AB
luego BJ = AG = BD = t, de donde D y J coinciden. Adems BE = CH y AF = CI.

G
A

b
b

H
b

Figura 7.107: Ejemplo 673.

674 Definicin (Semejanza) Dos figuras S y T se dicen semejantes si mediante una serie de rotaciones, traslaciones, reflexio-

nes, dilataciones o contracciones se puede hacer a la una coincidir con la otra. La constante de dilatacin o contraccin utilizada
se llama coeficiente de homotecia o de semejanza. Si S y T son semejantes se escribe S T .
675 Teorema (Criterio AAA) Dos tringulos son semejantes si y slo si sus ngulos homlogos son congruentes.

Demostracin: Slo es necesario mostrar la suficiencia.


= A,
= B
= C.
b B
b Superpngase al A BC sobre el ABC.
byC
Considrese el tringulo A BC en donde A
Como los ngulos coinciden, los lados homlogos son paralelos. El resultado ahora se sigue del teorema 670.

676 Teorema (Criterio LLL) Dos tringulos son semejantes si y slo si sus lados homlogos son proporcionales.

Demostracin: El teorema es resultado inmediato de la presuncin 671.


La siguiente presuncin es ahora inmediata.

Proporcionalidad y semejanza

131

677 Presuncin (Criterio LAL) Dos tringulos son semejantes si y slo dos de sus lados lados homlogos son proporcionales

y los ngulos comprendidos entre estos lados son congruentes.


Las siguientes aseveraciones son ahora evidentes.
678 Presuncin Si dos figuras tienen razn de semejanza 1 : , entonces:

1. ngulos homlogos son congruentes.


2. segmentos de recta homlogos llevan una razn de 1 : .
3. reas homlogas llevan una razn de 1 : 2 .
4. volmenes homlogos llevan una razn de 1 : 3 .
679 Teorema Si las rectas AB y PQ se intersecan en M entonces

[ABP]
PM
=
.
[ABQ] QM
Demostracin: Se observan cuatro casos, como en las figuras 7.108 a 7.111. Sin prdida de generalidad presmase que las reas involucradas no son degeneradas. As
[ABP]
[ABP] [AMP] [AMQ]
AB PM AM
PM
=

=
.
[ABQ] [AMP] [AMQ] [ABQ]
AM QM AB
QM

bM

Q
Figura 7.108: Teorema 679.

Figura 7.109: Teorema 679.

Figura 7.110: Teorema 679.

Figura 7.111: Teorema 679.

680 Ejemplo ABCD es un cuadriltero convexo tal que DA y CB se intersecan en K; AB y DC se intersecan en L; AC y KL se


intersecan en G; DB y KL se intersecan en F. Demostrar que

KF
KG
=
.
FL
GL
Resolucin: Aplicaciones sucesivas del teorema 679 producen
KF
[DBK] [DBK] [KBL] CD AK [ACD] [ACK] [ACK] KG
=
=

=
=
.
FL
[DBL]
[KBL] [DBL] CL AD
[ACL] [ACD]
[ACL]
GL

681 Ejemplo Sea P un punto en el interior del ABC. Las semirrectas AP, BP, CP intersecan a los lados BC, CA, AB en los
puntos A , B , C respectivamente, como en la figura 7.113. Demostrar que

PA PB PC
+
+
= 1.
AA BB CC

132

Captulo 7
Resolucin: Se tiene
1=

[ABP] + [BCP] + [CAP] [ABP] [BCP] [CAP] PC PA PB


=
+
+
=
+
+
,
[ABC]
[ABC] [ABC] [ABC] CC AA BB

gracias al teorema 679.


b

D
b

b b

G
L

Figura 7.112: Ejemplo 680

682 Ejemplo Tres lneas rectas, cada una de ellas paralelas a los lados del ABC, concurren en el punto M (figura 7.114). Si

las reas de
resultantes dentro del ABC son [EKM] = R, [MQF] = S y [PMN] = T , demustrese que
los tres
tringulos

[ABC] = ( R + S + T )2 .
Resolucin: Obsrvese que EKM MQF PMN. Entonces
R
EM 2
S
MF 2
T
PN 2
=
,
=
,
=
.
[ABC]
AC2 [ABC]
AC2
[ABC] AC2
Luego

EM =

R
AC, MF =
[ABC]

S
AC, PN =
[ABC]

T
AC.
[ABC]

A causa del paralelismo de rectas, EM = AP, y MF = NC. Esto conlleva a


EM + PN + MF = AP + PN + NC = AC.
De esta ltima igualdad se desprende que

R
AC +
[ABC]

S
AC +
[ABC]

T
AC = AC,
[ABC]

de donde se colige, al cancelar AC y resolver para [ABC] que

[ABC] = ( R + S + T )2 .

683 Ejemplo En el ABC, A , B , C son puntos en BC, CA, AB respectivamente tales que

AC
BA CB 1
= = = .

C B AC B A 3
K, L, M son las intersecciones de las rectas AA y CC ; BB y AA ; CC y BB , respectivamente. Si el rea del ABC es 1,
encuentre el rea del KLM.

Proporcionalidad y semejanza

133

Resolucin: Observe que


1 = [ABC] = [ABL] + [LBC] + [ALC].
s
Si [ABL] = s, por el teorema 679, se tiene [LBC] = y [ALC] = 3s. As,
3
s
3
s + + 3s = 1 = s = .
3
13
3
Con razonamientos semejantes se obtiene [BCM] = [CAK] = . Luego
13
[KLM] = [ABC] [ABL] [BCM] [CAK] =

4
.
13

K
b

A
b

Figura 7.113: Ejemplo 681.

B
b

bQ

b
b

K
b

bM

B
Figura 7.114: Ejemplo 682.

Figura 7.115: Ejemplo 683.

684 Teorema (Ptolomeo) Sea ABCD un cuadriltero. Entonces se cumple la desigualdad

AC BD AB CD + BC DA.

Igualdad ocurre si y solamente si ABCD es un cuadriltero cclico simple, esto es, si AC y BD se cortan en el interior del crculo.
Demostracin: Sea E el punto nico en el plano tal que ABE y ADC sean directamente semejantes, esto
EB
AB
CD
es, que existe una homotecia directa enviando ABE a ADC. Se tiene
=
, de donde BE = AB
.
CD
AD
AD
= BAD
y AE = AC , de donde los tringulos ACE y ADB son semejantes, y se
Por otra parte, se tiene EAC
AB
AD
BD
sigue que CE = AC
. De la desigualdad triangular aplicada al BCE se tiene CE CB + BE, con igualdad
AD
si y solamente si C, B, E son alineados, en este orden. En reemplazando BE y CE por los valores obtenidos, se
= ABE
= ADC,
esto es, A, B,C, D son
halla la desigualdad pedida. Igualdad ocurre si y solamente si ABC
co-cclicos en este orden.
685 Teorema (Potencia de un punto con respecto a un crculo) Considrese un crculo de centro O y radio r y un punto P

en el plano. Dada cualquier recta pasando por P y cortando el crculo en A y B, el producto PA PB depende solamente de P y
del crculo y no de la recta. Si A = B se considera la recta tangente al crculo en A.
Demostracin: Considrese cualquier otra recta pasando por P y cortando al crculo en C y D, como en la figura
7.117. Se tiene
= BAC
= BDC
= PDB.

PAC
As, los tringulos PAC y PDB son semejantes, de donde
PA
PC
=
,
PD PB
de donde se sigue el resultado.

bA

134

Captulo 7

El producto PA PB se llama la potencia de P con respecto al crculo. Se tiene


(OP + r)(OP r) = OP2 r2 .
Considrense dos crculos de centros O y O, como en la figura 7.118, de rayos respectivos r1 y r2 . El conjunto
{P R2 : PO2 r12 = PO2 r22 }

es una recta perpendicular a la recta OO , como se puede verificar en utilizando el teorema de Pitgoras, llamada el eje radical

de los crculos. Obsrvese que si los crculos se cortan en dos puntos A y B entonces su eje radical es la recta AB. Si los crculos
son tangentes en A entonces su eje radical es la tangente que los separa.
686 Teorema (Teorema de los ejes radicales) Considernse tres crculos 1 , 2 , 3 . Entonces sus ejes radicales 1 , 2 , 3

son o bien se confunden, o bien concurrentes, o bien paralelos.


Demostracin: Un punto en dos de los ejes radicales tiene la misma potencia con respecto a los tres crculos. Por
lo tanto, si dos de los ejes se confunden, tambin lo hace el tercero. Si dos de los ejes tienen un punto en comn,
este punto yace en el tercer eje tambin.

b
b

C
B

b
b

E
Figura 7.116: Teorema 684.

Figura 7.117: Teorema 685.

Figura 7.118: Eje radical.

Tarea

687 Problema En la figura 7.119, AD BC = E y [AB] k [EF] k [CD]. Demostrar que

Figura 7.119: Problema 687.

1
1
1
=
+
.
EF
AB CD

688 Problema ABCD es un paralelogramo. El punto E est sobre la recta CD ms all


de D. Se traza el segmento [BE], intersecando [AD] en F y la diagonal [AC] en G. Demostrar que
1
1
1
=
+
.
BG
BF
BE

689 Problema ABCD es un trapecio en el cual AB = 7 y CD = 10. Si E yace sobre

[AD] y F sobre [BC], y si

AE
BF
=
= 2, hallar EF.
ED
FC

Tarea

135

690 Problema El ABC tiene lados que 13, 14 y 15 unidades. El A BC est dentro
del ABC con lados paralelos al ABC y a 2 unidades de distancia de los lados de ste.
Hallar el rea del A BC .

691 Problema En el tringulo agudo ABC, considrese la altura [AHA ] y la mediana

A corta el segmento [BC] en D. Si AB = 11, AC = 8 y


[AMA ]. La bisectriz angular de b
ma = 1, hallar MA HA .

692 Problema En la figura 7.120, ABC es rectngulo en A y ADB es rectngulo en

D. El punto E es el punto de interseccin de los segmentos [AD] y [BC]. Si AC = 15,


AD = 16 y BD = 12, hllese el rea del ABE.

bM

b
b

Figura 7.122: Problema 696.

697 Problema En el ABC, se trazan paralelas a los lados [AC] y [AB] a travs de un

punto M que yace sobre el lado [BC]. El rea del paralelogramo resultante es

del ABC. Hallar la razn en que M divide al lado [BC].

E
b

5
del rea
18

698 Problema Un penacho triangular esta coloreado en verde (252 unidades), rojo (90

bD

unidades), magenta (120 unidades), cinico (105 unidades), amarillo y azul, como en la
figura 7.123, donde se presume que las cevianas concurren. De cuntas unidades cuadradas est pintado en azul y amarillo?

Figura 7.120: Problema 692.


b
693 Problema (AIME, 1992) Los puntos A , B ,C estn en los lados BC, CA y AB res-

pectivamente, del ABC. Dado que AA , BB y CC , concurren en O y que la suma


AO
BO
CO
AO BO CO
+
+
= 92, hllese el producto

.
OA
OB
OC
OA OB OC

694 Problema (Canad, 1971) [DB] es una cuerda de un crculo, E es un punto sobre
esta cuerda para el cual DE = 3 y EB = 5. Sea O el centro del crculo. nase [OE] y
extindase [OE] de tal manera que corte al crculo en C, como en la figura adjunta. Dado
que EC = 1, hallar el radio del crculo.

C
D

b
b

E
b

Figura 7.123: Problema 698.

699 Problema En la figura 7.124, cada uno de los tringulos ABC, FDC, GEC es

issceles. Adems AB = 3AC. El permetro del ABC es 84. D es el punto medio del segmento [BC]; E es el punto medio del segmento [DC]; F es el punto medio del segmento
[AC] y G es el punto medio del segmento [FC]. Hllese el permetro del cuadriltero
sombreado DEGF.

O
B

D
E

Figura 7.121: Problema 694.

695 Problema Se construye, exteriormente, cuadrados en cada lado del cuadrilte-

ro ABCD siendo P, Q, R, S los centros de los respectivos cuadrados. Demustrese que


PR = QS y que PR QS.

696 Problema En la figura 7.122, ABCD es un cuadrado, [AN] k [LC] y [OB] k [DM].

Adems, AL = MB = 2. Hallar el rea, en unidades cuadradas, de la regin sombreada


cruzada dentro del cuadrado.

F G

Figura 7.124: Problema 699.

700 Problema En el ABC, E y F yacen sobre el [AB], con E entre A y F, como en la

figura 7.125. Se satisface adems

AE : EF : FB = 1 : 2 : 3.

136

Captulo 7
Figura 7.125: Problema 700.

Los puntos G y D yacen sobre [CB] con G entre C y D. Se satisface


CG : GD : DB = 4 : 3 : 2.
Si[FG] interseca al [ED] en H, hallar la razn DH : HE.

b b

H K

D
b

3x

M
b

4x

2y

3y

D
b
b

tos medios de los lados [BC], [CD] y [AD], respectivamente. M es el punto medio del
a
segmento [QR]. Sea b el rea del tringulo APM. Hllese la fraccin .
b

2x

F
E

701 Problema En la figura 7.126, el rectngulo ABCD de rea a, P, Q y R son los pun-

Figura 7.126: Problema 701.

7.7. Construcciones con regla y comps


Se recogen aqu algunas construcciones fundamentales.
702 Construccin Dado un segmento de recta, copiarlo.

Resolucin: Para copiar el segmento [AB] en el segmento congruente [PQ], se observarn los siguientes pasos:
1. Mrquese un punto P, que ser un extremo de la copia.
2. Pngase una punta del comps sobre A.
3. Ajstese la anchura del comps, cosa de que la otra punta descanse sobre B. La anchura del comps es ahora
AB.
4. Sin cambiar la anchura del comps, pngase una punta sobre P.
5. Sin cambiar la anchura del comps, trcese un arco de centro P.
6. Escjase un punto Q sobre el arco, que ser el otro extremo del segmento.
7. Trcese una recta de P a Q.
8. Se concluye AB = PQ.

703 Construccin Dado un ngulo, copiarlo.


es dado y se le quiere copiar en el ngulo QPR.

Resolucin: El ngulo CAB

1. Pngase P en el lugar deseado y trcese una recta a travs de P en la direccin deseada.


2. Abra el comps a la anchura de la distancia AC y sin ajustarlo, pngalo sobre P y trcese un arco sobre la
recta. Llmese Q al punto de interseccin de la recta y el arco.
3. Con la punta del comps sobre A, pngase el otro extremo del comps sobre B, la anchura del comps ahora
siendo AB.
4. Sin ajustar el comps, ponga la punta sobre P y trace un arco sobre la recta.
5. Ponga la punta del comps sobre C y el otro extremo sobre B, siendo ahora la anchura del comps la distancia
CB.
6. Sin ajustar el comps, ponga la punta sobre Q y trace un arco cruzando el arco previamente trazado. Llmese
R a la interseccin de los arcos.

Construcciones con regla y comps

137

7. Usando la regla, trace el segmento de recta [PR].


= CAB.

8. Se tiene QPR

704 Construccin Dado un tringulo, copiarlo.

Resolucin: Dado el tringulo ABC, construir PQR, con PQR


= ABC.
1. Ponga el vrtice P en cualquier lugar deseado.
2. Ponga la punta del comps en A y el otro extremo en B, el comps tiene ahora una anchura AB.
3. Trace un arco con centro P y radio AB cerca de donde se quiera colocar el vrtice Q.
4. Marque un punto Q en el arco. Observe que PQ = AB.
5. Ponga la punta del comps en B y el otro extremo en C, el comps tiene ahora una anchura BC.
6. Trace un arco con centro Q y radio BC cerca de donde se quiera colocar el vrtice R.
7. Ponga la punta del comps en C y el otro extremo en A, el comps tiene ahora una anchura CA.
8. Trace un arco con centro P y radio CA cerca de donde se quiera colocar el vrtice R.
9. La interseccin de estos ltimos dos arcos es el vrtice R.
La construccin funciona porque PQR
= ABC ya que PQ = AB, QR = BC y RP = CA.

705 Construccin Dada una recta L y un punto R no en L , construir, con regla y comps, una recta L pasando por R y

paralela a L .

Resolucin: Sean P y Q puntos sobre L , e tal manera que PQ = L . Para trazar PQ k RS, se observarn los
siguientes pasos:
1. Trcese una recta pasando por R y cortando a PQ en un punto arbitrario que se llamar J.
2. Con el comps abierto un poco ms de la mitad de la distancia entre R y J, pngase un punto sobre J y

trcese un arco cortando a PQ en A y a RJ en B.


3. Sin cambiar la anchura del comps, colquese una punta del comps sobre R y trcese un arco, tal como en

el segundo paso, cortando a RJ en B .


4. Ajstese ahora el comps tan ancho como la distancia AB.
5. Pngase una punta del comps sobre B . Trcese un arco ahora que cruce el arco existente de centro R en el
punto S.

6. Trcese la recta RS

7. PQ k RS.
RS =
La construccin funciona porque la igualdad de los ngulos correspondientes B
BJA.

706 Construccin Construir la mediatriz de un segmento.

Resolucin: Para trazar la mediatriz del segmento [PQ], se observarn los siguientes pasos:
1. Pngase una punta del comps sobre P.
2. Abra el comps a un poco ms de la mitad de la distancia entre P y Q.
3. Sin cambiar la anchura del comps, trcense arcos por encima y por abajo del segmento con P como centro.

138

Captulo 7
4. Sin cambiar la anchura del comps, pero ahora con Q como centro, trcense arcos por encima y por abajo
del segmento, cortando al previo arco superior en A y al arco inferior previo en B.

5. Trcese la recta AB

6. AB es la mediatriz de [PQ].
La construccin funciona ya que PAQB es un rombo con diagonales [AB], [PQ] y las diagonales de un rombo son
mutuamente perpendiculares.

707 Construccin Dada una recta y un punto sobre ella, erigir una perpendicular a la recta desde el punto.

Resolucin: Se erige sobre el punto K de la recta AB, una recta perpendicular a AB pasando por K.
1. Abra el comps a una anchura promedio, en realidad no importa cuanto.
2. Sin cambiar la anchura del comps, marque dos arcos en la recta, en ambos rayos terminando en K, cortando

a AB en P y Q. Note que P y Q equidistan de K.


3. Ensanche el comps hasta casi el doble de su actual anchura.
4. Con centro en P marque ahora un arco encima de K.
5. Sin cambiar la anchura del comps, ponga una punta del comps sobre Q y marque ahora un arco encima K
con centro en Q. Este arco interseca ahora el arco previo con centro en P en el punto R.


6. Una ahora K con R. Entonces KR AB.

708 Construccin Construir una perpendicular a una recta que pase por un punto fuera de la recta.

Resolucin: Sea R el punto externo a la recta AB. Se construir una perpendicular a AB desde R.
1. Poner una punta del comps sobre R.

2. Abra el comps a un poco ms de la mitad de la distancia de R a AB.

3. Trace dos arcos con centro en R sobre AB a entrambos lados de R, intersecando a AB en P y Q.


4. Con la misma anchura en el comps, trace dos arcos uno con centro en P y otro con centro en Q, intersecndose en S.

5. La recta RS es la perpendicular a AB buscada.

709 Construccin Dividir a un segmento de recta en n sub-segmentos congruentes.

Resolucin: Para dividir el segmento [AB] en n partes iguales:


1. Desde el punto A, dibujse una recta en un ngulo agudo con el segmento de recta. La inclinacin exacta de
la recta no importa en tanto el ngulo sea agudo.
1
2. Ponga la punta del comps en el punto A y abra el comps a una anchura un poco menor que de la longitud
n
del nuevo segmento de recta.
3. Comenzando desde A, trace n arcos consecutivos sobre el nuevo segmento, creando n puntos sobre ella. El
centro del k-simo arco es el k 1-simo punto. Llmese al ltimo punto C.
4. Con el comps abierto a la anchura BC trace un arco con centro en A debajo de A.
5. Con el comps abierto a la anchura AC trace un arco centro en C intersecando al arco del paso anterior.
Llmese D al punto de interseccin.

Construcciones con regla y comps


6. Trace el segmento [DB].
7. Usando la misma anchura que la utilizada para trazar arcos en el segmento AC, trcense ahora n 1 arcos
consecutivos comenzando en D a lo largo de [DB].
8. nanse los puntos correspondientes a lo largo de [AC] y [DB].
9. Esto divide a [AB] en n partes iguales.

710 Construccin Dado un ngulos, construir su bisectriz angular.

Resolucin: Se desea bisecar al ngulo PQR.

1. Ponga el comps sobre el vrtice Q del ngulo.


2. Ajuste el comps a una anchura promedio. La anchura exacta no importa.
3. Sin alterar la anchura del comps, trace un arco en cada rayo del ngulo, intersecando al rayo [QP[ en A y
al rayo [QR] en B.
4. Ponga la punta del comps en A y trace un arco en el interior del ngulo. Haga ahora lo mismo poniendo la
punta en B. Llame a la interseccin de los arcos el punto S
5. Trace una recta desde el vrtice hasta el punto de interseccin de los arcos.
6. [QS] es la bisectriz buscada.
= BQS.

La construccin funciona porque QAS


= QBS, ya que QA = QB, AS = BS, QS = QS y por lo tanto AQS

711 Construccin Construir el incentro e el crculo inscrito de un tringulo dado.

Resolucin: Dado el ABC construir I, el centro de su crculo inscrito y r, su radio.


1. Construya las bisectrices de dos de los ngulos del tringulos utilizando la construccin 710. La interseccin
de las bisectrices es el incentro I.
2. Desde I, construya una perpendicular a cualquiera de los lados, dgase [AB], utilizando la construccin 708.
El punto de interseccin de esta perpendicular y el lado se denominar L. Entonces r = IL.
3. Trcese el crculo de centro I y radio r.

712 Construccin Construir el circuncentro y el circuncrculo de un tringulo dado.

Resolucin: Dado el ABC construir O, el centro de su circuncrculo y R, su radio.


1. Escogiendo cualesquiera dos de los lados, constryase la mediatriz de cada lado, utilizando la construccin
706.
2. Sea O el punto de interseccin de estas dos mediatrices.
3. Pngase la punta del comps en O y el otro extremo en cualquier vrtice del tringulo, por ejemplo, en A.
Entonces R = OA.
4. Trcese el crculo de centro O y radio R.

713 Construccin Dado crculo, hallar su centro.

139

140

Captulo 7
Resolucin:
1. Con una regla, trcense dos cuerdas, tan paralelas como sea posible, en el crculo.
2. Construya la mediatriz de cada uno de las cuerdas, utilizando la construccin 706.
3. El punto de interseccin de las mediatrices es el centro del crculo.

714 Construccin Dado un punto fuera de un crculo y un crculo, construir las tangentes al crculo desde el punto.

Resolucin: Se considera un crculo de centro O y un punto P fuera del crculo.


1. Trcese el segmento OP.
2. Hllese el punto medio de OP. Llmese M a este punto medio.
3. Trcese el crculo con centro M y radio MO. Observe que este crculo interseca al crculo de centro O en dos
puntos, que se denominarn A y B.
4. A y P son los puntos de tangencia desde P.
La construccin funciona porque POA es rectngulo en A y POB es rectngulo en B. As el radio OA es

perpendicular a la recta PA y por lo tanto PA es tangente al crculo de centro O. Se arguye de la misma manera
para el punto B.
715 Construccin Construir las tangentes mutuas de dos crculos.

Resolucin: Considrense crculos de centro O, O y de respectivos radios R, R . Presmase que R > R y en el


caso contrario, cmbiense los roles de O y O .
1. Constryase un crculo de centro O y radio R R.
2. Utilizando la construccin 714, construir tangentes a este crculo.
3. Se desplazan ahora estas tangentes R unidades arriba y abajo de los centros. Esto resultar en dos tangentes
externas a los crculos originales, que existen si OO > R R.
4. Se construye ahora un crculo de centro O y radio R + R .
5. Utilizando la construccin 714, construir tangentes a este crculo.
6. Se desplazan ahora estas tangentes R unidades arriba y abajo de los centros. Esto resultar en dos tangentes
internas a los crculos originales, que existen si OO > R + R .

716 Construccin Construir la cuarta proporcional de tres segmento de recta dados.

Resolucin: Dados segmentos de longitud a, b y c, se quiere hallar x tal que


x a
= .
c b
1. Construya dos rectas suficientemente largas formando un ngulo conveniente, intersecndose en un punto,
que se denominar O.
2. En una de las rectas, localicse un punto A tal que OA = a.
3. En la misma recta, localicse un punto B tal que AB = b.
4. En la otro recta, localicse un punto C tal que OC = c.
5. nase A y C, formando el segmento [AC].

Tarea

141

6. Usando la construccin 705, constryase una paralela a [AC] pasando por B. Esta recta intersecar a OC
en un punto, que se denominar X.
7. Trcese [BX].
8. Se tiene x = CX.
La construccin funciona gracias al teorema 669.

717 Construccin Dados un segmento de longitud a y otro de longitud b construir un segmento de longitud

ab.

Resolucin:
1. Sobre una recta, escjase un punto arbitrario O. Localicse un punto A tal que OA = a.
2. Localicse un punto B tal que AB = b.
3. Localicse el punto medio M del segmento [OA]. Trcese un semicrculo de centro M y radio MO.

4. Erjase una perpendicular a OB en A. Esta recta intersecar al semicrculo en un punto, que se denominar
X.

5. AX = ab.

Tarea
718 Problema Construir un tringulo equiltero con regla y comps, si la longitud de

sus lados es dada.

719 Problema Dada una recta y dos puntos del mismo lado de ella, construir un crculo
que pase por los dos puntos y tangente a la recta.

7.8. Repaso de Trigonometra


C
C

b
a

Figura 7.127: Notacin para tringulos.

b
c

Figura 7.128: Tringulo rectngulo.

Dado el tringulo ABC, la medida del segmento opuesto al vrtice se indicar con la minscula del vrtice correspondiente.
As pues, la medida del segmento [AB] es AB = c, la del segmento [BC] es BC = a y la del segmento [CA] es CA = b, como se
indica en la figura 7.127. Tanto el ngulo en el vrtice V como su medida algebraica se indicar por V .

142

Captulo 7

720 Definicin Se ABC un tringulo rectngulo en A, como en la figura 7.128. Se definen entonces las razones trigonomtricas seno, coseno, tangente, secante, cosecante y cotangente del ngulo B de la manera siguiente:

sen B

cos B

tan B

csc B

sec B

cot B

longitud del cateto opuesto a B


longitud de la hipotenusa
longitud del cateto adyacente a B
longitud de la hipotenusa
longitud del cateto opuesto a B
longitud del cateto adyacente a B
longitud de la hipotenusa
longitud del cateto opuesto a B
longitud de la hipotenusa
longitud del cateto adyacente a B
longitud del cateto adyacente a B
longitud del cateto opuesto a B

b
a
c
a
b
c
a
b
a
c
c
b

=
=
=
=
=
=

=
=
=
=

sen B
cos B
1
sen B
1
cos B
cos B
sen B

Las razones trigonomtricas del ngulo C se definen de manera semejante:


sen C

cos C

tan C

csc C

sec C

cot C

longitud del cateto opuesto a C


longitud de la hipotenusa
longitud del cateto adyacente a C
longitud de la hipotenusa
longitud del cateto opuesto a B
longitud del cateto adyacente a C
longitud de la hipotenusa
longitud del cateto opuesto a C
longitud de la hipotenusa
longitud del cateto adyacente a C
longitud del cateto adyacente a C
longitud del cateto opuesto a C

=
=
=
=
=
=

sen

cos

tan

1
2
2
2

3
2

3
2
2
2
1
2

3
3

c
a
b
a
c
b
a
c
a
b
b
c

=
=
=
=

sen C
cos C
1
sen C
1
cos C
cos C
sen C

cos B

sen B

cot B

sec B

csc B

tan B

Cuadro 7.1: ngulos especiales.

Los valores del seno, coseno y tangente de 0 = 0, 30 = , 45 = , 60 = y 90 = , son deducidos de inmediato al


6
4
3
2
considerar varios tringulos equilteros o issceles. Son utilizados frecuentemente y vale la pena el memorizarlos. Consltese
el cuadro 7.1.

Repaso de Trigonometra

143

Como es sabido, estas razones trigonmetricas se extienden a un ngulo de magnitud arbitraria mediante las siguientes
relaciones.
721 Teorema (Frmulas de simetra) Sea un ngulo arbitrario. Las funciones seno y tangente son impares:

sen( ) = sen ;

tan( ) = tan .

La funcin coseno es par:


cos( ) = cos .
722 Teorema (Frmulas de periodicidad) Sea un ngulo arbitrario, medido en radianes y n Z un entero arbitrario. En-

tonces

sen( + 2n ) = sen ;

cos( + 2n ) = cos ;

tan( + n ) = tan .

723 Teorema (Frmulas de complementareidad) Sea un ngulo arbitrario, medido en radianes. Entonces

sen

= cos ;

cos

= sen ;

tan

= cot .

En el tringulo ABC de la figura 7.128 se cumple la relacin de Pitgoras


b 2 + c2 = a 2 ,
de donde se obtiene el teorema siguiente.
724 Teorema (Frmulas Pitagricas) Sea un ngulo arbitrario. Entonces

sen2 + cos2 = 1;

1 + tan2 = sec2 ;

cot2 + 1 = csc2 .

725 Ejemplo Dado que

3 sen x + 4 cosx = 5,
encontrar sen x y cos x.
Resolucin: Se tiene
3 sen x + 4 cosx = 5 sen x =

5 4 cosx
.
3

Utilizando las relaciones pitagricas se obtiene

cos2 x +

5 4 cosx
3

=1

25 40 cosx + 16 cos2 x
=1
9

cos2 x +

9 cos2 x + 25 40 cosx + 16 cos2 x = 9

25 cos2 x 40 cosx + 16 = 0

(5 cos x 4)2 = 0

cos x =

4
5

Adems,
sen x =

5 4 cosx 5 16
3
5
=
= .
3
3
5

144

Captulo 7

726 Teorema (Frmulas de rea) Sea ABC un tringulo arbitrario. Dentese por [ABC] el rea del ABC. Entonces

[ABC] =

ab sen C bc sen A
ca sen B
=
=
.
2
2
2

Demostracin: Dentese por HA , HB y HC los pies de las perpendiculares a los segmentos [BC], [CA] y [AB],
respectivamente. Dentese por ha , hb y hc las medidas de las alturas [AHA ], [BHB ] y [CHC ], respectivamente.
Entonces
chc
[ABC] =
.
2
Pero de la figura 7.8 se ve que hc = a sen B y por tanto,
[ABC] =

chc ca sen B
=
,
2
2

probando una de las frmulas. Las otras frmulas se obtienen en permutando los lados.
727 Teorema (Frmulas de la adicin) Sean y ngulos arbitrarios. Entonces

sen( ) = sen cos sen cos ,


cos( ) = cos cos sen sen ,
tan( ) =

tan tan
.
1 (tan )(tan )

Demostracin: Tan slo se demostrar que


sen( + ) = sen cos + sen cos ,

> 0,

> 0,

+ < ,

C y = H

dejando el resto a cargo del lector. En la figura 7.8, C = + , con = ACH


CCB. Luego

ab sen( + )
2

[ABC]

[AHCC] + [CHC B]

ahc sen hc b sen


+
,
2
2

lo cual implica que


sen( + ) =

hc
hc
sen + sen = sen cos + sen cos ,
b
a

como se quera demostrar.


728 Teorema (Regla del coseno de Al-Kashi) En el tringulo ABC se observan las siguientes relaciones:

a2 = b2 + c2 2bc cos A,

b2 = c2 + a2 2ca cos B,

c2 = a2 + b2 2ab cos C.

Repaso de Trigonometra
Demostracin:
Pitgoras

145

Presmase primero que 0 < B , como en la figura 7.8. Obsrvese que por el teorema de
2
c2

(AHC + HC B)2

(AHC )2 + (HC B)2 + 2(AHC )(HC B)

AHC )
b2 (hc )2 + a2 (hc )2 + 2(b cos A)(c

b2 (hc )2 + a2 (hc )2 + 2bc cos A 2AHC b cos A

b2 + a2 + 2bc cos A 2(AHC )2 2(hc )2

b2 + a2 + 2bc cos A 2b2

b2 + a2 + 2bc cos A,

lo cual implica

a2 = c2 + b2 2bc cos A.

Presmase ahora que B > , como en la figura 7.130. Se tiene


2
c2

(AHC BHC )2

(AHC )2 + (HC B)2 2(AHC )(HC B)

b2 (hc )2 + a2 (hc )2 2(AHC c)(b cos A)

b2 (hc )2 + a2 (hc )2 + 2bc cos A 2bAHC cos A

b2 + a2 + 2bc cos A 2(AHC )2 2(hc )2

b2 + a2 + 2bc cos A 2b2

b2 + a2 + 2bc cos A,

lo cual implica

a2 = c2 + b2 2bc cos A.
Las otras identidades se obtienen en permutando los lados.
729 Teorema (Frmula de Hern) Si s =

a+b+c
es el semi-permetro del ABC, entonces su rea viene dada por
2
[ABC] =

s(s a)(s b)(s c).

Demostracin: Se tiene
1

[ABC] = ab sen C,
2
Ahora bien, sen2 C = 1 cos2 C,
de donde
y as 16[ABC]2 = 4a2 b2 sen2 C.

16[ABC]2 = 4a2b2 (1 cos2 C).

146

Captulo 7
Por la regla del coseno,
(c2 a2 b2 )2
.
cos2 C =
4a2 b2
De aqu se obtiene
16[ABC]2

= 4a2 b2 (c2 a2 b2 )2
= (2ab c2 + a2 + b2)(2ab + c2 a2 b2)
= ((a + b)2 c2 )(c2 (a b)2)
= (a + b + c)(a + b c)(c + a b)(c a + b)
= (2s)(2s 2c)(2s 2b)(2s 2a),

y al dividir por 16 y tomar races cuadradas se obtiene el resultado.


C

bC

b
hc

HC c

hc

B HC

Figura 7.129: Frmulas de rea.

Figura 7.130: Regla de Al-Kashi.

730 Teorema (Regla del seno) En el tringulo ABC se observan las siguientes relaciones:

a
b
c
=
=
.

sen A sen B sen C


Demostracin: Se considerarn por separado los casos cuando B es agudo u obtuso.

Presmase primero que 0 < B , como en la figura 7.8. Entonces


2
b
a
a sen B = hc = b sen A =
=
.

sen A sen B
c
Que estas dos cantidades son iguales a
se observa al considerar, por ejemplo, la altura BHB .
sen C

Presmase ahora que B > , como en la figura 7.130. Se tiene


2

= hc = b sen A.
a sen( B)
Pero por las frmulas de la adicin
= a sen cos B sen B cos = a sen B
a sen( B)
y as,
a sen B = b sen A =
obteniendo nuevamente el resultado.

a
b
=
,

sen A sen B

Repaso de Trigonometra

147

731 Ejemplo Hllese el valor exacto de cos

2
y cos .
5
5

Resolucin: Considrese un pentgono regular ABCDE. Sea x la longitud de uno de sus lados. Recurdese que
la seccin urea satisface

1+ 5
.
> 0,
=
= =

1+
2
= CED
y por lo tanto
Dentese por F el punto de interseccin de [AC] and [BE]. Como [AC] k [DE] , FCE
FCD DEC. As FC = CD = x. Obsrvese que FAB es issceles y semejante al FCE. Poniendo t = AF
y observando que CE = CA = x + t, se tiene
x
FA
BA
t
x
1
x
=
= =
= x = t x = = .
FC CE
x t +x
1
+
t
t
t

= CED
y BCA
= FCE,
se tiene BCA
= FCE
= CED
= 1 3 = . Esto quiere decir que
Ahora bien, como FCE
3 5
5
3

FCE =
y luego ABF = FAB = . Erigiendo una perpendicular desde F hasta [AB], se deduce del FAB,
5
5

1+ 5
2
cos = =
= =
.
5
t
2t
2
4

De aqu se sigue, en observando que 2 = + 1,


 2
2

2
2 2 1
= 2 cos2 1 = 2
1 =
1 =
=
=
cos
5
5
2
2
2
2

Cabe adems notar que

3
2
cos
= cos
5
5

51
.
4

2
1 5
= cos
=
.
5
4

Figura
731.

7.131:

Ejemplo

Figura
732.

7.132:

Ejemplo

732 Ejemplo Considrese un pentgono regular inscrito en un crculo de radio 1, y a su vez, el pentagrama regular obtenido al
unir alternadamente los vrtices del pentgono, como en la figura 7.132.

1. Calcule el rea del pentgono.


2. Calcule el permetro del pentgono.
3. Calcule el rea del pentagrama.

148

Captulo 7
Resolucin: El rea del pentgono es 5 veces el rea de cualquier tringulo formado por el centro del pentgono y dos de sus vrtices consecutivos. Ntese que tal tringulo es issceles, con ambos lados iguales de medida
2
. As pues, el rea del pentgono es
1 y con el ngulo entre estos lados de medida
5
1
2
5 1 1 sen .
2
5
Por el ejemplo 731,
2
sen
=
5

1 sen2

de donde el rea del pentgono es

2
=
5

51
4

5+ 5
=
,
8

25 + 5 5
.
16

Para obtener el permetro del pentgono, sea x la medida de uno de sus lados. Considerando el tringulo issceles
formado al unir los extremos de este lado con el centro del pentgono, y viendo que los lados iguales miden 1, se
obtiene, usando la regla de los cosenos, que
2
x2 = 12 + 12 2(1)(1) cos
= 22
5

51
4

5 5
5 5
=
= x =
.
2
2

C
b

Figura 7.133: Teorema de la bisectriz angular.

733 Teorema (Forma trigonomtrica del teorema de la bisectriz) En el ABC sea P un punto en la recta BC. Entonces

BP BA sen BAP
=
.
PC CA sen PAC

Demostracin: sese la ley de los senos en el APB:


BP

sen BAP

y en el APC:

PC

sen PAC

AB

sen APB

AC

sen CPA

Repaso de Trigonometra

149

Luego
sen APB

BP AB sen BAP/
=
.
PC
sen CPA

AC sen PAC/
+ CPA
= , se tiene sen APB
= sen CPA.
Cancelando
Ya que APB

BP AB sen BAP
=
.
PC

AC sen PAC

734 Ejemplo El cuadrado ABCD tiene lado 1. Se trazan cuatro cuartos-de-crculo con centros en los vrtices del cuadrado y

de radio 1. Hallar el rea del cuadriltero curvilneo formado por la interseccin de los arcos de crculo. Vase la figura 7.134.
Resolucin: Obsrvese
que AF = AE = 1, siendo radios de crculos unitarios. Adems, si O es el centro de la
2
es trisecado por [AE] y [AF]. El rea del sector circular
diagonal [AC], AO =
y AOE = porque el arco BD
2
6
AFE es as
1

(1) = .
2
6
12
r

Ahora bien, de | sen x| =

1 cos2x
, se obtiene
2

sen
=
12
El rea del AEO es

1 cos 6
=
2

2 3
.
2

2 2 3
1
2

31

1 sen
=
=
.
2 2
12
8
8

El rea buscada es finalmente

2
12

31

= 3 + 1.
8
3

E
b

b
b

b
b

Figura 7.134: Ejemplo 734.

Figura 7.135: Teorema 735.

150

Captulo 7

735 Teorema (Morley) Los pares adyacentes de las trisectrices de los ngulos de un tringulo siempre se encuentran en los

vrtices de un tringulo equiltero.


b = 3a, B
b = 3c. Sea R el circunradio del ABC. Por la ley de los senos,
b = 3b y C
Demostracin: Pngase A
BC = 2R sen(3a). Aplicando ahora dicha ley al BPC,

BP/ sen(c) =
=

BC/ sen( b c) = 2R sen(3a)/ sen(b + c)

2R sen(3a)/ sen( a),


3

de donde
BP = 2R sen(3a) sen(c)/ sen(

(7.2)
(7.3)

a) .
3

Combinando esto con la identidad anterior,


sen(3a) = 4 sen(a) sen(

+ a) sen( a)
3
3

y as se obtiene
BP = 8R sen(a) sen(c) sen(

+ a) .
3

BR = 8R sen(c) sen(a) sen(

+ c) .
3

De igual manera,

Por la ley de los cosenos,


PR2 = BP2 + BR2 2BP BR cos(b)

= 64 sen2 (a) sen2 (c)[sen2 ( + a) + sen2 ( + c) 2 sen( + a) sen( + c) cos(b)] .


3
3
3
3
Pero como
(
se sigue que
sen2 (b) = sen2 (

+ a) + ( + c) + b = .
3
3

+ a) + sen2 ( + c) 2 sen( + a) sen( + c) cos(b),


3
3
3
3

de donde
PR = 8R sen(a) sen(b) sen(c) .
Como esta expresin es simtrica en a, b y c, se colige que
PR = RQ = QP
as claimed.

Tarea
736 Problema Dos crculos de radio R son tangentes externamente. Ambos crculos son

Figura 7.136: Problema 736.

tambin tangentes internamente a un tringulo rectngulo de catetos de longitud 3 y 4,


como en la figura 7.136. Hallar R.

b
737 Problema El ABC es tal que la altura desde A mide lo mismo que la suma de las

longitudes de las alturas desde B y C. Demostrar que

sen b
A

1
sen b
B

1
.
b
sen C

Tarea

151

738 Problema Demostrar que si un tringulo de rea A es tal que el producto de las

longitudes de dos sus medianas es

749 Problema Probar que cot x 2 cot 2x = tan x y que

3
A, entonces estas dos medianas son perpendiculares.
2

n
X

739 Problema Si , , son los ngulos internos de un tringulo, demostrar que

cot + cot + cot = cot cot cot .


2
2
2
2
2
2
740 Problema (Eotvos, 1897) Si , , son los ngulos internos de un tringulo arbitrario, demostrar que

1
(sen )(sen )(sen ) .
2
2
2
8
Cundo se verifica la igualdad?

1
tan k = n cot n 2 cot 2 .
2
2
2k
2

k=0

750 Problema (AHSME, 1991) El tringulo equiltero ABC se ha doblado de tal manera que ahora el vrtice A descansa sobre el punto A del segmento [BC], tal como en la
figura 7.138. Si BA = 1 y AC = 2, hllese la longitud del doblez PQ.

741 Problema Sean , , , los ngulos internos de un tringulo. Dado que

tan( ) + tan( ) + tan( ) = 0,

demostrar que el tringulo es issceles.


742
Problema Un rombo tiene lados de longitud s y diagonales de longitud d, d . Si

s=

dd , hallar la medida del ngulo agudo entre los lados del rombo.

743 Problema Dos rombos congruentes son sobrepuestos de tal manera que la diago-

nal mayor del uno yace sobre la diagonal menor del otro. La interseccin resulta en un
octgono convexo de lados congruentes. Determine la razn de la diagonal mayor a la
diagonal menor de tal manera que el octgono sea regular.
b

b
b
b

751 Problema Demostrar que en el ABC se satisface

sen2 A + sen2 B = sen2 C

b
b

Figura 7.138: Problema 750.

si y solamente si ABC es un tringulo rectngulo.

752 Problema Sean , ngulos agudos. Demostrar que

sen2 + sen2 = sen2 ( + ),

b
si y slo si + =

Figura 7.137: Problema 743.

.
2

753 Problema Hallar el valor exacto de


744 Problema Demostrar que

sen2 1 + sen2 2 + + sen2 90 .

(1 + tan 1 )(1 + tan 2 ) (1 + tan 44 )(1 + tan 45 ) = 223 .


745 Problema Sean , , reales tales que + + = . Demstrese que

1 cos + cos + cos = 4 sen


y que
tan

cos cos ,
2
2
2

tan + tan tan + tan tan = 1.


2
2
2
2
2
2

754 Problema Sea x + y + z = 90 . Determnese

(tan x)(tan y) + (tan y)(tan z) + (tan z)(tan x).


755 Problema Si a + b + c = 36 , demostrar que

tan 5a + tan 5b + tan 5c (tan 5a)(tan 5b)(tan 5c) = 0.


746 Problema Calclese tan p tan q. Dedzcase
n
X

Sn =

756 Problema En el ABC, probar que

1
cos(k ) cos((k + 1) )

ab
sen A senB
=
.
a+b
sen A + senB

k=1

757 Problema Si en el ABC, a = 5, b = 4 y cos (A


B) =

con R.

1
cos C = and that c = 6.
8

747 Problema Demostrar que

tan 51 cot 51 = 2 tan 12 .

758 Problema Sea ABCDEFG un heptgono regular. Considrense las distancias =

AB, = AC, = AD. Demostrar que

748 Problema Demostrar que


n1
X

k=0

3k sen3

3k+1

1
4

3n sen

31
, demostrar que
32

sen .
3n

1
1
1
= + .

152

Captulo 7

7.9. Repaso de Geometra Analtica


En esta seccin se desarrollarn las ecuaciones cannicas de crculos y rectas en el plano.
759 Definicin El plano cartesiano R2 es el conjunto de pares ordenados

R2 = {(x, y) : x R, y R}.
Aqu x es la abscisa y3 y la ordenada. Un punto del plano A es un par ordenado de nmeros reales A = (a1 , a2 ).

B(x2 , y2 )

|y2 y1 |
b

A(x1 , y1 )

(x0 , y0 )

|x2 x1 |

C(x2 , y1 )
Figura 7.140: El crculo.

Figura 7.139: Distancia entre dos puntos.

Considrese dos puntos A = (x1 , y1 ), B = (x2 , y2 ) en el plano, como en la figura 7.139. Construyendo los segmentos [CA] y
sec BC con C = (x2 , y1 ), se puede hallar la longitud del segmento [AB], esto es, la distancia de A a B, en utilizando el Teorema
de Pitgoras:

AB2 = AC2 + BC2 = AB =

(x2 x1 )2 + (y2 y1 )2 .

Esto motiva la siguiente definicin:


760 Definicin Sean A(x1 , y1 ), B(x2 , y2 ) puntos en el plano cartesiano. La distancia eucldea entre A y B, o la longitud o norma

del segmento [AB], es la cantidad

AB = dh(x1 , y1 ), (x2 , y2 )i =

(x1 x2 )2 + (y1 y2 )2 .

(7.4)

Gracias a la frmula de la distancia se deduce el teorema siguiente.


761 Teorema La ecuacin cartesiana cannica de un crculo de radio R > 0 y centro (x0 , y0 ) es

(x x0 )2 + (y y0)2 = R2 .

(7.5)

Demostracin: El punto (x, y) pertenece a un crculo de radio R > 0 si y slo si su distancia al centro del crculo
es R. Luego se requiere

dh(x, y), (x0 , y0 )i


(x x0 )2 + (y y0)2

(x x0 )2 + (y y0)2

obteniendo el resultado. Vase la figura 7.140.


3

R ,

R2

Se lee la letra como ye y no i griega. Por tanto se utiliza la conjuncin y y no e.

Repaso de Geometra Analtica

153

762 Definicin Tres puntos A, B,C son colineales si los tres estn la misma recta.
763 Definicin El punto B est entre los puntos A y B si los tres puntos son colineales y

AB + BC = AC.
Sean a y b constantes reales. El conjunto
{(a, y) R2 : y R}
tiene abscisa constante, y recorre paralelamente al eje de y. De igual manera, el conjunto
{(x, b) R2 : x R}
tiene ordenada constante, y recorre paralelamente al eje de x. Esto conlleva a la siguiente definicin.
764 Definicin La ecuacin cannica de una recta vertical es x = a, en donde a R es una constante. La ecuacin cannica
de una recta horizontal es y = b, en donde b R es una constante.

(x1 , y1 )

b
y y1

(x, y)

y2 y1

(x2 , y2 )

b
x x1

x2 x1
Figura 7.141: Recta vertical.

Figura 7.142: Recta horizontal.

Figura 7.143: Teorema 765.

Se determinar ahora la ecuacin de una recta inclinada.


765 Teorema La ecuacin cartesiana de toda recta no vertical en el plano es de la forma y = mx + k, en donde m R y k R

son constantes.
Recprocamente, los puntos del plano ligados por la ecuacin y = ax + b, en donde a, b son constantes reales, forman una
recta.
Demostracin: Si la recta fuere paralela al eje de x, esto es, si fuere horizontal, entonces su ecuacin sera de la
forma y = b, end donde b es una constante real y entonces se puede tomar m = 0 y k = b. Considrese ahora una
recta no paralela a ninguno de los ejes, como en la figura 7.143, y pertenezcan los puntos (x, y), (x1 , y1 ), (x2 , y2 ) a
dicha recta. De tringulos semejantes se observa que
y2 y1 y y1
=
,
x2 x1 x x1
y masajeando un poco se colige que

y=

y2 y1
y2 y1
x x1
+ y1 ,
x2 x1
x2 x1

154

Captulo 7
de donde se puede tomar

m=

y2 y1
y2 y1
, k = x1
+ y1 .
x2 x1
x2 x1

Recprocamente, considrense nmeros reales x1 < x2 < x3 . Pngase P = (x1 , ax1 + b), Q = (x2 , ax2 + b) y R =
(x3 , ax3 + b), que pertenecen a la grfica de ecuacin y = ax + b. Se demostrar que
dhP, Qi + dhQ, Ri = dhP, Ri,
esto es, los puntos P, Q y R son colineales. Como los puntos P, Q, R son arbitrarios, se deduce pues que la grfica
de la ecuacin y = ax + b es una recta. En efecto, se tiene

dhP, Qi =

(x2 x1 )2 + (ax2 ax1 )2 = |x2 x1 | 1 + a2 = (x2 x1 ) 1 + a2,

dhQ, Ri =
dhP, Qi =

(x3 x2 )2 + (ax3 ax2 )2 = |x3 x2 | 1 + a2 = (x3 x2 ) 1 + a2,

(x3 x1 )2 + (ax3 ax1 )2 = |x3 x1 | 1 + a2 = (x3 x1 ) 1 + a2,

de donde
dhP, Qi + dhQ, Ri = dhP, Ri.

y2 y1
en el teorema 765 es la pendiente o gradiente de la recta que pasa por (x1 , y1 ) y
x2 x1
(x2 , y2 ). Como y = m(0) + k, el punto (0, k) es el intercepto del eje de y de la recta que pasa por (x1 , y1 ) y (x2 , y2 ).
766 Definicin La cantidad m =

Una recta horizontal tiene pendiente 0. Una recta vertical tiene pendiente infinita.
De las figuras 7.144 y 7.145 se puede percatar que si la recta L tiene ecuacin cartesiana L : y = mx + k y si es el ngulo
que L forma con la parte positiva del eje de x, entonces
m = tan ,
Si =

, entonces m = .
2

[0; [.

(7.6)

767 Definicin Sean A y B puntos distintos en el plano. La direccin de la recta AB es el ngulo que AB forma con la parte

positiva del eje de x.


Las observaciones anteriores proporcionan de inmediato el siguiente teorema.
768 Teorema Dos rectas son paralelas si y solamente si tienen la misma pendiente.

La condicin para perpendicularidad es un poco ms difcil de demostrar.


769 Teorema Dos rectas inclinadas son perpendiculares si y slo si el producto de sus pendientes es 1.

1
Demostracin: Se quiere demostrar que L1 : y = mx +k es perpendicular a L2 : y = m1 x +k1 si y slo si m1 = .
m
Refirase a la figura 7.146. Ya que el trasladar rectas no afecta el ngulo entre ellas, se podr asumir sin prdida
de generalidad que tanto L1 : y = mx + k como L2 : y = m1 x + k1 se intersecan en (0, 0), en cuyo caso k = k1 = 0.
Ahora bien, la recta y = mx se encuentra con la recta vertical x = 1 en (1, m) y la recta y = m1 x se encuentra con
esta misma vertical en (1, m1 ). Si las rectas fuesen perpendiculares el Teorema de Pitgoras dara
(m m1 )2 = (1 + m2) + (1 + m21),

Repaso de Geometra Analtica

155

que simplificando resulta en mm1 = 1.


La recproca se obtiene de la recproca del Teorema de Pitgoras.

y = mx

(1, m)

(1, m1 )
y = m1 x

Figura 7.144: ngulo con el eje de x.


.

Figura 7.145: ngulo con el eje de x.


.

Figura 7.146: Teorema 769.


.

Se desarrollar ahora una serie de resultados que culminarn en la demostracin de la desigualdad triangular en el plano.
770 Lema (Desigualdad de Cauchy) Sean a, b, x, y nmeros reales. Entonces

(ax by)2 (a2 + b2)(x2 + y2 ),


con igualdad si y slo si (a, x) y (b, y) son proporcionales.
Demostracin: Se tiene
(a2 + b2 )(x2 + y2 ) (ax by)2

a2 x2 + a2 y2 + b2 x2 + b2 y2 a2 x2 2axby b2y2

a2 y2 + b2 x2 2abxy

(ay bx)2

0.
a x
= , esto es (a, x) es proporcional a (b, y). Si b = 0,
b y
entonces ay = 0x, y, o bien a = 0 o bien y = 0. En uno y otro caso es
Igualdad ocurrir si y slo si ay = bx. Si by 6= 0, entonces

(a2 + b2 )(x2 + y2 ) = (ax by)2,


y se obtiene el resultado. Si y = 0 es tambin claro el resultado.
771 Lema Pngase A = (a1 , a2 ), B = (b1 , b2 ) y C = (c1 , c2 ). Entonces A, B y C son colineales si y slo si (c2 b2 , b2 a2 ) y
(c1 b1, b1 a1 ) son proporcionales.

Demostracin:
= Pase la recta de ecuacin y = mx + k por A, B y C. Luego
a2 = a1 x + k,

b2 = b1 x + k,

c2 = c1 x + k,

156

Captulo 7
y as
b2 a2 = m(b1 a1 ),

c2 b2 = m(c1 b2 ) = (c2 b2 , b2 a2 ) = m(c1 b1 , b1 a1 ),

estableciendo la proporcionalidad.
Supngase ahora que (c2 b2, b2 a2 ) y (c1 b1 , b1 a1 ) son proporcionales, dgase
(c2 b2 , b2 a2) = t(c1 b1, b1 a1 ).
Hay dos casos, o bien 0 = c1 b1 = b1 a1 en cuyo caso c1 = b1 = a1 y A, B, C estn alineados en la misma
recta vertical al tener todos la misma abscisa, o bien uno de entre c1 b1 y b1 a1 no es cero. Spongase
que c1 b1 6= 0, siendo el caso b1 a1 6= 0 de manejo idntico. Se tiene pues
c2 b 2
= t,
c1 b 1

b2 a2 = t(b1 a1)

Luego los puntos B y C pasan por la recta de pendiente t, dgase


y = tx + k.
Cuando x = b1 , deber ser y = b2 y as
b2 = tb1 + k = k = b2 b1t.
Luego B y C estn sobre la recta de ecuacin
y = tx + b2 b1t.
Falta ahora demostrar que A tambin est en esta recta. Pero para x = a1 se tiene
y = a1t + b2 b1t = b2 t(b1 a1 ) = b2 (b2 a2) = a2 ,
que quiere decir que (a1 , a2 ) = A tambin est sobre la recta, terminando la demostracin.

772 Teorema (Desigualdad del tringulo) Sean A, B, C puntos en el plano (no necesariamente colineales). Entonces

AB + BC AC.
La igualdad ocurre si y slo si los tres puntos son colineales.
Demostracin: Pngase A = (a1 , a2 ), B = (b1 , b2 ) y C = (c1 , c2 ). Por el lema 770,

(c1 b1)(b1 a1) + (c2 b2 )(b2 a2 )

((c1 b1 )2 + (c2 b2)2 )((b1 a1 )2 + (b2 a2 )2 ).

Igualdad ocurre si y slo si (c2 b2 , b2 a2 ) y (c1 b1 , b1 a1 ) son proporcionales, y en vista del lema 771,
igualdad ocurre si y slo si A, B y C son colineales.
Ahora bien,
AC2

(c1 a1 )2 + (c2 a2 )2

(c1 b1 + b1 a1)2 + (c2 b2 + b2 a2 )2

(c1 b1 )2 + 2(c1 b1 )(b1 a1 ) + (b1 a1 )2 + (c2 b2 )2 + 2(c2 b2 )(b2 a2 ) + (b2 a2 )2

(c1 b1 )2 + (c2 b2 )2 + 2(c1 b1 )(b1 a1 ) + 2(c2 b2 )(b2 a2 ) + (b1 a1 )2 + (b2 a2 )2

(c1 b1 )2 + (c2 b2 )2 + 2 ((c1 b1)2 + (c2 b2)2 )((b1 a1 )2 + (b2 a2 )2 ) + (b1 a1 )2 + (b2 a2 )2

=
=

(c1 b1 )2 + (c2 b2 )2 +
(AB + BC)2,

(b1 a1 )2 + (b2 a2 )2

2

Tarea

157
demostrando el teorema. Vanse las figuras 7.147 y 7.148.

Figura 7.148: AB + BC > AC.

Figura 7.147: AB + BC = AC.

Tarea
773 Problema Osama la cucaracha comienza a viajar desde el punto (1, 1) y quiere
llegar al punto (2, 1). En cada cuadrante y tambin, en los ejes, se mueve a razn de 1
de distancia por minuto, excepto en el segundo cuadrante, en donde se mueve a razn de
media unidad de distancia por minuto. Qu ruta deber tomar Osama para minimizar el
tiempo del recorrido? La respuesta no es una recta de (1, 1) a (2, 1)!

774 Problema (Desigualdad de Minkowski) Demustrese que si (a, b), (c, d) son

Considrese
Sn = mn

n
X
p

(2k 1)2 + a2k ,

k=1

en donde el mnimo es tomado sobre todas las particiones P. Demustrese que exactamente una de entre S2 , S3 , . . . ,Sn , . . . es un entero y hllese cual lo es.

puntos en el plano, entonces

(a + c)2 + (b + d)2

a2 + b2 +

777 Problema Al parmetro real arbitrario t, asciese la recta Lt de ecuacin

c2 + d 2 .

Lt :

Ocurre igualdad si y slo si ad = bc.


775 Problema (Generalizacin de la desigualdad de Minkowski) Sea ninguno de

los puntos del plano (ak , bk ), 1 k n sobre los ejes. Entonces

n
X
p

a2k + b2k

k=1

Igualdad ocurre si y slo si

ak

bk

Lt es paralela a al eje de x.

Lt es paralela al eje de y.

k=1

Lt es paralela a la recta de ecuacin x 2y 6 = 0.

a2
an
a1
=
= = .
b1
b2
bn

1
Lt es normal (perpendicular) a la recta de ecuacin y = x 5.
4

776 Problema (AIME, 1991) Sea P = {a1 , a2 , . . . ,an } una coleccin de puntos con

0 < a1 < a2 < < an < 17.

En los casos siguientes , encuentre el valor de t y Lt que satisface las condiciones estipuladas.
Lt pasa por (2, 3).

n 2 n 2
X
X
k=1

(t 2)x + (t + 3)y + 10t 5 = 0.

Existe acaso un punto (x0 , y0 ) que pertenece a todas las Lt , para no importa qu
valor de t?

7.10. Vectores

778 Definicin Dados dos puntos A(a1 , a2 ) y B(b1 , b2 ) en el plano, el vector AB asociado al segmento dirigido [AB] es
2
3

6b1 a1 7
7.
AB = 6
4
5
b2 a2

158

Captulo 7

Obsrvese tambin que AB = BA.

u
b

2
u

B
b

u +
v
b

C
1

u
2

Figura 7.149: Suma de vectores.


Figura 7.150: Multiplicacin escalar de vectores.

779 Ejemplo Dos segmentos diferentes pueden asociarse al mismo vector. Por ejemplo, si A = (1, 2), B = (3, 4), C = (0, 1),

D = (2, 3), entonces

2 3

637
7
AB = 6
4 5 = CD.
2

Se interpreta pues la nocin de vector como un conjunto de instrucciones: si el vector


2 3
6a7

7
u =6
4 5,
b

la instruccin es desplazarse a unidades en el eje de x, y b unidades en el eje de y. Esto es, un vector es un objeto
que nos indica magnitud (norma), direccin (en las coordenadas del vector) y sentido (desde el punto inicial hasta
el punto final).
2 3

607
7
780 Definicin El vector cero es 0 = 6
4 5.
0
781 Definicin Se llamar escalar a todo nmero real R.
782 Definicin Dados vectores

6u1 7

7
u =6
4 5,
u2

2 3

v1 7

v = 6
6 7,
4 5
v2

Vectores

159

y el escalar R, se define la suma de vectores

v = 6
6
u +
4

u 1 + v2 7
u 2 + v2

y la multiplicacin de vectores por un escalar

7,
5

6 u1 7
7.
5

u =6
4

u2

v se busca
Ntese que geomtricamente la suma y el producto de vectores se interpreta de manera siguiente. Para sumar
u a

un segmento arbitrario [AB] tal que AB = u . Luego se busca al (segmento nico) [BC] tal que v = BC. Finalmente, la suma se
obtiene por la regla del paralelogramo

v =
u +
AB + BC = AC.
Vase la figura 7.149. La multiplicacin de vectores por un escalar tiene interpretacin semejante. Vase la figura 7.150.
Lo arriba indicado pone en evidencia la Regla de Chasles.
783 Teorema (Regla de Chasles vectorial) Dados tres puntos arbitrarios A, B,C en el plano,

AB + BC = AC.
Se notarn aqu, para futura referencia, algunas propiedades del lgebra vectorial. La demostracin es inmediata, al apelar
a las coordenadas de los vectores.

784 Teorema (lgebra de vectores) Sean


a , b ,
c , vectores en el plano y sean y escalares. Entonces se cumplen las

siguientes propiedades:

Conmutatividad
Asociatividad

Distributividad
Distributividad

a + b = b +
a

(7.7)

(
a + b )+
c =
a +( b +
c)

(7.8)

a + 0 =
a + 0 =
a

(7.9)


a tal que
a + (
a ) = (
a )+
a = 0

(7.10)

(
a + b ) =
a + b

(7.11)

( + )
a =
a +
a

(7.12)

( )
a = (
a)

(7.13)



785 Definicin La norma o longitud del vector AB es simplemente AB = (b1 a1 )2 + (b2 a2)2 = AB. Si A 6= B, la

direccin del vector AB es la direccin de la recta AB, esto es, el ngulo [0; [ que la recta AB hace con el eje de x.

160

Captulo 7

Obsrvese que si R, entonces


y gracias a la desigualdad del tringulo, 772

||
u || = | | ||
u ||

(7.14)

v || .
||
u +
u || ||
u || + ||

(7.15)

Si u 6= 0 , entonces el vector ||uu || tiene norma 1 y la misma direccin del vector u . El vector nulo 0 tiene

norma 0 y no posee direccin.4

786 Definicin A un vector de norma 1 se le llama vector unitario.

787 Definicin
Sea
u =
6 0 . Pngase R
u = {
u : R} y sea A R2 , A = (a1 , a2 ). La recta afn con vector director
2 3
6u 1 7

7
u =6
4 5 y pasando por A es el conjunto de puntos en el plano
u2

A + R
u = {(x, y) R2 : x = a1 + tu1,

y = a2 + tu2,

t R}.

Si u1 = 0, la recta afn arriba definida es vertical, ya que x es constante. Si u1 6= 0, entonces


x a1
(x a1 )
= t = y = a2 +
u2 ,
u1
u1
esto es, la recta afn es una recta cartesiana con pendiente
entonces

2 3

u2
. De manera semejante, si y = mx + k es una recta cartesiana,
u1

2 3

2 3

6x 7 6 1 7
60 7
6 7 = 6 7t + 6 7,
4 5 4 5
4 5

k
2 3

617
7
esto es, toda recta cartesiana es tambin una recta afn, y se puede tomar al vector 6
4 5 como su vector director.
m

v son paralelos si las rectas R

v son paralelas. De aqu, dos vectores

v si
788 Definicin Dos vectores
u y
u y R
u y

v . Se indicar que

v mediante la notacin

v .

existe un escalar R tal que


u =
u es paralelo a
u k

0 es paralelo a todo vector.

v .
> 0 tal que u = v . El vector v tiene sentido opuesto al vector
u si existe algn escalar < 0 tal que
u =

v tiene el mismo sentido que el vector

789 Definicin Dado un vector


u 6= 0 , se dice que el vector
u si existe algn escalar

790 Definicin Sean A y B puntos en el plano y


u un vector unitario. Si AB =
u , entonces es la distancia dirigida o

medida algebraica del segmento [AB] con respecto al vector


u . Se denotar esta por AB
u , o si el vector u se sobrentiende,
por AB. Obsrvese que AB = BA.
La medida algebraica ser particularmente til al considerar razones de longitudes de segmentos. Considrese tres puntos

colineales A y B, A 6= B, fijos y X variando sobre la recta AB. Supngase, sin perdida de generalidad, que yacen sobre una recta
4

Y por eso canta: No soy de aqu, ni soy de all, no tengo edad, ni porvenir. . . .

Vectores

161

horizontal, como en la figura 7.151, siendo positiva la direccin de AB. As pues, si X est estrictamente a la izquierda de A (y
por consiguiente, a la izquierda de B), entonces AX < 0, XB > 0. Si X est estrictamente entre A y B, entonces AX > 0, XB > 0.
Finalmente, si X est estrictamente a la derecha de B (y por consiguiente, a la izquierda de B), entonces AX > 0, XB < 0.
Ntese que si X = M, el punto medio del segmento [AB], entonces AX = XB. Si X = A, entonces AX = 0 y XB
. = AB. Si X = B,
AX
1. La grfica de la funcin
entonces AX = AB y XB
. = 0. Cuando X ,
XB
X 7

AX
XB

aparece en la figura 7.152.

AX > 0, XB > 0

AX > 0, XB < 0

AX < 0, XB > 0

tal).

AX
XB

X =B

X =A

Figura 7.152: Razn

Figura 7.151: Distancias dirigidas.

(eje vertical) contra X (eje horizon-

Sea X AB, X 6= A, X 6= B. Entonces


AX
XB
=
.
XB
AX
Los puntos A, B,C estn alineados si y slo si se cumple la Relacin de Chasles:
AB + BC = AC.

(7.16)

La relacin de Chasles muestra que


AA = AB + BA = AB AB = 0,
para cualquier punto A. Adems
AX = AP AX + PA = 0 PX = 0 X = P.

(7.17)

162

Captulo 7

791 Teorema (Teorema de Euler) Sean A, B,C, D cuatro puntos alineados. Entonces

AB CD + AC BD + AD BC = 0.
Demostracin: Utilizando la relacin de Chasles,
AB = AD + DB = DB DA,

AC = AD + DC = DC DA,

BC = BD + DC = DC DB.

De aqu,
AB CD + AC BD + AD BC

(DB DA) CD + (DC DA) BD + AD (DC DB)

DB CD DA CD + DC BD DA BD + AD DC AD DB

DB CD DA CD DC DB DA BD + AD DC + AD BD

0.

A
B
C

D
b

A
b

B
b

A
C

Figura 7.153: Teorema de Thales.

C
b

A
b

Figura 7.154: Corolario del teorema de Thales

792 Teorema (Teorema de Thales de Mileto) Sean D y D dos rectas distintas. Sean A, B,C puntos distintos en D, A , B ,C


puntos distintos en D , A 6= A , B 6= B , C 6= C , A 6= B, A 6= B . Sea AA k BB . Entonces


AC AC
AA k CC
=
.
AB A B

Demostracin: Refirase a la figura 7.10. Por un lado, por ser vectores unitarios en la direccin de una misma
recta,

AB AC
A B A C
=
;
=
.
AB AC
A B A C
Por otro lado, por la Regla de Chasles,




BB = BA + AA + A B = (A B AB) + AA .

Vectores

163

Ya que AA k BB , existe un escalar R tal que

A B = AB + AA .
Ensamblando estos resultados,


CC = CA + AA + AC

AC
AC 

AB + AA + AB + AA
A
B
 AB


AC AC

AC
AB
+
1
+

AA .

A B AB
A B

Como AA no es paralela a AB, la igualdad anterior revela que


AC AC

= 0,
AA k CC
AB A B
mostrando el teorema.5
Del teorema anterior, se obtiene de inmediato el siguiente corolario. (Vase la figura 7.10.)
793 Corolario Sean D y D dos rectas distintas, intersecndose en el punto nico C. Sean A, B, puntos en D, A , B , puntos en

D . Entonces

CB CB
AA k BB
=
.
CA CA

El clculo vectorial hasta ahora desarrollado es particularmente til al resolver problemas que demandan demostrar si tal
recta es paralela a otra recta, etc. Se vern algunos ejemplos.

794 Ejemplo Dado un pentgono ABCDE, hllese AB + BC + CD + DE + EA.

Resolucin: Utilizando la Regla de Chasles varias veces:





0 = AA = AB + BC + CD + DE + EA,
como se quera demostrar.
795 Ejemplo Sea ABC un tringulo en el plano. Demustrese que el segmento de recta que une a los puntos medios de dos
de los lados es paralelo al tercer lado y que mide la mitad de ste.
5

Si tres o ms paralelas (Si tres o ms parale-le-le-las)


Si tres o ms paralelas (Si tres o ms parale-le-le-las)
Son cortadas, son cortadas (por dos transversales, dos transversales)
Son cortadas, son cortadas (por dos transversales, dos transversales)
Si tres o ms parale-le-le-las Si tres o ms parale-le-le-las
Son cortadas, son cortadas Son cortadas, son cortadas...
Dos segmentos de una de estas,
dos segmentos cualesquiera
Dos segmentos de una de estas
son proporcionales
A los segmentos correspondientes de la oootraaa....
Les Luthiers.

164

Captulo 7

Resolucin: Sean los puntos medios de [AB] y [CA], MC y MB , respectivamente. Se demostrar que BC =

2MC MB . Se tiene 2AMC = AB y 2AMB = AC. As

BC =

BA + AC

AB + AC

2AMC + 2AMB

2MC A + 2AMB


2(MC A + AMB)

2MC MB ,

como se haba de demostrar.


796 Ejemplo En el ABC, sea MC el punto medio de [AB]. Prubese que

1
CMC =
CA + CB .
2

Resolucin: Como AMC = MC B, se tiene


CA + CB = CMC + MC A + CMC + MC B
=


2CMC AMC + MC B

2CMC ,

de donde se deduce el resultado.


797 Ejemplo Si las medianas [AMA ] and [BMB ] del tringulo no degenerado ABC se intersecan en el punto G, demostrar que

AG = 2GMA ;

BG = 2GMB .


Resolucin: Como el tringulo no es degenerado, las rectas AMA y BMB no son paralelas y por tanto se

encuentran en un punto, llammosle G. Luego AG y GMA son paralelos, y por lo tanto existe un escalar a tal que

AG = aGMA . De igual manera, existe un escalar b tal que BG = bGMB . Por el ejemplo 795 ,

2MA MB =

y as

BA


BG + GA

bGMB aGMA

bGMA + bMAMB aGMA ,

(2 b)MAMB = (b a)GMA.

Tarea

165

Al no ser ABC degenerado6, MA MB y GMA no son paralelos, de donde
2 b = 0,

b a = 0,

= a = b = 2.

798 Ejemplo En todo tringulo no degenerado ABC, las medianas concurren. El punto de concurrencia G se llama el bari-

centro o centroide del tringulo.

Resolucin: Sea G como en el ejemplo 797. Se tiene que demostrar que CMC tambin pasa por G. Encuntrense

las rectas CMC y BMC en G . Por el ejemplo ya citado,

AG = 2GMA ;

BG = 2GMB ;

BG = 2G MB ;

CG = 2G MC .

As,

GG =

Resulta pues que


GB + BG

2GMB + 2G MB


2(MB G + G MB )

2G G.

GG = 2GG = 3GG = 0 = GG = 0 = G = G ,

demostrando lo pedido.

Tarea
799 Problema Dados en el plano son el cuadriltero ABCD y el punto M. Sean

N , P , Q , R , respectivamente, los puntos medios de los lados [AB], [BC], [CD], [DA].
Constryase ahora puntos N, P, Q, R, que son las imgenes al ser M reflejado con respecto
a los puntos N , P , Q , R . Demostrar que NPQR es un paralelogramo. (Vase la figura
7.155.)

801 Problema Sean A, B,C, D cuatro puntos distintos en el plano, no tres de ellos alinea-

dos. Se dir que ABCD es un paralelogramo si AB = DC. Demostrar que las condiciones
siguientes son equivalentes:

bM

b
b
b

ABCD es un paralelogramo.

[AC] y [BD] se bisecan.

Db

N
R

lado [BC] interseca la diagonal [BD] en el punto M. De igual manera, La recta trazada a
travs de B y paralela al lado [DA] interseca la diagonal [AC] en el punto N. Demostrar


que MN k CD.

AB k DC y AD k BC.
802 Problema En un cuadriltero arbitrario no degenerado ABCD, M, N, P y Q son,

P
b

respectivamente, los puntos medios de los segmentos [AB], [CD], [BC] y [AD]. Sean E
y F los puntos medios de las diagonales [AC] y [BD] respectivamente. Demostrar que
EQFP es un paralelogramo.
803 Problema Sean I, A, B,C puntos en el plano, y sean A , B , C puntos simtricos a


A, B, C respectivamente, con respecto a I, esto es, AI = IA , etc. Demostrar que

Figura 7.155: Problema 799.



AB + AC + AB + AC = 2AA .
Demostrar tambin que para un punto arbitrario del plano M se cumple

800 Problema Dado un cuadriltero ABCD. La recta trazada a travs de A y paralela al


6

No suele frecuentar antros de vicio.

MA + MB + MC + MA + MB + MC = 6MI.

166

Captulo 7

804 Problema Sobre el lado [AB] del ABC resta el punto P, del cual se han trazado
rectas paralelas a las medianas AMA y BMA , que intersecan a los lados del tringulo en
los puntos A (sobre [BC]) y B (sobre [CA]). Sea E el punto medio de A B . Demostrar
que P, E y el baricentro G del ABC estn alineados.

[CD] y [DA] del cuadrado ABCD en E, F, K y L respectivamente. Demostrar que


EK = FL. (Figura 7.156.)

805 Problema Sean p, q enteros estrictamente positivos, y sea n = p + q. Sean

A1 , A2 , . . . ,An n puntos distintos (dos a dos). Demostrar que existe una recta L en el plano
tal que exactamente p de los puntos yacen en un semiplano de sta, y los otros q puntos
en el otro semiplano, ninguno de los puntos yaciendo en L.

E
806 Problema Sea S el punto medio del segmento finito AB, y sea M cualquier punto

que yace en la recta infinita que contiene al segmento AB. Demustrese que
MA2 + MB2 = 2SA2 + 2SM 2 .

Figura 7.156: Problema 807.


807 Problema Dos rectas mutuamente perpendiculares intersecan los lados [AB], [BC],

7.11. Baricentros
808 Lema Sean A1 , A2 , . . . , An n puntos en el plano. Sean a1 , a2 , . . . an n nmeros reales. A todo punto O del plano se le asocia
la suma ponderada de vectores

v = a
1 OA1 + a2 OA2 + + an OAn .

Entonces

v es independiente de O.
1. Si a1 + a2 + + an = 0,

2. Si a1 + a2 + + an = 1, el punto G tal que OG =
v es independiente de O.
Demostracin: Para algn otro punto O pngase

v = a 1 O A 1 + a 2 O A 2 + + a n O A n .

Por la regla de Chasles, OO = OAk O Ak y luego

v
v = a1 (OA1 O A1 ) + a2(OA2 O A2 ) + + an (OAn O An ) = (a1 + a2 + + an )OO .

v =
De aqu, si a1 + a2 + + an = 0,
v y v no depende de O.


v
Si a1 + a2 + + an = 1,
v = OO , entonces sean G y G puntos tales que OG =
v y OG = v . Luego

GG = GO + OO + O G = OO (
v v ) = 0 ,
lo cual establece que G es independiente de O.
809 Lema Sea (A1 , x1 ), (A2 , x2 ),. . . , (An , xn ), un sistema de puntos ponderados, con Ak R2 , xk R. Si x1 + x2 + + xn 6= 0

entonces existe un punto nico G tal que

x1 GA1 + x2 GA2 + + xn GAn = 0 .

Demostracin: Pngase
a1 =
Entonces

x1
,
x1 + x2 + + xn

a2 =

x2
,
x1 + x2 + + xn

a1 + a2 + + an =
y el resultado se deduce del lema 808.

...,

an =

x1 + x2 + + xn
= 1,
x1 + x2 + + xn

xn
.
x1 + x2 + + xn

Baricentros

167

810 Definicin El punto G definido por el lema 809 es llamado el baricentro de los puntos ponderados

(A1 , x1 ), (A2 , x2 ), . . . , (An , xn ),


denotado por

G = bar

A1

A2

An

x1

x2

xn

Si x1 = x2 = = xn = x, entonces el baricentro de (A1 , x), (A2 , x), . . . , (An , x) se conoce como iso-baricentro.
811 Lema El baricentro no cambia si se reemplazan sus coeficientes por coeficientes proporcionales, esto es, si k 6= 0 es una
constante real,

bar

A1

A2

An

x1

x2

xn

= bar

A1

A2

An

kx1

kx2

kxn

Demostracin: Por definicin del baricentro de los puntos ponderados (A1 , x1 ), (A2 , x2 ), . . . , (An , xn ) se tiene

x1 GA1 + x2 GA2 + + xn GAn = 0 ,


y esto implica que

(kx1 )GA1 + (kx2 )GA2 + + (xan )GAn = k 0 = 0 ,

de donde se destila el resultado.

El baricentro coincide con la nocin fsica de centro de gravedad, tambin llamado centro de masa. Esto es, si en el punto Ak se
pone una masa de ak unidades, el baricentro es el punto de equilibrio de todas estas masas.
812 Ejemplo Sean A y B dos puntos distintos del plano. El baricentro de (A, 1) y (B, 1) es el punto G del plano tal que

GA + GB = 0 .
1

Esto implica que AG = GB = AB y por lo tanto G es el punto medio del segmento [AB].
2
813 Ejemplo Sean A y B dos puntos distintos del plano y a + b 6= 0. El baricentro de (A, a) y (B, b) es el punto G del plano tal

que

Esto implica que AG =

aGA + bGB = 0 .
b

a

AB y BG =
BA. Obsrvese que G est en la recta AB.
a+b
a+b

El ejemplo anterior provee un algoritmo geomtrico para localizar el baricentro de dos puntos (A, a) y (B, b): se divide al
segmento [AB] en a + b partes iguales. El baricentro est a b unidades de A y a unidades de B. Vase la figura 7.157 para varios

ejemplos. De igual manera, si el punto X 6= B est sobre la recta AB, se observa que
XA


= k XA kXB = 0 XA kXB = 0 ,
XB
esto es, se puede interpretar a X como el baricentro de los puntos ponderados (A, 1) y (B, k). Se ve entonces que
[AB] = {tA + (1 t)B : t [0 ; 1]},

(7.18)

esto es, el segmento [AB] no es otra cosa que una coleccin de baricentros de sus extremos, con coeficientes que suman a 1.
Inclusive, se puede demostrar fcilmente que si ab 0, entonces aA + bB est entre los puntos A y B, yaciendo sobre [AB].

168

Captulo 7
G

(A, 1)

(B, 1)

(A, 2)

G
b

(B, 4)

(A, 1)

(B, 7)

Figura 7.157: Varios baricentros.

Se puede percatar entonces que para dos puntos A R2 , B R2 fijos, la coleccin de baricentros
{G R2 : G = aA + bB, a R, b R, a + b 6= 0}

no es otra cosa si no la recta AB.


Si Ak = (xk , yk ) y G = (g1 , g2 ) es el baricentro de los puntos ponderados (A1 , a1 ), (A2 , a2 ), . . . , (An , an ) entonces
2

2 3

6x1 g1 7
6 x2 g 1 7
6xn g1 7 607
7 +a2 6
7 + +an 6
7 = 6 7 = g1 = a1 x1 + a2 x2 + + an xn ,
a1 6
4
5
4
5
4
5 4 5
a1 + a2 + + an
y1 g 2
y2 g 2
yn g 2
0

g2 =

a 1 y1 + a 2 y2 + + a n yn
.
a1 + a2 + + an

En general, como el baricentro no cambia si se reemplazan sus coeficientes por coeficientes proporcionales, no se puede decir
que
(a1 , a2 , . . . , an )
son las coordenadas baricntricas, ya que no son nicas. Una manera de obtener unicidad es imponiendo una condicin extra,
tal como en la siguiente definicin.
814 Definicin (Coordenadas baricntricas) Si G es el baricentro de los puntos ponderados (A1 , a1 ), (A2 , a2 ), . . . , (An , an )

y
a1 + a2 + + an = 1,
entonces se dice que
(a1 , a2 , . . . , an )
son las coordenadas baricntricas de G con respecto a A1 , A2 , . . . , An y se escribe
G = a 1 A1 + a 2 A2 + + a n An .
815 Teorema (Asociatividad de baricentros) El baricentro de n puntos ponderados no cambia si se reemplaza una parte de

ellos por su baricentro (si existiere) guardando los mismos coeficientes de los puntos restantes.
Demostracin: Sea G el baricentro de los puntos ponderados (A1 , a1 ), (A2 , a2 ), . . . , (An , an ). Si a1 + a2 + +
am 6= 0, se puede definir el baricentro H de los puntos ponderados (A1 , a1 ), (A2 , a2 ), . . . , (Am , am ). Luego

a1 GA1 + a2GA2 + + anGAn = 0 ,

a1 HA1 + a2 HA2 + + an HAm = 0 .


Utilizando la regla de Chasles para 1 k m, GH = GAk Ak H, de donde

a1 GA1 + a2 GA2 + + a pGA p = (a1 + a2 + + am )GH,


de donde

(a1 + a2 + + am )GH + am+1 GA p+1 + + an GAn = 0

y por lo tanto G es tambin el baricentro de los n m + 1 puntos ponderados (H, a1 + a2 + + am ), (Am+1 , am+1 ),
. . . , (An , an ).

Baricentros

169

816 Ejemplo Por el teorema anterior, el baricentro de tres puntos no alineados (A, 1), (B, 1), (C, 1) se puede hallar de la
siguiente manera: primero se halla el baricentro (H, 2) de (B, 1) y (C, 1), que no es otra cosa si no el punto medio del segmento
[BC]. Luego se halla el baricentro de (A, 1) y (H, 2), que est a 2 unidades de A y 1 de H. Esto es, el baricentro de los tres
puntos (A, 1), (B, 1), (C, 1) es en efecto el baricentro del tringulo ABC. En otras palabras,

A B C

A H
= bar

bar
1

= G.
1

Vase la figura 7.158.

a>0
b<0
c<0

b
/

a>0
b>0
c<0

a>0
b>0
c>0

a<0
b>0
c<0

Figura 7.158: Baricentro de un tringulo.

a>0
b<0
c>0

a<0
b>0
c>0

a<0
b<0
c>0

Figura 7.159: aA + bB + cC, a + b + c = 1.

Si el tringulo ABC no es degenerado y si a + b + c 6= 0, los baricentros aA + bB + cC son puntos en el plano interiores,


sobre, o exteriores al ABC. Vase la figura 7.159. Los signos son determinados de la manera siguiente: para saber el signo
de a, considrese el punto A y la recta del tringulo opuesta a A. Esta recta divide al plano en dos semiplanos: uno conteniendo
al punto A y el otro no. En el semiplano que contiene al punto A se observa que a > 0, en el semiplano que no contiene a A se
observa que a < 0. Sobre la recta a = 0. De igual manera se determinan los signos de b y c.
La asociatividad de baricentros provee el siguiente criterio para alineacin de puntos: dados tres puntos ponderados (A, a),
(B, b), (C, c) con G como su baricentro. Si b + c 6= 0 y F es el baricentro de (B, b) y (C, c), entonces los puntos A, G, F estn
alineados.
Adems, la asociatividad de baricentros provee el siguiente criterio para concurrencia de rectas: tengan tres puntos ponderados (A, a), (B, b), (C, c) baricentro G. Si (a + b)(b + c)(c + a) 6= 0 y si
E es el baricentro de (A, a) y (B, b).
F es el baricentro de (B, b) y (C, c).
K es el baricentro de (A, a) y (B, b).

entonces las rectas CE, AF y BK son concurrentes en G.

170

Captulo 7

Tarea
821 Problema (Teorema de Menelao7 ) A un punto P que yaga sobre la recta determi-

817 Problema Dado un tringulo no degenerado ABC:

nada por un lado del ABC se le llama punto menelaico de este lado. Si el punto no es
un vrtice del tringulo entonces es un punto menelaico propio.
Sean L, M, N puntos menelaicos de los lados [BC], [CA] y [AB] del ABC. L, M, N
son colineales si y solamente si

Construir el baricentro K de (A, 1), (B, 2), (C, 3).


Construir el baricentro K de (A, 1), (B, 3), (C, 3).

Demostrar que AL k BC

BL CM AN

= 1.
LC MA NB

818 Problema ABCD es un trapecio, con [AB] k [DC]. Las rectas AD y BC se cortan en

822 Problema En el ABC sea L un punto en el lado [BC]. Demostrar que

E.

1. Si

EA
ED

BL

= x, demostrar que E es el baricentro de (A, 1) y (D, x).

LC

2. Demostrar que E es tambin el baricentro de (B, 1) y (C, x).


3. Demostrar que

denota la medida del ngulo dirigido, positiva si medido en sentido levgiro


Aqu LAC

823 Problema (Menelao trigonomtrico) Sean L, M, N puntos menelaicos de los lados


[BC], [CA] y [AB] del ABC. L, M, N son colineales si y solamente si

Deducir que E es el baricentro de (A, 1), (B, 1), (C, x) y (D, x).
4. Sea I el punto medio de [AB] y J el punto medio de [DC]. Sea F la interseccin
de las diagonales [AC] y [DB]. Demostrar que I, J, E, F estn alineados.

sen CBM
d sen A
d
sen BAL
CN

= 1.
sen M
d
d
sen LAC
BA sen N
CB

819 Problema ABC es rectngulo en A. Sea A el pie de la perpendicular desde A

hasta el lado [BC]. Pngase BC = a, CA = b, AB = c.


2

AB sen BAL

CA sen LAC

= C
d
y negativa en sentido dextrgiro, con la propiedad LAC
AL.

EA + EB xED xEC = 0 .

824 Problema Demostrar que las rectas tangentes al circuncentro de un tringulo en sus

vrtices se cortan en tres puntos alineados.

1. Demostrar que A es el baricentro de (B, b ) y (C, c ).


2. Sea L el punto medio de [AA ]. Demostrar que L es el baricentro de (A, a2 ),
(B, b2 ) y (C, c2 ).

825 Problema (Teorema de Desargues) Dos tringulos ABC y A BC se dicen



copolares si las tres rectas AA , BB y CC son concurrentes. Los tringulos son coaxia


les si los tres puntos L (interseccin de BC y BC ), M (interseccin de CA y C A ), N


(interseccin de AB y A B ) son colineales. Demostrar que dos tringulos en el plano son
copolares si y slo si son coaxiales.

820 Problema El ABC tiene todos sus ngulos agudos. Sea M un punto del lado [BC].

Pngase BC = a, CA = b, AB = c.

1. Demostrar que M es el baricentro de (B, [MAC]) y (C, [MAB]).

826 Problema (Teorema de Ceva, 1678) Una recta que pasa por el vrtice de un trin-

gulo se llama ceviana de este vrtice. La ceviana es propia si no coincide con un lado del
tringulo. Las tres cevianas [AA ], [BB ] y [CC ] de un ABC concurren si y slo si

2. Si M es el punto medio de [BC], a qu conclusin se llega?

3. Si AM es la bisectriz del B
AC, a qu conclusin se llega?

AC BA CB

= +1.
C B AC B A

4. Deducir que el baricentro de (A, a), (B, b), (C, c) es el punto de concurrencia
de las bisectrices del ABC.

7.12. Transformaciones geomtricas

827 Definicin Una traslacin por un vector


u es una funcin
R2

R2

7 M

T
u :

tal que MM =
u . El punto M se llama punto homlogo del punto M con respecto a la traslacin T
u.
es una funcin que no mueve ningn punto, esto es, la identidad. Adems se observa la ley de composicin
Se observa que T
0
1

conmutativa T

u T
v = T
u +
v = T
v +
u = T
v T
u y la ley de inversos T
u = T
u
Es fcil de verificar que la traslacin satisface las siguientes propiedades:

1. La traslacin enva a un segmento [AB] a otro A B , igual en longitud y paralelo a ste, siendo A homlogo a A y B
homlogo a B.

Transformaciones geomtricas

171

2. La traslacin transforma a una recta a otra paralela a la original.


3. La traslacin transforma un tringulo ABC en otro igual A BC siendo A , B ,C homlogos, respectivamente, a
A, B,C.
4. La traslacin transforma a un ngulo en otro ngulo, siendo los vrtices puntos homlogos y los lados de correspondientes
de los ngulos, paralelos.
5. La traslacin transforma a un crculo en otro igual, preservando los radios y siendo los centros de los crculos puntos
homlogos.
828 Ejemplo Dadas las longitudes AB, BC, CD y DA, construir un trapecio ABCD, de bases AB y DC.
, con T
(D) =
Resolucin: Vase la figura 7.160. Supuesto resuelto el problema, considrese la traslacin T
DC
DC
(A) = M. Obsrvese que CM = DA, es longitud conocida. As pues, una posible construccin es la siguiente:
C, T
DC
conocidas las longitudes de los lados [CM], [MB] y [BC] del MBC, se construye este tringulo.
los
Para
obtener


vrtices A y D, se trasladan los puntos homlogos M y C por un vector de longitud mn( DC , AB ) y con

direccin y sentido del vector BM.


D
C
b

b
b

A
b

Figura 7.160: Ejemplo 828.

Figura 7.161: Ejemplo 829.

829 Ejemplo Sea un crculo dado y D una recta dada. Construir una cuerda de paralela a D de longitud l dada.

Resolucin: Vase la figura 7.161. Sea


u un vector director de D, de longitud l. Considrese la imagen de

mediante la traslacin T
u . Sean A y B las intersecciones de y . La recta paralela a D y pasando por A

cumple las condiciones del problema. En efecto, si T


u (A) = C, entonces AC = l y ||AC|| = l, como requerido. Si R
es el radio del crculo, ntese que hay dos soluciones si l < 2R, una solucin si l = 2R y ninguna si l > 2R.
830 Definicin Una rotacin (o giro) de centro O y ngulo de giro es una funcin

R2

R2

7 M

GO, :


tal que OM = OM y MOM
= . El punto M se llama punto homlogo del punto M con respecto a la rotacin GO, .
Se observa que GO,0 es una funcin que no mueve ningn punto, esto es, la identidad. Adems se observa la ley de composicin
conmutativa GO, GO, = GO, + = GO, + = GO, GO, y la ley de inversos GO, = G1
O, .

172

Captulo 7

Es fcil de verificar que la rotacin satisface las siguientes propiedades:




1. La rotacin enva a un segmento [AB] a otro A B , igual en longitud y paralelo a ste, siendo A homlogo a A, B


homlogo a B y (AB, A B ) = .
2. La rotacin transforma a una recta en otra, siendo el ngulo entre ellas .
3. La rotacin transforma un tringulo ABC en otro igual A BC siendo A , B ,C homlogos, respectivamente, a A, B,C.
4. La rotacin transforma a un ngulo en otro igual e igualmente orientado.
5. La rotacin transforma a un crculo en otro igual, siendo los centros de los crculos puntos homlogos.

60

b
b

60
b

L
L

b
b
b

B
b

N
Figura 7.162: Ejemplo 831.

Figura 7.163: Ejemplo 832.

831 Ejemplo Dado un punto A y dos rectas L y L , hallar un tringulo ABC, equiltero, que tenga el vrtice B sobre L y el

vrtice C sobre L .

Resolucin: Supuesto el problema resuelto, considrese la rotacin GA,60 . Se tendr GA,60 (B) = C. Cmo
encontrar C? Sea C la interseccin de la recta L y GA,60 (L). Pngase ahora GA, 60 (C) = B. Es fcil percatarse
que C y B cumplen las condiciones requeridas.
832 Ejemplo Considrese rectas paralelas L, M, N. Constryase un tringulo equiltero ABC tal que A yaga sobre L, B yaga

sobre M y C yaga sobre N.

Resolucin: Vase la figura 7.163. Fjese un punto A arbitrario de L. Considrese la recta GA,60 (M). Sea C la
interseccin de N y GA,60 (M). Pngase ahora GA,60 (C) = B.
833 Ejemplo Se construye, exteriormente, cuadrados en los lados AB y BC del tringulo ABC, siendo P y Q los centros de

los respectivos cuadrados, como en la figura 7.164. Sea M el punto medio del lado AB. Demustrese que PM = QM y que
PM QM.

Transformaciones geomtricas

173

Resolucin: Sean U y V las esquinas de los cuadrados opuestas a A y B, respectivamente. Ntese que una
rotacin de 90 en torno a C toma al ACV al UCB. Luego PV y UB son iguales y perpendiculares. En PUB
vemos que we see PM es la mitad de larga y paralela a UB. De igual manera, MQ es la mitad de larga y paralela
a PV . Por lo tanto, PM y MQ son iguales y perpendiculares.

bU
b

C
b

b
b

Figura 7.164: Ejemplo 833.

834 Definicin Una reflexin con respecto al punto O o simetra central de centro O es una funcin

R2
RO :
M


R2
7 M

tal que O es el punto medio de MM . El punto M se llama punto homlogo del punto M con respecto a la simetra central RO .

Una simetra central no es otra cosa que el giro G

O,180

y por lo tanto tiene las mismas propiedades de ste.


A

b b

B
b

E
Figura 7.165: Ejemplo 840.

Figura 7.166: Ejemplo 841.

835 Ejemplo Por un punto X, situado en el interior de un ngulo, trcese una recta cortando a los lados del ngulo en dos

puntos equidistantes de X.

174

Captulo 7
Resolucin: Constryase la recta , simtrica con respecto a X de la recta (uno de los lados del ngulo). Para
hacer esto, eljanse dos puntos cualesquiera de y constryanse sus simtricos con respecto a X, luego trcese la
recta que los une. Sea S la interseccin de con el otro lado del ngulo. Ntese que X est a igual distancia de
y . Constryase ahora S , homlogo a S con respecto a una simetra central de centro X. Obsrvese que S est
sobre . Esto resuelve el problema.

Dado un polgono de n lados, est claro que se puede construir otro polgono cuyos vrtices son los puntos medios M1 , . . . , Mn
de los lados. El siguiente ejemplo considera el problema recproco.
836 Ejemplo Dados los puntos M1 , . . . , Mn , existe acaso un polgono cuyos puntos medios de los lados son los Mk ?

Resolucin: Sea RMk una reflexin de centro Mk . Pngase RMk (Ak ) = Ak+1 , con An+1 = A1 . Una vez A1 es
elegido, los otros Ak quedan determinados. Se ver ahora como elegir a A1 .
Obsrvese que A1 es un punto fijo de la composicin de reflexiones centrales R = RMn . . . RM2 RM1 . Ntese
adems que ya que una reflexin central es un giro con ngulo de rotacin , un nmero par de composiciones es
efectivamente una traslacin, mientras que un nmero impar es una reflexin central.
As pues, si la composicin es impar, va existir un punto fijo, llammosle A. Se pondr entonces A1 = A. Si la
composicin es par, tal punto fijo no existe necesariamente y el problema no tiene solucin.

b
b

b
b

O
b

b
b

Figura 7.167: Ejemplo 835

Figura 7.168: Ejemplo 838

837 Definicin Una simetra axial con eje , o reflexin con respecto a la recta es una funcin

R2

R2

7 M

S :


tal que es la mediatriz de MM . El punto M se llama punto homlogo del punto M con respecto a la simetra axial S .
Se observa que S S es la identidad. Es fcil de verificar que la simetra axial satisface las siguientes propiedades:


1. La simetra axial enva a un segmento [AB] a otro A B , igual en longitud, siendo A homlogo a A, B homlogo a B.

2. La simetra axial transforma a una recta en otra.


3. La simetra axial transforma un tringulo ABC en otro igual A BC siendo A , B ,C homlogos, respectivamente, a
A, B,C, pero cambia el sentido de orientacin de los vrtices.

Tarea

175

4. La simetra axial transforma a un ngulo en otro igual pero contrariamente orientado.


5. La simetra axial transforma a un crculo en otro igual, siendo los centros de los crculos puntos homlogos.
838 Ejemplo Dadas dos circunferencias de centro O, O y una recta L, hallar sobre L un punto del cual las tangentes trazadas

hasta las circunferencias desde este punto formen ngulos idnticos con L.
Resolucin: Vase la figura 7.168. Se refleja el centro O a travs de la recta L y se construye un crculo
=
homlogo con centro O . Se construyen ahora tangentes a los crculos O y O . Por ser opuestos al vrtice BSG

FSA. Por ser reflexin axial, G SB = BSG, de donde se deduce que S es el punto buscado. Hay cuatro soluciones,
ya que hay cuatro tangentes. En la solucin se consider el caso cuando ambos crculos yacan en el mismo lado
de la recta. Est claro que la solucin se simplifica si ambos crculos yacen en lados opuestos de la recta.
839 Definicin Llmase homotecia de centro O y constante de similitud a una funcin

R2
HO, :
M

R2
7 M

tal que OM = OM . Si > 0 la homotecia se dice directa y si < 0, inversa.


Es fcil de ver que HO,1 es la identidad y que HO,1 es una reflexin de centro O. Se observan adems las siguientes propiedades:
1. Una recta es homottica a otra recta.
2. Un tringulo es homottico a otro tringulo semejante al primero.
3. Un ngulo es homottico a otro ngulo igual al primero.
4. Un crculo es homottico a otro crculo concntrico.
840 Ejemplo Dados dos crculos de centros O y O , hallar los centros y las constantes de similitud de las homotecias que

transforman el uno al otro.


Resolucin: Trcese la recta pasando por los centros O y O . Trcense ahora dos radios OA y O A paralelos y


en el mismo sentido, en uno y otro crculo. Sea O la interseccin de las rectas AA y OO . Es fcil ver ahora que
HO , R es una homotecia directa que transforma al crculo de centro O y radio R en el crculo de centro O y radio
R .

Para obtener la homotecia inversa HO , R , simplemente trcense dos radios OA y O A paralelos y en distinto
R
sentido y sgase el procedimiento anterior. Vase la figura 840.
841 Ejemplo En el ABC construir, mediante regla y comps, un cuadrado tal que uno de sus vrtices pertenezca al lado [AB],

otro al lado [AC] y los otros dos al lado [BC].

Resolucin: Constryase primero el cuadrado BCDE exterior al lado [BC] del tringulo. Sean D y E las

intersecciones de las rectas AD y AE con el lado [BC]. Sea HA, la homotecia de centro A tal que HA, (D) = D
y HA, (E) = E . Sea BC D E la imagen del cuadrado BCDE bajo esta homotecia. Por construccin, BC D E
cuadrado es. Ahora bien, los puntos A, B, B estn alineados, al igual que los puntos A,C,C , de donde BC D E es
el cuadrado deseado. Vase la figura 7.166

Tarea

176

Captulo 7

842 Problema Dado un punto A, una recta L y un crculo , constryase un tringulo

ABC tal que B yaga sobre L y C yaga sobre .


[AC], respectivamente. Las rectas Les BN y CM se cortan en P. Demostrar que

MN k BC A
PM = A
PN.

843 Problema Encontris un antiguo mapa de tesoro en el bal del tatarabuelo. El dia-

grama indica que partiendo del muelle debis caminar hasta el roble, doblar a la derecha
90 y caminar igual distancia, clavando una estaca aqu, marcada con una X . Luego regresis al muelle, caminis hasta el pino, doblis a la izquierda 90 , caminis igual distancia,
y clavis una estaca marcada con una Y . El tesoro se halla en el punto medio del segmento [XY ]. Como encontris fciles las instrucciones, alquilis un barco y viajis hasta los
mares del sur. Al llegar a la costa de tan pequea isla, sis capaz de encontrar el roble y
el pino, pero, ay caramba!, del muelle no hay rastro porque se lo ha comido el salitre.
Cmo encontraris el tesoro?

845 Problema Considrese el A1 A2 A3 y sea P0 un punto en el plano. Defnase

As = As3 para s 4. Ahora se construye la sucesin de puntos P1 , P2 , P3 , . . . de tal


manera que GAk+1 ,120 (Pk1 ) = Pk . Demustrese que si P2004 = P0 , entonces A1 A2 A3
es equiltero.

846 Problema Dados son tres crculos disjuntos dos a dos. Trcense los tres puntos de
844 Problema Sea ABC issceles en A. Sean M y N puntos sobre los lados [AB] y

interseccin de las tres tangentes exteriores comunes. Demustrese que esto puntos estn
alineados.

7.13. Teoremas de Ceva y de Menelao


847 Teorema (Teorema de Ceva, 1678) Las tres cevianas AA , BB y CC de un ABC concurren si y slo si

AC BA CB

= +1.
C B A C B A

Demostracin: = Usando el teorema 679 se obtiene



AC BA CB [APC] [APB] [PCB]

= 1.
=
C B AC BA
[PCB] [APC] [ABP]

Presmase que AA y BB se intersecan en P. nase CP y extindase hasta AB, intersecndola en C . Como


AA , BB y CC son concurrentes, por la mitad del teorema ya demostrada se tiene
AC BA CB

= +1.
C B A C B A
Pero por hiptesis

Esto significa que

AC BA CB

= +1.
C B A C B A
AC
AC
= ,

CB C B

de donde se colige que C = C .

Como el teorema 679 no depende de la posicin de los puntos involucrados, la demostracin anterior es
vlida aun cuando el punto P yace fuera del tringulo ABC.
848 Teorema (Forma Trigonomtrica del Teorema de Ceva) Sea P un punto arbitrario en el plano y en el ABC sea A el

punto de interseccin de las rectas AP y BC, B el punto de interseccin de las rectas BP y CA, y C el punto de interseccin de
las rectas CP y AB. Entonces
sen CBB

sen
ACC sen BAA

= 1.
AC sen B
BA
CB sen A

sen C
De manera recproca, si A , B ,C son puntos en los lados BC,CA, AB respectivamente, y si
sen BAA
sen CBB

sen ACC

=1
AC sen B
BA
CB sen A

sen C

entonces AA , BB ,CC son concurrentes.

Teoremas de Ceva y de Menelao

177

Demostracin: Se quiere demostrar que


sen BAA
sen CBB

sen ACC

=1
AC sen B
BA
CB sen A

sen C

s y slo si
AZ BX CY

= 1.
ZB XC YA
Ahora bien,
AZ CA sen
ACC
=
,
CB
ZB BC sen C

BX
AB sen BAA
=
,
AC
XC CA sen A

CY
BC sen CBB
=
.
BA
YA
AB sen B

Multiplicando y cancelando en estas tres igualdades se obtiene el resultado.


849 Teorema (Teorema de Menelao) Sean L, M, N puntos menelaicos de los lados BC, CA y AB del ABC. L, M, N son

colineales si y solamente si

BL CM AN

= 1.
LC MA NB
Demostracin: = Sean X,Y puntos arbitrarios de la recta LMN. Entonces


BL CM AN [BXY ] [CXY ] [AXY ]

= 1.

=

LC MA NB

[CXY ] [AXY ] [BXY ]

Presmase ahora que la recta MN corta a AC en L . Por la mitad del teorema ya demostrada se tiene
BL CM AN

= 1.
LC MA NB
Pero por hiptesis
BL CM AN

= 1.
LC MA NB
As

BL
BL
=
,
LC LC

de donde L = L .
850 Ejemplo (IMO 1982) Las diagonales AC y CE de un hexgono regular ABCDEF son dividas interiormente en los puntos

M y N, respectivamente, de tal manera que


AM CN
=
= r.
AC
CE
Determnese r si B, M, N son colineales.
Resolucin: nase BE intersecando AC en P. Aplcando el teorema de Menelao al CPE y a la recta BMN,
se tiene
CM
1r
= AB = BE = PB = 1 .
=
;
PB = AB cos ABP
1
MP r 2
2
4
BE
4

178

Captulo 7
Adems

de donde se colige que

EN
1r
=
,
NC
r

3
2 2r 1 1 r

= 1 = r =
.
2r 1 4
r
3

M
D

P
b

E
F
851 Teorema [Invariancia bajo perspectiva] Sean L1 y L2 dos rectas distintas sobre el plano. Si A, B, C, D son puntos distintos
sobre L1 y si A , B , C , D son puntos distintos sobre L2 y si las rectas AA , BB , CC , DD son concurrentes, entonces
AC BD AC B D
=
.
CB DA C B D A
Demostracin: Si AA , BB , CC , DD se intersecan en O y si P es el punto de interseccin de L1 y L2 (si L1 ||L2 ,
entonces P es el punto en infinito). Vase la figure 7.169.
Aplicando el teorema de Menelao a ABA , A PA, B PB, BPB intersecndose con las rectas CC , DD , CC ,
y DD respectivamente,
AC PC A O

= 1,
CP C A OA
A D PD AO

= 1,
D P DA OA
BD PD B O

= 1.
DP D B OB
En multiplicando estas cuatro igualdades se colige
AC A D BC BD
= 1,
C A DA CB DB
de donde

AC BD AC B D
=
.
CB DA C B D A

852 Teorema [Teorema de Simson] Sea P un punto en el circuncrculo del ABC. Sean D, E, F los pies de las perpendiculares
de P a BC, CA y AB respectivamente. Entonces C, E, F son colineales.

Demostracin: Para demostrar que D, E, F son colineales, se necesita demostrar que


AF BD CE

= 1.
FB DC EA

Teoremas de Ceva y de Menelao

179

FB = PB cos PBF,
CE = PC cos PCE,
EA =
BD = PB cos PBD,
DC = PC cos PCD,
Ntese que AF = PA cos PAF,

PA cos PAE. Por lo tanto,

PBD)(cos
PCE)
AF BD CE
(cos PAC)(cos

=
.
FB DC EA (cos PBF)(cos

PCD)(cos
PAE)
= PCD,
PBD
PCE
= PBF,
= PAE,
de donde se consigue el resultado.
Ahora bien, PAF

C
O

C
b

E
b

bC

F
b

B C

Figura 7.169: Teorema 851.

Figura 7.170: Teorema 852.

bD

Figura 7.171: Ejemplo 853.

853 Ejemplo Refirase a la figura 7.171. D, E, F son, respectivamente, los pies de las perpendiculares de A a BC, B a CA y de C

a AB. Trcense las rectas perpendiculares de D a AB, CA, BE, CF y sean P, Q, M, N los respectivos pies de las perpendiculares.
Demustrese que P, Q, M, N son colineales.
Demostracin: El cuadriltero BDHF es cclico y la recta de Simson de D pasa por P, M, N. En otras palabras
P, M, N son colineales. Arguyendo de manera semejante se demuestra que Q, M, N son colineales.
854 Ejemplo Sea H el ortocentro, O el circuncentro y R el circunradio del ABC. Sea D la reflexin de A a travs de la recta

BC, E la reflexin de B a travs de CA y F aquella de C a travs de AB. Demustrese que D, E y F son colineales si y slo si
OH = 2R. (Figura 7.172.)
Demostracin: Sea PQR el tringulo cuyo tringulo de medianas es ABC (esto es A es el punto medio
de QR, B el de RP y C el de PQ). Desde O, trcense rectas perpendiculares a QR, RP y PQ, cuyos pies de las
perpendiculares son D , E , F respectivamente. Por el teorema de Simson, D , E , F son colineales si y slo si O
yace en el circuncrculo del PQR. Ntese que el circuncentro del PQR es el ortocentro del ABC, esto es, H.
As, O yace en el circuncrculo del PQR si y slo si OH es igual al PQR que es 2R. Vase la figure 7.173

855 Teorema (Teorema de Ptolomeo) Dados cuatro puntos cualesquiera en posicin general (esto es, ninguno de ellos coin-

cide, y no tres de ellos son colineales),


AB CD + AD BC AC BD.
La igualdad se cumple si y slo si el cuadriltero ABCD es cclico.
Demostracin: Sean L, M, N, respectivamente, los pies de las perpendiculares de D a BC, CA, AB. Como
= CMD
= 90 , los puntos L, C, D, M son cocclicos. La figura 7.174 muestra uno de los casos posibles. De
CLD
todas maneras se tiene
= CD AB ,
LM = CD sen BCA
2R

180

Captulo 7
en donde R denota el circunradio del ABC. De la misma guisa,
MN =

AD BC
,
2R

LN =

BD AC
.
2R

En virtud de la desigualdad del tringulo, LM + MN LN. Utilizando las expresiones para LM, MN y LN se
obtiene
CD AB AD BC BD AC
+

.
2R
2R
2R
Esto demuestra la desigualdad deseada. La igualdad es satisfecha si y solamente si L, M, N son colineales y
gracias al teorema de Simson esto sucede si y solamente si D yace en el circuncrculo del ABC.

F
b

D
b

R
b

C
A

F
b

B
A

E
b

D
b

Figura 7.173: Ejemplo 854.

Figura 7.172: Ejemplo 854.

Demustrese que PA = PB + PC.


856 Ejemplo Sea P un punto en el circuncrculo del ABC yaciendo en el arco BC.

Resolucin: El resultado se consigue de inmediato al aplicar el teorema de Ptolomeo al cuadriltero cclico


ABPC (figura 7.175).
857 Ejemplo Si un crculo pasando por A corta dos lados y una diagonal del paralelogramo ABCD en los puntos P, Q, R tal
como en la figura , demustrese que
AP AB + AR AD = AQ AC.

Resolucin: Aplquese el teorema de Ptolomeo al cuadriltero cclico APQR obteniendo


AP RQ + AR PQ = AQ RP.

Teoremas de Ceva y de Menelao

181

Vase la figura 7.176. Obsrvese que ABC RQP. Al multiplicar esta ltima igualdad por la constante

AB
RQ

se obtiene

AP AB + AR CB = AQ AC.
Reemplazando CB por AD, se colige el resultado
AP AB + AR AD = AQ AC.

b
b

b
b

NA

b
b

b
b

Figura 7.174: Teorema 855.

Figura 7.175: Ejemplo 856.

Figura 7.176: Ejemplo 857.

858 Ejemplo Sean A, B, C, D, E, F, G vrtices, nombrados en orden, adyacentes de un heptgono regular. Demustrese que

1
1
1
+
=
.
AC AD AB
Resolucin: En la figura 7.177, aplicando el teorema de Ptolomeo al cuadriltero cclico ABCF
AC BF = AB CF + BC FA.
Substituyendo BF por AD, BC por AB y FA por AC en la ltima desigualdad se obtiene
AC AD = AB AD + AB AC =

1
1
1
+
=
,
AC AD AB

obtenindose el resultado.
859 Ejemplo En el ABC, BC > CA > AB. D yace en el lado BC y E yace en la prolongacin de BA producida ms all de A

de tal manera que BD = BE = CA. Si P yace en el lado AC de tal manera que E, B, D, P son cocclicos y sea Q el otro punto de
interseccin de BP con el cicuncrculo del ABC. Demustrese que
AQ +CQ = BP.
Vase la figura 7.178.

182

Captulo 7
Resolucin: Obsrvese que AQC EPD porque

= CBQ
= DEP

CAQ

= 180 ABD
= EPD.
Por otra parte, el teorema de Ptolomeo implica que
y AQC

BP DE = BE DP + BE EP.
As pues
BP = BE

DP
EP
CQ
AQ
+ BD
= CA
+CA
= AQ +CD.
DE
DE
CA
CA

D
b

bQ

Figura 7.177: Ejemplo 858.

Figura 7.178: Ejemplo 859.

Tarea
860 Problema (Teorema de Van Aubel) Demostrar que si el ABC tiene cevianas

AA , BB ,CC que concurren en P entonces

CP
CA CB
= + .
PC
AB
BA

7.14. Puntos y rectas notables de un tringulo


861 Definicin Dado un ABC, si MA , MB , MC son, respectivamente, los puntos medios de los lados BC, CA y AB, los

segmentos AMA , BMB , CMC son las medianas del tringulo.

862 Teorema Las medianas de un tringulo concurren.

Demostracin: Dentense los puntos medios de los lados BC,CA, AB por MA , MB , MC . Refirase a la figura 7.179.
Por definicin de las medianas
BMA = MAC,

CMB = MB A,

AMC = MC B

y as
AMC BMA CMB

= 1.
MC B MAC MB A
Luego las medianas concurren gracias al Teorema de Ceva (847).

Puntos y rectas notables de un tringulo

183

863 Definicin El punto de concurrencia de las medianas de un tringulo se llama el centroide o baricentro.
864 Teorema Todo tringulo es dividido por sus medianas en seis tringulos de rea igual.

Demostracin: Como los GBMA y GMAC tienen bases de igual longitud y las misma altura, se tiene
[GBMA ] = [GMAC] = x,
digamos. De la misma manera
[GCMB ] = [GMB A] = y,

[GAMC ] = [GMC B] = z.

Ahora bien
[CAMC ] = [CMC B] = 2y + z = z + 2x = x = y,
[ABMA ] = [AMAC] = 2z + x = 2y + x = y = z,
y por lo tanto x = y = z.
865 Corolario El baricentro G del ABC triseca cada mediana. En efecto, AG : GMA = BG : GMB = CG : GMC = 2 : 1.

Demostracin: Por el teorema 864, [BMA G] = 2[BMA A]. Como ambos tringulos tienen la misma altura,
[BMA G] = 2[BMAA] = MA G = 2GA = AG : GMA = 2 : 1.
Las otras relaciones se demuestran de manera semejante.
866 Teorema Sea G el baricentro del ABC y L una recta. Entonces

GG =

AA + BB +CC
,
3

en donde A , B ,C denotan, respectivamente, los pies de las perpendiculares desde A, B,C en L.


Demostracin: Sea M el punto medio de CA y N el punto medio de BG, como en la figura 7.180. Entonces
2GG = MM + NN ,
de donde
4GG = 2MM + 2NN = (AA +CC ) + (BB + GG )
y por lo tanto, 3GG = AA + BB +CC .
867 Corolario Sea G el baricentro del ABC y sea P un punto arbitrario. Entonces


PG =
PA + PB + PC .
3

An ms, si A = (a1 , a2 ), B = (b1 , b2 ), C = (c1 , c2 ), entonces

G=

a 1 + b 1 + c1 a 2 + b 2 + c2
,
.
3
3

Demostracin: El resultado se obtiene de inmediato del teorema 866.


868 Definicin El tringulo medial de un tringulo dado es el tringulo obtenido al unir los pies de las medianas del tringulo

dado.

184

Captulo 7

869 Teorema Los lados del tringulo medial son paralelos a los lados del tringulo original y miden la mitad de los lados

correspondientes en el tringulo original. El rea del tringulo medial es un cuarto del rea del tringulo original.
b se tiene que
Demostracin: Como 2AMC = AB y 2AMB = AC, y los tringulos AMC MB y ABC comparten el A,

AMC MB ABC. Luego AMC MB = ABC y por lo tanto BC y MC MB son segmentos paralelos. Un argumento
semejante demuestra que AB k MA MB y AC k MA MC . Se sigue, adems que MA MB MC ABC y que 2MA MB =
AB, 2MA MC = AC, 2MC MB = CB.

Ahora bien, como MA MB MC ABC, y 2MB MC = BC, las alturas del ABC son el doble de las alturas las
alturas del MA MB MC . As pues [ABC] = 4[MA MB MC ].

A
MC
B

b
b

MA

MB
b

b
bC

bb

bM

AG

Figura 7.179: El baricentro. Teorema 862.

MC
B

MB

MA

Figura 7.181: El tringulo medial.


Figura 7.180: Teorema 866.

870 Teorema En todo tringulo, el radio del circuncrculo de su tringulo medial es

original.

R
, el radio del circuncrculo del tringulo
2

Demostracin: El resultado sigue inmediatamente del teorema 869.


871 Teorema (Generalizacin de la ley de los senos) En el ABC pngase a = BC, b = AC y c = AB. Sea R el radio del

crculo circunscrito al ABC. Entonces

a
b
sen A

b
sen Bb

c
sen Cb

= 2R.

b is agudo u obtuso.
Demostracin: Se considerarn dos casos: si A

b es agudo agudo. Sea DC un dimetro. Entonces CBD


= al estar inscrito en un
Presmase primero que A
2
semicrculo. As
a
a
b=
sen D
=
.
DC 2R
b ya que ambos subtienden el mismo arco. Por lo tanto,
b = A,
Ahora bien, D
b = sen D
b=
sen A

a
.
2R

De la misma manera se demuestra que


sen Bb =
b agudo.
demostrando el teorema en el caso A

b
,
2R

sen Cb =

c
,
2R

Puntos y rectas notables de un tringulo

185

b es obtuso. Se tiene que


Presmase ahora que A
b=
sen D

a
.
2R

b porque ngulos opuestos inscritos en un cuadriltero son suplementarios. Como


b = A,
En este caso, D
b = sen( A)
b = sen(A)
b = sen A,
b
sen( A)

se deduce que
b = sen( A)
b = sen D
b=
sen A

a
,
2R

de donde queda demostrado el teorema.

A
B

C
O

Figura 7.182: Generalizacin de la ley de los


b agudo.
senos. A

Figura 7.183: Generalizacin de la ley de los


b obtuso.
senos. A

872 Teorema Sea R el radio del crculo circunscrito del ABC y sean las longitudes de sus lados a = BC, b = AC y c = AB.

Entonces [ABC] =

abc
.
4R

Demostracin: Se tiene
[ABC] =

ab sen Cb abc
=
,
2
4R

por la ley de los senos.


873 Teorema Las alturas de un tringulo concurren.

Demostracin: Obsrvese que


b
AHC cot A
=
,
HC B cot Bb

BHA
cot Bb
=
,
HAC cot Cb

CHB cot Cb
AHC BHA CHB
=
=

= 1.
b
HB A cot A
HC B HAC HB A

Luego las alturas concurren gracias al Teorema de Ceva (Teorema 847).

874 Definicin El punto de concurrencia de las alturas de un tringulo se llama el ortocentro del tringulo.
875 Definicin El tringulo rtico del ABC es el tringulo cuyos vrtices son los pies de las alturas del ABC.
876 Teorema Las bisectrices angulares de un tringulo concurren.

186

Captulo 7
Demostracin: Gracias al Teorema de la bisectriz (Teorema 672),
AC
CB
= ,

AC
CB

AB
AC
= ,

BA
AC

BC
AB
AC BA CB
=
,
=

= 1,
CB B A
C B AC B A

y el resultado se obtiene por el Teorema de Ceva (teorema 847).


877 Definicin El punto de concurrencia de las bisectrices angulares de un tringulo se llama el incentro.
878 Teorema Sea r el radio del crculo inscrito del ABC y sean las longitudes de sus lados a = BC, b = AC y c = AB. Sea

s=

a+b+c
el semi-permetro. Entonces [ABC] = sr.
2
Demostracin: De a figura 7.79 se tiene

[ABC] = [IBC] + [ICA] + [IAB] =

ar br cr
+ + = sr.
2
2
2

F
C

b
b

E
b

b
b

b
bb

B
A

b
b

D
C

Figura 7.184: El incentro. Teorema


876.

Figura 7.185: Punto de Gergonne y


tringulo de Gergonne.

879 Teorema Las cevianas a los puntos de contacto de un tringulo con su crculo inscrito concurren.

Demostracin: En la figura 7.185, AF = EA, FB = BD y CE = EA, ya que tangentes desde un punto a un crculo
son congruentes. Luego
AF BD CE

=1
FB DC EA
y las cevianas concurren gracias al Teorema de Ceva (teorema 847).
880 Definicin El punto de concurrencia de las cevianas a los puntos de contacto de los lados de un tringulo con su crculo

inscrito es el punto de Gergonne.


881 Definicin El tringulo de Gergonne del ABC es el tringulo cuyos vrtices son los pies de las las cevianas a los puntos
de contacto de los lados del ABC.
882 Teorema (Recta de Euler) En un tringulo dado, el ortocentro H, el circuncentro O y el baricentro G son colineales y se

satisface

OG 1
= .
GH
2

Puntos y rectas notables de un tringulo

187

Demostracin: Sea HB la proyeccin del ortocentro H en el lado AC. Luego


AH =

AHB
c cos A
=
= 2R cosA = 2OMA .
senC
senC

Si OH y AMA se intersecan en G entonces AG H MA G O y AG = 2G MA . Esto implica que G = G, el


baricentro de ABC.
883 Teorema Los pies de las tres alturas de cualquier tringulo, los puntos medios de los tres lados, y los puntos medios
de los tres segmentos de los vrtices al ortocentro todos yacen en el mismo crculo, llamado el crculo de nueve puntos de
Euler-Feuerbach.

Demostracin: Como BC es un lado comn al ABC y HBC, cuyos otros dos lados son bisecados respectivamente por MC , MB y J, K ambos segmentos MB MC y JK son paralelos a BC y 2MB MC = 2JK = BC. De igual
manera, AH es lado comn a BAH y CAH, de donde MC J y MB K son paralelos a AH y 2MC J = 2MB K = AH.
De aqu, MB MC JK es un paralelogramo y como BC AH, MB MC JK es un rectngulo. De igual manera, MA MB IJ

y MC MA KI son rectngulos. Luego MA I, MB J y MC K son tres dimetros de un crculo. Como M


, el
A HA I =
2
crculo con MA I como dimetro pasa por HA . De igual manera se demuestra que pasa por HB y por HC .
A

bH
B

HC
MC

HC
b

bM
B
b

MC

G
b

b
rs

MA

884 Teorema El radio del crculo de nueve puntos es

bM
B

HA

H
b

Figura 7.186: Recta de Euler.

del tringulo medial.

bHB

K
b

HA

MA

Figura 7.187: Crculo de nueve puntos de Euler-Feuerbach.

R
, la mitad del radio del crculo circunscrito. Su centro es el circuncentro
2

Demostracin: Al pasar el circuncrculo medial y el crculo de nueve puntos por MA , MB y MC , ambos crculos
R
coinciden, de donde se deriva la asercin, ya que el radio del circuncrculo del tringulo medial es .
2
885 Ejemplo Dos crculos C1 (con centro O1 ) y C2 (con centro O2 ) se intersecan en los puntos A y B, como en la figura . Al

extender la recta O1 B, esta se interseca con C2 en E. Al extender la recta O2 B, esta se interseca con C1 en F. Se construye
una recta paralela a EF a travs de B cortando a C1 en M y a C2 en N. Demustrese que B ese el incentro del AEF y que
MN = AE + AF.
Resolucin: Obsrvese que O1 AO2 O1 BO2 y por tanto

O
1 AO2 = O1 BO2 = 180 O1 BF = 180 O1 FO2

y as AO1 FO2 es un cuadriltero cclico. Por simetra, E tambin yace en este crculo, de donde se ha demostrado
que B es el incentro del AEF.

188

Captulo 7

Ahora bien, AO
1 B = 2AFB y MO1 F = 2MBF = 2EFB. Como B yace el bisector angular interior del AFE, se ha
demostrado que AF = MB. De igual manera se demuestra que AE = NB por lo tanto MN = AE + AF.

886 Teorema Un punto P dentro del ABC es el incentro del tringulo si y slo si
y
1. P yace en el bisector angular del ngulo CAB
=
2. BPC

180 + CAB
.
2

Demostracin: Es claro que el incentro de un tringulo satisface las condiciones mencionadas. Por otra parte,
el conjunto de puntos en el plano que satisface la primera condicin es un segmento de recta dentro del tringulo,
en tanto el conjunto de puntos que satisface la segunda condicin es un arco. La interseccin de estos dos lugares
geomtricos es obviamente nica, el incentro del tringulo.
B

O1 b

O
b 2
b

b
b

P
b

Figura 7.188: Ejemplo 885.

Figura 7.189: Teoremas 886 y


thm:criterio-ortocentro.

Figura 7.190: Teorema 888.

La siguiente aserciones son evidentes.


887 Teorema Un punto P dentro del tringulo agudo ABC es el ortocentro del tringulo si y slo si

1. AP BC y
+ CPB
= 180 .
2. CAB
Un punto P en el segmento AD es en el
888 Teorema Sea D un punto en el circuncrculo del ABC tal que AD biseca al CAB.

incentro del tringulo si y slo si DB = DP (o lo que es equivalente, DC = DP). En este caso, D es el circuncentro del BPC.

> 60 . Sea
889 Ejemplo (IMO 2002) Sea BC un dimetro del crculo C con centro O. Sea A un punto en C tal que AOC
que no contiene a C. La recta a travs de O paralela a DA interseca a AC en J. La mediatriz
D el punto medio del arco AB

perpendicular de OA interseca con C en E y F. Demostrar que J es el incentro del CEF.


Como EA = EO y OE = OA, OAE es equiltero. Ya que AOC > 60 , F yace en el arco
Se sigue que ACF
= AEF
= 1 = 30 y ACE
= 1 AOE
= 30 = ACF,
de donde J yace en el bisector
menor AC.
2
2
Resolucin:

Puntos y rectas notables de un tringulo

189

Ahora bien, ACB


= 1 AOB
= DOB,
luego AC k DO y por lo tanto ADOJ es un paralelogramo. Luego
angular ECF.
2
AJ = DO = AO = AE. Por el teorema 888, J es el incentro del CEF.

A
D
E b
B b

b
b

J
b

b
b

D
Figura 7.191: Ejemplo 889.

Figura 7.192: Ejemplo 890.


Figura 7.193: Teorema 891.

b > B,
es obtuso. Sea H el ortocentro
b D es un punto en BC tal que ADB
890 Ejemplo (CMO 1999) En el tringulo agudo ABC, C

del ABD. El punto F est en el interior del ABC y en el circuncrculo del ABD. Demustrese que F es el ortocentro del
ABC si y slo si HD k CF y H yace en el circuncrculo del ABC.
Resolucin: Por el teorema 887 se necesita demostrar que
1. CF AB

+ BFA
= 180 .
2. BCA

Como H es el ortocentro del ABD, se tiene HD AB. As (1) es equivalente a HD CF. Se necesita demostrar
= ADB
=
ahora que (2) es satisfecha si y slo si H yace en el circuncrculo del ABC. Obsrvese que AFB

180 AHB. As, (2) se convierte en ACB = AHB, que equivale a decir que H yace en el circuncrculo del ABC.

891 Teorema (Teorema de los tres crculos de Carnot) Tres crculos de radio unidad se encuentran en el punto P y las otras

intersecciones de cada dos de ellos son los puntos A, B y C. Demustrese que P es el ortocentro del ABC y que circunradio
de ABC es 1.
=
Demostracin: Sean D, E, F los puntos en cada crculo diametralmente opuestos a P. Obsrvese que PAE

PAF = 90 y PE = PF = 2, lo que implica que A yace en el segmento EF y PA es la mediatriz perpendicular de


EF. De igual manera, PB y PC son las mediatrices perpendiculares de PD y DE respectivamente.

En adicin, el ABC es el tringulo medial del DEF, y por lo tanto el circunradio del ABC es la mitad del
circunradio del DEF. Como P es el circuncentro del DEF, P es adems el ortocentro del tringulo medial del
ABC y el circunradio del DEF es 2. Luego, el circunradio del ABC es 1.

190

Captulo 7

Tarea
Demostrar que los baricentros de los tringulos ABC y LMN coinciden.

892 Problema Demustrese que el circuncentro de un tringulo coincide con el orto-

centro de su tringulo medial.

894 Problema Por el baricentro G de un tringulo se traza una recta que corta al lado
893 Problema En el ABC, los puntos L, M =, N yacen sobre los segmentos [AB],

AB en P y al lado AC en Q. Demustrese que

[BC] y [CA], respectivamente, satisfaciendo

PB QC
1

.
PA QA
4

BM
CN
AL
=
=
.
AB
BC
CA

7.15. Potencia de un punto con respecto a un crculo


A
A
B

A
b

B
B
Figura
895.

7.194:

Teorema

Figura
895.

7.195:

Teorema

Figura
900.

7.196:

Ejemplo

895 Teorema (Interseccin de cuerdas) Sea C un crculo y P un punto no en C . Sea L una recta pasando por P que interseca

a C en los puntos A y B. Entonces la cantidad PA PB es independiente de L, esto es, si L es otra recta que pasa por P y que
interseca a C en A y B , se tiene
PA PB = PA PB.
Recprocamente, si A, B, A , B, son cuatro puntos no alineados, si las rectas AB y A B se encuentran en P y si PA PB =
PA PB como segmentos dirigidos, entonces A, B, A , B yacen en el mismo crculo.

P = PBB

Demostracin: Supngase primero que P yace en el interior del crculo. De la figura 7.194 se tiene AA

= BPB
, as PAA PB B, dando PA/PB = PA /PB, esto es PA PB = PA PB .
y APA

Supngase ahora que P est en el exterior del crculo. Sea T un punto de tangencia desde P, como en la figura
7.195. Entonces PTA PBT y PBA PB A. As
PT
PB
=
;
PA
PT

PB
PB
=
= PT 2 = PA PB = PA PB .

PA
PA

Recprocamente, sea C el circuncrculo del tringulo ABA . La recta PA corta C en un punto C tal que PA PB =
PA PC. Luego
PA PB = PA PB , PA PB = PA PC = PB = PC.
Como PB y PC son segmentos dirigidos, se tiene B = C, completando la demostracin.

896 Definicin La cantidad PA PB en el teorema 895 se llama potencia de P con respecto al crculo C .
897 Teorema Un crculo de radio r con centro I est dentro de un crculo de radio R > r y centro O. Sea A un punto arbitrario
en el crculo mayor, y sean AB y AC
dos cuerdas en el crculo mayor, ambas tangentes al crculo menor. Entonces BC es tangente
al crcul menor si y slo si IO = R(R 2r).

Potencia de un punto con respecto a un crculo

191

Trcese CI y CS.
Demostracin: Sea S un punto en el crculo mayor tal que AS es la bisectriz angular del BAC.
d = ICA.
d A su vez, esto sucede si y slo si SCI
d = CIS,
b
BC es tangente al crculo menor si y slo si BCI
ya que
d = ICA
d + IAC
d = ICA
d + SCB.
Adems, SCI
d = CIS
d si y slo si SC = SI. Sea MN = 2R el dimetro del crculo
CIS
r
mayor que pasa por I y O. Entonces SC = SI si y slo si SI IA = SC IA = 2R sen
= 2rR, en donde
sen

= CAS.

En virtud del teorema , SI IA = MI IN = (R d)(R + d), en donde d = IO. Luego se tiene SI IA = 2rR si y slo
si (R d)(R + d) = 2rR, lo que equivale a d 2 = R2 2rR.
898 Corolario (Teorema de Euler) Sea I el centro del crculo inscrito y O el centro del crculo circunscrito al tringulo ABC.

Entonces

OI 2 = R2 2Rr.
899 Corolario Dos reales r > 0 y R > 0 son el radio del crculo inscrito y el radio del crculo circunscrito del ABC, respectivamente, si y slo si R 2r. Adems, R = 2r si y slo si el ABC es equiltero. Si R > 2r entonces existe un nmero infinito
de tringulos no semejantes teniendo a R como circunradio y r como inradio.

M
B

b
b

O
b

Figura 7.197: Teorema 897.

b del ABC con D yaciendo en AC. El circuncrculo del BDC


900 Ejemplo (SPCMO 1996) Sea BD la bisectriz angular del B
interseca a AB en E y el circuncrculo del ABD interseca a BC en F. Demustrese que AE = CF.

Resolucin: Por el teorema 895,


AD AE
=
,
AC
AB

CD CF
AE
AD CB
=
=
=
= 1,
CA CB
CF
AB CD

pero esta ltima cantidad es 1, por el Teorema de la bisectriz angular.


901 Ejemplo Sean A, B,C, D cuatro puntos alineados en este orden. Los crculos con dimetros AC y BD se intersecan en los

puntos X y Y . La recta XY interseca la recta BC en el punto Z. Sea P un punto en XY distinto de Z. La recta CP interseca al
crculo con dimetro AC en los puntos C y M. La recta CP interseca al crculo con dimetro AC en los puntos C y M. La recta
BP interseca al crculo con dimetro BD en los puntos B y N. Demustrese que las rectas AM, DN y XY son concurrentes.

192

Captulo 7
Resolucin: Trcese DE paralelo a CM, intersecando XY en E. Trcese AE paralelo a BN, intersecando XY
en E . Se quiere demostrar que E = E . Obsrvese que
ZE ZE ZP ZA ZC
=

.
ZE
ZP ZE
ZB ZD

(7.19)

Por el teorema 895


ZA ZC = ZX ZY = ZB ZD.

Por lo tanto (7.19) da ZE = ZE . Se concluye que AM, DN, XY son las alturas del ADE y por lo tanto, concurrentes.

b
b

B2

P
Z

N
b

bB

A1

b b

A2
bC
1

bC
2

Y
Figura
901.

7.198:

Ejemplo

Figura
902.

7.199:

Ejemplo

902 Ejemplo AB es una cuerda de un crculo que no es un dimetro. Las cuerdas A1 B1 y A2 B2 se intersecan en el punto

medio P de AB. Las tangentes al crculo desde A1 y B1 se intersecan en C1 y las tangentes desde A2 y B2 se intersecan en C2 .
Demustrese que C1C2 k AB.
Resolucin: Sea O el centro del crculo, dgase que OC1 interseca a A1 B1 en M, OC2 interseca a A2 B2 en
N y OC1 interseca a AB en K. Evidentemente, OM y ON son, respectivamente, las mediatrices perpendiculares
= ONP
= 90 , por lo que O, M, P, N yacen en el mismo crculo. Esto implica que
de A1 B1 y A2 B2 . As, OMP

= OPM
= 90 MOP
= OKA.

ONM
Ahora se demostrar que M, C1 , C2 , N yacen en el mismo crculo. Obsrvese que OA1C1 y OB2C2 son tringulos rectngulos, de donde
OM OC1 = OA21 = OB22 = ON OC2 .

Se tiene pues que OC


1C2 = ONM = OKA, completando la demostracin.

Tarea

Tarea
903 Problema Demostrar que si las cuerdas A1 B1 and B1C1 en el circuncrculo del
ABC son tangentes al incrculo, la cuerda C1 A1 (en el circuncrculo) es tambin tan-

193
gente al incrculo.


Apendice

Indicaciones y respuestas
8 Presmase que AC BC y colquese D en el segmento de recta AC de tal manera que AD = BD. Luego ADB es issceles en D y se
b= B
b, contradiccin.
tendr A

9 Si a entonces 2 , lo que implica que (1 ) 0, desigualdad imposible al ser 0 < < 1.


10 Se tiene 1

1
< < 1. En cuadrando,
102000
1

Ya que

1
1
+
< 0, se tiene
102000 104000
1

1
102000

2
1
+
< 2.
102000 104000

< 1

1
102000

1
102000

1
104000

< 2.

11 Se tiene
x4 + 2x2 + 2x + 2

(x2 + ax + b)(x2 + cx + d)

x4 + (a + c)x3 + (d + b + ac)x2 + (ad + bc)x + bd.

As
bd = 2, ad + bc = 2, d + b + bc = 2, a + c = 2.
Presmase que a, b, c, d son ntegros. Entonces como bd = 2, b y d deben ser de paridad opuesta, el uno par y el otro non. Luego d + b sera
non, y como d + b + bc = 2, bc debera ser non, lo que hace tanto a b como a c nones, de donde d es par. As pues ad es par y ad + bc = 2 no
puede ser, ya que ad es par y bc non, contradiccin.
20 Decompngase el conjunto en los n pares
{1, 2}, {2, 3}, . . . , {2n 1, 2n}.

Como siempre se tomarn dos enteros consecutivos, estos sern relativamente primos.
21 Cada entero es de la forma 2a m en donde a 0 es entero y m es impar. Como solamente hay n enteros impares en el conjunto {1, 2, . . . , 2n},
de los n + 1 enteros tomados, dos tendrn la misma parte impar, digamos 2a m y 2b m. Luego si a < b se tendr que 2a m dividir a 2b m. Se
dar otra solucin en el problema 47
22 Hay n residuos posibles diferentes cuando se divide a un entero por n, as entre n + 1 enteros diferentes habr dos dejando el mismo
residuo al dividirse por n. Su diferencia ser divisible por n.
23 20
24 Divdase al cuadrado en 4 subcuadrados congruentes, con lados paralelos al cuadrado
original. Dos de los cinco puntos caern en un

subcuadrado. Pero como la distancia mxima en un subcuadrado es su diagonal de 2/2 unidades de longitud, el resultado se cumple.
38 Hay 27 sumas distintas. Las sumas 1 y 27 se obtienen de manera nica (en 100 y 999). Cada una de las otras 25 sumas aparece al menos
tres veces. Luego si se sacan 27 + 25 + 1 = 53 boletas, al menos 3 tendrn la misma suma.

194

Indicaciones y respuestas

195

45 Sea P(n) la aseveracin: (1 + 2)2n + (1 2)2n es par y (1 + 2)2n (1 2)2n = b 2 para algn b N. Si n = 1, entonces vemos
que

(1 + 2)2 + (1 2)2 = 6,
que es par y que

(1 + 2)2 (1 2)2 = 4 2.

As P(1) cierta es. Presmase que P(n 1) es cierta para n > 1, i.e., presmase que

(1 + 2)2(n1) + (1 2)2(n1) = 2N
para algn entero N y que

(1 + 2)2(n1) (1 2)2(n1) = a 2

para algn entero positivo a.


Considrese ahora

(1 + 2)2n + (1 2)2n = (1 + 2)2 (1 + 2)2n2 + (1 2)2 (1 2)2n2 .

Esto es

(3 + 2 2)(1 + 2)2n2 + (3 2 2)(1 2)2n2 .

En utilizando P(n 1), esto simplifica a


12N + 2 2a 2 = 2(6N + 2a),

un entero par y de manera semejante

(1 + 2)2n (1 2)2n = 3a 2 + 2 2(2N) = (3a + 4N) 2,


estableciendo la veracidad de P(n).
46 Para n = 1, la aseveracin es cierta, ya que k2 1 = (k 1)(k + 1) es el producto de dos enteros pares consecutivos, y por lo tanto divisible
n
n+1
n+1
n
n
por 8. Hgase la hiptesis que 2n+2 |k2 1, y demostremos que 2n+3 |k2 1. Como k2 1 = (k2 1)(k2 + 1), vemos que 2n+2 divide
a(k2n 1), de donde el problema se reduce a demostrar que 2|(k2n + 1). Pero esto es obvio ya que el impar k2n torna a k2n + 1 par.
47 Este es el problema 47. Aqu se dar una solucin por induccin. Ntese que la solucin utilizando el principio de las pichoneras es ms
sucinta.
Supngase que de entre los 2n nmeros 1, 2, . . . , 2n, con n 2 , se han encontrado n + 1 nmeros tales que ninguno de ellos es divisible
por cualquier otro. Dentese este conjunto de n + 1 nmeros por Mn+1 . Se demostrar que si as occurriese sera posible seleccionar de entre
los 2n 2 nmeros 1, 2, . . . , 2n 2, un conjunto conteniendo n nmeros tales que ninguno de los n nmeros sea divisible por cualquier otro.
Se observan los siguientes casos:
1. Mn+1 no contiene al nmero 2n 1 ni al nmero 2n.
2. Mn+1 contiene a 2n 1 pero no a 2n.
3. Mn+1 contiene a 2n pero no a 2n 1.
4. Mn+1 contiene tanto a 2n 1 como a 2n.
Veamos
1. Qutese un nmero arbitrario del conjunto Mn+1 . Entonces quedan n nmeros ninguno de los cuales es mayor que 2n 2. Ninguno de
stos es divisible por cualquier otro.
2. Qutese el nmero 2n 1 del conjunto Mn+1 . Efectivamente, de nuevo, entre los n nmeros restantes, ninguno es mayor que 2n 2 y
ninguno de ellos es divisible por otro cualquiera.
3. Qutese el nmero 2n del conjunto Mn+1 ; el resultado es el mismo que en los casos 1 y 2.
4. Antes que todo, obsrvese que el nmero n no puede pertenecer al conjunto Mn+1 ; en caso contrario, el conjunto Mn+1 contendra a
dos los nmeros n y 2n; y 2n es divisible entre n. Qutese ahora los dos nmeros 2n 1 y 2n del conjunto Mn+1 . Dentese por Mn1
al conjunto de los n 1 nmeros que quedan. A continuacin agrguese el nmero n al conjunto Mn1 , obteniendo de este modo un
conjunto de n nmeros, ninguno de los cuales es mayor que 2n 2. Falta demostrar que de estos n nmeros, ninguno ser divisible
por cualquier otro. Como el conjunto Mn+1 no contuvo dos nmeros de los cuales uno fuera divisible por el otro, el conjunto Mn1
tampoco contendr tales nmeros. Por lo tanto, es slo necesario el demostrar que no existe dos nmeros tales, an cuando se agrega
el nmero n al conjunto Mn1 . Para hacerlo, basta demostrar (I) Que ningn nmero en Mn1 es divisible por n y (II) Que n no es
divisible por nmero alguno en Mn1 . La primera proposicin se deduce del hecho de que de los nmeros en Mn1 , ninguno es mayor
que 2n 2. La segunda se deduce del hecho de que 2n no es divisible por nmero alguno de en Mn1 . As se ha demostrado que si la
proposicin es falsa para los 2(n 1) nmeros 1, 2, . . . , 2n 2. De aqu que, si la proposicin es verdadera para los 2(n 1) nmeros
1, 2, . . . , 2n 2, tambin debe ser verdadera para los 2n nmeros 1, 2, . . . , 2n. La proposicin es verdadera para los dos nmeros 1 y 2;
de aqu que es verdadera para todos los conjuntos de 2n nmeros 1, 2, . . . , 2n, donde n es un nmero natural.

196

Anexo A

48 Para n = 1, tenemos
0 1 = f0 f1 = 12 (1)1 = f12 (1)1 ,

de donde la asercin es cierta para n = 1. Supongamos que n > 1, y que la asercin es cierta para n, esto es
fn1 fn+1 = fn2 + (1)n .
Usando fn+2 = fn+1 + fn , y por la hiptesis de induccin, fn2 = fn1 fn+1 (1)n . Esto significa que
fn fn+2

fn ( fn+1 + fn )

fn fn+1 + fn2

fn fn+1 + fn1 fn+1 (1)n

fn+1 ( fn + fn1 ) + (1)n+1

fn+1 fn+1 + (1)n+1 ,

de donde se colige el resultado.


49 Utilizarase induccin robusta. Obsrvese que 8 = 3 + 5, 9 = 3 + 3 + 3, 10 = 5 + 5, de donde se puede pagar 8, 9, o 10 pesos con las
susodichas monedas. Presmase que se puede pagar n 3, n 2, y n 1 pesos, esto es, que 3x + 5y = k tiene soluciones no negativas para
k = n 3, n 2 y n 1. Demostrarase que tambin se pueden obtener soluciones para 3x + 5y = k con k = n, n + 1 y n + 2. Ahora,
3x + 5y = n 3 = 3(x + 1) + 5y = n,
3x1 + 5y1 = n 2 = 3(x1 + 1) + 5y1 = n + 1,
3x2 + 5y2 = n 1 = 3(x2 + 1) + 5y2 = n + 2,
y as si las cantidades n 3, n 2, n 1 se pueden pagar, tambin se puede pagar las cantidades n, n+ 1, n+ 2. La aseveracin queda demostrada
por induccin robusta.
1
1+2
=
. Presmase que para k > 1
4 2(1 + 1)




1
1
1
k+2
1
1
1
=
.
4
9
2(k + 1)
(k + 1)2

50 El resultado es inmediato para n = 1 ya que 1

Por la hiptesis de induccin,


1

1
4



1
1
1
1
9
(k + 2)2

=
=
=
=

k+2
1
1
2(k + 1)
(k + 2)2
2

k+2
k + 4k + 3
2(k + 1)
(k + 2)2

k+2
(k + 1)(k + 3)
2(k + 1)
(k + 2)2
k+3
,
2(k + 2)

estableciendo el resultado para k + 1.


51 Para n = 0 esto es cierto, ya que 03 + 12 + 23 = 9. Presmase que k3 + (k + 1)3 + (k + 2)3 = 9N, en donde N es un entero. Se demostrar
que (k + 1)3 + (k + 2)3 + (k + 3)3 es tambin un mltiplo de 9. Pero
(k + 1)3 + (k + 2)3 + (k + 3)3 = 9N + (k + 3)3 k3 ,
en virtud de la hiptesis de induccin. Esto es
9N + (k + 3)3 k3 = 9N + (k3 + 9k2 + 27k + 27) k3 = 9N + 9k2 + 27k + 27 = 9(N + k2 + 3k + 3),
mltiplo de 9, como se quera demostrar.

Indicaciones y respuestas

197

52 El enunciado es obvio para n = 1. Si


1+

1
1
1
++ 2 < 2
2
n
2
n

para n > 1 entonces


1+

1
1
1
1
++
< 2 +
=.
n (n + 1)2
22
(n + 1)2

Se demostrar que para n > 1


2
Pero esto de inmediato resulta de la desigualdad
n(n + 1) < (n + 1)2 =

1
1
1
< 2
+
.
n (n + 1)2
n+1



1
1
1 1
1
1
1

<

=
1

=
.
2
n
n(n
+
1)
n
n
n
+
1
n
+
1
(n + 1)

53 Primero demostramos que todos los elementos de C poseen la misma paridad. De cierto, sea x C y sean A (con suma de elementos a)
y B (con suma de elementos b) dos subconjuntos con n elementos cada uno verificando

C \ {x} = A B;

A B = ;

a = b.

De manera semejante, sea y C y sean A (con suma de elementos a ) y B (con suma de elementos b ) dos subconjuntos con n elementos
cada uno verificando
C \ {x} = A B ;
A B = ;
a = b .

Si c es la suma de todos los elementos en C entonces c = x + a + b = x + 2a y tambin c = y + a + b = y + 2a . As y x = 2(a a) y x, y


tienen la misma paridad.
Ahora demostraremos la igualdad de todos los elementos por induccin, donde n se mantendr fijo y se inducir en el mximo de los
elementos de
C = {c1 , c2 , . . . , c2n+1 }.
Si

max

1i2n+1

= 1 entonces c1 = c2 = . . . = c2n+1 = 1, por ser todos los elementos estrictamente positivos. Presumse pues que el enunciado

es cierto cuando

max

1i2n+1

ci = t > 1. Sea
F = { f1 , f2 , . . . , f2n+1 }

un conjunto de enteros positivos no nulos verificando la propiedad del enunciado con

max

1i2n+1

fi = t + 1. . Tenemos dos casos: o bien todos

los elementos de F son pares o bien todos nones.


Si todos los elementos de F son pares, aplica la hiptesis de induccin a { f1 /2, f2 /2, . . . , f2n+1 /2} porque
max

1i2n+1

fi /2 = (t + 1)/2 < t

y al ser todas las fi /2 idnticas tambin lo sern las fi .


Si todos los elementos de F son nones, aplica la hiptesis de induccin a {( f1 + 1)/2, ( f2 + 1)/2, . . . , ( f2n+1 + 1)/2} porque
max ( fi + 1)/2 = (t + 2)/2 < t

1i2n+1

y al ser todas las ( fi + 1)/2 idnticas tambin lo sern las fi .


54 Razonamos por induccin sobre m + n. Como
n a1 + a2 + + an < mn,

se sigue que m > 1. De igual manera se demuestra que n > 1. Para m + n = 4 se tiene m = n = 2, y las igualdades posibles son 1 + 1 = 1 + 1
y 1 + 2 = 2 + 1, de donde se colige el resultado. Supngase que el resultado es cierto para k = m + n 4 y considrese
a1 + a2 + + an = b1 + b2 + + bn < mn,
donde m + n = k + 1. Sin perdida de generalidad se puede presumir que a1 es la mayor de todas las ai y que b1 es la mayor de todas las bi . Si
a1 = b1 no cabr nada que demostrar, pues se podrn suprimir estos trminos y se lograr una suma restante idntica. Si a1 > b1 entonces
(a1 b1 ) + a2 + + an = b2 + + bn ,
y

mn
= n(m 1).
m
Como n + (m 1) = k, podemos aplicar la hptesis de induccin, obteniendo el resultado.
b2 + + bn < mn

198

Anexo A

55 Las figuras A.1, A.2, y A.3 proveen una descomposicin en 4, 6, y 10 subtringulos, respectivamente. As pues, dado n, se puede construir
o bien n + 3 tringulos o n + 5 tringulos. Obsrvese ahora que toda n 6 puede ser escrita de la manera 3x + 5y = n, lo que se puede
demostrar con otra induccin, a la manera del problema 49.

Figura A.1: 4 subtringulos equilteros.

Figura A.2: 6 subtringulos equilteros.

Figura A.3: 10 subtringulos equilteros.

56 Si s fuese una potencia de 2 entonces no hay nada que demostrar. Si s yace estrictamente entre dos potencias de 2, digamos 2r1 < s < 2r ,
entonces s < 2r < 2s < 2r+1 , por lo que el intervalo [s; 2s] contiene a 2r , una potencia de 2.
61 Si la tuviere entonces
a2m+1 a2m = a2m a2m1

m = 2, 3, 4, . . . .

Como claramente se tiene am+1 + am > am + am1 se deber tener


am+1 am < am am1
para m 2. Esto se puede escribir como

a2 a1 > a3 a2 > a4 a3 > . . .

una sucesin infinita de enteros positivos estrictamente decreciente, lo que es imposible.


65 Sea O el centro del disco, y sean A1 , A2 , . . . A7 los siete puntos en cuestin. Si ninguno de ellos es el centro del disco, entonces el menor

entre los ngulos A


i OA j es estrictamente inferior a 60 . Sean A y B los puntos correspondientes a este ngulo. Por la ley de los cosenos de
Al-Kashi

AB2 = OA2 + OB2 2OA OB cos A


OB = AB < 1,
lo que es una contradiccin.
81 Ponga x = 123456789. Entonces x2 (x + 2)(x 2) = 4.
82 52
84 Pista:
22225555 + 55552222 = (22225555 + 45555 ) + (55552222 42222 ) (45555 42222 ).
158 Pista: n2 + 15n + 122 n2 + 3n + 2 = (n + 1)(n + 2) mod 6.
160 63
166 Pista: Demuestre primero que 320 1 mod 100.
179 2
180 $0,73
194 Los puntos 16, 17, . . . , 48 son 33 en total y estn del mismo lado del dimetro que une al punto 15 con el 49. Para cada uno de estos hay
un punto correspondiente y opuesto en la circunferencia. As pues hay un total de 2 33 + 2 = 68 puntos en total.
195 Los factores de 295 son 1, 2, 22 , . . . , 295 . Observemos que 210 = 1024 y por lo tanto 220 = 1048576. Luego 219 = 524288 < 1000000 <
1048576 = 220 . Por lo tanto, son los factores 220 , 221 , . . . 295 los mayores de 1000000. Estos constituyen un total de 95 20 + 1 = 76 factores.
196 En utilizando

Indicaciones y respuestas

199

91 = 9

enteros de 1-dgito,

90 2 = 180

enteros de 2-dgitos,

900 3 = 2700

enteros de 3-dgitos,

un total de 9 + 180 + 2700 = 2889 dgitos han sido utilizados, as pues el 3000-avo dgito debe de pertenecer a los enteros positivos de
4-dgitos. Quedan pues 3000 2889 = 111 dgitos para ser usados y como 111 = 4 27 + 3, el 3000-avo dgito es el tercer dgito del 28-avo
entero positivo de 4-dgitos, esto es, el tercer dgito de 4027, es decir, el 2.
197 Presumiremos conocido el que los enteros naturales se pueden factorizar en factores primos de una manera nica. Entonces pues, al
expandir el producto
(1 + 2 + 22 + + 28 )(1 + 3 + 32 + + 39 )(1 + 5 + 52 )

obtenemos todos los factores de 28 39 52 y slo factores de este nmero. As pues, hay tantos factores como trminos en el producto. Por lo
tanto, hay (1 + 8)(1 + 9)(1 + 3) = 320 factores.
La suma de los divisores la obtenemos sumando las tres series geomtricas anteriores:
29 1 310 1 53 1

= 467689684.
21 31 51
En general, si n = pa11 pa22 pas s , donde las ps son primos distintos y si d(n), (n) denotan, el respectivamente, el nmero de divisores
positivos de n y la suma de los divisores positivos de n, el razonamiento anterior nos dice que
d(n) = (a1 + 1)(a2 + 1) (as + 1)
y

(n) =

pa11 +1 1 pa22 +1 1
pas +1 1

s
.
p1 1
p2 1
ps 1

198 Para escribir las primeras nueve pginas, se utilizaron nueve dgitos. Para escribir las 99 10 + 1 = 90 pginas entre la 10 y 99 inclusas,
se utilizaron 2 90 = 180 dgitos. Hasta ahora hemos utilizado 189 dgitos. Si el libro llegase hasta la pgina 999, las 999 100 + 1 = 900
pginas de tres dgitos utilizaran 3 900 = 2700 dgitos, que es mucho ms que la cantidad de dgitos prescrita. As pues, el nmero de
pginas es un nmero de tres dgitos. Nos quedan 1890 189 = 1701 dgitos que usar, que nos dan para 1701/3 = 567 pginas ms. As pues,
contamos 567 pginas a partir de la 100. Esto quiere decir que el libro tiene 666 pginas.
200 800
269 3030
270 93
272

5973
1993

308 4
309 a = 998 = b
310 2400
311 9
312 1
314 384
315 580
317 Pista: x3 1 = (x 1)(x2 x + 1)
319 Pista: Demuestre primero que csc 2x = cot x cot 2x.

200

Anexo A

320 Pista: Observe que


y
1
1
=

.
2
1

y
1y
1 y2
324 Pista: De la identidad
tan x tan y =

tan x tan y
1 + tan x tan y

deduzca que
arctan a arctan b = arctan
325 Pista: De la identidad

ab
.
1 + ab

1
,
k+1 k =
k+1+ k

deduzca

1
2 k + 1 2 k < < 2 k 2 k 1.
k

Haciendo k = 2, 3, . . . , n, luego

n
X

1
< 2 n 2.
2 n+12 2 <
k
k=2

Como 2 2 < 3 y n + 1 > n se sigue que

X 1

< 2 n 1.
2 n2 <
k
n

k=1

Poniendo n = 1000000 se obtiene el resultado.


340 Pista: Poner un = cos vn .
343 Se tiene
( f (x))2 f
de donde

Substituya x por

1x
1+x


= 64x,

 
 2
1x
( f (x))4 f
) = 642 x2
1+x
1x
. Entonces
1+x


f

1x
1+x

2


f (x) = 64

Divida (I) por (II),


f (x)3 = 64x2


1x
.
1+x

(I)

(II)


1+x
,
1x

de donde se destila el resultado.


362 Pista: cos2 x = 1 sen2 x
370 Observe que

(x 1)x(x + 1)(x + 2) + 1 = (x2 + x)(x2 + x 2) + 1 = (x2 + x)2 2(x2 + x) + 1 = (x2 + x 1)2 = x2 + x 1.


As

30 31 32 33 + 1 = 312 + 31 1 = 991.

371 Se tiene
xx

.
x.

= 2 = x2 = 2 = x =

2,

ya que x es positivo.
372 Se tiene

q
x+

x+

x + = 2 = x + 2 = 2 = x + 2 = 4 = x = 2.

376 Pista: Ponga y = mx y divida las ecuaciones as obtenidas. Resuelva para m.

Indicaciones y respuestas

201

377 Pista: Escriba la ecuacin como


(x2 9x 1)10 10x9 (x2 9x 1) + 9x10 = 0.
383 Pista: Ponga u = x + 2, v = y + 3. Divida una ecuacin por la otra.
384 Pista: Ponga u = x + y, v = x y.
436 Como aB < Ab se tiene a(b+ B) = ab+ aB < ab+ Ab = (a+ A)b as,
A(b + B) y as,

a+A
A
< .
b+B
B

a a+A
<
. De manera semejante, B(a+ A) = aB+ AB < Ab+ AB =
b b+B

Por otra parte,


7
11
7
18 11
7
25 18 11
<
=
<
<
=
<
<
< .
10 15
10 25 15
10 35 25 15
25 5
5 11
4
7
Como
= , se tiene q 7. Podra ser q menor? Obsrvese que >
y que < . As, considerando caso a caso con denominadores
35 7
6 15
6 10
q = 1, 2, 3, 4, 5, 6, se ve que ninguna de estas fracciones yace en el intervalo deseado. Luego entonces el menor denominador es 7.
439 Obsrvese que para un entero k, 1 < k < n,

k(n k + 1) = k(n k) + k > 1(n k) + k = n. As,

n!2 = (1 n)(2 (n 1))(3 (n 2)) ((n 1) 2)(n 1) > n n n n = nn .


440 Sea
A=

9999
1 3 5

2 4 6
10000

B=

2 4 6
10000

.
3 5 7
10001

Evidentemente, x2 1 < x2 para todo real x. Esto implica que


x1
x
<
x
x+1
en tanto cada uno de los cuatro factores sea positivo. Luego
1/2

<

2/3

3/4

<

4/5

5/6

<

6/7

..
.

..
.

..
.

9999/10000

<

10000/10001

Como todos los nmeros involucrados son positivos, se multiplican ahora una y otra columna para obtener
1 3 5
9999
2 4 6
10000

<
,
2 4 6
10000
3 5 7
10001
o sea, A < B. Esto da A2 = A A < A B. Ahora bien,
1 2 3 4 5 6 7
9999 10000
1

=
,
2 3 4 5 6 7 8
10000 10001 10001

y en consecuencia A2 < A B = 1/10001. Se deduce que A < 1/ 10001 < 1/100.


AB =

202

Anexo A

441 Para i fijo,


n | ai |=| (n 1) ai (ai ) |=|

X
i6= j

 X
| ai a j |
ai a j |
i6= j

Sumando sobre i, se tiene,


n

n
X
X
| ai | 2
| ai a j |,
i< j

i=1

de donde se destila el resultado.


n
Obsrvese que para a1 = a2 = . . . = an1 = x y an (n 1) x, se obtiene igualdad, por lo que no puede ser reemplazada por un valor
2
mayor.
442 Pngase
Tm =

ak

1km

ak .

m<kn

Claramente, T0 = Tn . Como la sucesin T0 , T1 , . . . , Tn cambia de signo, eljase un ndice p tal que Tp1 y Tp tengan signos diferentes. O bien
Tp1 Tp = 2|a p |, o bien Tp Tp1 = 2|a p |. Se asevera que


mn Tp1 , Tp = max |ak | .
1kn




Si de caso contrario se tuviese Tp1 > max |ak | y Tp > max |ak |, entonces 2|a p | = |Tp1 Tp | > 2 max |ak |, contradiccin.
1kn

1kn

455 De las igualdades dadas se deduce que

1kn

n
X
(x2k xk )2 = 0.
k=1

Como una suma de cuadrados es 0 si y slo si cada trmino es 0, el resultado es inmediato.


456 De las igualdades dadas se deduce

1
(x1 x2 )2 + (x2 x3 )2 + + (xn1 xn )2 + (xn x1 )2 = 0.
2
Como una suma de cuadrados es 0 si y slo si cada trmino es 0, el resultado es inmediato.
457 Se tiene
(r s + t)2 t 2 = (r s + t t)(r s + t + t) = (r s)(r s + 2t).
Como t s 0, r s + 2t = r + s + 2(t s) r + s y as,
(r s + t)2 t 2 (r s)(r + s) = r2 s2
lo que resulta en
(r s + t)2 r2 s2 + t 2 .
458 Si a = b, no hay nada que demostrar. Presmase que a 6= b. Como

(b a)2 = ( b a)2 ( b + a)2 ,


se tiene

a+b
ab
2

=
=
=

Se nota ahora que 2 a b + a 2 b y por lo tanto

a + b 2 ab
2
( b a)2
2
(b a)2

.
2( b + a)2

1 (b a)2
(b a)2
1 (b a)2


2
8
b
8
a
2( b + a)

Indicaciones y respuestas

203

459 Como el cuadrado de todo real es positivo


(a 1)2 0 = a2 + 1 2a.

De la misma manera,

b2 + 1 2b,

Multiplicando estas tres desigualdades,

c2 + 1 2c.

(a2 + 1)(b2 + 1)(c2 + 1) 8abc,

como se quera demostrar.


460 Sean x 0, y 0 con x + y = 100. Entonces

x+y

xy
= xy 50 = xy 2500,
2

de donde el mximo producto es 2500.


461 De la desigualdad de la media para tres nmeros,


1=

a b c

b c a

1/3

a b c
+ +
a b c
b c a = 3 + + ,
3
b c a

demostrando la desigualdad deseada.


462 Usando la identidad
x3 + y3 = (x + y)3 3xy(x + y)
dos veces, se tiene,
a3 + b3 + c3 3abc

(a + b)3 + c3 3ab(a + b) 3abc

(a + b + c)3 3(a + b)c(a + b + c) 3ab(a + b + c)

(a + b + c)((a + b + c)2 3ac 3bc 3ab)

(a + b + c)(a2 + b2 + c2 ab bc ca)

Si a, b, c son positivos, entonces a + b + c 0 y adems como a2 + b2 + c2 ab bc ca 0 por la desigualdad 6.6. De aqu se colige que
a3 + b3 + c3
abc.
3
La desigualdad deseada se obtiene en poniendo u = a3 , v = b3 , w = c3 .
463 Como x 7 x sen x es estrictamente positiva en dicho intervalo,




4
4
9x2 sen2 x + 4
= 9x sen x +
2 (9x sen x)
= 2 9 4 = 12.
x sen x
x sen x
x sen x

Luego el valor mnimo es 12, que se obtiene cuando


9x sen x =
464 Ver que x2 x +

4
1
1
= x2 sen2 x =
= x sen x = .
x sen x
36
6



1
1 2
1
= x
0. Si todos los productos fuesen > hllese una contradiccin.
4
2
4

465 Se tiene


2 p
2
0 x y2 + 1 + y x2 + 1

p
x2 + y2 + 1 x y2 + 1 + y x2 + 1.

de donde

8
>
<
Habr igualdad si y slo si

x=

y2 + 1

p
>
: y = x2 + 1

, de donde x2 + y2 = x2 + y2 + 2, pero como esto es imposible, habr siempre desigualdad.

204

Anexo A

466 Por el ejemplo 451, se tiene


(a + b) (b + c) (c + a) 8abc.
Adems,

De la misma manera,
b

a2 (b c)2 =

a + b c a b + c.

b + a c b + c a,

c + a b c + b a.

As pues
abc (a + b c) (b + c a) (c + a b) ,
de donde se deduce el resultado.
467 La desigualdad propuesta es equivalente a

x y

y z


+

y z

z x

2
+

z x

x y

2
0,

que es trivial.
Obsrvese que igualdad ocurre si y slo si x = y = z.
468 La igualdad siniestra equivale a
(a + b + c)2 3 (ab + bc + ca) 0,
esto es,
1
[(a b)2 + (b c)2 + (c a)2 ] 0.
2
De aqu se obtiene el resultado, con igualdad si y slo si a = b = c. Ntese que esta igualdad es vlida cualesquiera sean los valores
estrictamente positivos a, b, c.
Para la desigualdad diestra, como a, b, c son las longitudes de los lados de un tringulo,
|a b| < c,

|b c| < a,

|c a| < b.

Entonces
4 (ab + bc + ca) (a + b + c)2 = c2 (a b)2 + a2 (b c)2 + b2 (c a)2 > 0.
469 Pngase x = a + b c, y = b + c a, z = c + a b. Entonces x > 0, y > 0, z > 0 y
a=
Ntese que

x+z
y+x
z+y
,b =
,c =
.
2
2
2

2

2 (x + y) x + y + 2 xy =
x+ y ,

cumplindose la igualdad si y slo si x = y. Luego,


a + b c + b + c a = x + y 2 x + y = 2 b.
De la misma manera

b + c a + c + a b 2 c,

c + a b + a + b c 2 a.

Sumando se obtiene la desigualdad deseada. La igualdad se cumple si y slo si


x = y = z, a = b = c.
470 Pngase a = x + y, b = y + z, c = z + x. Entonces a > 0, b > 0, c > 0 y
x2 z2 y2 x2 z2 y2
+
+
y+z
z+x
x+y

=
=

Ahora bien,
1
2

ac ba
+
b
c

(a b) c (b c) a (c a) b
+
+
b
c
a
ac ba bc
+
+ a b c.
b
c
a


=a

b2 + c2
a
2bc

Indicaciones y respuestas

205

con igualdad si y slo si b = c. De la misma manera




1 ac bc
+
c,
2 b
a

1
2

ba bc
+
c
a


b.

Sumando se obtiene

ac ba bc
+
+ a b c 0,
b
c
a
de donde se obtiene el resultado. Hay igualdad si y slo si a = b = c, esto es, si x = y = z.
471 Supngase primero que cada factor siniestro es positivo. Se tiene,




1
1
1
1
b1+ = b 1 +
= b 1+a
.
c
b bc
b
En consecuencia,

De aqu





1
1
1 2
a1+
b1+
c1+
(abc)2 = 1,
b
c
a





1
1
1 2
2
a1+
b1+
= b a 1
ba2 .
b
c
b
Procediendo de igual manera con los otros factores se deduce que






1
1
1
1
b1+
c1+
cb2 ,
c1+
a1+
ac2 .
c
a
a
b

obteniendo la desigualdad deseada.


1
1
1
Ahora bien, si por ejemplo, a 1 + < 0, entonces a < 1 y b > 1. Bajo estas condiciones b 1 + > 0 y c 1 + > 0, esto es, slo
b
c
a
uno de los factores es negativo, de donde nuevamente se obtiene la conclusin.
472 Sin prdida de generalidad se puede suponer que a b c. Es suficiente entonces demostrar que a + b > c.
Ahora bien,

a2 + b2 + c2 > 2 a4 + b4 + c4
si y slo si
(a + b + c) (a + b c) (b + c a) (c + a b) > 0.
Ya que tres de los factores son positivos, tambin lo habr de ser el cuarto, de donde se llega a la conclusin.
482 Aplicando la desigualdad de las medias a 1, 2, . . . , n:
n!1/n (1 2 n)1/n <

1+2+ +n n+1
=
,
n
2

habiendo desigualdad estricta cuando n > 1.


488 Usando CBS en

n
X

(ak bk )ck una vez, se obtiene

k=1
n
X
k=1

Usando CBS de nuevo

n
X

ak bk ck

n
X

!1/2
a2k b2k

n
X

k=1

!1/2
c2k

k=1

!1/2
a2k b2k

se obtiene

k=1
n
X
k=1

ak bk ck

lo que resulta en la desigualdad deseada.

n
X

!1/2
a2k b2k

k=1
n
X
k=1

!1/4
a4k

n
X
k=1
n
X
k=1

b4k

!1/2
c2k

!1/4

n
X
k=1

!1/2
c2k

206

Anexo A

489 Por CBS,

(1 x1 + 1 x2 + + 1 xn )2 12 + 12 + + 12 x21 + x22 + + x2n ,

de donde se obtiene el resultado.


490 El caso n = 3 ya se ha visto en el problema 472. Supngase ahora que n 4. Por simetra, es suficiente demostrar que a1 , a2 , a3 son las
longitudes de los lados de un tringulo. Por la desigualdad de CBS,

(n 1) a41 + a42 + . . . + a4n

<
=

De aqu,

2
a21 + a22 + . . . + a2n

!2
n
a21 + a22 + a23 a21 + a22 + a23 X 4
+
+
ak
2
2
k=4
!
2
2
n
X
a21 + a22 + a23
a21 + a22 + a23
4
(n 1)
+
+
ak .
4
4
k=4


2
2 a41 + a42 + a43 < a21 + a22 + a23 ,

obteniendo el resultado.
491 Sin prdida de generalidad presmase que a2 + b2 = 1. Entonces c2 + d 2 = 1. As,
3

2
c
d3
(ac + bd) c2 + d 2 = 1.
+
a
b
Ahora bien,
ac + bd

1 2
a2 + c2 b2 + d 2
+
=
a + b2 + c2 + d 2 = 1.
2
2
2

Esto quiere decir que


c3 d 3
1
+

1.
a
b
ac + bd
492 De CBS se obtiene

r
p

x1 y1 z1
x1+ y1+ z1 x+y+z
+
+
x
y
z

Ahora bien,
x1 y1 z1
+
+
= 3
x
y
z

1 1 1
+ +
x y z

= 1,

de donde se deduce la conclusin.


Habr igualdad si y slo si
x1 y1 z1
= 2 = 2 ,
x2
y
z

1 1 1
+ + = 2,
x y z

3
esto es, si x = y = z = .
2
1
1
1 1 1
1
493 Pngase a = , b = , c = . La restriccin xyz xy + yz + zx, es equivalente a + + 1. De aqu, a + b + c 1. Por otra parte,
x
y
z
x y z
xyz 3 (x + y + z) ab + bc + ca 1.
Ahora bien,
Gracias a CBS, se tiene

1 (a + b + c)2 = a2 + b2 + c2 + 2 (ab + bc + ca) .


ab + bc + ca

a2 + b2 + c2

b2 + c2 + a2 ,

lo que quiere decir que


As pues,

a2 + b2 + c2 ab + bc + ca.
1 3 (ab + bc + ca) ,

llegando a la conclusin.

Indicaciones y respuestas

207

494 Por CBS se tiene


S2 a2i

1
(S1 ai )2 .
n1

Se sigue que
S2 a2i
1

(S1 ai )
S1 ai
n1
y entonces
n
n
X
S2 a2k
1 X

(S1 ak ) = S1 .
S1 ak
n1
k=1

k=1

505 Por la desigualdad del reordenamiento,


n
X
a

k
k2

k=1

n
X
a
k=1

k
k2

n
X
1
k=1

ya que ak k y las as siendo enteros distintos estrictamente positivos.


506 Pngase
S=

b3
c3
d3
a3
+
+
+
,
b+c+d c+d +a d +a+b a+b+c

x = b + c + d,

y = c + d + a,

Por simetra y sin prdida de generalidad se puede presumir que a b c d. Entonces


an bn cn d n
para un entero n > 0 y
1 1 1 1
.
x
y
z
t
Ahora bien, por la desigualdad del reordenamiento,
a2 + b2 + c2 + d 2 ab + bc + cd + da = 1
y por la desigualdad de Chebyshev
S

1 1 1 1
1 3
+ + +
a + b3 + c3 + d 3
.
4
x y z t

Usando otra vez Chebyshev,


a3 + b3 + c3 + d 3

1 2
a + b2 + c2 + d 2 (a + b + c + d) .
4

De a2 + b2 + c2 + d 2 1 y 3 (a + b + c + d) = x + y + z + t se concluye,

1
1 1 1 1
16 1
S
(x + y + z + t)
+ + +

= .
48
x y z t
48 3

507 La desigualdad deseada es equivalente a


n
X
i=1

n
X
xi zi
xi yi ,
i=1

que se sigue inmediatamente de la desigualdad de reordenamiento.


508 Pngase an+1 = a1 . De la desigualdad de reordenamiento se tiene,
n
n
n
X
X
a2i
a2i X

=
ai ,
ai+1
ai
i=1

dando la conclusin.

i=1

i=1

z = d + a + b,t = a + b + c.

208

Anexo A

509 Por simetra se puede suponer, sin prdida de generalidad, que a b c. Entonces an bn cn y
desigualdad de reordenamiento

1
1
1

. Por la
b+c
c+a
a+b

an
bn
cn
an
bn
cn
+
+

+
+
b+c c+a a+b
a+b b+c c+a

an
bn
cn
an
bn
cn
+
+

+
+
.
b+c c+a a+b c+a a+b b+c

En resumen,
an
bn
cn
1
+
+

b+c c+a a+b 2


Por la desigualdad de Chebyshev,
an + bn

an + bn bn + cn cn + an
+
+
a+b
b+c
c+a

1 n1
a
+ bn1 (a + b) ,
2

de donde

an + bn
1 n1

a
+ bn1 .
a+b
2

De igual manera,

1 n1
bn + cn

b
+ cn1
b+c
2

cn + an
1 n1

c
+ an1
c+a
2

y as,
bn
cn
an1 + bn1 + cn1
an
+
+

.
b+c c+a a+b
2
510 Por simetra se puede suponer, sin prdida de generalidad que (xi ) la sucesin es creciente. Entonces tambin lo ser (ln xi ). Por la
desigualdad de Chebyshev
!
!
n
n
n
X
X
1 X
xi ln xi
xi
ln xi ,
n
i=1

i=1

i=1

o lo que es lo mismo,


n
Y
i=1

1
n

n
Y
xxi i
xi

n
P

xi
i=1


.

i=1

511 Por simetra se puede suponer, sin prdida de generalidad, que x y z. Entonces
1
1
1

.
(1 + y) (1 + z) (1 + z) (1 + x) (1 + x) (1 + y)
Por la desigualdad de Chebyshev,
x3
y3
z3
+
+
(1 + y) (1 + z) (1 + z) (1 + x) (1 + x) (1 + y)

Pngase

x3 + y3 + z3
1
1
1
[
+
+
]
3
(1 + y) (1 + z) (1 + z) (1 + x) (1 + x) (1 + y)
3
3
3
x +y +z
3+x+y+z

3
(1 + y) (1 + z) (1 + x)

x+y+z
= a. Luego por la desigualdad de las medias,
3
x3 + y3 + z3
a3 ,
3

3a = x + y + z 3 (xyz) 3 = 3,

(1 + y) (1 + z) (1 + x)

(1 + x) + (1 + y) + (1 + z)
3

As,
x3
y3
z3
6
.
+
+
a3
(1 + y) (1 + z) (1 + z) (1 + x) (1 + x) (1 + y)
(1 + a)3
Luego pues, es suficiente demostrar que
6a3
(1 + a)3

3
.
4

3
= (1 + a)3 .

Indicaciones y respuestas

Ya que a 1 y como

209

6a3


= 6 1

1
1+a

3

, el percatarse de que
(1 + a)

3
1
a 7 6 1
es estrictamente creciente sobre el intervalo ]0; +[, asegura la conclusin.
1+a
3

360
= 6 , y
582 Mdanse los ngulos en sentido dextrgiro, siendo el origen (0 ), a las 12 : 00. Cada minuto corrido por el minutero cuenta
60
40 2
as, de hora a hora, las manecillas viajan 5(6 ) = 30 . Cuando el minutero est en el 8, ste ha viajado
= de la circunferencia, esto es,
60 3
2
360 = 240
3
2
y el horario se ha movido del camino desde el 4 al 5, habiendo viajado
3


2
4+
(30 ) = 140 .
3
Luego, el ngulo entre una y otra manecilla es 240 140 = 100 .
583 La suma de los ngulos internos de un pentgono es 3 180 = 540 . Cada uno de los ngulos internos mide pues 540 5 = 108 . As
180 162

= 360 108 90 = 162 . Como AEF es issceles, se desprende que E


A
EF = 360 A
ED EDF
AF =
= 9 .
2

=A

+A

584 De la figura A.4 se percata que ADH


CT y que T HC es rectngulo issceles. Luego ADH
CH = 45 .

Figura A.4: Problema 584.

360
. El ngulo exterior debe ser lo suficientemente pequeo para que
n
360
cuando la figura rote 40 o 60 , se obtenga una figura homloga. Esto se satisfar cuando
divida al mximo comn divisor de 40 y
n

360
60 , que es 20 . Como
= 20 tiene la solucin n = 18, ste es el mnimo requerido.
n

585 Sea n el nmero de lados requerido. Cada ngulo exterior mide

591 Se tiene
AB

de donde, en resolviendo para DB, se obtiene el resultado.

AD + DB

AE + DB

AC EC + DB

AC CF + DB

AC (BC FB) + DB

AB BC + DB + DB

AB BC + 2DB,

210

Anexo A

592 Las tangentes a un crculo desde un punto son congruentes. As, AC = 20 + r y CB = 6 + r. La igualdad de reas revela que





1
1
1
(20 + r)(6 + r) = r2 + 2
(20)(r) + 2
(6)(r) = 120 26r r2 = 0 = (30 + r)(4 r) = 0.
2
2
2
Como r > 0 se tiene r = 4.
593

594 Extindanse los lados del hexgono, como en la figura A.5. Como los ngulos del hexgono son 120 cada uno, el XY Z es equiltero.
Luego
ZA + AF + FX = XE + ED + DY,
y en restando los lados de los tringulos menores,
AB + AF + EF = EF + DE +CD = AB DE = CD FA,
dando una de las igualdades. La otra se obtiene de manera semejante.

F
X

B
b

b
b

C
b

Figura A.5: Problema 594.

= MAB.
De la misma manera, MAC es issceles y MCA
= MAC.
As pues
616 Como BAM es issceles, MBA
+ MCA
= MAB
+ MAC
= BAC.
d
MBA
Pero como la suma de los ngulos internos del ABC es

d
+ MCA
= ,
B
AC + MBA
d=
se deduce BAC

.
2

617 Obsrvese primero que por el teorema de Pitgoras, la longitud de cualquier cateto es menor que la longitud de la hipotenusa. Luego
AHA < AB,

BHB < BC,

CHC < CA = AHA + BHB +CHC < AB + BC +CA,

dando el resultado.
619 El AHC es rectngulo en H. [AH], [AM], [MC] son todos congruentes (en efecto, radios del circuncrculo del AHC) y por lo tanto
= 30 .
MHC

Indicaciones y respuestas

211

C = COM
B . Por lo tanto, BOE
639 En la figura A.6, BO = CO, OMC = OMB , gracias al teorema 633 y BOM
= COD. As BMC = CD y
AB = AC.

MB
b

bMC
b

Figura A.6: Problema 639.

649 Pngase AB = a, MN = x y CD = b. Sea h = h1 + h2 la altura del trapecio, siendo h1 la altura del trapecio ABNM y h2 la altura del
trapecio MNCD. Entonces
r
a+b
h1
a+b
h2
a+b
a+b
a2 + b2
= ,
=
=
= 1
= x =
.
2(a + x)
h
2(b + x)
h
2(a + x)
2(b + x)
2

650 6 3 2 .
653 7.
654 Sean D, E, F, los pies de las perpendiculares a los lados [BC], [CA] y [AB] respectivamente. Entonces PF = a, PE = 2a, PF = 3a, y
[ABC] = [APB] + [APC] + [CPA].

Por lo tanto

dandoa =

3
.
4

1
3
2

32

 2
1
1
1
3
= 3 a + 3 3a + 3 2a,
2
2
2
2

655 Supngase que c1 tiene lado c. Entonces se ve que c2 tiene lado c + 1, c3 tiene lado c + 2 y c4 tiene lado c + 3. De aqu se sigue que c5
tiene lado 4. Continuando este proceso se deduce que c6 tiene lado 2c + 1. El largo del rectngulo es pues
(2c + 1) + (c + 1) + (c + 2) = 4c + 4.
El cuadrado c7 tiene lado
c + 3 + 4 = c + 7.
El ancho del rectngulo es pues
(c + 7) + (c + 3) + (c + 2) = 3c + 12.
El cuadrado c8 tiene lado
c + 7 + 4 = c + 11.
Finalmente, dos lados opuestos del rectngulo tienen dimensiones 4c + 4 y
(c + 7) + (c + 11) = 2c + 18.
Como han de ser iguales se deduce que 4c + 4 = 2c + 18 = c = 7. Conclusin: el rectngulo tiene rea 33 32 = 1056.
657 Sea O el centro del crculo. Obsrvese que
[ABCA BC ] = 2[AOB] + 2[BOC] + 2[COA ].
Se tiene
[AOB] = [B OA],

[BOC] = [COB ],

[COA ] = [AOC].

As,
[ABCA BC ] = 2([B OA] + [COB ] + [AOC]) = 2[ACB ] = 2.

212

Anexo A

A
3A
658 Se A el rea del circulo exterior. La informacin dada estipula que las cuerdas dividen a A en dos regiones: una de rea y otra de
.
4
4
Divdase el rea A del crculo exterior en nueve regiones: cuatro rincones como el rea sombreada que se denominar 4P, cuatro entre los
rincones, que se denominar 4Q y el cuadrado, que se denominar S. As
A = 4P + 4Q + S.
Las cuerdas dividen ahora al crculo mayor en tres bandas, dos de rea 2P + Q y una de rea 2Q + S. El rea de la banda central y una de las
bandas exteriores se pueden ahora expresar como
2Q + S =

A
,
2

2P + Q =

A
.
4

Luego
2Q + S = 2(2P + Q) = S = 4P.
Si x es el lado del cuadrado central entonces P =

x2
S x2
= . El rea del crculo interior es I =
. Se colige que
4
4
2
x2
4
x2
2

P
=
I

1
.
2

659 Sea y en la figura adjunta, el rea de la regin amarilla, r el rea de la regin roja y b el rea de la regin azul. Se quiere r + b. Ahora
bien,
r + b + 2y =

R2
,
4

y+b =

La ltima igualdad resulta en


2y =
y substitutyendo,
r+b =

(R/2)2
R2
=
.
2
8

R2
2b,
4

R2
2y = 2b,
4

de donde r = b.
Se desea ahora hallar el rea de la regin azul. Para lograr esto, obsrvese que OCD es rectngulo en C. La mitad del rea de la porcin
azul se puede hallar en substrayendo del rea OCD el rea del OCD:

 2
R2 ( 2)
b 1
R
1 R R
=
.
= b =
2 4
2
2 2 2
8

El rea buscada es entonces


b + r = 2b =

( 2)R2
.
4

bD

Indicaciones y respuestas

213

663 nanse los centros A y C como en la figura 7.100. El punto B, directamente abajo del punto A, es la interseccin de las de rectas paralelas
d

ABC = y ACB
= , ABC es issceles. Evidentemente, AC = 2 + r, AB = 2 r,
a los lados del cuadrado que pasan por A y C. Como
2
4
BC = 2 r. Por el teorema de Pitgoras,
(2 r)2 + (2 r)2 = (2 + r)2 .
Esto simplifica a
r2 12r + 4 = 0,

de donde r = 6 4 2, escogiendo la raz menor que 2. Se puede tambin obtener a r observando que

2r
2

= sen =
.
2
4
2+r
664 Sea [AD] tangente al crculo P en E. Entonces DE = DB y

AE 2 = AP2 PE 2 = 36 4 = 32 = AE = 4 2.
Adems,
que a su vez

AD2 = BD2 AB2 = (AE + BD)2 = 64 + BD2 = (4 2 + BD)2 = 64 + BD2 ,

= 32 + 8BD 2 + BD2 = 64 + BD2 = BD = 2 2.

665 Sean R y rlos radios del crculo externo e interno, respectivamente. El rea buscada es (R2 r2 ). Por el Teorema de Pitgoras, R2 r2 =
20
( )2 = 100. El rea deseada es pues 100 .
2

O
b

C
666 nase los centros de ambos crculos, como en la figura. Obsrvese que R = a + 2r, en donde r es el radio del crculo menor y a es la
medida del segmento de recta fuera del crculo menor desde el centro del crculo mayor. Trcense dos radios del crculo menor, paralelos a
los dos radios del crculo mayor, obteniendo un cuadrado de lado r. En utilizando el teorema de Pitgoras,

r2 + r2 = (a + r)2 = a = ( 2 1)r,
ya que a > 0. Finalmente,

R
R = a + 2r = ( 2 1 + 2)r = r =
= ( 2 1)R.
2+1

689 Trace [KL] k [AB] entre [KL] y [EF]. Use el teorema 632. Resulta que EF = 9.
690 Gracias a la frmula de Hern, [ABC] = 84. ABC A BC y por lo tanto [A BC ] = 84k2 donde k es la constante de semejanza. El
rea del A BC tambin puede obtenerse al substraer l rea de los tres trapezoides de altura 2 formados al unir los vrtices correspondientes
de los tringulos. As pues
1
84k2 = 84 41(k + 1) = k = ,
2
84
al descartarse una raz extraa. El rea buscada es entonces
= 21.
4

214

Anexo A

b, se tiene por el teorema de la bisectriz,


691 Vase la figura A.7. Como [AD] es la bisectriz angular de A
8
CD
= .
DB 11
Sea CD = 8a y DB = 11a para alguna constante a. Como CM = MB, se tiene CM = 9,5a. Ahora bien, DM = 1,5a. Como DM = 1 se tiene
2
16
22
a = . Luego CD =
y DB = . Sea x = HD. Luego
3
3
3
82 (

22
16
x)2 = AHA2 = 112 ( + x)2 = HA MA = 1 + x = 2,25.
3
3

b b

HA

DMA

Figura A.7: Problema 691.

Indicaciones y respuestas

215
b

692 Sea E la proyeccin perpendicular de E sobre el segmento [AB]. Por Pitgoras,


AB =

AD2 + BD2 =

162 + 122 =

42

42 + 32 = 20.

Since
BD 3 AC
= =
,
AD 4 AB

AE = A
BE y por lo tanto AE = EB. Se sigue que ABE es issceles en E. Como EE es la altura a la base de 20 de
ABC DAB. As, B

un tringulo issceles, AE = E B = 10. Por otro lado, AE E BAC y por lo tanto


EE
AE
1
15
=
= = EE = .
AC
AB
2
2
Para completar,
[AEB] =

1
15
1
AB EE = 20
= 75.
2
2
2

693 Pngase
KA = [BCO]

KB = [CAO]

KC = [ABO].

Entonces
AO
[ABO]
[CAO]
[ABO] + [CAO]
KB + KC
=
=
=
=
.
OA
[BOA ] [OAC] [BOA ] + [OAC]
KA
De manera semejante se puede demostrar que
KA + KC
BO
=
OB
KB

CO
KA + KB
=
.
OC
KC

Se deduce que
92 =

KB + KC KA + KC KA + KB
+
+
= KA2 KB + KA2 KC + KB2 KA + KB2 KC + KC2 KA + KC2 KB = 92KA KB KC .
KA
KB
KC

As
KA2 KB + KA2 KC + KB2 KA + KB2 KC + KC2 KA + KC2 KB + 2KA KB KC
AO BO CO
KB + KC KA + KC KA + KB

=
= 94.
OA OB OC
KA
KB
KC
KA KB KC

ya que son opuestos por el vrtice. Tambin B

EF = CED,
DC = B
FC, ya que subtienden el mismo arco. As
694 En la figura 7.121, B
EFB EDC. Luego
FE
EB
=
ED EC
o
FE
5
= ,
3
1
de donde FE = 15. Si r es el radio del crculo entonces 2r = FC = FE + EC = 16, de donde r = 8.
695 Sea M el punto medio de BD. Del ejemplo 833, MP y MS son iguales y perpendiculares, y tambin MQ y MR son iguales y perpendiculares. Un giro de 90 toma al PMR al QMS. Por lo tanto, PR y QS son iguales y perpendiculares.
696 Sea A el rea de uno de los tringulos rectngulos de los rincones (con hipotenusa 3). El rea buscada es 25 4A. Ahora, cada uno de
estos pequeos tringulos es semejante a los tringulos rectngulos mayores (de catetos 5 y 3). Como la hipotenusa de estos tringulos est en

15
15
135
135 290
proporcin, 3k = 34, para alguna constante k. Luego, Ak2 = . Esto resulta en A = 2 =
. Finalmente 25 4A = 25
=
.
2
68
17
17
2k
697 5 : 1

216

Anexo A

698 Sea b el nmero de unidades cuadradas pintadas en azul y y el nmero en amarillo. De tringulos semjantes,
b
120 + 90 + b
=
252 252 + 105 + y
y
252 + y + b
y
=
.
105 120 + 105 + 90
Luego y = 210, b = 168.
AC
BC
699 Sea x = AC. Entonces x + 3x + 3x = 84 = x = 12. De aqu FC =
= 6 y FG = 3. Adems DC =
= 18. Se deduce que el
2
2
permetro buscado es 39.

700 Tmese K sobre FG tal que [DK] k [EF]. Ya que GDK GBF, se sigue que
DK
DG
9y
=
= DK = .
BF
BG
5
Como HFE HKD, se sigue que

La razn requerida es

DH
DK
DK
3 DK
3 3x
9
=
=
=
=
= .
HE
EF
2y
2 3y
2 5x 10

9
.
10

701 Sea N el punto medio del segmento [PQ]. Por tringulos semejantes, RD = 2MN, de donde se infiere que MN =
Obsrvese que a = (AD)(DC) = (AB)(BC) y que
[ABCD]

=
=
=
=
=

As,

DA
CD
y DN =
.
4
4

[AMND] + [APM] + [CPMN] + [APB]






DA + MN
CP + MN
AB PB
DN + b +
NC +
2
2
2




5DA CD
3CB 3CD AB BC
+b+
+
8
4
8
4
4
5a
9a a
+b+
+
32
32 4
22a
+b
32

a 16
= .
b
5

D
b

M
R

Q
b

719 Sean los dos puntos A y B y la recta L . Si AB k L , constryase la mediatriz de [AB]. Esta intersecar a L en algn punto, dgase P, y
a [AB] en su punto medio, llmese Q. Sea O el punto medio de [PQ]. El crculo de centro O y radio OP cumple las condiciones deseadas.

Si AB no es paralela a L entonces las dos rectas se intersecarn en un punto, llmese T . Se tiene que localizar el punto de tangencia P
del crculo con la recta. Para esto, obsrvese que P debe satisfacer
T P2 = TA T B,
luego P se construye con el proceso usado para construir la media proporcional, conocidos T, A, B. Observe que hay dos posibilidades para P.

Erjase ahora la perpendicular 1 a L desde P y la mediatriz 2 del [AB]. El centro del crculo buscado es O = 1 2 .

Indicaciones y respuestas

217

736 Sean P y P las proyecciones perpendiculares del centro del crculo superior sobre los segmentos [AB] y [AC] respectivamente. Sean Q y
Q las proyecciones perpendiculares del centro del crculo superior sobre los segmentos [AC] y [BC] respectivamente. Obsrvese que
AP = AP ,

CQ = CQ ,

AP + 2R +CQ = 5.

Pngase AP = AP = x y CQ = CQ = y. En considerando tringulos rectngulos apropiados se obtiene

b
x
A
= cot ,
R
2
Luego,
x + 2R + y = 5 = R cot

Adems,

b
y
C
= cot .
R
2

b
b
A
C
+ 2R + R cot = 5 = R =
2
2

5
.
b
A
Cb
cot + cot + 2
2
2

b
A
2
1
1 + cos A
b cos 2
A
2
2
2+5
cot =
=
=
= 3.
2
b
12
1 cos A
A
2
5
2
2
sin
2

De la misma guisa,

De aqu, R =

Cb
1
3
1 + cosC
b cos 2
C
2 + 10
2
2
=
= 2.
=
cot =
2
b
cosC
3
1
1
C
2 10
2 2
sin
2

5
5
= .
2+3+2 7

Aliter: Tengan los crculos centros O y O como en la figura adjunta. Sea S la proyeccin perpendicular de O sobre [AB] y sea T la
proyeccin perpendicular de O sobre [SO ]. Se tiene
6 = [ABC] = [O BC] + [O AC] + [O AB].
Porque el crculo de centro O es tangente a los lados [BC] y [AC], se tiene
5
[O AC] = R,
2

[O BC] = 2R.

1
Se hallar ahora [O AB] = (AB)(SO ). Obsrvese que ABC OT O . Ahora bien,
2
O T
OO
O T
2R
8R
=
=
=
= O T =
.
BC
AC
4
5
5
As,
SO = ST + T O = R +

8R 13R
=
5
5

y
1
3 13R 39R
[O AB] = (AB)(SO ) =
=
.
2
2 5
10
Finalmente,
6=

b
b

P
b

39R 5R
5
+
+ 2R = R = .
10
2
7

S
b

218

Anexo A

742 Si A es este ngulo, hallando el rea de dos maneras distintas,


s2 sen A =

dd
1

= sen A = = A = .
2
2
6

(8 2)
3
=
. Observe que cuatro de los vrtices del octgono provienen directa8
8
3

y los otros dos sern . Sean d > d las diagonales de los rombos. Como
mente de los rombos. As, dos de los ngulos del rombo sern
8
4
las diagonales de un rombo se bisecan en ngulos rectos y como bisecan los ngulos en los vrtices, se obtiene de la figura adjunta que
q

p
1 + 1 cos

cos
2
2
d

2+ 2
4
8
=
q
=
p
=
cot
=
= 2 + 1.

d
8
1 1 cos
2 2
sen
2
2
8
4

743 Cada ngulo interno de un octgono regular mide

d /2

/8

d/2
sen(p q)
.
746 tan p tan q =
cos p cos q

8
tan((n + 1) ) tan
>
>
>
sen
<
Sn =
n
>
>
>
: n
750

si sen 6= 0
si 0 mod 2
si

mod 2 .

7
21
20

758 Poner =

1
1
2 cos 2
1
. Entonces

=
=
.
7
sen sen 3
sen 3
sen 2

773 Osama deber viajar sobre dos segmentos de recta: primero desde (1, 1) hasta el origen (0, 0), y luego desde (0, 0) hasta (2, 1), evitando
a toda costa el segundo cuadrante. Esto es porque si a > 0,b > 0 entonces la recta que une a (b, 0) y (a, 0) yace en el segundo cuadrante
p
a2 + b2
y mide a2 + b2 unidades. La cucaracha pasa aqu unos
minutos. Pero en el camino sobre los ejes desde (b, 0) hasta (a, 0) es
2
a + b unidades de largo y la cucaracha slo invierte unos a + b minutos aqu. As pues, en tanto
a+b

a2 + b2
,
2

la cucaracha deber evitar el segundo cuadrante a toda costa. Pero por la desigualdad de la media
2ab a2 + b2 = (a + b)2 = a2 + 2ab + b2 2a2 + 2b2 = a + b

p
2 a2 + b2 ,

1
1
1
lo que significa que en tanto la velocidad de la cucaracha en el segundo cuadrante sea < ser mejor evitarlo. Ya que < , se sigue lo
2
2
2
anunciado.
774 Es suficiente demostrar la desigualdad en el caso de que todos a, b, c, d sean positivos. Para esto, pngase O = (0, 0), L = (a, b) y
M = (a + c, b + d). Por la desigualdad del tringulo OM OL + LM, en donde ocurre igualdad si y slo si los puntos son colineales. Pero
entonces,

p
p
(a + c)2 + (b + d)2 = OM OL + LM = a2 + b2 + c2 + d 2 ,
a
c
y la igualdad ocurre si y slo si los puntos son colineales, esto es, si = .
b d

Indicaciones y respuestas

219

776 sese la generalizacin del teorema de Minkowski y el hecho que 172 + 1442 = 1452 . El valor buscado es S12 .
777 Se tiene
Si Lt pasa por (2, 3) entonces
(t 2)(2) + (t + 3)(3) + 10t 5 = 0,
8
de donde t = . En este caso la recta es
11

25
135
30
x+ y
= 0.
11
11
11

Lt ser paralela al eje de x si el coeficiente de x desaparece, lo que ocurrir cuando t 2 = 0, o sea, t = 2. En este caso la recta es
y = 3.
Lt ser paralela al eje de y si el coeficiente de y es cero, lo cual necesita t + 3 = 0, o sea, t = 3. En este caso la recta es
x = 7.
La recta de ecuacin x 2y 6 = 0 tiene pendiente
t = 1/3. En este caso la ecuacin de la recta es

1
2t
2t
1
y Lt tiene pendiente
. Las rectas sern paralelas cuando
= or
2
t +3
t +3
2
10
5
5
x + y = 0.
3
3
3

1
2t
2t
1
. Las rectas sern perpendiculares cuando
= 4 or t = 2. En
La recta y = x 5 tiene pendiente y Lt tiene pendiente
4
4
t +3
t +3
este caso la recta pedida es
4x + y 25 = 0.
Si tal punto existiese, entonces pasara por las rectas verticales y horizontales de la familia de Lt arriba obtenidas. Por lo tanto (x0 , y0 ) =
(7, 3) es un candidato para tal punto. Como
(t 2)(7) + (t + 3)(3) + 10t 5 = 7t + 14 3t 9 + 10t 5 = 0
el punto (7, 3) pasa por todas las rectas Lt no importa cual valor de t.


799 Se tiene, por definicin de N , AN = N B, y por construccin, MN = N N. As,

MN

Igualmente,


MP = MB + MC,

Como


MN + N N


MN + MN


MA + AN + MB + BN


MA + MB


MQ = MC + MD,


NR = MR MN,


MR = MD + MA.


PQ = MQ MP,

se deduce,

NR PQ



(MR MN) (MQ MP)



(MD + MA MA MB) (MC + MD MB MC)

0.


De manera semejante se establece que NP = RQ. Luego, NPQR es un paralelogramo.

220

Anexo A


n
en nmero, existe una recta en el plano, llammosla D que no es paralela a ninguna de ellas. De
2
igual manera, existe una recta en el plano, llammosla D que no es paralela a D. Desplazando D paralelamente a si misma en la direccin de
D se pasan los puntos Ai uno a uno. Es suficiente detenerse cuando se hallan pasado p puntos.

805 Como las rectas Ai A j , i 6= j son

821 Por ser baricentros de sus respectivas rectas, existen reales a, b, c con
L = aB + (1 a)C,

M = bC + (1 b)A,

Se procede ahora de manera formal. Se tiene


2 3

2 3

6L7
6 7
6 7
6M 7 =
6 7
4 5
N

N = cA + (1 c)B.

1a

1b

1c

6A 7
6 7
6 7
6B 7 .
6 7
4 5
C

Para este sistema tener solucin no trivial es necesario y suficiente que

det

1a

1b

1c

= 0 abc + (1 a)(1 b)(1 c) = 0,

Como
BL 1 a
=
,
a
LC

CM
1b
=
,
b
MA

AN
1c
,
=
c
NB

de donde se obtiene el resultado.


824 Se tiene por la ley de los senos,

BL
AB sen BAL
AB sen Cb
(AB)2
=
=
=
.
(CA)2
LC CA sen LAC

CA sen
ABC
Como la divisin del segmento [BC] es externa, se debe tomar el signo negativo. De igual manera
CM
(BC)2
,
=
(AB)2
MA

AB
(CA)2
.
=
(BC)2
NB

El resultado se deduce ahora en multiplicando estas razones y en utilizando el Teorema de Menelao.


825 Sean ABC y A BC copolares en O. Aplquese el Teorema de Menelao al OBC (con puntos menelaicos L,C , B ), OCA (con
puntos menelaicos M, A ,C ) y al OAB (con puntos menelaicos N, B , A ). Entonces
BL CC OB

= 1,
LC C O B B

CM AA OC

= 1,
MA A O CC

AN BB OA
BL CM AN

= 1,

= 1 =

NB B O A A
LC MA NB

de donde L, M, N son colineales.


Recprocamente, sean ABC y A BC coaxiales en L, M, N. Encuntrense BB y CC en O. Ahora bien, MCC y NBB son copolares
en L. Luego, por lo ya demostrado en la primera parte de este problema, dichos tringulos son coaxiales, esto es los puntos A, A , O son
colineales. Se recoge que los tringulos ABC y A BC son copolares en O.
826 Por ser baricentros de sus respectivas rectas, existen reales a, b, c con

0 = aA B + (1 a)AC = bBC + (1 b)B A = cC A + (1 c)C B.


Luego
A B
A C

1a
,
a

B C
B A

1b
,
b

C A
C B

1c
c

Indicaciones y respuestas
y

221

AA = aAB + (1 a)AC,

BB = AB + bAC,

CC = (1 c)AB AC.

Obsrvese que de las igualdades arriba obtenidas se desprende que


BA CB AC
(1 a)(1 b)(1 c)

= +1
= +1.
abc
A C B A C B

Se distingue dos casos, dependiendo de si AA k BB o no.


Si AA k BB entonces ab = a 1 lo que implica que b(c 1) + 1 = 0, que a su vez implica que AA k CC .

si

Si AA no es paralelo a BB , entonces ab 6= a 1. Un punto P tal que AP = xAB + yAC yacer sobre las tres rectas AA , BB y CC si y slo
(1 a)x ay = 0,

bx + y = b,

x (1 c)y = c 1.

Este es un sistema de dos variables y tres ecuaciones, por lo que una de ellas es redundante. Ahora bien,
(1 a)x ay = 0,

bx + y = b = x =

ab
,
ab + 1 a

y=

b(1 a)
.
ab + 1 a

Estas soluciones debern satisfacer la tercera ecuacin y por lo tanto,

ab
b(1 a)(1 c)

= c1
ab + 1 a
ab + 1 a

ab b(1 a)(1 c) = (c 1)(ab + 1 a)

ab b + bc 2abc c + ca + 1 a = 0

(1 a)(1 b)(1 a) abc = 0

(1 a)(1 b)(1 c)
= +1,
abc

como se quera demostrar.



843 Sean OM, OR y OP vectores, desde un origen O arbitrario hasta el muelle, el roble y el pino, respectivamente. Sea r una rotacin de 90

hacia la derecha. Las condiciones del problema requieren vectores OX y OY que satisfagan


OX = OR + r(OR OM),



OY = OP r(OP OM).

As pues



OR + OP r(OR OP)
OX OY
=
+
2
2
2
es independiente de la posicin del muelle. Esto proporciona un algoritmo para encontrar el tesoro: tomad a P como el origen, luego el tesoro

PR + r(PR)
.
est en
2

BC. Luego
d y = B
860 Sea = ABB

CB CB sen
=
B A
AB sen

en el ABC

CP
CB sen
=
PC C B sen

en el C BC.

De aqu
CB AB CP C B
=
B A
PC
y as
CP
CB AB
=
.

PC
B A C B

222

Anexo A

Como AB = AC +C B, en substituyendo se tiene


CB AB
CB (AC +C B)
=

B A C B
B A C B

CB AC CB C B
=
+
B A C B B A C B
CB AC CB
=
+
B A C B B A
Por el Teorema de Ceva,

luego

de donde se obtiene el resultado.

CB AC BA

=1
B A C B AC
CB AC CA
=
,
B A C B A B

También podría gustarte